Chapter: Neuroanatomy; Topic: Brainstem Circulation; Subtopic: Arterial Supply of the Medulla Oblongata
Keyword Definitions:
Medulla oblongata: The lowest part of the brainstem, controlling vital functions like breathing, heart rate, and reflexes such as swallowing and coughing.
Vertebral artery: A branch of the subclavian artery that ascends through the cervical vertebrae to supply the brainstem and cerebellum.
Anterior spinal artery: Formed by branches of the vertebral arteries, supplying the anterior two-thirds of the medulla and spinal cord.
Basilar artery: Formed by the union of the two vertebral arteries, supplying the pons and upper medulla.
Posterior cerebral artery: A branch of the basilar artery that supplies the midbrain and occipital lobes but not the medulla.
Lead Question – 2014
Medulla is supplied by all except?
a) Basilar artery
b) Anterior spinal artery
c) Vertebral artery
d) Posterior cerebral artery
Explanation:
The medulla oblongata is supplied mainly by the vertebral arteries, anterior spinal artery, and posterior inferior cerebellar artery (PICA). The basilar artery contributes minimally to the upper medulla. However, the posterior cerebral artery supplies the midbrain and occipital lobes but does not supply the medulla. Hence, the correct answer is d) Posterior cerebral artery. This vascular arrangement is vital for maintaining vital autonomic functions.
1) Which artery forms the anterior spinal artery?
a) Vertebral artery
b) Basilar artery
c) Posterior inferior cerebellar artery
d) Posterior spinal artery
Explanation: The anterior spinal artery is formed by branches of the vertebral arteries near their junction at the medulla. It runs along the anterior median fissure and supplies the anterior two-thirds of the spinal cord and medulla. Hence, the correct answer is a) Vertebral artery.
2) The posterior inferior cerebellar artery (PICA) is a branch of which artery?
a) Basilar artery
b) Vertebral artery
c) Posterior cerebral artery
d) Internal carotid artery
Explanation: The PICA arises from the vertebral artery before it merges into the basilar artery. It supplies the dorsolateral medulla and inferior part of the cerebellum. Its occlusion results in lateral medullary (Wallenberg’s) syndrome. The correct answer is b) Vertebral artery.
3) Occlusion of which artery causes medial medullary syndrome?
a) Anterior spinal artery
b) Posterior spinal artery
c) Basilar artery
d) Posterior inferior cerebellar artery
Explanation: Medial medullary syndrome occurs due to occlusion of the anterior spinal artery. It affects the pyramids, medial lemniscus, and hypoglossal nerve, leading to contralateral hemiplegia, loss of proprioception, and ipsilateral tongue paralysis. Hence, the correct answer is a) Anterior spinal artery.
4) Which artery does not directly contribute to the blood supply of the medulla?
a) Vertebral artery
b) Posterior cerebral artery
c) Basilar artery
d) Anterior spinal artery
Explanation: The posterior cerebral artery supplies the midbrain, thalamus, and occipital cortex, but not the medulla. The vertebral, basilar, and anterior spinal arteries all supply portions of the medulla. Hence, the correct answer is b) Posterior cerebral artery.
5) A patient presents with hoarseness, loss of gag reflex, and contralateral body sensory loss. Which artery is most likely affected?
a) Anterior spinal artery
b) Posterior inferior cerebellar artery
c) Basilar artery
d) Posterior cerebral artery
Explanation: These are classic features of lateral medullary (Wallenberg’s) syndrome due to occlusion of the PICA. It affects the nucleus ambiguus, spinal tract of the trigeminal nerve, and spinothalamic tract. Hence, the correct answer is b) Posterior inferior cerebellar artery.
6) The basilar artery is formed by the union of which arteries?
a) Internal carotid arteries
b) Vertebral arteries
c) Anterior spinal arteries
d) Posterior cerebral arteries
Explanation: The basilar artery is formed by the union of the two vertebral arteries at the level of the pontomedullary junction. It ascends on the ventral surface of the pons and gives off branches to the pons, cerebellum, and inner ear. Hence, the correct answer is b) Vertebral arteries.
7) A 60-year-old man presents with tongue deviation to one side and contralateral hemiplegia. The lesion involves which artery?
a) Basilar artery
b) Anterior spinal artery
c) Posterior inferior cerebellar artery
d) Posterior cerebral artery
Explanation: The symptoms indicate medial medullary syndrome due to occlusion of the anterior spinal artery. It damages the hypoglossal nerve, pyramid, and medial lemniscus. Tongue deviation occurs ipsilaterally, and contralateral weakness occurs due to corticospinal tract involvement. Correct answer: b) Anterior spinal artery.
8) Which of the following arteries supplies the dorsal medulla including the gracile and cuneate nuclei?
a) Posterior spinal artery
b) Anterior spinal artery
c) Vertebral artery
d) Basilar artery
Explanation: The posterior spinal arteries supply the dorsal medulla, particularly the gracile and cuneate nuclei. These arteries arise from either the vertebral arteries or posterior inferior cerebellar arteries. Their occlusion leads to sensory loss for fine touch and proprioception. Hence, the correct answer is a) Posterior spinal artery.
9) A stroke involving the vertebral artery may present with which of the following symptoms?
a) Contralateral limb paralysis and ipsilateral facial weakness
b) Dysphagia, hoarseness, and ataxia
c) Aphasia and hemianopia
d) Facial numbness and upper limb weakness
Explanation: Vertebral artery occlusion can cause lateral medullary syndrome with dysphagia, hoarseness, ataxia, and loss of pain and temperature sensation. These features result from involvement of the nucleus ambiguus and inferior cerebellar peduncle. The correct answer is b) Dysphagia, hoarseness, and ataxia.
10) Which artery is commonly affected in brainstem infarction at the pontomedullary junction?
a) Basilar artery
b) Posterior cerebral artery
c) Anterior spinal artery
d) Posterior inferior cerebellar artery
Explanation: The basilar artery supplies the pons and upper medulla at the pontomedullary junction. Infarction here may cause cranial nerve deficits (VI, VII) and contralateral motor weakness. The posterior cerebral artery does not supply this region. Hence, the correct answer is a) Basilar artery.
Chapter: Neuroanatomy; Topic: Cerebral Circulation; Subtopic: Blood Supply of Basal Ganglia
Keyword Definitions:
Basal ganglia: A group of subcortical nuclei involved in motor control, emotion, and learning, including caudate nucleus, putamen, and globus pallidus.
Putamen: The lateral part of the lentiform nucleus, involved in regulating voluntary motor movements and learning.
Lenticulostriate arteries: Small penetrating branches from the middle cerebral artery that supply the putamen and internal capsule.
Anterior choroidal artery: A branch of the internal carotid artery supplying the optic tract, internal capsule, and hippocampus.
Posterior communicating artery: Connects the internal carotid and posterior cerebral arteries, forming part of the Circle of Willis.
Lead Question – 2014
Blood supply of putamen includes all except?
a) Medial striate arteries
b) Lateral striate arteries
c) Anterior choroidal artery
d) Posterior communicating artery
Explanation:
The putamen receives its major blood supply from the lateral striate arteries (branches of the middle cerebral artery) and medial striate arteries (from the anterior cerebral artery). The anterior choroidal artery contributes to its posterior part. The posterior communicating artery does not supply the putamen. Hence, the correct answer is d) Posterior communicating artery. The putamen is highly vascular and susceptible to infarction in MCA occlusion.
1) The lateral striate arteries are branches of which main artery?
a) Anterior cerebral artery
b) Middle cerebral artery
c) Posterior cerebral artery
d) Basilar artery
Explanation: The lateral striate arteries originate from the middle cerebral artery and supply deep structures like the putamen, caudate nucleus, and internal capsule. These arteries are often termed “arteries of stroke” because they are prone to rupture due to hypertension. The correct answer is b) Middle cerebral artery.
2) The anterior choroidal artery is a branch of which vessel?
a) Middle cerebral artery
b) Internal carotid artery
c) Posterior cerebral artery
d) Vertebral artery
Explanation: The anterior choroidal artery arises from the internal carotid artery just before it bifurcates. It supplies the optic tract, posterior limb of the internal capsule, and choroid plexus of the lateral ventricle. Infarction can cause contralateral hemiplegia and hemianopia. The correct answer is b) Internal carotid artery.
3) A hypertensive patient develops hemiplegia due to rupture of lenticulostriate arteries. Which structure is most affected?
a) Thalamus
b) Putamen
c) Cerebellum
d) Corpus callosum
Explanation: The lenticulostriate arteries supply the putamen and internal capsule. Hypertension may cause rupture, leading to intracerebral hemorrhage and contralateral hemiplegia. This clinical condition is known as “stroke in the internal capsule.” The correct answer is b) Putamen.
4) The posterior communicating artery connects which two major arteries?
a) Internal carotid and middle cerebral
b) Internal carotid and posterior cerebral
c) Anterior cerebral and basilar
d) Vertebral and posterior inferior cerebellar
Explanation: The posterior communicating artery forms a link between the internal carotid artery and posterior cerebral artery, completing the Circle of Willis. It mainly supplies the hypothalamus and optic chiasma regions but does not contribute to the putamen supply. Hence, correct answer is b) Internal carotid and posterior cerebral.
5) Which artery is most frequently involved in lacunar infarcts affecting the putamen?
a) Posterior cerebral artery
b) Middle cerebral artery
c) Anterior cerebral artery
d) Basilar artery
Explanation: The middle cerebral artery and its deep branches (lateral striate arteries) are the most common sources of lacunar infarcts in the putamen. These small vessel occlusions result in pure motor strokes or dysarthria. Hence, the correct answer is b) Middle cerebral artery.
6) A 65-year-old man presents with contralateral weakness and hemianopia. MRI shows infarction in the region supplied by the anterior choroidal artery. Which structure is involved?
a) Pons
b) Internal capsule
c) Cerebellum
d) Thalamus
Explanation: The anterior choroidal artery supplies the posterior limb of the internal capsule, optic tract, and choroid plexus. Infarction results in contralateral hemiplegia and homonymous hemianopia. Hence, the correct answer is b) Internal capsule. This artery plays an essential role in deep cerebral circulation.
7) Which part of the Circle of Willis directly contributes to the putamen blood supply?
a) Anterior cerebral artery
b) Middle cerebral artery
c) Posterior communicating artery
d) Vertebral artery
Explanation: The middle cerebral artery and anterior cerebral artery contribute through their striate branches to the putamen. The posterior communicating artery does not supply this structure. Hence, the correct answer is b) Middle cerebral artery.
8) A 52-year-old male with chronic hypertension presents with sudden hemiplegia and CT shows bleed in the lentiform nucleus. Which artery rupture is most likely?
a) Lateral striate artery
b) Posterior inferior cerebellar artery
c) Basilar artery
d) Vertebral artery
Explanation: The lateral striate arteries (MCA branches) supply the putamen and internal capsule. Their rupture causes hemorrhage in the lentiform nucleus, leading to contralateral paralysis. The correct answer is a) Lateral striate artery.
9) Which structure does the anterior choroidal artery not supply?
a) Optic tract
b) Hippocampus
c) Putamen
d) Choroid plexus
Explanation: The anterior choroidal artery supplies the optic tract, posterior limb of the internal capsule, hippocampus, and choroid plexus of the lateral ventricle. It does not significantly supply the putamen, which mainly receives blood from the lenticulostriate arteries. Hence, the correct answer is c) Putamen.
10) A patient with occlusion of the lateral striate arteries will most likely show which symptom?
a) Ipsilateral paralysis
b) Contralateral paralysis
c) Cerebellar ataxia
d) Facial nerve palsy
Explanation: Occlusion of lateral striate arteries damages the internal capsule and putamen, producing contralateral hemiplegia and sensory loss. These arteries are commonly involved in hypertensive hemorrhages. Hence, the correct answer is b) Contralateral paralysis.
Topic: Renal Circulation; Subtopic: Blood Supply of Kidney in Fetal and Neonatal Life
Keyword Definitions:
• Renal Artery: Major artery supplying blood to the kidney, usually arising from the abdominal aorta.
• Common Iliac Artery: Terminal branch of the aorta that divides into internal and external iliac arteries.
• Fetal Circulation: The circulatory system in the fetus, which includes temporary vessels like the umbilical arteries.
• Neonate: A newborn child, especially within the first 28 days after birth.
Lead Question – 2014
In a neonate, kidney is supplied by?
a) Internal pudendal artery
b) External iliac artery
c) Common iliac artery
d) Aorta
Explanation:
In the neonate, the kidney receives its blood supply from the common iliac artery. During embryonic development, the kidneys ascend from the pelvic region to the lumbar region, and their arterial supply changes from the common iliac to the aorta. However, in neonates, remnants of this early pattern persist temporarily before stabilization occurs. Thus, the answer is common iliac artery.
1. During fetal development, the kidneys receive blood supply from which arteries initially?
a) Umbilical arteries
b) Common iliac arteries
c) Internal iliac arteries
d) Median sacral artery
2. Which of the following statements about the development of renal arteries is true?
a) They arise from the thoracic aorta initially
b) They remain constant in position during development
c) They change position as kidneys ascend
d) They arise from internal iliac arteries permanently
3. A 1-month-old infant presents with an accessory renal artery. This is due to:
a) Persistence of embryonic vessels
b) Renal agenesis
c) Congenital adrenal hyperplasia
d) Malrotation of kidney
4. The final arterial supply of adult kidneys originates from:
a) Common iliac artery
b) Abdominal aorta
c) External iliac artery
d) Median sacral artery
5. Which artery directly gives rise to segmental arteries in the kidney?
a) Interlobar artery
b) Renal artery
c) Afferent arteriole
d) Arcuate artery
6. In fetal life, the kidneys are located in which region?
a) Pelvic region
b) Lumbar region
c) Thoracic region
d) Sacral region
7. A neonate’s renal artery originates lower than normal. What does this indicate?
a) Incomplete renal ascent
b) Polycystic kidney disease
c) Nephroptosis
d) Ectopic kidney
8. Which of the following arteries supplies the suprarenal gland?
a) Renal artery
b) Superior mesenteric artery
c) Median sacral artery
d) Common iliac artery
9. A 3-day-old newborn is found to have reduced renal perfusion. Which vessel’s constriction could cause this?
a) Common iliac artery
b) Umbilical artery
c) Renal artery
d) Internal pudendal artery
10. Which of the following changes occur in renal blood supply after birth?
a) Blood supply shifts from common iliac to aortic origin
b) Blood supply remains same
c) Blood supply shifts to external iliac artery
d) Blood supply decreases due to closure of umbilical arteries
11. A clinical case of ectopic kidney shows its blood supply derived from which artery?
a) Common iliac artery
b) Abdominal aorta
c) Superior mesenteric artery
d) Renal artery
Explanation:
The common iliac artery supplies the kidney in early fetal life. As the kidney ascends during development, its arterial source changes sequentially—from common iliac to abdominal aorta. Clinical remnants may persist as accessory renal arteries. Understanding these variations is crucial for renal surgery, transplantation, and diagnosing congenital anomalies.
Topic: Development of Kidneys and Their Blood Supply
Subtopic: Embryological Origin of Renal Arteries
Keyword Definitions:
Renal Arteries: Paired arteries arising from the abdominal aorta to supply the kidneys.
Common Iliac Artery: One of the terminal branches of the abdominal aorta dividing into internal and external iliac arteries.
Mesonephros: The embryonic kidney that functions temporarily before the metanephros develops.
Metanephros: The definitive kidney, developing in the pelvis and ascending during fetal life.
Aorta: The main arterial trunk supplying oxygenated blood to the systemic circulation.
Lead Question – 2014
Initially, renal arteries are branches of?
a) Internal pudendal artery
b) External iliac artery
c) Common iliac artery
d) Aorta
Explanation: In early embryonic life, kidneys develop in the pelvic region and receive their blood supply from branches of the common iliac arteries. As kidneys ascend, their arterial supply shifts sequentially to higher levels, finally deriving from the abdominal aorta. The transient arteries regress as the kidney ascends. Hence, the correct answer is c) Common iliac artery.
1. During kidney ascent, which artery becomes the final source of its blood supply?
a) Common iliac artery
b) Abdominal aorta
c) Internal iliac artery
d) Median sacral artery
Explanation: Initially supplied by branches from the common iliac artery, the kidney ascends to the lumbar region where it ultimately receives blood from the abdominal aorta. Lower vessels regress, and the renal arteries from the aorta become the definitive arterial supply. Therefore, the correct answer is b) Abdominal aorta.
2. Accessory renal arteries arise due to:
a) Failure of regression of primitive renal arteries
b) Abnormal division of the aorta
c) Persistent mesonephric arteries
d) Recanalization defects
Explanation: Accessory renal arteries result from the persistence of embryonic renal arteries that normally regress as the kidney ascends. They may arise from the aorta or iliac arteries and are common anatomical variations. These vessels are functionally important and supply specific kidney segments. Thus, the correct answer is a) Failure of regression of primitive renal arteries.
3. The kidney ascends from the pelvis to the lumbar region during which week of development?
a) 3rd week
b) 4th–5th week
c) 6th–9th week
d) 10th–12th week
Explanation: The metanephric kidneys initially lie in the pelvic region and ascend to their lumbar position between the 6th and 9th weeks of gestation. The ascent is due to body growth and decreased curvature of the embryo. Hence, the correct answer is c) 6th–9th week.
4. A patient with a pelvic kidney has its arterial supply most likely from:
a) Abdominal aorta
b) Common iliac artery
c) Superior mesenteric artery
d) Inferior mesenteric artery
Explanation: A pelvic kidney results when the kidney fails to ascend during development. It retains its early blood supply from the common iliac artery. The renal vessels are shorter, and this anomaly is often incidental but may cause ureteral obstruction. The correct answer is b) Common iliac artery.
5. During embryonic development, which structure gives rise to the definitive kidney?
a) Pronephros
b) Mesonephros
c) Metanephros
d) Wolffian duct
Explanation: The metanephros forms the permanent kidney. It appears in the 5th week, derived from the ureteric bud (collecting system) and metanephric blastema (nephrons). The pronephros and mesonephros regress. Thus, the answer is c) Metanephros.
6. (Clinical) An aberrant renal artery crossing the ureter anteriorly may cause:
a) Hydronephrosis
b) Renal vein thrombosis
c) Pyelonephritis
d) Renal failure
Explanation: An aberrant renal artery may cross the ureter anteriorly, compressing it and obstructing urine flow, leading to hydronephrosis. This condition causes flank pain and renal pelvis dilation visible on imaging. Thus, the correct answer is a) Hydronephrosis.
7. (Clinical) A 30-year-old male with pelvic kidney presents with hematuria. Imaging shows accessory arteries from the common iliac. The cause is:
a) Ectopic kidney with persistent early blood supply
b) Renal artery stenosis
c) Inferior mesenteric artery aneurysm
d) Abnormal ureteral insertion
Explanation: A pelvic kidney receives its blood supply from the arteries at its developmental level, usually the common iliac. Persistence of embryonic vessels leads to multiple accessory arteries. Hence, the correct answer is a) Ectopic kidney with persistent early blood supply.
8. (Clinical) A patient undergoing renal transplantation has the donor renal artery anastomosed with:
a) Internal iliac artery
b) Common iliac artery
c) External iliac artery
d) Inferior epigastric artery
Explanation: In renal transplantation, the donor renal artery is usually anastomosed to the external iliac artery because of its accessibility and size match. The renal vein is connected to the external iliac vein. This provides optimal graft perfusion. Therefore, the correct answer is c) External iliac artery.
9. (Clinical) During aortic aneurysm repair, which artery must be preserved to maintain kidney perfusion?
a) Mesenteric artery
b) Renal artery
c) Lumbar artery
d) Celiac trunk
Explanation: The kidneys are supplied by the renal arteries directly from the abdominal aorta. During aneurysm repair near the renal hilum, preserving these arteries is crucial to prevent renal ischemia or infarction. Hence, the correct answer is b) Renal artery.
10. (Clinical) A 40-year-old patient with renovascular hypertension is found to have stenosis of the renal artery. The cause of hypertension is due to:
a) Increased renin release
b) Decreased angiotensin II
c) Low aldosterone
d) Increased GFR
Explanation: Renal artery stenosis reduces renal perfusion pressure, stimulating the juxtaglomerular cells to release renin. This activates the renin-angiotensin-aldosterone system, increasing blood pressure. Hence, renovascular hypertension develops. The correct answer is a) Increased renin release.
Topic: Large Intestine Blood Supply
Subtopic: Watershed Areas and Ischemic Zones
Keyword Definitions:
• Watershed zone: Region of intestine between two arterial supplies prone to ischemia.
• Rectosigmoid junction: Area between inferior mesenteric and internal iliac artery supplies.
• Marginal artery of Drummond: Continuous arterial circle along colon’s inner border.
• Ischemic colitis: Inflammation caused by reduced blood flow in watershed areas.
Lead Question - 2014
Watershed zone of large intestine?
a) Cecum
b) Ascending colon
c) Rectosigmoid
d) Transverse colon
Explanation:
The correct answer is c) Rectosigmoid. The rectosigmoid junction represents a classic watershed area between the territories of the inferior mesenteric and internal iliac arteries. Due to dual supply borders, it is highly susceptible to ischemic colitis during low perfusion states, hypotension, or shock conditions, making it clinically significant.
1. The artery forming marginal artery of the colon is:
a) Superior mesenteric artery
b) Inferior mesenteric artery
c) Both a and b
d) Internal iliac artery
Explanation:
The correct answer is c) Both a and b. The marginal artery of Drummond is formed by anastomosis between branches of the superior and inferior mesenteric arteries. This continuous arterial arcade supplies the colon and provides collateral circulation, reducing the risk of ischemia except at the watershed areas like splenic flexure.
2. The splenic flexure is supplied by:
a) Superior mesenteric artery only
b) Inferior mesenteric artery only
c) Both SMA and IMA
d) Celiac artery
Explanation:
The correct answer is c) Both SMA and IMA. The splenic flexure represents another watershed zone, located between the terminal branches of the superior and inferior mesenteric arteries. Because of this dual supply, it becomes vulnerable during systemic hypotension and is a common site for ischemic colitis or mucosal necrosis.
3. The blood supply to the rectum is mainly from:
a) Superior rectal artery
b) Middle rectal artery
c) Inferior rectal artery
d) All of the above
Explanation:
The correct answer is d) All of the above. The rectum has a rich blood supply from three sources: the superior rectal artery (from IMA), middle rectal artery (from internal iliac), and inferior rectal artery (from internal pudendal). This overlapping supply helps maintain perfusion except in rectosigmoid ischemia zones.
4. Clinical case: A 65-year-old with hypotension develops abdominal pain and bloody stool. The most likely site of ischemia is:
a) Sigmoid colon
b) Rectosigmoid junction
c) Cecum
d) Descending colon
Explanation:
The correct answer is b) Rectosigmoid junction. In elderly or hypotensive patients, ischemia often occurs at watershed regions where dual arterial supplies meet, such as the rectosigmoid junction. The lack of sufficient collateral flow during low perfusion states leads to mucosal necrosis and abdominal pain with bloody diarrhea.
5. Which of the following arteries is a branch of the inferior mesenteric artery?
a) Ileocolic artery
b) Right colic artery
c) Left colic artery
d) Middle colic artery
Explanation:
The correct answer is c) Left colic artery. The inferior mesenteric artery gives off three main branches: left colic, sigmoid, and superior rectal arteries. The left colic artery supplies the descending colon and forms an anastomosis with the middle colic artery, contributing to the marginal artery of Drummond’s loop.
6. Clinical case: A patient post-surgery develops ischemia at the splenic flexure. Which vessel is involved?
a) Left colic artery
b) Middle colic artery
c) Both a and b
d) Right colic artery
Explanation:
The correct answer is c) Both a and b. The splenic flexure is supplied by terminal branches of the middle colic (SMA) and left colic (IMA) arteries. Any compromise in either vessel or systemic hypoperfusion can lead to ischemia, making this flexure another classical watershed region of the colon.
7. Which vessel directly continues as the superior rectal artery?
a) Internal iliac artery
b) Inferior mesenteric artery
c) External iliac artery
d) Common iliac artery
Explanation:
The correct answer is b) Inferior mesenteric artery. The superior rectal artery is the terminal continuation of the inferior mesenteric artery. It descends into the pelvis to supply the upper rectum. Its anastomoses with the middle and inferior rectal arteries maintain blood flow to the rectal region during reduced perfusion.
8. Clinical case: A patient presents with segmental ischemic colitis after shock. Which part of colon is most affected?
a) Transverse colon
b) Sigmoid colon
c) Splenic flexure
d) Ascending colon
Explanation:
The correct answer is c) Splenic flexure. The splenic flexure is a typical site for ischemic colitis because it lies between the blood supplies of SMA and IMA. Reduced perfusion during shock or atherosclerosis leads to necrosis in this zone, presenting clinically with abdominal pain and bloody stool.
9. The inferior mesenteric artery arises from:
a) Abdominal aorta at L1
b) Abdominal aorta at L3
c) Common iliac artery
d) Internal iliac artery
Explanation:
The correct answer is b) Abdominal aorta at L3. The inferior mesenteric artery originates from the anterior surface of the abdominal aorta at the level of L3 vertebra. It supplies the hindgut structures including the descending colon, sigmoid colon, and upper rectum through its terminal branches.
10. Clinical case: In a CT angiogram showing reduced flow through the inferior mesenteric artery, which area is least affected due to collaterals?
a) Splenic flexure
b) Rectosigmoid junction
c) Ascending colon
d) Descending colon
Explanation:
The correct answer is d) Descending colon. The descending colon receives rich collateral blood flow from both the left colic and middle colic arteries through the marginal artery of Drummond. Hence, it is relatively protected from ischemia compared to the splenic flexure and rectosigmoid regions in arterial compromise.
Topic: Spleen
Subtopic: Accessory Spleen
Keyword Definitions:
Accessory spleen: A small nodule of splenic tissue separate from the main spleen, often congenital.
Hilum of spleen: Site where splenic vessels enter and leave the spleen.
Splenic artery: Main arterial supply of the spleen, a branch of the celiac trunk.
Splenectomy: Surgical removal of the spleen, sometimes revealing accessory spleens.
Lead Question (2014): Most common location of accessory spleen?
a) Hilum of spleen
b) Greater omentum
c) Lesser omentum
d) None
Explanation: The most common location of an accessory spleen is at the hilum of the spleen or within the gastrosplenic ligament. These arise from failure of fusion of splenic tissue during embryonic development and are usually asymptomatic. Answer: a) Hilum of spleen
1. Which artery supplies an accessory spleen?
a) Left gastric artery
b) Splenic artery
c) Short gastric artery
d) Left gastroepiploic artery
Explanation: Accessory spleens are supplied by branches of the splenic artery, similar to the main spleen. This ensures their functional similarity and ability to take over partial splenic functions if required. Answer: b) Splenic artery
2. During splenectomy, failure to remove an accessory spleen can lead to?
a) Hypertension
b) Recurrence of hematologic disease
c) Portal thrombosis
d) Renal failure
Explanation: Failure to remove accessory spleens during splenectomy may lead to recurrence of hematologic disorders like ITP because the residual splenic tissue continues its immunologic functions. Answer: b) Recurrence of hematologic disease
3. Accessory spleen is most commonly found in which ligament?
a) Gastrosplenic ligament
b) Hepatogastric ligament
c) Falciform ligament
d) Phrenicocolic ligament
Explanation: Accessory spleens are frequently found in the gastrosplenic ligament, which connects the spleen to the greater curvature of the stomach. Answer: a) Gastrosplenic ligament
4. A 40-year-old male underwent splenectomy for ITP, but platelet count remains low. The cause could be?
a) Missed accessory spleen
b) Liver dysfunction
c) Bone marrow failure
d) Infection
Explanation: Persistence of low platelet count after splenectomy often indicates a missed accessory spleen, which continues to sequester platelets. Answer: a) Missed accessory spleen
5. Which imaging modality best detects accessory spleen?
a) CT scan
b) MRI
c) Ultrasonography
d) PET scan
Explanation: CT scan with contrast is the best imaging technique for detecting accessory spleens because it differentiates splenic tissue based on enhancement patterns. Answer: a) CT scan
6. Accessory spleen is derived from which embryological structure?
a) Dorsal mesogastrium
b) Ventral mesogastrium
c) Midgut loop
d) Septum transversum
Explanation: The spleen and accessory spleens develop from mesenchymal condensations in the dorsal mesogastrium during the 5th week of embryogenesis. Answer: a) Dorsal mesogastrium
7. The size of accessory spleen is usually less than?
a) 1 cm
b) 2 cm
c) 3 cm
d) 5 cm
Explanation: Accessory spleens are usually small, less than 2 cm in diameter, and composed of normal splenic tissue. Answer: b) 2 cm
8. In thalassemia patients after splenectomy, an accessory spleen may cause?
a) Persistent anemia
b) Jaundice
c) Hypertension
d) Renal failure
Explanation: In thalassemia, an accessory spleen may cause persistent anemia after splenectomy due to continued sequestration and destruction of red blood cells. Answer: a) Persistent anemia
9. Which of the following statements about accessory spleen is TRUE?
a) It is functionally inactive
b) It can undergo torsion
c) It never enlarges in hematologic disease
d) It always lies in retroperitoneum
Explanation: Accessory spleens are functionally active and may enlarge in hematologic disorders. Rarely, they can undergo torsion causing acute abdominal pain. Answer: b) It can undergo torsion
10. A surgeon accidentally finds a small splenic nodule near the pancreas tail. It represents?
a) Pancreatic pseudocyst
b) Accessory spleen
c) Lymph node
d) Neuroendocrine tumor
Explanation: A small, well-circumscribed nodule near the tail of the pancreas is typically an accessory spleen, arising due to splenic tissue inclusion during development. Answer: b) Accessory spleen
Topic: Blood Supply of Rectum and Anal Canal
Subtopic: Venous Drainage of Rectum
Keyword Definitions:
Superior rectal vein: Drains blood from the upper rectum and continues as the inferior mesenteric vein.
Inferior mesenteric vein: Drains into the splenic vein and joins the portal venous system.
Internal iliac vein: Drains blood from pelvic organs and walls.
Internal pudendal vein: Drains perineum and external genitalia.
Lead Question (2014): Superior rectal vein drains into?
a) Inferior mesenteric vein
b) External iliac vein
c) Internal iliac vein
d) Internal pudendal vein
Explanation: Superior rectal vein drains into the inferior mesenteric vein, which is a part of the portal venous system. Middle and inferior rectal veins drain into systemic veins. Hence, the superior rectal vein forms an important portosystemic anastomosis site. Answer: a) Inferior mesenteric vein
1. Middle rectal vein drains into?
a) Internal pudendal vein
b) Internal iliac vein
c) Inferior mesenteric vein
d) External iliac vein
Explanation: The middle rectal vein drains into the internal iliac vein. It connects the rectal venous plexus with the systemic venous system. This contributes to portosystemic anastomosis. Answer: b) Internal iliac vein
2. Inferior rectal vein drains into?
a) Internal pudendal vein
b) External iliac vein
c) Superior rectal vein
d) Internal iliac vein
Explanation: The inferior rectal vein drains into the internal pudendal vein. This vein communicates between the lower rectum and perineal venous plexus. Answer: a) Internal pudendal vein
3. Which of the following veins forms part of the portosystemic anastomosis?
a) Superior rectal vein
b) Internal pudendal vein
c) Femoral vein
d) Obturator vein
Explanation: Superior rectal vein forms part of the portosystemic anastomosis by connecting the portal and systemic circulations through the middle and inferior rectal veins. Answer: a) Superior rectal vein
4. During portal hypertension, which condition may occur due to venous congestion in the rectal plexus?
a) Hemorrhoids
b) Fistula
c) Abscess
d) Anal fissure
Explanation: Portal hypertension causes venous congestion in the superior rectal vein, leading to dilatation of rectal veins and development of hemorrhoids (piles). Answer: a) Hemorrhoids
5. The superior rectal vein belongs to which venous system?
a) Systemic venous system
b) Portal venous system
c) Caval venous system
d) Vertebral venous system
Explanation: The superior rectal vein belongs to the portal venous system because it drains into the inferior mesenteric vein, which joins the splenic vein to form the portal vein. Answer: b) Portal venous system
6. A patient with portal hypertension develops rectal varices. Which veins are dilated?
a) Superior rectal veins
b) Internal iliac veins
c) Middle rectal veins
d) Inferior rectal veins
Explanation: Rectal varices result from dilation of superior rectal veins due to portal hypertension, leading to communication with systemic veins via the middle and inferior rectal veins. Answer: a) Superior rectal veins
7. Which vein drains the upper anal canal?
a) Superior rectal vein
b) Middle rectal vein
c) Inferior rectal vein
d) Internal pudendal vein
Explanation: The upper anal canal, derived from endoderm, is drained by the superior rectal vein, which connects to the portal venous system via the inferior mesenteric vein. Answer: a) Superior rectal vein
8. In portal hypertension, which type of hemorrhoids are formed due to superior rectal vein involvement?
a) Internal hemorrhoids
b) External hemorrhoids
c) Mixed hemorrhoids
d) None
Explanation: Internal hemorrhoids develop due to dilation of veins in the upper anal canal, involving the superior rectal veins, which belong to the portal venous system. Answer: a) Internal hemorrhoids
9. The inferior mesenteric vein drains into which of the following?
a) Splenic vein
b) Portal vein directly
c) Superior mesenteric vein
d) Hepatic vein
Explanation: The inferior mesenteric vein drains into the splenic vein, which later joins the superior mesenteric vein to form the portal vein. Answer: a) Splenic vein
10. In a surgery involving sigmoid colon, the superior rectal vein is injured. Which complication can occur?
a) Rectal bleeding
b) Constipation
c) Intestinal perforation
d) Hematuria
Explanation: Injury to the superior rectal vein can lead to rectal bleeding due to disruption of venous drainage from the upper rectum. Prompt hemostasis is necessary to prevent hemorrhage. Answer: a) Rectal bleeding
Topic: Venous Drainage of the Anterior Abdominal Wall
Subtopic: Inferior Epigastric Vein
Keyword Definitions:
Inferior epigastric vein: A vein that accompanies the inferior epigastric artery, draining blood from the lower part of the anterior abdominal wall.
External iliac vein: Major pelvic vein that continues as the femoral vein below the inguinal ligament and receives tributaries including the inferior epigastric vein.
Rectus sheath: Fibrous sheath enclosing the rectus abdominis muscle, formed by aponeuroses of the abdominal wall muscles.
Superficial epigastric vein: Vein accompanying the superficial epigastric artery, draining into the great saphenous vein.
Portosystemic anastomosis: Connection between portal and systemic venous systems, important in portal hypertension.
Linea alba: Fibrous midline structure where abdominal aponeuroses fuse, serving as a landmark for venous drainage zones.
Lead Question – 2014
Inferior epigastric vein drains into?
a) Femoral vein
b) External iliac vein
c) Internal iliac vein
d) Internal pudendal vein
Explanation: The inferior epigastric vein accompanies its artery and drains into the external iliac vein just above the inguinal ligament. It plays a role in collateral circulation between the femoral and internal thoracic veins. (Answer: b)
1) The inferior epigastric artery arises from:
a) External iliac artery
b) Internal iliac artery
c) Femoral artery
d) Inferior mesenteric artery
Explanation: The inferior epigastric artery arises from the external iliac artery just above the inguinal ligament and runs upward in the rectus sheath behind the rectus abdominis. (Answer: a)
2) (Clinical) Injury to inferior epigastric vessels during surgery occurs in:
a) Umbilical region
b) Lateral to deep inguinal ring
c) Medial to deep inguinal ring
d) Hypogastric region
Explanation: The inferior epigastric vessels lie just medial to the deep inguinal ring and may be injured during hernia repair or laparoscopic trocar insertion. (Answer: c)
3) The inferior epigastric vein forms an anastomosis with:
a) Superior epigastric vein
b) Superficial epigastric vein
c) Paraumbilical vein
d) Great saphenous vein
Explanation: The inferior epigastric vein anastomoses with the superior epigastric vein within the rectus sheath, forming a communication between femoral and internal thoracic venous systems. (Answer: a)
4) (Clinical) During portal hypertension, which vein may become dilated near the umbilicus?
a) Inferior epigastric vein
b) Paraumbilical vein
c) Great saphenous vein
d) Internal iliac vein
Explanation: In portal hypertension, the paraumbilical veins connect with the inferior and superior epigastric veins, leading to caput medusae appearance. (Answer: b)
5) The inferior epigastric vein lies:
a) Superficial to rectus abdominis
b) Within rectus abdominis
c) Behind rectus abdominis
d) Deep to transversalis fascia only
Explanation: The inferior epigastric vein lies behind the rectus abdominis muscle within the rectus sheath and accompanies its corresponding artery. (Answer: c)
6) (Clinical) A laceration below the arcuate line may cause bleeding from:
a) Superior epigastric artery
b) Inferior epigastric vessels
c) Deep circumflex iliac vessels
d) Internal thoracic artery
Explanation: Below the arcuate line, only the transversalis fascia covers the inferior epigastric vessels, making them prone to injury and significant bleeding in trauma or surgery. (Answer: b)
7) Inferior epigastric vein joins the external iliac vein:
a) Above the inguinal ligament
b) Below the inguinal ligament
c) At the level of pubic tubercle
d) Near the umbilicus
Explanation: The inferior epigastric vein joins the external iliac vein above the inguinal ligament, accompanying its artery through the transversalis fascia. (Answer: a)
8) (Clinical) In an indirect inguinal hernia, inferior epigastric vessels are located:
a) Medial to hernial sac
b) Lateral to hernial sac
c) Posterior to hernial sac
d) Superior to hernial sac
Explanation: In indirect inguinal hernia, the inferior epigastric vessels lie medial to the neck of the hernial sac, serving as an important landmark in differentiating hernia types. (Answer: b)
9) The inferior epigastric vein is formed by:
a) Two venae comitantes
b) A single trunk
c) A plexus of small veins
d) Fusion with superficial veins
Explanation: The inferior epigastric vein is formed by two venae comitantes accompanying the artery, which unite before draining into the external iliac vein. (Answer: a)
10) (Clinical) During laparoscopic trocar placement near the midline, which vessel must be avoided?
a) Inferior epigastric vessels
b) Superficial circumflex iliac vessels
c) Obturator vessels
d) Deep circumflex iliac artery
Explanation: Surgeons must avoid the inferior epigastric vessels located just lateral to the umbilicus and rectus abdominis to prevent bleeding during trocar insertion. (Answer: a)
Topic: Arteries of the Anterior Abdominal Wall
Subtopic: Superficial Epigastric Artery
Keyword Definitions:
Superficial epigastric artery: A small branch of the femoral artery that supplies the lower part of the anterior abdominal wall and skin over the inguinal region.
Femoral artery: Main artery of the thigh, a continuation of the external iliac artery below the inguinal ligament.
Inferior epigastric artery: A deep branch of the external iliac artery that supplies the rectus abdominis muscle and anastomoses with the superior epigastric artery.
External pudendal artery: A branch of the femoral artery that supplies the perineum and external genitalia.
Superficial fascia: The fatty and membranous layer beneath the skin, especially well developed in the lower abdomen.
Inguinal ligament: Fibrous band extending from the anterior superior iliac spine to the pubic tubercle forming the base of the inguinal canal.
Lead Question – 2014
Superficial epigastric artery is a branch of?
a) Internal pudendal artery
b) External pudendal artery
c) Internal iliac artery
d) Femoral artery
Explanation: The superficial epigastric artery arises from the femoral artery just below the inguinal ligament. It runs upward and medially in the superficial fascia to supply the lower abdominal wall and skin over the inguinal region. (Answer: d)
1) The femoral artery begins at the level of:
a) Inguinal ligament
b) Pubic symphysis
c) Mid-inguinal point
d) Iliac crest
Explanation: The femoral artery begins at the mid-inguinal point, midway between the anterior superior iliac spine and the pubic symphysis, as a continuation of the external iliac artery. (Answer: c)
2) The superficial epigastric artery pierces which fascia?
a) Scarpa’s fascia
b) Camper’s fascia
c) Deep fascia
d) Fascia lata
Explanation: The superficial epigastric artery pierces the cribriform fascia (part of the fascia lata) just below the inguinal ligament before ascending in the superficial fascia of the lower abdominal wall. (Answer: d)
3) (Clinical) During hernia surgery, superficial epigastric vein injury leads to bleeding in which layer?
a) Deep fascia
b) Superficial fascia
c) Transversalis fascia
d) Peritoneum
Explanation: Bleeding from the superficial epigastric vein occurs in the superficial fascia, which lies beneath the skin. This vein accompanies the artery and drains into the femoral vein. (Answer: b)
4) The superficial epigastric artery anastomoses with:
a) Superior epigastric artery
b) Inferior epigastric artery
c) Deep circumflex iliac artery
d) Obturator artery
Explanation: The superficial epigastric artery anastomoses with branches of the inferior epigastric artery in the anterior abdominal wall, contributing to collateral blood supply between femoral and external iliac systems. (Answer: b)
5) (Clinical) A patient with obstruction of the femoral artery may retain some blood flow through:
a) Superficial epigastric and inferior epigastric anastomosis
b) External pudendal artery
c) Deep femoral artery
d) Obturator artery
Explanation: Collateral circulation via the superficial epigastric and inferior epigastric arteries can maintain blood flow to the lower abdominal wall when femoral artery flow is compromised. (Answer: a)
6) The superficial epigastric artery supplies:
a) Deep abdominal muscles
b) Lower part of anterior abdominal wall
c) Rectus abdominis muscle
d) Peritoneum
Explanation: The superficial epigastric artery supplies the skin and superficial fascia of the lower anterior abdominal wall and the inguinal region. (Answer: b)
7) (Clinical) Dilated superficial epigastric veins are seen in:
a) Deep vein thrombosis
b) Portal hypertension
c) Varicocele
d) Aneurysm of aorta
Explanation: Dilated superficial epigastric veins may be seen in portal hypertension due to portosystemic anastomosis between the paraumbilical and superficial epigastric veins. (Answer: b)
8) The superficial epigastric artery arises:
a) Above inguinal ligament
b) Below inguinal ligament
c) At mid-inguinal point
d) At pubic tubercle
Explanation: The superficial epigastric artery arises from the femoral artery below the inguinal ligament, near the saphenous opening. (Answer: b)
9) (Clinical) A surgeon performing varicose vein surgery near the saphenous opening must identify:
a) Superficial epigastric vein
b) Inferior epigastric vein
c) Obturator vein
d) Deep circumflex iliac vein
Explanation: The superficial epigastric vein joins the great saphenous vein near the saphenous opening, an important landmark in varicose vein surgery to prevent hemorrhage. (Answer: a)
10) (Clinical) A penetrating wound just below the inguinal ligament may damage which artery?
a) Inferior epigastric artery
b) Superficial epigastric artery
c) Deep circumflex iliac artery
d) Obturator artery
Explanation: A wound just below the inguinal ligament may injure the superficial epigastric artery as it arises from the femoral artery and pierces the fascia lata in this region. (Answer: b)
Subtopic: Cremasteric Artery
Keyword Definitions:
Cremasteric artery: A branch of the inferior epigastric artery that supplies the cremaster muscle and coverings of the spermatic cord.
Inferior epigastric artery: A branch of the external iliac artery supplying the lower anterior abdominal wall.
Cremaster muscle: A skeletal muscle that raises and lowers the testis to regulate temperature.
Spermatic cord: A bundle containing vas deferens, arteries, veins, and nerves that pass through the inguinal canal.
External iliac artery: Main artery of lower limb that gives rise to inferior epigastric and deep circumflex iliac arteries.
Pampiniform plexus: Venous network in spermatic cord aiding testicular thermoregulation.
Lead Question – 2014
Cremasteric artery is a branch of?
a) Internal pudendal artery
b) External pudendal artery
c) Inferior epigastric artery
d) Superior epigastric artery
Explanation: The cremasteric artery arises from the inferior epigastric artery, a branch of the external iliac artery. It supplies the cremaster muscle and coverings of the spermatic cord, anastomosing with the testicular and artery of the vas deferens. (Answer: c)
1) The inferior epigastric artery arises from:
a) Internal iliac artery
b) External iliac artery
c) Common iliac artery
d) Femoral artery
Explanation: The inferior epigastric artery arises from the external iliac artery just above the inguinal ligament and ascends to supply the rectus abdominis muscle and overlying skin. (Answer: b)
2) The cremaster muscle is derived from:
a) External oblique
b) Internal oblique
c) Transversus abdominis
d) Rectus abdominis
Explanation: The cremaster muscle is derived from the internal oblique muscle and its fascia. It covers the spermatic cord and helps elevate the testes during the cremasteric reflex. (Answer: b)
3) (Clinical) Injury to the inferior epigastric artery can lead to hematoma in:
a) Perineum
b) Rectus sheath
c) Inguinal canal
d) Scrotum
Explanation: Injury to the inferior epigastric artery causes bleeding into the rectus sheath, leading to a rectus sheath hematoma, often seen after trauma or laparoscopic surgery. (Answer: b)
4) The cremasteric artery anastomoses with:
a) Testicular artery and artery to vas deferens
b) Deep epigastric artery
c) Obturator artery
d) External pudendal artery
Explanation: The cremasteric artery forms anastomoses with the testicular artery and the artery to vas deferens, ensuring collateral circulation to the spermatic cord and testis. (Answer: a)
5) Cremasteric reflex involves contraction of:
a) Dartos muscle
b) Cremaster muscle
c) External oblique
d) Rectus abdominis
Explanation: The cremasteric reflex is the upward pull of the testis upon stroking the inner thigh, mediated by the cremaster muscle supplied by the genital branch of the genitofemoral nerve. (Answer: b)
6) (Clinical) In testicular torsion, cremasteric reflex is:
a) Exaggerated
b) Normal
c) Absent
d) Hyperactive
Explanation: In testicular torsion, the cremasteric reflex is absent due to ischemia and irritation of the genitofemoral nerve, which normally mediates this reflex. (Answer: c)
7) (Clinical) A surgeon ligating the inferior epigastric artery must be cautious of which nearby structure?
a) Vas deferens
b) Deep inguinal ring
c) Femoral vein
d) External pudendal vein
Explanation: The inferior epigastric artery runs just medial to the deep inguinal ring, an important landmark during hernia repairs. Injury here can cause bleeding and hematoma formation. (Answer: b)
8) External pudendal artery supplies:
a) Perineum and scrotal skin
b) Cremaster muscle
c) Testis
d) Internal oblique muscle
Explanation: The external pudendal artery, a branch of the femoral artery, supplies the perineum, penis, and scrotal or labial skin, not the cremaster muscle. (Answer: a)
9) (Clinical) A patient has a deep inguinal hernia. The artery lying medial to the deep inguinal ring is:
a) Inferior epigastric artery
b) Superficial epigastric artery
c) External pudendal artery
d) Deep circumflex iliac artery
Explanation: The inferior epigastric artery lies medial to the deep inguinal ring. This relationship helps differentiate direct from indirect inguinal hernias clinically. (Answer: a)
10) (Clinical) During orchiopexy, which artery must be preserved for testicular viability?
a) Cremasteric artery
b) Testicular artery
c) Artery to vas deferens
d) All of the above
Explanation: During orchiopexy for undescended testis, the testicular artery is the major blood supply, but preservation of all three arteries—testicular, cremasteric, and artery to vas deferens—ensures adequate perfusion and prevents ischemia. (Answer: d)
Topic: Blood Supply of Anal Canal
Subtopic: Inferior Rectal Artery
Keyword Definitions:
Inferior rectal artery: Branch of internal pudendal artery supplying anal canal below pectinate line and perianal skin.
Internal pudendal artery: Branch of internal iliac artery supplying perineum, external genitalia, and anal canal.
Superior rectal artery: Terminal branch of inferior mesenteric artery supplying rectum above pectinate line.
Pectinate line: Anatomical line dividing upper (visceral) and lower (somatic) anal canal.
Anal canal: Terminal part of large intestine extending from rectum to anus.
Inferior mesenteric artery: Branch of abdominal aorta supplying hindgut structures including upper rectum.
Lead Question – 2014
Inferior rectal artery is a branch of?
a) Inferior mesenteric artery
b) Superior mesenteric artery
c) Coeliac trunk
d) Internal pudendal artery
Explanation: The inferior rectal artery arises from the internal pudendal artery, a branch of the internal iliac artery. It supplies the lower anal canal below the pectinate line, anal sphincters, and perianal skin. The superior rectal artery arises from inferior mesenteric artery. (Answer: d)
1) Superior rectal artery is a continuation of:
a) Inferior mesenteric artery
b) Internal pudendal artery
c) Coeliac trunk
d) Superior mesenteric artery
Explanation: The superior rectal artery is the terminal branch of the inferior mesenteric artery. It supplies rectum above pectinate line, anastomosing with middle and inferior rectal arteries. (Answer: a)
2) Middle rectal artery arises from:
a) Internal iliac artery
b) External iliac artery
c) Inferior mesenteric artery
d) Superior mesenteric artery
Explanation: The middle rectal artery arises from internal iliac artery, supplying rectum, prostate, seminal vesicles in males, and vagina in females. It forms anastomoses with superior and inferior rectal arteries. (Answer: a)
3) (Clinical) A patient with internal hemorrhoids has bleeding above pectinate line. Blood supply is mainly from:
a) Inferior rectal artery
b) Superior rectal artery
c) Middle rectal artery
d) Internal pudendal artery
Explanation: Internal hemorrhoids occur above the pectinate line. Their arterial supply is from the superior rectal artery, a branch of inferior mesenteric artery, which forms a rich anastomotic network. (Answer: b)
4) Inferior rectal artery supplies:
a) Upper rectum
b) Lower anal canal and perianal skin
c) Sigmoid colon
d) Rectosigmoid junction
Explanation: The inferior rectal artery supplies the lower anal canal below pectinate line, anal sphincters, and perianal skin. It is a branch of internal pudendal artery. (Answer: b)
5) Pectinate line divides:
a) Upper and lower rectum
b) Visceral and somatic innervation
c) Ascending and descending colon
d) Sigmoid and rectum
Explanation: The pectinate line marks the junction of hindgut and proctodeum, separating visceral (above) and somatic (below) innervation, lymphatic drainage, and arterial supply. Inferior rectal artery supplies below it. (Answer: b)
6) (Clinical) A surgeon ligates inferior rectal artery during perianal surgery. This artery is a branch of:
a) Inferior mesenteric artery
b) Internal pudendal artery
c) External iliac artery
d) Middle rectal artery
Explanation: During perianal procedures, the inferior rectal artery from internal pudendal artery may be ligated to control bleeding. It supplies lower anal canal, anal sphincters, and skin. (Answer: b)
7) (Clinical) Patient has external hemorrhoids. Painful swelling is supplied by:
a) Superior rectal artery
b) Inferior rectal artery
c) Middle rectal artery
d) Superior mesenteric artery
Explanation: External hemorrhoids are located below the pectinate line, supplied by the inferior rectal artery, which is a branch of the internal pudendal artery. They are painful due to somatic innervation. (Answer: b)
8) Internal pudendal artery arises from:
a) External iliac artery
b) Internal iliac artery
c) Common iliac artery
d) Inferior mesenteric artery
Explanation: The internal pudendal artery branches from the internal iliac artery, exiting the pelvis via the greater sciatic foramen, supplying perineum, external genitalia, and inferior rectal artery. (Answer: b)
9) (Clinical) Bleeding from lower anal canal is supplied by:
a) Superior rectal artery
b) Inferior rectal artery
c) Middle rectal artery
d) Coeliac trunk
Explanation: The inferior rectal artery supplies the lower anal canal below the pectinate line and perianal skin. Bleeding in this region originates from this branch of internal pudendal artery. (Answer: b)
10) Anastomosis of rectal arteries ensures:
a) Only venous drainage
b) Collateral circulation to anal canal
c) Supply to small intestine
d) Supply to sigmoid colon only
Explanation: Superior, middle, and inferior rectal arteries form an anastomotic network ensuring collateral arterial supply to the anal canal, providing protection against ischemia. Inferior rectal artery contributes to lower anal canal supply. (Answer: b)
Subtopic: Pericardium and Pericardial Sinuses
Keyword Definitions:
Pericardium: The fibroserous sac enclosing the heart and the roots of great vessels.
Transverse pericardial sinus: A passage between arterial and venous ends of the heart formed by pericardial reflections.
Oblique pericardial sinus: A blind recess behind the left atrium formed by serous pericardium.
Great vessels: Major arteries and veins entering and leaving the heart, such as aorta, pulmonary trunk, SVC, and IVC.
Lead Question (2014):
Posterior to transverse pericardial sinus?
a) Aorta
b) Pulmonary trunk
c) SVC
d) Left atrium
Explanation:
The transverse pericardial sinus lies between the arterial and venous ends of the heart. Anteriorly it’s related to the aorta and pulmonary trunk, and posteriorly to the left atrium. Answer: d) Left atrium. Surgeons can pass a finger through this sinus during cardiac procedures to isolate great vessels.
1)
The transverse pericardial sinus is located between which two groups of structures?
a) Arteries and veins
b) Veins and nerves
c) Right and left atria
d) Aorta and pulmonary veins
The transverse sinus separates the arterial trunks (aorta and pulmonary trunk) anteriorly from the venous structures (SVC and left atrium) posteriorly. Answer: a) Arteries and veins. It forms due to pericardial reflections around the great vessels during cardiac development.
2)
During cardiac surgery, a surgeon can pass a finger behind which vessels through the transverse pericardial sinus?
a) Aorta and pulmonary trunk
b) SVC and IVC
c) Pulmonary veins
d) Coronary sinus
The surgeon passes a finger behind the aorta and pulmonary trunk through the transverse sinus, isolating them from venous structures. Answer: a) Aorta and pulmonary trunk. This helps in clamping or cannulating the great arteries during cardiopulmonary bypass surgery.
3)
Which of the following structures forms the anterior boundary of the transverse pericardial sinus?
a) SVC
b) Aorta and pulmonary trunk
c) Left atrium
d) Pulmonary veins
The transverse pericardial sinus is bounded anteriorly by the ascending aorta and pulmonary trunk. Answer: b) Aorta and pulmonary trunk. These arterial trunks pass upward from the heart and are covered by serous pericardium forming the anterior wall of this sinus.
4)
The oblique pericardial sinus is located posterior to which cardiac chamber?
a) Right atrium
b) Left atrium
c) Right ventricle
d) Left ventricle
The oblique pericardial sinus lies behind the left atrium, formed by reflection of serous pericardium around pulmonary veins and IVC. Answer: b) Left atrium. It forms a cul-de-sac where pericardial fluid may collect in pericardial effusion cases.
5)
A cardiac surgeon uses the transverse pericardial sinus to control blood flow. What vessels are clamped during this procedure?
a) SVC and IVC
b) Pulmonary veins
c) Aorta and pulmonary trunk
d) Coronary arteries
During cardiac surgery, the transverse sinus allows passage of a clamp behind the ascending aorta and pulmonary trunk to control arterial outflow. Answer: c) Aorta and pulmonary trunk. This isolation is vital during cardiopulmonary bypass procedures for safe cardiac manipulation.
6)
The pericardial cavity is located between which two layers?
a) Parietal and visceral pericardium
b) Fibrous and parietal pericardium
c) Serous and fibrous pericardium
d) Epicardium and myocardium
The pericardial cavity is the potential space between the parietal and visceral layers of serous pericardium containing lubricating fluid. Answer: a) Parietal and visceral pericardium. It allows frictionless cardiac movement within the pericardial sac during heartbeats.
7)
A pericardial effusion compressing the left atrium would most likely accumulate in which sinus?
a) Coronary sinus
b) Oblique pericardial sinus
c) Transverse pericardial sinus
d) Costomediastinal recess
Fluid tends to accumulate in the oblique pericardial sinus posterior to the left atrium, especially in supine patients. Answer: b) Oblique pericardial sinus. This collection can compress pulmonary veins and impair cardiac filling, producing symptoms of cardiac tamponade.
8)
Which pericardial sinus lies behind the left atrium and between the pulmonary veins?
a) Oblique pericardial sinus
b) Transverse pericardial sinus
c) Coronary sinus
d) Pleural recess
The oblique pericardial sinus lies behind the left atrium and between pulmonary veins. Answer: a) Oblique pericardial sinus. It forms a blind recess, allowing expansion of the left atrium during increased venous return without friction against pericardial surfaces.
9)
In a patient with pericardial effusion, which structure allows surgical drainage without injuring pleura?
a) Left 2nd intercostal space
b) Right 5th intercostal space
c) Infrasternal angle
d) Left midaxillary line
The infrasternal angle or subxiphoid approach allows needle insertion into the pericardial cavity without damaging pleura. Answer: c) Infrasternal angle. This route provides direct access to pericardial fluid in emergencies such as cardiac tamponade.
10)
A 50-year-old male undergoing open-heart surgery—what is the clinical significance of the transverse pericardial sinus?
a) Allows passage of coronary vessels
b) Used to clamp great arteries during bypass
c) Site of venous drainage
d) Receives pulmonary veins
The transverse pericardial sinus enables surgeons to pass a clamp or tube behind great arteries to isolate them from veins. Answer: b) Used to clamp great arteries during bypass. It serves as an important landmark during cardiac surgeries involving cardiopulmonary bypass setup.
Topic: Lungs and Mediastinum
Subtopic: Hilum of Lungs
Keyword Definitions:
Hilum of lung: Depression on the mediastinal surface where bronchi, vessels, and nerves enter.
Azygous vein: Drains thoracic wall, arches over root of right lung into SVC.
Vagus nerve: Provides parasympathetic fibers to lungs.
SVC: Superior vena cava, drains blood from upper body to heart.
Arch of aorta: Curved portion of aorta, related to left lung hilum.
Lead Question - 2014
Not related to hilum of right lung?
a) Azygous vein
b) Vagus nerve
c) SVC
d) Arch of aorta
Explanation: The hilum of the right lung is related to the azygous vein, superior vena cava, and vagus nerve. However, the arch of the aorta is a relation of the left lung hilum, not the right. Thus, the correct answer is Arch of aorta. This distinction is important in thoracic anatomy.
Guessed Questions for NEET PG:
1) Structure most commonly arching over the root of the right lung?
a) Pulmonary trunk
b) Azygous vein
c) Right phrenic nerve
d) Right brachiocephalic vein
Explanation: The azygous vein arches over the root of the right lung before draining into the superior vena cava. This anatomical landmark is unique to the right side. The correct answer is Azygous vein. This relation is crucial during mediastinal dissections and radiological imaging for diagnosis.
2) In a patient undergoing lung surgery, which nerve must be preserved near hilum of right lung?
a) Phrenic nerve
b) Vagus nerve
c) Hypoglossal nerve
d) Sympathetic trunk
Explanation: The vagus nerve passes posterior to the hilum of the right lung and provides parasympathetic innervation. Injury to it can cause impaired bronchoconstriction and secretory reflexes. Thus, the correct answer is Vagus nerve. Preservation is critical during lung resections and mediastinal lymph node dissections.
3) Which vessel lies anterior to the right lung hilum?
a) Superior vena cava
b) Azygous vein
c) Pulmonary artery
d) Arch of aorta
Explanation: The superior vena cava lies anterior to the hilum of the right lung, draining venous blood into the right atrium. This relation helps surgeons and radiologists distinguish normal vascular landmarks. The correct answer is Superior vena cava. Its injury can cause rapid hemodynamic compromise during thoracic procedures.
4) Clinical case: A 50-year-old with carcinoma lung has enlarged lymph nodes compressing the structure arching over right hilum. Which is affected?
a) Aortic arch
b) Left subclavian artery
c) Azygous vein
d) Right pulmonary artery
Explanation: Lymphadenopathy around the right lung hilum often compresses the azygous vein, which arches over it. Compression may cause venous congestion. The correct answer is Azygous vein. Recognition of this relation helps in staging lung carcinoma and planning surgery or radiotherapy.
5) Right pulmonary artery in relation to hilum lies:
a) Superior to right bronchus
b) Inferior to right bronchus
c) Anterior to right bronchus
d) Posterior to right bronchus
Explanation: In the right lung, the pulmonary artery lies anterior to the bronchus at the hilum. This arrangement differs from the left lung, where the artery is superior to bronchus. The correct answer is Anterior to right bronchus. This anatomical difference is remembered by “RALS” (Right Anterior, Left Superior).
6) Clinical case: During mediastinoscopy, the surgeon encounters a nerve behind right lung hilum. Likely identity?
a) Phrenic nerve
b) Vagus nerve
c) Intercostal nerve
d) Accessory nerve
Explanation: The vagus nerve runs posterior to the root of the lung, unlike the phrenic nerve, which runs anterior. Identifying this during thoracic procedures prevents iatrogenic injury. The correct answer is Vagus nerve. Injury may cause autonomic dysfunction of bronchi and impaired airway regulation.
7) Which structure is not found in hilum of lungs?
a) Pulmonary artery
b) Pulmonary veins
c) Main bronchus
d) Thoracic duct
Explanation: The hilum transmits bronchi, pulmonary arteries, and pulmonary veins. Thoracic duct does not pass through the hilum, instead ascends in the posterior mediastinum. Therefore, the correct answer is Thoracic duct. Differentiating contents is key for interpreting chest radiographs and CT scans.
8) Clinical scenario: A stab wound injures a structure anterior to right hilum. Which is likely damaged?
a) Superior vena cava
b) Aortic arch
c) Azygous vein
d) Descending thoracic aorta
Explanation: Anterior to the right lung hilum lies the superior vena cava. A penetrating injury here may cause fatal venous hemorrhage. The correct answer is Superior vena cava. Identifying this anatomical relation helps trauma surgeons anticipate bleeding sources in thoracic injuries.
9) Which structure is posterior to both lung hila?
a) Azygous vein
b) Descending aorta
c) Vagus nerve
d) Pulmonary veins
Explanation: The vagus nerve is consistently posterior to the hilum of both lungs. This relation is surgically important in thoracic approaches. Thus, the correct answer is Vagus nerve. Damage may result in autonomic dysfunction including impaired bronchial tone regulation.
10) Clinical case: CT chest shows a mass compressing right hilum. Which vessel arching above hilum must be checked?
a) Aortic arch
b) Azygous vein
c) Pulmonary artery
d) Superior vena cava
Explanation: The azygous vein arches above the right lung hilum. On CT scans, compression of this vein may indicate mediastinal pathology or hilar mass. The correct answer is Azygous vein. Recognizing this landmark helps radiologists in staging lung cancers and evaluating mediastinal syndromes.
Topic: Thorax
Subtopic: Relations of Lung Hilum
Keyword Definitions:
Hilum of lung: Area on the medial surface of lung where bronchi, vessels, and nerves enter or leave.
Vagus nerve: Cranial nerve X, provides parasympathetic supply to thoracic and abdominal organs.
Azygos vein: Vein draining posterior thoracic wall, arches over right lung root.
SVC: Large vein returning venous blood from upper body to right atrium.
Arch of aorta: Major arterial curve from ascending to descending aorta, giving major branches.
Lead Question - 2014
Posterior relation of hilum of lung ?
a) Azygous vein
b) SVC
c) Vagus nerve
d) Arch of aorta
Explanation: The posterior relation of the lung hilum is the vagus nerve, which passes behind the root of both lungs. Azygos vein arches over right lung root anteriorly, and arch of aorta lies superiorly. Correct answer: c) Vagus nerve.
Guessed Questions for NEET PG:
1) Which nerve lies anterior to the lung hilum?
a) Vagus
b) Phrenic
c) Sympathetic trunk
d) Intercostal
Explanation: The phrenic nerve passes anterior to the root of the lung, while the vagus nerve lies posterior. This relationship is crucial during thoracic surgeries. Correct answer: b) Phrenic.
2) Which vessel arches over the right lung hilum?
a) Hemiazygos vein
b) Azygos vein
c) Superior vena cava
d) Internal thoracic vein
Explanation: The azygos vein arches over the root of the right lung to drain into the superior vena cava. This is a classical radiological landmark in chest imaging. Correct answer: b) Azygos vein.
3) Which structure crosses the arch of aorta anteriorly?
a) Left phrenic nerve
b) Left vagus nerve
c) Left recurrent laryngeal nerve
d) Left subclavian vein
Explanation: The left phrenic nerve crosses the arch of aorta anteriorly, while the left recurrent laryngeal nerve hooks around ligamentum arteriosum posteriorly. Correct answer: a) Left phrenic nerve.
4) A patient with left lung tumor compresses the posterior relation of hilum. Which nerve is likely affected?
a) Left phrenic
b) Left vagus
c) Sympathetic chain
d) Intercostal nerve
Explanation: Tumors at the left lung hilum posteriorly may compress the vagus nerve, leading to parasympathetic dysfunction, hoarseness, or cardiac reflex changes. Correct answer: b) Left vagus.
5) In right lung hilum, which lies superior among structures?
a) Pulmonary vein
b) Pulmonary artery
c) Main bronchus
d) Azygos vein
Explanation: At the right hilum, the bronchus lies posterior, pulmonary artery superior, and pulmonary veins anterior-inferior. This arrangement is remembered as "B-A-V" (Back to front). Correct answer: b) Pulmonary artery.
6) Left recurrent laryngeal nerve hooks around?
a) Right subclavian artery
b) Arch of aorta
c) Left pulmonary artery
d) Superior vena cava
Explanation: The left recurrent laryngeal nerve, branch of vagus, hooks around the arch of aorta near ligamentum arteriosum. Correct answer: b) Arch of aorta.
7) Which bronchus is more prone to foreign body aspiration?
a) Left main bronchus
b) Right main bronchus
c) Both equal
d) Depends on position
Explanation: The right main bronchus is shorter, wider, and more vertical, making it more prone to aspiration of foreign bodies. Correct answer: b) Right main bronchus.
8) During hilar lymph node dissection, which nerve injury can cause hoarseness of voice?
a) Phrenic
b) Sympathetic
c) Recurrent laryngeal
d) Intercostal
Explanation: Injury to recurrent laryngeal nerve during lymph node dissection causes vocal cord paralysis and hoarseness. Correct answer: c) Recurrent laryngeal.
9) In chest radiograph, hilar shadow mainly represents?
a) Bronchi
b) Pulmonary arteries
c) Lymph nodes
d) Pulmonary veins
Explanation: On radiographs, hilar shadows are predominantly formed by pulmonary arteries. Bronchi are air-filled and less visible. Correct answer: b) Pulmonary arteries.
10) A 60-year-old smoker has left hilar mass compressing anterior relation of hilum. Which nerve is affected?
a) Left vagus
b) Left phrenic
c) Left recurrent laryngeal
d) Sympathetic chain
Explanation: The anterior relation of the lung hilum is the phrenic nerve. Compression leads to diaphragmatic palsy, breathlessness, and raised hemidiaphragm. Correct answer: b) Left phrenic.
Topic: Lymphatic System
Subtopic: Termination of Thoracic Duct
Keyword Definitions:
Thoracic duct: Largest lymphatic channel, draining lymph from most of the body except right upper quadrant.
Venous angle: Junction of internal jugular vein and subclavian vein.
Subclavian vein: Major vein draining blood from the upper limb.
Internal jugular vein: Vein draining blood from brain, face, and neck.
Brachiocephalic vein: Large vein formed by union of subclavian and internal jugular veins.
Lead Question - 2014
Thoracic duct opens into ?
a) Subclavian vein
b) Internal jugular vein
c) Right brachiocephalic vein
d) Left brachiocephalic vein
Explanation: The thoracic duct terminates at the left venous angle, i.e., the junction of the left subclavian and left internal jugular veins. This anatomical site is critical in surgeries of the neck and mediastinum. Correct answer: a) Subclavian vein (at its junction with internal jugular vein).
Guessed Questions for NEET PG:
1) Which side of the venous angle receives thoracic duct?
a) Right
b) Left
c) Both sides
d) Variable
Explanation: The thoracic duct consistently terminates at the left venous angle, formed by left internal jugular and left subclavian veins. This is a fixed anatomical feature with great surgical relevance. Correct answer: b) Left.
2) Right lymphatic duct opens into?
a) Left venous angle
b) Right venous angle
c) Superior vena cava
d) Azygos vein
Explanation: The right lymphatic duct drains lymph from right upper limb, right thorax, and right side of head and neck. It opens into the right venous angle, i.e., junction of right internal jugular and subclavian veins. Correct answer: b) Right venous angle.
3) Length of thoracic duct is?
a) 10 cm
b) 20 cm
c) 40 cm
d) 60 cm
Explanation: The thoracic duct measures approximately 40 cm in adults. It extends from cisterna chyli at L1-L2 vertebrae to the left venous angle in the root of the neck. Correct answer: c) 40 cm.
4) Thoracic duct crosses from right to left at the level of?
a) T2
b) T4
c) T6
d) T8
Explanation: The thoracic duct ascends on the right side of vertebral column and crosses to the left side at the level of T4-T6 vertebrae, continuing upward to terminate in the left venous angle. Correct answer: b) T4.
5) A 45-year-old man develops chylothorax after oesophagectomy. Which structure is injured?
a) Azygos vein
b) Thoracic duct
c) Hemiazygos vein
d) Vagus nerve
Explanation: Chylothorax results from thoracic duct injury during mediastinal or esophageal surgery. Milky fluid rich in triglycerides accumulates in the pleural cavity, requiring drainage. Correct answer: b) Thoracic duct.
6) Cisterna chyli is located at?
a) T10
b) T12
c) L1-L2
d) S1
Explanation: The cisterna chyli is located anterior to the bodies of L1 and L2 vertebrae. It collects lymph from lumbar and intestinal trunks and continues as thoracic duct. Correct answer: c) L1-L2.
7) Thoracic duct drains all except?
a) Right lower limb
b) Left upper limb
c) Right thorax
d) Left thorax
Explanation: The thoracic duct drains both lower limbs, abdomen, left thorax, left upper limb, and left head and neck. The right thorax, right upper limb, and right head and neck are drained by the right lymphatic duct. Correct answer: c) Right thorax.
8) In a left neck dissection, accidental thoracic duct injury causes leakage of?
a) Blood
b) Serous fluid
c) Chyle
d) Bile
Explanation: Injury to thoracic duct causes leakage of chyle, a milky lymph rich in fats. Chylous fistula is a known complication in neck surgeries, particularly near left venous angle. Correct answer: c) Chyle.
9) In lymphoma, obstruction of thoracic duct may cause?
a) Pleural effusion
b) Ascites
c) Chylous ascites
d) Pericardial effusion
Explanation: Thoracic duct obstruction by lymphoma or tumor can cause chylous ascites, characterized by milky fluid in the peritoneal cavity due to blocked lymphatic flow. Correct answer: c) Chylous ascites.
10) During ligation of thoracic duct, surgeon aims to prevent?
a) Air embolism
b) Chylothorax
c) Pneumothorax
d) Pulmonary embolism
Explanation: Ligation of thoracic duct is done to prevent persistent chylothorax, which results from continuous leakage of chyle into pleural cavity. This is life-threatening due to nutritional loss. Correct answer: b) Chylothorax.
Subtopic: Thoracic Duct Formation
Keyword Definitions:
Thoracic duct: The largest lymphatic vessel in the human body draining lymph from most areas.
Cisterna chyli: Dilated sac at the lower end of thoracic duct collecting lymph from abdomen.
Subclavian vein: Vein that drains blood from upper limb into brachiocephalic vein.
Jugular vein: Vein draining blood from head and neck.
Brachiocephalic vein: Large vein formed by subclavian and internal jugular veins.
Lead Question - 2014
Thoracic duct is formed by?
a) Union of left subclavian and left internal jugular vein.
b) Union of brachiocephalic vein and internal jugular vein
c) Continuation of upper end of cisterna chyli
d) None of the above
Explanation: The thoracic duct originates as the continuation of the cisterna chyli at the level of L1-L2 vertebrae, ascending through the thorax. It drains into the venous system at the junction of the left internal jugular and left subclavian veins. Correct answer: c) Continuation of upper end of cisterna chyli.
Guessed Questions for NEET PG:
1) Length of thoracic duct is approximately?
a) 10 cm
b) 20 cm
c) 40 cm
d) 50 cm
Explanation: The thoracic duct measures about 40 cm in adults. It starts from cisterna chyli in the abdomen and ascends to the venous angle. Its long course makes it prone to injury during surgery. Correct answer: c) 40 cm.
2) Thoracic duct pierces diaphragm through?
a) Aortic hiatus
b) Caval opening
c) Esophageal hiatus
d) None
Explanation: The thoracic duct passes through the diaphragm along with the aorta at the aortic hiatus at the level of T12 vertebra. This is a key anatomical relation during abdominal and thoracic surgeries. Correct answer: a) Aortic hiatus.
3) Which vein receives terminal drainage of thoracic duct?
a) Right subclavian vein
b) Left brachiocephalic vein
c) At junction of left internal jugular and left subclavian vein
d) Superior vena cava
Explanation: The thoracic duct terminates into the venous system at the left venous angle, i.e., the junction of the left subclavian vein and left internal jugular vein. This is a key anatomical landmark. Correct answer: c) Junction of left internal jugular and subclavian vein.
4) Thoracic duct drains all except?
a) Left upper limb
b) Right thorax
c) Left abdomen
d) Left thorax
Explanation: The thoracic duct drains lymph from entire body except the right upper limb, right thorax, right side of head and neck, which are drained by the right lymphatic duct. Correct answer: b) Right thorax.
5) In a neck surgery, thoracic duct injury leads to leakage of?
a) Blood
b) Bile
c) Chyle
d) Lymphocyte-depleted fluid
Explanation: Injury to thoracic duct leads to chylous fistula, with leakage of milky chyle rich in triglycerides. This complication is common during left neck dissections near the venous angle. Correct answer: c) Chyle.
6) Cisterna chyli is located at?
a) T8-T9
b) L1-L2
c) S1-S2
d) T12
Explanation: The cisterna chyli is located anterior to the bodies of L1 and L2 vertebrae, behind the right crus of diaphragm. It acts as the reservoir for intestinal and lumbar lymph trunks. Correct answer: b) L1-L2.
7) In chylothorax, fluid accumulates in?
a) Pleural cavity
b) Peritoneal cavity
c) Pericardial cavity
d) Subarachnoid space
Explanation: Chylothorax occurs when the thoracic duct is injured, leading to leakage of chyle into the pleural cavity. It is a serious surgical complication, requiring drainage and repair. Correct answer: a) Pleural cavity.
8) Right lymphatic duct drains lymph from?
a) Right upper limb
b) Right thorax
c) Right side of head and neck
d) All of the above
Explanation: The right lymphatic duct drains lymph from right upper limb, right thorax, and right side of head and neck. It terminates into the right venous angle. Correct answer: d) All of the above.
9) In case of lymphoma, thoracic duct obstruction may cause?
a) Ascites
b) Chylothorax
c) Chylous ascites
d) Edema
Explanation: Thoracic duct obstruction due to malignancy such as lymphoma may cause chylous ascites, characterized by milky fluid in peritoneum. This is a clinical indicator of lymphatic obstruction. Correct answer: c) Chylous ascites.
10) During oesophageal carcinoma surgery, thoracic duct is at risk at level of?
a) T2
b) T4
c) T8
d) T12
Explanation: Thoracic duct runs posterior to oesophagus in thorax, closely related at T4 to T8 levels. Surgical manipulation in esophagectomy carries risk of injury. Correct answer: c) T8.
Chapter: Thorax Anatomy
Topic: Intercostal Arteries
Subtopic: Anterior Intercostal Arteries
Keyword Definitions:
Anterior intercostal artery: Branches supplying anterior part of intercostal spaces.
Internal thoracic artery: Branch of subclavian artery giving rise to anterior intercostals.
Posterior intercostal artery: Branch of thoracic aorta or superior intercostal artery supplying posterior chest wall.
Intercostal space: Gap between two ribs occupied by muscles, vessels, and nerves.
Aorta: Main artery arising from left ventricle supplying systemic circulation.
Lead Question - 2014
True about anterior intercostal artery ?
a) Present in 1st to 11th intercostal space
b) Each intercostal space has two anterior intercostal arteries
c) Branch of internal thoracic artery
d) Branch of aorta
Explanation: Anterior intercostal arteries are branches of internal thoracic artery. Each of the first six spaces has two anterior intercostal arteries. Lower spaces are supplied by musculophrenic artery. They are not branches of aorta. Correct answer is c) Branch of internal thoracic artery. Knowledge is vital for thoracic surgery. (50 words)
1. Posterior intercostal arteries of lower nine spaces arise from:
a) Internal thoracic artery
b) Thoracic aorta
c) Subclavian artery
d) Musculophrenic artery
Explanation: Posterior intercostal arteries supplying the 3rd to 11th spaces arise directly from thoracic aorta. First two posterior intercostal arteries originate from superior intercostal artery. Correct answer is b) Thoracic aorta. These arteries run in costal grooves with intercostal nerves. Anatomy is clinically important during chest drain insertion. (50 words)
2. Which artery supplies anterior part of lower intercostal spaces?
a) Superior intercostal artery
b) Musculophrenic artery
c) Internal thoracic artery
d) Aorta
Explanation: The musculophrenic artery, a terminal branch of the internal thoracic artery, supplies anterior intercostal arteries of lower intercostal spaces. Internal thoracic supplies upper six. Correct answer is b) Musculophrenic artery. This distribution is significant during coronary bypass surgeries where internal thoracic artery is harvested. (50 words)
3. A stab injury in 5th intercostal space anteriorly may damage anterior intercostal branch of:
a) Thoracic aorta
b) Internal thoracic artery
c) Subclavian artery
d) Superior intercostal artery
Explanation: Anterior intercostal arteries in upper six spaces are branches of internal thoracic artery. A stab wound in 5th space anteriorly may damage its branch. Correct answer is b) Internal thoracic artery. Knowledge of intercostal vessels prevents complications during chest wall surgeries or thoracostomy. (50 words)
4. Internal thoracic artery terminates as:
a) Superior epigastric and musculophrenic arteries
b) Inferior epigastric and superior phrenic arteries
c) Musculophrenic and inferior phrenic arteries
d) Superior and inferior epigastric arteries
Explanation: Internal thoracic artery ends in 6th intercostal space dividing into superior epigastric and musculophrenic arteries. Superior epigastric continues to supply anterior abdominal wall, musculophrenic supplies lower intercostal spaces. Correct answer is a) Superior epigastric and musculophrenic arteries. This is important in reconstructive and coronary bypass surgeries. (50 words)
5. A patient with chest wall surgery developed bleeding from anterior part of 4th space. Which vessel is most likely injured?
a) Posterior intercostal artery
b) Anterior intercostal artery
c) Thoracic aorta
d) Superior intercostal artery
Explanation: Bleeding from anterior part of intercostal space indicates anterior intercostal artery injury, which is branch of internal thoracic artery. Posterior arteries lie close to vertebrae. Correct answer is b) Anterior intercostal artery. Such injuries are controlled by cauterization or ligation during surgery. (50 words)
6. Which intercostal space lacks anterior intercostal artery?
a) 1st
b) 7th
c) 10th
d) 11th
Explanation: The 11th intercostal space does not have anterior intercostal arteries as it ends posteriorly. Anterior intercostal arteries are absent in 11th and 12th spaces. Correct answer is d) 11th. Clinical relevance includes safe placement of lower chest drains avoiding vessel injury. (50 words)
7. A patient with coronary artery disease undergoes CABG. Which artery is commonly harvested?
a) Internal thoracic artery
b) Musculophrenic artery
c) Superior intercostal artery
d) Inferior phrenic artery
Explanation: Internal thoracic artery is commonly harvested for coronary artery bypass graft due to superior long-term patency. It supplies anterior intercostal arteries of upper spaces. Correct answer is a) Internal thoracic artery. Harvesting preserves musculophrenic and superior epigastric branches while maintaining collateral supply. (50 words)
8. Collateral circulation between anterior and posterior intercostal arteries occurs in:
a) Intercostal muscles
b) Costal groove
c) Midaxillary line
d) Parasternal region
Explanation: Anterior and posterior intercostal arteries anastomose within intercostal spaces, providing collateral circulation. This is particularly significant in coarctation of aorta where these collaterals enlarge. Correct answer is b) Costal groove. The vessels lie in neurovascular plane under ribs. This ensures perfusion of thoracic wall. (50 words)
9. During thoracentesis, which precaution prevents injury to intercostal arteries?
a) Needle insertion close to rib upper border
b) Needle insertion close to rib lower border
c) Midline approach only
d) Posterior approach only
Explanation: Intercostal vessels run along lower border of each rib in costal groove. To avoid injury, procedures like thoracentesis require needle insertion just above rib upper border. Correct answer is a) Needle insertion close to rib upper border. This prevents bleeding and nerve damage during clinical procedures. (50 words)
10. First two posterior intercostal arteries arise from:
a) Aorta
b) Superior intercostal artery
c) Internal thoracic artery
d) Subclavian artery
Explanation: First and second posterior intercostal arteries arise from superior intercostal artery, which is a branch of costocervical trunk from subclavian artery. Remaining posterior intercostals arise from thoracic aorta. Correct answer is b) Superior intercostal artery. This variation explains differences in collateral circulation in upper thorax. (50 words)
Topic: Superficial Veins of Lower Limb
Subtopic: Short Saphenous Vein
Keyword Definitions:
Short saphenous vein: Superficial vein of the leg, begins behind the lateral malleolus, drains into popliteal vein.
Long saphenous vein: Largest superficial vein of the leg, runs medially from foot to femoral vein.
Lateral malleolus: Bony prominence of distal fibula, landmark for short saphenous vein course.
Sural nerve: Cutaneous nerve of posterior leg, often runs with short saphenous vein.
Achilles tendon: Tendon of gastrocnemius and soleus, medial to short saphenous vein at ankle level.
Lead Question - 2014
All are true about short saphenous vein except?
a) Runs behind lateral malleolus
b) Runs on lateral side of leg
c) Accompanied by sural nerve
d) Achillis tendon is medial to vein
Explanation: The short saphenous vein runs behind the lateral malleolus, courses along the posterior aspect of the leg, is accompanied by the sural nerve, and lies lateral to the Achilles tendon. Therefore, the incorrect statement is that Achilles tendon is medial to vein. Correct answer: Achilles tendon is medial to vein.
1) The short saphenous vein drains into?
a) Femoral vein
b) Popliteal vein
c) External iliac vein
d) Posterior tibial vein
Explanation: The short saphenous vein ascends along the posterior leg and pierces deep fascia at popliteal fossa to drain into the popliteal vein. This makes the popliteal vein the correct answer.
2) Varicosity of short saphenous vein most commonly presents at?
a) Groin
b) Popliteal fossa
c) Medial malleolus
d) Lateral thigh
Explanation: Varicosities of short saphenous vein usually present in the posterior calf and popliteal region, as the vein terminates into the popliteal vein there. Correct answer: Popliteal fossa.
3) Which nerve accompanies the short saphenous vein?
a) Tibial nerve
b) Sural nerve
c) Superficial peroneal nerve
d) Deep peroneal nerve
Explanation: The sural nerve, a sensory branch formed by tibial and common peroneal nerves, accompanies the short saphenous vein along the posterior leg. Correct answer: Sural nerve.
4) During surgery for varicose veins, short saphenous vein is ligated at?
a) Groin
b) Lateral malleolus
c) Popliteal fossa
d) Medial thigh
Explanation: The short saphenous vein drains into the popliteal vein at the popliteal fossa. Therefore, surgical ligation is performed at the popliteal fossa. Correct answer: Popliteal fossa.
5) Short saphenous vein originates from?
a) Medial marginal vein
b) Lateral marginal vein
c) Dorsal venous arch
d) Femoral vein
Explanation: The short saphenous vein begins from the lateral end of dorsal venous arch of foot, via the lateral marginal vein. Correct answer: Lateral marginal vein.
6) A patient with sural nerve injury may show sensory loss in which region?
a) Medial leg
b) Anterior leg
c) Posterolateral leg and lateral foot
d) Medial thigh
Explanation: The sural nerve supplies sensation to posterolateral part of leg and lateral border of foot. Injury results in sensory loss over this distribution. Correct answer: Posterolateral leg and lateral foot.
7) Which is true about communication of short saphenous vein?
a) Communicates with long saphenous via perforators
b) Has no communication
c) Drains directly into femoral vein
d) Communicates only with deep femoral vein
Explanation: The short saphenous vein communicates with the long saphenous vein through perforating veins, allowing collateral venous return. Correct answer: Communicates with long saphenous via perforators.
8) Short saphenous vein lies in relation to Achilles tendon as?
a) Medial
b) Lateral
c) Posterior
d) Deep
Explanation: The short saphenous vein lies lateral to the Achilles tendon as it passes upwards behind the lateral malleolus. Correct answer: Lateral.
9) Short saphenous vein pierces deep fascia at?
a) Groin
b) Popliteal fossa
c) Ankle
d) Medial thigh
Explanation: The short saphenous vein ascends along posterior calf and pierces the popliteal fossa to drain into popliteal vein. Correct answer: Popliteal fossa.
10) Clinical significance of short saphenous vein in DVT diagnosis?
a) Used for arterial cannulation
b) Assessed for varicosities
c) Doppler used to trace flow into popliteal vein
d) Used for dialysis access
Explanation: In suspected DVT, Doppler ultrasound assesses venous flow of short saphenous vein into popliteal vein. Its patency and reflux are clinically significant for diagnosis. Correct answer: Doppler used to trace flow into popliteal vein.
Topic: Wrist and Hand Vasculature
Subtopic: Scaphoid Blood Supply
Keyword Definitions:
Scaphoid: Carpal bone located on radial side of wrist, prone to fracture.
Radial artery: Main arterial supply to scaphoid, particularly dorsal branch.
Ulnar artery: Supplies medial side of hand, not major source for scaphoid.
Dorsal branch: Provides retrograde blood flow to proximal scaphoid.
Avascular necrosis: Bone death due to inadequate blood supply, common in proximal scaphoid fractures.
Lead Question - 2014
True about blood supply of scaphoid?
a) Mainly through ulnar artery
b) Major supply from ventral surface
c) Major supply from dorsal surface
d) Proximal supply in antegrade fashion
Explanation: The scaphoid receives its blood supply predominantly from the dorsal branch of the radial artery. This supply enters distally and runs retrograde to the proximal pole, making the proximal part vulnerable to avascular necrosis. Hence, the correct answer is Major supply from dorsal surface.
1) Which carpal bone is most commonly fractured?
a) Scaphoid
b) Lunate
c) Triquetrum
d) Pisiform
Explanation: Among carpal bones, the scaphoid is most frequently fractured, especially after falls on an outstretched hand. Due to retrograde blood flow, its proximal fragment is at high risk of avascular necrosis. The correct answer is Scaphoid.
2) A patient presents with tenderness in the anatomical snuffbox. Which bone is likely fractured?
a) Scaphoid
b) Lunate
c) Hamate
d) Capitate
Explanation: Anatomical snuffbox tenderness is highly suggestive of scaphoid fracture. Clinical suspicion requires imaging since fracture may be radiologically occult initially. The correct answer is Scaphoid.
3) Main arterial supply to lunate bone is from?
a) Radial artery
b) Ulnar artery
c) Interosseous artery
d) Both radial and ulnar arteries
Explanation: The lunate receives dual supply from both radial and ulnar arteries. Despite this, it may still undergo avascular necrosis in Kienbock’s disease. Correct answer is Both radial and ulnar arteries.
4) In scaphoid fracture, which part is more prone to avascular necrosis?
a) Distal pole
b) Waist
c) Proximal pole
d) Tubercle
Explanation: The proximal pole of scaphoid has retrograde blood supply from the distal entry of radial artery branches. Hence, fracture often compromises proximal circulation leading to avascular necrosis. Correct answer is Proximal pole.
5) Which artery enters scaphoid through its dorsal ridge?
a) Ulnar artery
b) Radial artery
c) Anterior interosseous artery
d) Posterior interosseous artery
Explanation: The dorsal branch of radial artery enters scaphoid through its dorsal ridge and supplies most of the proximal part. Correct answer is Radial artery.
6) Patient with delayed healing of scaphoid fracture – what complication is likely?
a) Osteoarthritis
b) Avascular necrosis
c) Osteomyelitis
d) Carpal tunnel syndrome
Explanation: Due to retrograde blood flow, proximal scaphoid is prone to ischemia after fracture, leading to avascular necrosis. Hence, the major complication of scaphoid fracture is Avascular necrosis.
7) Which carpal bone dislocates most frequently?
a) Scaphoid
b) Lunate
c) Capitate
d) Hamate
Explanation: The lunate, located centrally in the proximal row of carpals, is the most frequently dislocated carpal bone. Correct answer is Lunate.
8) Which nerve may be compressed in lunate dislocation?
a) Ulnar nerve
b) Radial nerve
c) Median nerve
d) Musculocutaneous nerve
Explanation: Lunate dislocation pushes into the carpal tunnel, compressing the median nerve. This leads to sensory and motor symptoms in hand. Correct answer is Median nerve.
9) Which ligament stabilizes the scaphoid and lunate together?
a) Scapholunate ligament
b) Radiocarpal ligament
c) Intercarpal ligament
d) Palmar carpal ligament
Explanation: The scapholunate ligament holds scaphoid and lunate in close articulation. Its rupture leads to carpal instability and may mimic scaphoid fracture symptoms. Correct answer is Scapholunate ligament.
10) Pain in anatomical snuffbox after fall but normal X-ray – best next step?
a) Ignore
b) Cast immobilization
c) Immediate surgery
d) Nerve conduction test
Explanation: Scaphoid fracture may be radiologically occult initially. If clinical suspicion exists, immobilization in thumb spica cast is done, and repeat imaging after 10–14 days confirms fracture. Correct answer is Cast immobilization.
Topic: Arteries of Forearm
Subtopic: Common Interosseous Artery
Keyword Definitions:
Brachial artery: Main artery of arm, continuation of axillary artery.
Radial artery: Lateral terminal branch of brachial artery.
Ulnar artery: Medial terminal branch of brachial artery, supplies forearm.
Common interosseous artery: Short branch of ulnar artery, dividing into anterior and posterior interosseous arteries.
Profunda brachii artery: Deep artery of arm, runs with radial nerve.
Lead Question - 2014
Common interosseous artery is a branch of -
a) Brachial artery
b) Radial artery
c) Ulnar artery
d) Profunda brachii artery
Explanation: The common interosseous artery is a short thick branch arising from the ulnar artery just below the cubital fossa. It quickly divides into anterior and posterior interosseous arteries. These supply the deep flexor and extensor compartments. Hence, the correct answer is Ulnar artery.
1) Which artery accompanies the median nerve in forearm?
a) Radial
b) Ulnar
d) Posterior interosseous
Explanation: The anterior interosseous artery, branch of common interosseous, accompanies the anterior interosseous nerve, a branch of the median nerve, in forearm. It supplies deep flexor muscles. The correct answer is Anterior interosseous.
2) Posterior interosseous artery passes through which structure?
a) Carpal tunnel
b) Interosseous membrane
c) Cubital tunnel
d) Supinator arch
Explanation: The posterior interosseous artery arises from the common interosseous and passes through an opening in the interosseous membrane to reach posterior compartment of forearm. It supplies extensor muscles. The correct answer is Interosseous membrane.
3) Anterior interosseous artery supplies all except?
a) Flexor digitorum profundus
b) Flexor pollicis longus
c) Pronator quadratus
d) Extensor carpi ulnaris
Explanation: The anterior interosseous artery supplies deep flexor muscles like FDP, FPL, pronator quadratus. Extensor carpi ulnaris is located in posterior compartment, supplied by posterior interosseous artery. Hence, the correct answer is Extensor carpi ulnaris.
4) A patient with posterior compartment forearm ischemia likely has obstruction of?
a) Anterior interosseous artery
b) Posterior interosseous artery
c) Radial recurrent artery
d) Profunda brachii artery
Explanation: Ischemia of extensor compartment indicates loss of posterior interosseous artery flow. This artery supplies most extensors and arises from the common interosseous. Thus, the correct answer is Posterior interosseous artery.
5) Which artery contributes to dorsal carpal arch?
a) Anterior interosseous
b) Posterior interosseous
c) Radial
d) Ulnar
Explanation: The posterior interosseous artery anastomoses with branches of radial and ulnar arteries to form the dorsal carpal arch supplying dorsal hand. Therefore, the correct answer is Posterior interosseous.
6) In Volkmann ischemic contracture, compromised artery is?
a) Ulnar
b) Radial
c) Brachial
d) Anterior interosseous
Explanation: Volkmann ischemic contracture results from brachial artery obstruction, reducing blood flow to forearm including interosseous branches. This leads to muscle ischemia and fibrosis. The correct answer is Brachial artery.
7) The nutrient artery to radius is derived from?
a) Anterior interosseous
b) Posterior interosseous
c) Radial
d) Ulnar
Explanation: The anterior interosseous artery gives nutrient branches to radius and ulna. Hence, the nutrient artery to radius comes from Anterior interosseous artery.
8) Injury to posterior interosseous artery during fracture of proximal radius leads to?
a) Loss of flexion
b) Loss of pronation
c) Loss of extension
d) Loss of supination
Explanation: Posterior interosseous artery supplies extensors. Injury leads to extensor weakness or loss. Therefore, correct answer is Loss of extension.
9) Which branch of common interosseous artery participates in palmar carpal arch?
a) Anterior interosseous
b) Posterior interosseous
c) Radial recurrent
d) Deep radial branch
Explanation: The anterior interosseous artery contributes to palmar carpal arch by anastomosing with branches of radial and ulnar arteries. The correct answer is Anterior interosseous.
10) Patient with compartment syndrome of deep anterior forearm – which artery is primarily affected?
a) Radial
b) Ulnar
c) Anterior interosseous
d) Posterior interosseous
Explanation: Deep anterior compartment of forearm is mainly supplied by anterior interosseous artery. Compartment syndrome compromises its blood flow, affecting deep flexors. Thus, the correct answer is Anterior interosseous artery.
Topic: Axilla and Axillary Artery
Subtopic: Divisions of Axillary Artery
Keyword Definitions:
Axillary artery: Continuation of the subclavian artery, supplying the upper limb.
Pectoralis minor: A muscle that divides axillary artery into three parts.
Clavicle: Collar bone, superior boundary of axilla.
Teres minor: Muscle of rotator cuff, posterior boundary near axilla.
1st rib: Marks transition from subclavian to axillary artery.
Quadrangular space: Anatomical interval transmitting axillary nerve and posterior circumflex humeral artery.
Spiral groove: Groove on humerus for radial nerve and profunda brachii artery.
Bicipital aponeurosis: Fibrous extension of biceps tendon protecting underlying neurovascular structures.
Lead Question - 2014
Axillary artery is divided into three parts by?
a) 1st rib
b) Clavicle
c) Pectoralis minor muscle
d) Teres minor muscle
Explanation: The pectoralis minor divides the axillary artery into three parts: first part proximal, second part posterior, and third part distal to the muscle. Other structures like 1st rib and clavicle mark transition of vessels but do not divide axillary artery into parts. Correct answer is c) Pectoralis minor muscle.
1) Branch of the first part of axillary artery?
a) Superior thoracic artery
b) Lateral thoracic artery
d) Thoracoacromial artery
Explanation: The first part of the axillary artery gives only one branch: the superior thoracic artery. The other branches arise from second and third parts. Knowledge of these branches is essential for surgical interventions in axilla. Correct answer is a) Superior thoracic artery.
2) A patient with humeral neck fracture injures the artery closely related to quadrangular space. Which is it?
a) Subscapular artery
b) Posterior circumflex humeral artery
c) Lateral thoracic artery
d) Profunda brachii artery
Explanation: The posterior circumflex humeral artery runs through the quadrangular space with the axillary nerve. It is often injured in surgical neck fractures of humerus. This leads to bleeding and compromised deltoid perfusion. Correct answer is b) Posterior circumflex humeral artery.
3) Which artery accompanies radial nerve in spiral groove?
a) Subscapular artery
b) Profunda brachii artery
c) Circumflex scapular artery
d) Lateral thoracic artery
Explanation: The profunda brachii artery accompanies radial nerve in spiral groove. Both are vulnerable in midshaft humeral fractures, leading to wrist drop and bleeding. Correct answer is b) Profunda brachii artery.
4) Which structure lies superficial to bicipital aponeurosis in cubital fossa?
a) Median nerve
b) Radial nerve
c) Median cubital vein
d) Brachial artery
Explanation: The median cubital vein lies superficial to bicipital aponeurosis, making it accessible for venipuncture. The aponeurosis protects deeper structures like brachial artery and median nerve. Correct answer is c) Median cubital vein.
5) Which branch arises from the second part of axillary artery?
a) Superior thoracic artery
b) Subscapular artery
c) Thoracoacromial artery
d) Posterior circumflex humeral artery
Explanation: The thoracoacromial artery and lateral thoracic artery arise from the second part of axillary artery. The thoracoacromial artery further divides into four branches supplying pectoral and deltoid regions. Correct answer is c) Thoracoacromial artery.
6) A patient with axillary lymph node dissection has bleeding from the largest branch of axillary artery. Which is it?
a) Superior thoracic artery
b) Subscapular artery
c) Thoracoacromial artery
d) Posterior circumflex humeral artery
Explanation: The subscapular artery is the largest branch of the axillary artery. It supplies latissimus dorsi via thoracodorsal artery and scapular region via circumflex scapular branch. Correct answer is b) Subscapular artery.
7) Which muscle divides the axilla into anterior and posterior folds?
a) Teres major
b) Latissimus dorsi
c) Pectoralis major
d) Deltoid
Explanation: The pectoralis major forms the anterior axillary fold, and the latissimus dorsi with teres major form the posterior axillary fold. These landmarks are important clinically for axillary palpation. Correct answer is c) Pectoralis major.
8) A stab wound injures the third part of axillary artery. Which branch is most likely affected?
a) Lateral thoracic artery
b) Subscapular artery
c) Superior thoracic artery
d) Thoracoacromial artery
Explanation: The third part of axillary artery gives rise to subscapular, anterior and posterior circumflex humeral arteries. Subscapular is the largest and most clinically significant. Correct answer is b) Subscapular artery.
9) Which nerve is closely related to the second part of axillary artery?
a) Ulnar nerve
b) Median nerve
c) Cord of brachial plexus
d) Musculocutaneous nerve
Explanation: The cords of brachial plexus are named according to their relation to the second part of axillary artery: lateral, medial, and posterior cords. This relation is important in axillary blocks. Correct answer is c) Cord of brachial plexus.
10) Which artery participates in scapular anastomosis?
a) Subscapular artery
b) Brachial artery
c) Radial artery
d) Ulnar artery
Explanation: The subscapular artery, through its circumflex scapular branch, forms an important part of scapular anastomosis with suprascapular and dorsal scapular arteries. This collateral circulation is crucial if axillary artery is obstructed. Correct answer is a) Subscapular artery.
Topic: Upper Limb
Subtopic: Cubital Fossa Structures
Keyword Definitions:
Bicipital aponeurosis: Fibrous expansion from biceps tendon crossing cubital fossa, protecting underlying brachial artery and median nerve.
Cubital fossa: Triangular area anterior to elbow joint, containing important nerves, arteries, and veins.
Brachial artery: Main artery of upper arm, continuation of axillary artery, lies deep to bicipital aponeurosis.
Veins: Superficial veins like median cubital, cephalic, and basilic veins lie above bicipital aponeurosis.
Ulnar nerve: Passes medial to cubital fossa, not over bicipital aponeurosis.
Radial nerve: Lateral to cubital fossa, deep to brachioradialis, not over aponeurosis.
Lead Question - 2014
Structure over bicipital aponeurosis in cubital fossa?
a) Ulnar nerve
b) Radial nerve
c) Brachial artery
d) Veins
Explanation: Superficial veins such as median cubital vein lie over the bicipital aponeurosis in the cubital fossa, making them easily accessible for venipuncture. Deep structures like brachial artery and median nerve lie beneath the aponeurosis. Correct answer is Veins.
Guessed Questions
1. Median cubital vein connects?
a) Cephalic and basilic veins
b) Brachial and radial arteries
c) Ulnar and radial nerves
d) Median and ulnar nerves
Explanation: The median cubital vein forms a connection between the superficial cephalic and basilic veins in the cubital fossa. It lies above the bicipital aponeurosis and is the preferred site for venipuncture. Correct answer is Cephalic and basilic veins.
2. Which nerve is deep to bicipital aponeurosis?
a) Radial nerve
b) Median nerve
c) Ulnar nerve
d) Musculocutaneous nerve
Explanation: The median nerve passes deep to the bicipital aponeurosis along with the brachial artery in the cubital fossa, protecting it from superficial trauma. Correct answer is Median nerve.
3. Brachial artery lies in relation to bicipital aponeurosis?
a) Superficial
b) Deep
c) Lateral
d) Medial
Explanation: The brachial artery lies deep to the bicipital aponeurosis in the cubital fossa, making the aponeurosis a protective layer during trauma or venipuncture. Correct answer is Deep.
4. Clinical importance of bicipital aponeurosis?
a) Protects brachial artery
b) Guides venipuncture
c) Prevents nerve injury
d) All of the above
Explanation: The bicipital aponeurosis protects underlying structures, including the brachial artery and median nerve, serves as landmark for venipuncture, and minimizes risk of vascular or nerve injury. Correct answer is All of the above.
5. Cephalic vein lies relative to bicipital aponeurosis?
a) Superficial
b) Deep
c) Lateral
d) Medial
Explanation: The cephalic vein is a superficial vein lying above the bicipital aponeurosis, allowing easy access for venipuncture. Deep structures remain protected. Correct answer is Superficial.
6. Structures NOT protected by bicipital aponeurosis?
a) Brachial artery
b) Median nerve
c) Superficial veins
d) Median cubital vein
Explanation: Superficial veins like the median cubital vein lie above the bicipital aponeurosis, and thus are not protected by it, making them vulnerable but easily accessible for venipuncture. Correct answer is Superficial veins.
7. Radial nerve in cubital fossa lies?
a) Medial to biceps tendon
b) Lateral and deep to brachioradialis
c) Superficial to brachialis
d) Over bicipital aponeurosis
Explanation: The radial nerve passes lateral to the cubital fossa, deep to brachioradialis, and does not lie over or under the bicipital aponeurosis. Correct answer is Lateral and deep to brachioradialis.
8. Injury above bicipital aponeurosis can affect?
a) Superficial veins
b) Median nerve
c) Brachial artery
d) Anterior interosseous artery
Explanation: Superficial lacerations above the bicipital aponeurosis primarily affect veins such as median cubital, while deeper structures are protected. Correct answer is Superficial veins.
9. Landmark for venipuncture in cubital fossa?
a) Brachial artery
b) Median cubital vein
c) Bicipital aponeurosis
d) Radial nerve
Explanation: The median cubital vein, superficial to the bicipital aponeurosis, is the preferred landmark for venipuncture because it is easily palpable and accessible without risk to deeper structures. Correct answer is Median cubital vein.
10. Structure lying deep to bicipital aponeurosis?
a) Cephalic vein
b) Median nerve
c) Superficial radial nerve
d) Basilic vein
Explanation: The median nerve lies deep to the bicipital aponeurosis in the cubital fossa along with brachial artery, protected from superficial injury. Superficial veins lie above the aponeurosis. Correct answer is Median nerve.
Topic: Iron Metabolism
Subtopic: Ferritin and Iron Storage
Keyword Definitions:
• Ferritin: Intracellular protein that stores iron and releases it in a controlled fashion.
• Serum ferritin: Circulating ferritin that reflects body iron stores.
• Iron deficiency: Condition with low ferritin, hemoglobin, and iron stores.
• Iron overload: Condition with elevated ferritin and iron deposition in organs.
• Adult male normal range: Standard reference values for clinical interpretation.
Lead Question - 2013
Normal ferritin level in adult male ?
a) 5-10 ng/ml
b) 100-200 ng/ml
c) 500-700 ng/ml
d) 800-900 ng/ml
Explanation: Normal serum ferritin in adult males ranges from 100–200 ng/ml, reflecting adequate iron stores. Low levels indicate iron deficiency, while very high levels may indicate iron overload or inflammation. Answer: b) 100-200 ng/ml.
1) Guess Question:
Low ferritin is most commonly caused by:
a) Hemolysis
b) Chronic blood loss
c) Vitamin B12 deficiency
d) Renal failure
Explanation: Chronic blood loss is the most common cause of low ferritin, leading to iron deficiency anemia. Hemolysis can cause normal or high ferritin, and vitamin B12 deficiency or renal failure do not directly lower ferritin. Answer: b) Chronic blood loss.
2) Guess Question:
Which lab test best reflects total body iron stores?
a) Serum iron
b) Transferrin saturation
c) Serum ferritin
d) Hemoglobin
Explanation: Serum ferritin is the most reliable marker of body iron stores. Serum iron and transferrin saturation fluctuate with diet and inflammation, and hemoglobin changes are late indicators. Answer: c) Serum ferritin.
3) Guess Question:
A patient with chronic inflammation may have:
a) Low ferritin
b) High ferritin
c) Normal ferritin
d) Undetectable ferritin
Explanation: Ferritin is an acute phase reactant, and its levels rise in chronic inflammation or infection, even if iron stores are low. This can mask iron deficiency. Answer: b) High ferritin.
4) Guess Question:
Iron overload disorders are associated with:
a) Ferritin b) Ferritin 100-200 ng/ml
c) Ferritin >500 ng/ml
d) Ferritin
Explanation: Ferritin levels above 500 ng/ml suggest iron overload, as seen in hereditary hemochromatosis or repeated transfusions. Answer: c) Ferritin >500 ng/ml.
5) Guess Question:
Which organ primarily stores ferritin-bound iron?
a) Brain
b) Liver
c) Kidney
d) Heart
Explanation: The liver is the major organ storing iron as ferritin. The spleen and bone marrow also store iron, while the brain and kidney store minimal amounts. Answer: b) Liver.
6) Guess Question:
A 35-year-old man presents with fatigue. Labs: Hb 11 g/dl, ferritin 8 ng/ml. Diagnosis is:
a) Iron deficiency anemia
b) Thalassemia
c) Vitamin B12 deficiency
d) Aplastic anemia
Explanation: Very low ferritin (8 ng/ml) confirms iron deficiency anemia. Hb is mildly reduced, consistent with early anemia. Other causes like thalassemia or B12 deficiency show normal or high ferritin. Answer: a) Iron deficiency anemia.
7) Guess Question:
Which factor falsely elevates ferritin levels?
a) Hemochromatosis
b) Infection
c) Liver disease
d) All of the above
Explanation: Ferritin is an acute phase reactant; it rises in infection, inflammation, liver disease, and genetic iron overload disorders, potentially misleading iron status assessment. Answer: d) All of the above.
8) Guess Question:
Normal ferritin in adult females is generally:
a) 15-150 ng/ml
b) 100-200 ng/ml
c) 500-700 ng/ml
d) 5-10 ng/ml
Explanation: Adult females typically have lower ferritin than males, around 15–150 ng/ml, due to menstrual blood loss and lower iron stores. Answer: a) 15-150 ng/ml.
9) Guess Question:
A patient with ferritin 600 ng/ml and transferrin saturation 80% likely has:
a) Iron deficiency anemia
b) Iron overload
c) Vitamin B12 deficiency
d) Normal iron stores
Explanation: High ferritin and transferrin saturation indicate iron overload, such as hereditary hemochromatosis or repeated transfusions. Answer: b) Iron overload.
10) Guess Question:
Ferritin is measured primarily by:
a) ELISA
b) Western blot
c) PCR
d) Blood smear
Explanation: Serum ferritin is quantified using ELISA, which detects protein levels accurately. Western blot and PCR are not used for routine ferritin measurement. Answer: a) ELISA.
Topic: Blood Physiology
Subtopic: Erythropoiesis and Gender Differences
Keyword Definitions:
• RBC count: Number of red blood cells per microliter of blood.
• Erythropoietin: Hormone from kidneys stimulating RBC production.
• Estrogen: Female sex hormone influencing hematopoiesis.
• Menstrual blood loss: Regular shedding of blood during menstruation.
• Stem cells: Bone marrow progenitors giving rise to blood cells.
Lead Question - 2013
Females have low RBC count compared to males due to ?
a) Low erythropoietin
b) Menstural blood loss
c) High estrogen
d) Low stem cells
Explanation: Females have lower RBC count than males mainly due to high estrogen, which suppresses erythropoiesis, and menstrual blood loss further contributing. Testosterone in males stimulates erythropoietin secretion, enhancing RBC production. The correct answer is c) High estrogen, as it plays a primary physiological role in reduced counts.
1) A 25-year-old woman presents with chronic fatigue. Her lab shows Hb 10 g/dl. What is the likely cause?
a) Low erythropoietin
b) Menstrual blood loss
c) High estrogen
d) Low stem cells
Explanation: In young menstruating women, chronic mild anemia commonly arises from menstrual blood loss. Estrogen also reduces erythropoiesis, but menstrual loss is clinically more relevant here. The correct answer is b) Menstrual blood loss, explaining the patient’s low hemoglobin with otherwise normal erythropoietin function.
2) Which hormone increases RBC production significantly in males?
a) Progesterone
b) Estrogen
c) Testosterone
d) Cortisol
Explanation: Testosterone stimulates erythropoietin secretion and directly promotes bone marrow activity. This leads to higher RBC counts in males compared to females. The correct answer is c) Testosterone, explaining the gender difference in hemoglobin and hematocrit values.
3) In chronic renal failure, anemia occurs mainly due to deficiency of?
a) Iron
b) Erythropoietin
c) Vitamin B12
d) Folic acid
Explanation: Chronic renal failure leads to reduced erythropoietin secretion from diseased kidneys, resulting in normocytic normochromic anemia. The correct answer is b) Erythropoietin, which is the most important factor impaired in such patients.
4) A 30-year-old female has low Hb, normal iron studies, normal B12 and folate. Most likely cause is?
a) Low stem cells
b) Menstrual blood loss
c) Low erythropoietin
d) High cortisol
Explanation: With normal nutritional and hormonal markers, menstrual blood loss is the commonest explanation for anemia in reproductive-age women. The correct answer is b) Menstrual blood loss.
5) Which of the following reduces erythropoiesis?
a) Hypoxia
b) Testosterone
c) Estrogen
d) Erythropoietin
Explanation: Estrogen inhibits erythropoiesis, unlike hypoxia, testosterone, and erythropoietin, which stimulate RBC production. Hence, females naturally have lower counts than males. The correct answer is c) Estrogen.
6) A 22-year-old female presents with pallor, Hb 9 g/dl, MCV 72 fl, microcytosis. Most probable cause is?
a) Estrogen suppression
b) Iron deficiency from menstruation
c) Stem cell depletion
d) Low erythropoietin
Explanation: Microcytic anemia with low hemoglobin strongly suggests iron deficiency, commonly due to chronic menstrual blood loss. The correct answer is b) Iron deficiency from menstruation.
7) Which is the normal RBC count in adult females?
a) 3.2–3.6 million/cu mm
b) 3.8–4.5 million/cu mm
c) 4.5–5.5 million/cu mm
d) 5.5–6.0 million/cu mm
Explanation: Normal RBC count in females is about 3.8–4.5 million/cu mm, while in males it is 4.5–5.5 million/cu mm. The correct answer is b) 3.8–4.5 million/cu mm, highlighting the physiological difference.
8) A 55-year-old postmenopausal woman shows improved hemoglobin levels compared to her earlier reports. Which factor explains this?
a) Decreased estrogen effect
b) Increased menstrual loss
c) Decreased testosterone
d) Increased cortisol
Explanation: After menopause, estrogen levels decline, reducing its inhibitory effect on erythropoiesis. This leads to improved hemoglobin and RBC count in postmenopausal women. The correct answer is a) Decreased estrogen effect.
9) Which vitamin is essential for RBC DNA synthesis?
a) Vitamin B6
b) Vitamin B12
c) Vitamin C
d) Vitamin K
Explanation: Vitamin B12 is essential for DNA synthesis during RBC maturation. Its deficiency causes megaloblastic anemia. The correct answer is b) Vitamin B12.
10) A 40-year-old man with chronic alcoholism develops macrocytic anemia. The deficiency is most likely?
a) Vitamin B12
b) Folate
c) Iron
d) Erythropoietin
Explanation: Chronic alcoholics frequently develop folate deficiency due to poor diet and malabsorption, leading to macrocytic anemia. The correct answer is b) Folate.
Chapter: Hematology
Topic: Hemostasis
Subtopic: Von Willebrand Factor
Keyword Definitions:
Von Willebrand factor (vWF): Glycoprotein important for platelet adhesion and factor VIII stabilization.
Endothelial cells: Cells lining blood vessels that synthesize and release vWF.
Megakaryocytes: Large bone marrow cells that give rise to platelets and store vWF.
Hepatocytes: Liver cells primarily involved in clotting factor synthesis but not vWF.
Hemostasis: Process preventing blood loss involving vasoconstriction, platelets, and clotting factors.
Lead Question (2013):
Von Willebrand factor is synthesized by all except?
a) Endothelial cells
b) Megakaryocytes
c) Hepatocytes
d) None
Explanation: Von Willebrand factor is synthesized by endothelial cells and stored in Weibel-Palade bodies, and also by megakaryocytes stored in platelet α-granules. Hepatocytes do not produce vWF, though they synthesize other clotting factors. Answer: c) Hepatocytes.
1) Guess Question:
Von Willebrand factor primarily mediates:
a) Platelet adhesion
b) Fibrin degradation
c) Prothrombin activation
d) Plasminogen activation
Explanation: vWF is critical for platelet adhesion to subendothelial collagen at injury sites. It also carries and stabilizes factor VIII, but it does not directly activate prothrombin or plasminogen. Answer: a) Platelet adhesion.
2) Guess Question:
A patient with recurrent nosebleeds has prolonged bleeding time but normal platelet count. The likely cause is:
a) Hemophilia A
b) Von Willebrand disease
c) Thrombocytopenia
d) Vitamin K deficiency
Explanation: Von Willebrand disease causes defective platelet adhesion and prolongs bleeding time, with normal platelet count. Hemophilia A shows factor VIII deficiency, thrombocytopenia lowers platelet count, and vitamin K deficiency affects clotting factor synthesis. Answer: b) Von Willebrand disease.
3) Guess Question:
Von Willebrand factor is stored in endothelial cells within:
a) Lysosomes
b) Weibel-Palade bodies
c) Ribosomes
d) Mitochondria
Explanation: Endothelial cells store vWF in Weibel-Palade bodies, specialized storage organelles. Platelets store vWF in α-granules. Ribosomes and mitochondria are not storage sites for vWF. Answer: b) Weibel-Palade bodies.
4) Guess Question:
A 16-year-old girl presents with menorrhagia and prolonged bleeding after dental extraction. Which lab finding is expected?
a) Increased bleeding time, prolonged aPTT
b) Decreased bleeding time, normal aPTT
c) Increased platelet count, prolonged PT
d) Normal coagulation profile
Explanation: In Von Willebrand disease, bleeding time is prolonged due to defective platelet adhesion, and aPTT may be prolonged due to decreased factor VIII stabilization. Platelet count is normal. Answer: a) Increased bleeding time, prolonged aPTT.
5) Guess Question:
Which factor is stabilized in plasma by vWF?
a) Factor V
b) Factor VII
c) Factor VIII
d) Factor IX
Explanation: vWF binds and stabilizes factor VIII in plasma, protecting it from proteolytic degradation. This allows effective participation in the intrinsic coagulation pathway. Answer: c) Factor VIII.
6) Guess Question:
A 22-year-old male with vWF disease presents with excessive bleeding after surgery. First-line therapy is:
a) Vitamin K
b) Desmopressin (DDAVP)
c) Warfarin
d) Heparin
Explanation: Desmopressin stimulates endothelial cells to release stored vWF, improving hemostasis in mild to moderate vWF disease. Vitamin K, warfarin, and heparin are not useful here. Answer: b) Desmopressin (DDAVP).
7) Guess Question:
Von Willebrand disease is inherited as:
a) Autosomal dominant or recessive
b) X-linked dominant
c) X-linked recessive
d) Mitochondrial
Explanation: Most forms of Von Willebrand disease are inherited as autosomal dominant, though rare recessive forms exist. It is not X-linked like hemophilia A or B, nor mitochondrial. Answer: a) Autosomal dominant or recessive.
8) Guess Question:
A 30-year-old woman with vWF disease is scheduled for delivery. To prevent bleeding, the drug of choice is:
a) Desmopressin
b) Aspirin
c) Heparin
d) Warfarin
Explanation: Desmopressin is effective in releasing vWF from endothelial cells during surgery or delivery, reducing bleeding risk. Aspirin increases bleeding, while warfarin and heparin are anticoagulants. Answer: a) Desmopressin.
9) Guess Question:
Which test is most sensitive for screening vWF disease?
a) Platelet count
b) PT test
c) Ristocetin cofactor activity
d) Thrombin time
Explanation: Ristocetin cofactor activity measures platelet aggregation in presence of ristocetin, highly sensitive for vWF disease. Platelet count is normal, PT and thrombin time are usually not prolonged. Answer: c) Ristocetin cofactor activity.
10) Guess Question:
Von Willebrand disease is associated with defective:
a) Platelet aggregation
b) Platelet adhesion
c) Prothrombin activation
d) Factor XIII stabilization
Explanation: vWF deficiency leads to defective platelet adhesion to subendothelial collagen at vascular injury sites. Platelet aggregation is normal, while prothrombin activation and factor XIII stabilization are unaffected. Answer: b) Platelet adhesion.
Subtopic: Hepatic Blood Flow
Keyword Definitions:
• Liver: Largest internal organ, vital for metabolism and detoxification.
• Blood supply: Dual supply via hepatic artery and portal vein.
• Portal vein: Carries nutrient-rich blood from intestines.
• Hepatic artery: Supplies oxygen-rich blood.
• Perfusion rate: Blood flow through tissue per unit weight.
Lead Question - 2013
Blood supply of liver [ml/100g/min]
a) 1500-2000
b) 1000-1500
c) 50-60
d) 250-300
Explanation:
The liver receives dual blood supply, about 25% from the hepatic artery and 75% from the portal vein. Normal perfusion of liver tissue is approximately 1000–1500 ml/100g/min. This ensures metabolic, detoxification, and synthetic functions. Answer: b) 1000–1500.
1) Which vessel supplies 75% of blood flow to the liver?
a) Hepatic artery
b) Portal vein
c) Hepatic vein
d) Inferior vena cava
Explanation:
The portal vein carries nutrient-rich blood from intestines and supplies nearly 75% of hepatic blood flow. The hepatic artery contributes the remaining 25%. This dual system supports liver’s metabolic activity. Answer: b) Portal vein.
2) During portal vein thrombosis, which vessel compensates blood flow?
a) Inferior vena cava
b) Hepatic artery
c) Renal vein
d) Splenic vein
Explanation:
When the portal vein is obstructed, the hepatic artery increases blood flow to maintain perfusion. This autoregulatory mechanism protects hepatocytes temporarily but cannot fully replace portal supply. Answer: b) Hepatic artery.
3) In cirrhosis, portal hypertension occurs due to:
a) Increased hepatic artery flow
b) Increased resistance in liver sinusoids
c) Decreased portal vein inflow
d) Increased hepatic vein drainage
Explanation:
Cirrhosis increases resistance within liver sinusoids and portal tracts, leading to portal hypertension. This causes ascites, varices, and splenomegaly. Answer: b) Increased resistance in liver sinusoids.
4) A 50-year-old man with cirrhosis presents with hematemesis. Most likely cause?
a) Gastric ulcer
b) Variceal bleed
c) Mallory-Weiss tear
d) Duodenal ulcer
Explanation:
In cirrhosis with portal hypertension, esophageal varices rupture commonly causing hematemesis. Portal-systemic shunts lead to dilated veins that rupture easily. Answer: b) Variceal bleed.
5) Oxygen content of hepatic arterial blood is:
a) High
b) Low
c) Moderate
d) Absent
Explanation:
Hepatic arterial blood is oxygen-rich, supplying hepatocytes with required oxygen. Portal vein blood is nutrient-rich but relatively low in oxygen. Answer: a) High.
6) A patient with Budd-Chiari syndrome presents with hepatomegaly. Blockage site is:
a) Hepatic veins
b) Portal vein
c) Hepatic artery
d) Inferior vena cava below diaphragm
Explanation:
Budd-Chiari syndrome results from obstruction of hepatic veins draining the liver, leading to hepatomegaly, ascites, and abdominal pain. Answer: a) Hepatic veins.
7) Functional unit of liver is:
a) Lobule
b) Acinus
c) Sinusoid
d) Portal triad
Explanation:
The hepatic acinus is the functional unit based on blood supply, oxygenation, and metabolism, divided into zones 1–3. Answer: b) Acinus.
8) A 45-year-old alcoholic develops ascites. Primary mechanism?
a) Decreased albumin
b) Increased portal pressure
c) Increased aldosterone
d) Lymphatic obstruction
Explanation:
Portal hypertension due to cirrhosis increases hydrostatic pressure, pushing fluid into peritoneal cavity. Hypoalbuminemia and aldosterone excess worsen ascites. Answer: b) Increased portal pressure.
9) Liver receives what percent of cardiac output?
a) 10%
b) 15%
c) 25%
d) 40%
Explanation:
Liver receives approximately 25% of cardiac output via portal vein and hepatic artery, essential for metabolism and detoxification. Answer: c) 25%.
10) In hepatic encephalopathy, toxin accumulation is mainly:
a) Ammonia
b) Urea
c) Bilirubin
d) Lactate
Explanation:
Ammonia accumulates due to impaired detoxification in liver failure, crossing blood-brain barrier and altering neurotransmission. This causes hepatic encephalopathy. Answer: a) Ammonia.
Topic: Body Fluid Compartments
Subtopic: Measurement of Plasma Volume
Keyword Definitions:
• Plasma volume: Volume of plasma in blood, measured using dye dilution techniques.
• Evans blue: Dye binding to plasma proteins, used to measure plasma volume.
• Inulin: Polysaccharide used for GFR measurement, not plasma volume.
• Mannitol: Used to measure extracellular fluid volume.
• D2O (Deuterium oxide): Used to measure total body water.
Lead Question - 2013
Plasma volume is measured by ?
a) Inulin
b) Evans blue
c) Mannitol
d) D2O
Explanation: Plasma volume is best measured by Evans blue dye, which binds to plasma proteins like albumin, remaining confined within the vascular compartment. Inulin is for GFR, Mannitol for ECF, and D2O for total body water. Correct answer: Evans blue.
1) Plasma osmolality is mainly determined by?
a) Sodium
b) Potassium
c) Calcium
d) Glucose
Explanation: Plasma osmolality primarily depends on sodium and its associated anions, as sodium is the major extracellular cation. Glucose and urea contribute less significantly under normal conditions. Correct answer: Sodium.
2) A patient receives IV mannitol infusion. What compartment expands most?
a) Plasma volume
b) Extracellular fluid
c) Intracellular fluid
d) Total body water
Explanation: Mannitol distributes only in extracellular fluid, not intracellular fluid. Thus, it expands ECF volume, increasing osmotic gradient and drawing water out of cells. Correct answer: Extracellular fluid.
3) Extracellular fluid volume is measured using?
a) Mannitol
b) Inulin
c) Evans blue
d) D2O
Explanation: Mannitol, sucrose, or thiosulfate can be used to measure extracellular fluid volume since they distribute in both plasma and interstitial fluid compartments but not intracellularly. Correct answer: Mannitol.
4) A patient presents with edema. Plasma volume estimation is best done by?
a) Radio-iodinated albumin
b) Inulin
c) Mannitol
d) D2O
Explanation: Plasma volume is accurately measured by dyes like Evans blue or radio-iodinated albumin, which bind plasma proteins. They do not cross into interstitial fluid. Correct answer: Radio-iodinated albumin.
5) Total body water is best measured by?
a) Inulin
b) D2O
c) Evans blue
d) Mannitol
Explanation: Deuterium oxide (D2O) or tritiated water distributes throughout all compartments, making them ideal for total body water measurement. Correct answer: D2O.
6) A child with diarrhea and dehydration is admitted. Which compartment is lost most?
a) Plasma volume
b) Interstitial fluid
c) Intracellular fluid
d) Extracellular fluid
Explanation: Diarrhea causes loss of extracellular fluid (plasma + interstitial fluid). Severe dehydration may later involve intracellular fluid. Correct answer: Extracellular fluid.
7) Interstitial fluid volume is calculated as?
a) Total body water – Plasma volume
b) Extracellular fluid – Plasma volume
c) Intracellular fluid – Plasma volume
d) Plasma volume – ECF
Explanation: Interstitial fluid cannot be measured directly but is calculated as extracellular fluid volume minus plasma volume. Correct answer: Extracellular fluid – Plasma volume.
8) A burn patient develops hypovolemia. Which compartment is primarily lost?
a) Intracellular fluid
b) Plasma volume
c) Interstitial fluid
d) Extracellular fluid
Explanation: Burns cause increased capillary permeability, leading to plasma protein and fluid loss into interstitial space, reducing plasma volume and overall ECF. Correct answer: Plasma volume.
9) Inulin clearance is used to measure?
a) Plasma volume
b) GFR
c) ECF volume
d) Total body water
Explanation: Inulin is freely filtered by glomeruli, not reabsorbed or secreted. Its clearance accurately measures glomerular filtration rate (GFR), not body fluid compartments. Correct answer: GFR.
10) A 65-year-old hypertensive patient on diuretics shows hyponatremia. Which compartment shrinks most?
a) Intracellular fluid
b) Extracellular fluid
c) Plasma volume
d) Interstitial fluid
Explanation: Diuretic use with sodium loss primarily reduces extracellular fluid volume, including plasma and interstitial compartments, leading to hypovolemia and electrolyte imbalance. Correct answer: Extracellular fluid.
11) D2O method helps in calculating which compartment?
a) Plasma volume
b) Extracellular fluid
c) Total body water
d) Interstitial fluid
Explanation: D2O distributes across all fluid compartments (plasma, interstitial, intracellular). Therefore, it is used to measure total body water. Correct answer: Total body water.
Topic: Hypothalamic Functions
Subtopic: Temperature Regulation
Temperature Centre: Area in the hypothalamus regulating body temperature homeostasis.
Supraoptic Nucleus: Produces ADH and oxytocin involved in water balance and lactation.
Paraventricular Nucleus: Controls autonomic functions and synthesizes oxytocin and ADH.
Preoptic Nucleus: Acts as the primary thermoregulatory center, integrating temperature signals.
Suprachiasmatic Nucleus: Regulates circadian rhythms via light input from the retina.
Lead Question - 2013 (September 2008)
Temperature centre is?
a) Supraoptic nucleus of hypothalamus
b) Paraventricular nucleus of hypothalamus
c) Preoptic nucleus of hypothalamus
d) Suprachiasmatic nucleus of hypothalamus
Answer: c) Preoptic nucleus of hypothalamus
Explanation: The preoptic nucleus of the hypothalamus serves as the primary temperature regulation center in humans. It integrates input from thermoreceptors and initiates appropriate responses such as sweating or shivering to maintain body temperature homeostasis. Dysfunction may result in hyperthermia or hypothermia.
1. Guessed Question
Which hypothalamic nucleus controls circadian rhythm?
a) Supraoptic nucleus
b) Paraventricular nucleus
c) Preoptic nucleus
d) Suprachiasmatic nucleus
Answer: d) Suprachiasmatic nucleus
Explanation: The suprachiasmatic nucleus (SCN) in the hypothalamus functions as the body’s master clock, regulating circadian rhythms. It receives light signals via the retinohypothalamic tract and adjusts sleep-wake cycles, hormone secretion, and body temperature accordingly. Disruption leads to sleep disorders and metabolic dysfunction.
2. Guessed Question
Which hormone is secreted by the supraoptic nucleus?
a) ADH
b) Oxytocin
c) Both ADH and Oxytocin
d) TSH
Answer: c) Both ADH and Oxytocin
Explanation: The supraoptic nucleus of the hypothalamus synthesizes both antidiuretic hormone (ADH) and oxytocin. ADH regulates water retention in kidneys, while oxytocin controls uterine contractions and milk ejection. These hormones travel via axonal transport to the posterior pituitary for release into circulation.
3. Guessed Question
Function of paraventricular nucleus includes?
a) ADH production
b) Oxytocin secretion
c) Autonomic regulation
d) All of the above
Answer: d) All of the above
Explanation: The paraventricular nucleus of the hypothalamus is involved in secreting ADH and oxytocin and controlling autonomic functions such as blood pressure and digestion. It integrates neural and endocrine signals to maintain homeostasis, demonstrating its critical role in neuroendocrine regulation.
4. Guessed Question
Preoptic nucleus stimulates which thermoregulatory responses during hyperthermia?
a) Vasoconstriction
b) Shivering
c) Sweating
d) Increased appetite
Answer: c) Sweating
Explanation: During hyperthermia, the preoptic nucleus activates efferent pathways to promote sweating and cutaneous vasodilation. Sweating increases heat loss via evaporation, aiding thermoregulation. Proper functioning is essential for maintaining body temperature and preventing heat stroke in elevated environmental temperatures or fever.
5. Guessed Question
Which structure provides primary input to the preoptic nucleus?
a) Thermoreceptors in the skin
b) Central thermoreceptors in hypothalamus
c) Retinal photoreceptors
d) Olfactory receptors
Answer: a) Thermoreceptors in the skin
Explanation: Thermoreceptors in the skin detect environmental temperature changes and relay signals to the preoptic nucleus of the hypothalamus. This integration triggers thermoregulatory responses such as vasodilation, vasoconstriction, and sweating, maintaining core body temperature within a narrow physiological range.
6. Guessed Question
Lesion in the preoptic area causes?
a) Hypothermia
b) Hyperthermia
c) Bradycardia
d) Hypertension
Answer: b) Hyperthermia
Explanation: Lesion of the preoptic nucleus of the hypothalamus impairs thermoregulatory control, leading to inability to dissipate heat. Consequently, the patient suffers from hyperthermia due to lack of sweating and vasodilation, emphasizing the nucleus's crucial role in temperature homeostasis.
7. Guessed Question
Which hypothalamic nucleus regulates thirst?
a) Supraoptic nucleus
b) Paraventricular nucleus
c) Preoptic nucleus
d) Lateral hypothalamic area
Answer: c) Preoptic nucleus
Explanation: The preoptic nucleus of the hypothalamus integrates osmolarity signals and regulates thirst, prompting increased water intake when plasma osmolality rises. This function is vital in preventing dehydration and maintaining electrolyte balance, with dysfunction causing disturbances in fluid homeostasis.
8. Guessed Question
Which neurotransmitter is predominantly used in preoptic nucleus thermoregulation?
a) GABA
b) Glutamate
c) Dopamine
d) Acetylcholine
Answer: b) Glutamate
Explanation: Glutamate acts as the primary excitatory neurotransmitter in the preoptic nucleus for thermoregulation. It mediates activation of downstream pathways responsible for vasodilation and sweating in response to increased body temperature, maintaining homeostasis by integrating sensory input and initiating effectors.
9. Guessed Question
Which nucleus is involved in antidiuretic hormone (ADH) release?
a) Preoptic nucleus
b) Supraoptic nucleus
c) Paraventricular nucleus
d) Suprachiasmatic nucleus
Answer: b) Supraoptic nucleus
Explanation: The supraoptic nucleus of the hypothalamus synthesizes ADH, which regulates water reabsorption in the kidneys. ADH release occurs in response to increased plasma osmolality or decreased blood volume, conserving water to maintain homeostasis. Disorders lead to diabetes insipidus or water retention abnormalities.
10. Guessed Question
Which hypothalamic nucleus is primarily responsible for thermoregulatory cooling?
a) Suprachiasmatic nucleus
b) Paraventricular nucleus
c) Preoptic nucleus
d) Supraoptic nucleus
Answer: c) Preoptic nucleus
Explanation: The preoptic nucleus of the hypothalamus mediates thermoregulatory cooling via vasodilation and sweating. It integrates signals from peripheral and central thermoreceptors and activates pathways to dissipate heat. Failure in this mechanism leads to hyperthermia and impaired thermoregulation, especially during febrile illnesses.
Topic: Cardiovascular System
Subtopic: Hemodynamics
Blood Velocity: Distance blood travels in vessels per unit time.
Laminar Flow: Smooth, parallel movement of blood in vessels.
Flow Rate: Volume of blood passing through a vessel per unit time.
Cross-sectional Area: Affects blood velocity inversely.
Lead Question - 2013
Normal velocity of blood is ? (September 2008)
a) 40-50 cm/sec
b) 100-150 cm/sec
c) 200-250 cm/sec
d) 250-300 cm/sec
Answer & Explanation: The correct answer is (b) 100-150 cm/sec. In large arteries such as the aorta, the normal blood velocity ranges approximately from 100 to 150 cm/sec under physiological conditions. This is due to the combined effects of cardiac output and vessel cross-sectional area, ensuring efficient circulation.
Guessed Questions
Blood Velocity: Distance blood travels in vessels per unit time.
Laminar Flow: Smooth, parallel movement of blood in vessels.
Flow Rate: Volume of blood passing through a vessel per unit time.
Cross-sectional Area: Affects blood velocity inversely.
Blood velocity in capillaries is approximately:
a) 0.1 cm/sec
b) 1 cm/sec
c) 10 cm/sec
d) 50 cm/sec
Answer & Explanation: The correct answer is (a) 0.1 cm/sec. Capillaries have the largest total cross-sectional area, causing a marked decrease in blood velocity, allowing time for efficient exchange of gases and nutrients between blood and tissues during the microcirculatory process.
Factors determining blood velocity include:
a) Blood viscosity
b) Vessel diameter
c) Pressure gradient
d) All of the above
Answer & Explanation: The correct answer is (d) All of the above. Blood velocity depends on multiple factors: pressure gradient drives flow, vessel diameter determines resistance per Poiseuille’s law, and blood viscosity affects internal friction. Together, these control hemodynamics ensuring adequate perfusion throughout the body.
During exercise, blood velocity in muscles:
a) Decreases
b) Remains unchanged
c) Increases
d) Ceases
Answer & Explanation: The correct answer is (c) Increases. Exercise induces vasodilation of skeletal muscle arterioles, leading to an increased blood flow and velocity to meet metabolic demands. This enhances oxygen and nutrient delivery while removing waste, critical for sustaining physical activity and muscle performance.
In atherosclerosis, blood velocity typically:
a) Increases due to narrowed lumen
b) Decreases due to obstruction
c) Remains same
d) Reverses direction
Answer & Explanation: The correct answer is (a) Increases due to narrowed lumen. Atherosclerosis narrows arterial lumen, increasing resistance but also locally increasing blood velocity per the continuity equation, potentially leading to turbulence and promoting further vascular damage and plaque formation over time.
Bernoulli's principle relates to blood velocity how?
a) Higher velocity lowers pressure
b) Higher velocity increases pressure
c) No relationship
d) Only in veins
Answer & Explanation: The correct answer is (a) Higher velocity lowers pressure. According to Bernoulli's principle, as blood velocity increases in narrowed sections of vessels, intravascular pressure drops. This explains phenomena like aneurysm risk or vessel collapse under certain pathological conditions.
Clinical condition associated with low blood velocity:
a) Thrombosis
b) Hypertension
c) Tachycardia
d) None
Answer & Explanation: The correct answer is (a) Thrombosis. Slow or stagnant blood flow promotes thrombus formation by allowing clotting factors to accumulate and platelets to adhere to vessel walls, increasing the risk of vascular occlusion and embolic events, particularly in veins.
Normal blood velocity in veins compared to arteries is:
a) Higher
b) Lower
c) Same
d) Variable
Answer & Explanation: The correct answer is (b) Lower. Veins have larger cross-sectional areas than arteries, leading to reduced blood velocity despite lower pressure gradients. This slow flow favors venous return through skeletal muscle pumps and valves rather than relying solely on pressure.
Laminar blood flow means:
a) Turbulent flow
b) Disorganized flow
c) Parallel flow
d) High velocity flow
Answer & Explanation: The correct answer is (c) Parallel flow. Laminar flow describes smooth, orderly movement of blood in parallel layers, minimizing energy loss and preventing vascular damage. Turbulent flow, in contrast, is disorganized and can occur in pathological states like atherosclerosis.
Hemodynamic equation relating flow, pressure, and resistance:
a) Q = P/R
b) Q = R/P
c) Q = P × R
d) Q = P + R
Answer & Explanation: The correct answer is (a) Q = P/R. This fundamental equation of hemodynamics expresses flow (Q) as the ratio of pressure difference (P) to resistance (R). It explains blood flow changes under various physiological and pathological states and guides clinical understanding of circulation.
Topic: Cardiovascular System
Subtopic: Baroreceptor Reflex
Baroreceptors: Pressure-sensitive nerve endings detecting blood pressure changes.
Sino-aortic Nerves: Carry signals from baroreceptors to brainstem.
Nucleus Ambiguus: Brainstem nucleus mediating parasympathetic output to heart.
Vagal Discharge: Parasympathetic stimulation reducing heart rate.
Lead Question - 2013
All are true about baroreceptors, except? (September 2008)
a) Stimulated when BP decreases
b) Afferents are through sino-aortic nerves
c) Stimulation causes increased vagal discharge
d) Stimulate nucleus ambiguous
Answer & Explanation: The correct answer is (a) Stimulated when BP decreases. Baroreceptors are mechanoreceptors that are stimulated when blood pressure increases, not decreases. Increased BP stretches their walls, enhancing afferent signaling to the brainstem, which increases vagal tone and decreases sympathetic output to lower blood pressure, maintaining homeostasis.
Guessed Questions
Baroreceptors: Pressure-sensitive nerve endings detecting blood pressure changes.
Sino-aortic Nerves: Carry signals from baroreceptors to brainstem.
Nucleus Ambiguus: Brainstem nucleus mediating parasympathetic output to heart.
Vagal Discharge: Parasympathetic stimulation reducing heart rate.
Baroreceptors are located in:
a) Carotid sinus and aortic arch
b) Pulmonary artery
c) Left ventricle
d) Cerebral cortex
Answer & Explanation: The correct answer is (a) Carotid sinus and aortic arch. These locations are primary sites where baroreceptors detect stretch due to changes in blood pressure. Signals are relayed via the glossopharyngeal and vagus nerves to modulate autonomic responses and maintain hemodynamic stability effectively.
Baroreceptor stimulation results in:
a) Increased heart rate
b) Decreased heart rate
c) Increased cardiac contractility
d) Increased sympathetic tone
Answer & Explanation: The correct answer is (b) Decreased heart rate. Increased arterial pressure activates baroreceptors, which send afferent signals to the medulla, leading to increased parasympathetic (vagal) discharge and decreased sympathetic outflow, thus reducing heart rate and helping lower blood pressure.
Baroreceptors primarily detect changes in:
a) Blood oxygen levels
b) Blood pressure
c) Blood pH
d) Body temperature
Answer & Explanation: The correct answer is (b) Blood pressure. Baroreceptors are sensitive to arterial wall stretch caused by changes in blood pressure. This allows rapid detection and correction of acute blood pressure variations, crucial for cardiovascular homeostasis during positional changes and circulatory stress.
Which nerve carries afferent impulses from carotid baroreceptors?
a) Vagus nerve
b) Glossopharyngeal nerve
c) Phrenic nerve
d) Hypoglossal nerve
Answer & Explanation: The correct answer is (b) Glossopharyngeal nerve. The carotid sinus baroreceptors send afferent signals via the glossopharyngeal nerve (cranial nerve IX) to the nucleus tractus solitarius in the brainstem, playing a critical role in short-term blood pressure regulation.
Clinical importance of baroreceptor reflex includes:
a) Long-term BP regulation
b) Immediate BP adjustment
c) Hormonal control of BP
d) Structural heart disease
Answer & Explanation: The correct answer is (b) Immediate BP adjustment. Baroreceptors enable rapid buffering of acute blood pressure changes through autonomic reflexes. They are essential in preventing hypotension upon standing or during acute stress, providing a critical short-term mechanism of cardiovascular stability.
Baroreceptor dysfunction may cause:
a) Stable BP
b) Labile hypertension
c) Bradycardia
d) Hypoglycemia
Answer & Explanation: The correct answer is (b) Labile hypertension. Impaired baroreceptor sensitivity leads to poor regulation of blood pressure, resulting in labile or fluctuating hypertension, orthostatic hypotension, and increased risk of cardiovascular complications due to inadequate feedback control of blood pressure.
Which brain region integrates baroreceptor signals?
a) Cerebellum
b) Hypothalamus
c) Nucleus tractus solitarius (NTS)
d) Medullary reticular formation
Answer & Explanation: The correct answer is (c) Nucleus tractus solitarius (NTS). The NTS in the medulla integrates afferent inputs from baroreceptors and coordinates autonomic output, modulating sympathetic and parasympathetic activities to maintain blood pressure homeostasis effectively.
Baroreceptor reflex prevents sudden changes in:
a) Blood pH
b) Body temperature
c) Blood pressure
d) Heart rate only
Answer & Explanation: The correct answer is (c) Blood pressure. The baroreceptor reflex responds to acute fluctuations in blood pressure by adjusting heart rate and vascular tone, thus preventing dangerous extremes of hypertension or hypotension and maintaining circulatory equilibrium during daily activities.
Baroreceptor afferent pathways include:
a) Only vagus nerve
b) Glossopharyngeal and vagus nerves
c) Phrenic nerve only
d) Sympathetic chain
Answer & Explanation: The correct answer is (b) Glossopharyngeal and vagus nerves. Carotid baroreceptors use the glossopharyngeal nerve, and aortic baroreceptors use the vagus nerve to transmit pressure information to the brainstem for processing and reflex regulation of cardiovascular function.
Topic: Cardiovascular System
Subtopic: Microcirculation
Critical Closing Pressure: Pressure at which small vessels collapse and stop blood flow.
Capillary Pressure: Pressure within capillaries driving fluid exchange.
Venous Pressure: Pressure in the venous system returning blood to the heart.
Arterial Pressure: Blood pressure in arteries supplying organs.
Lead Question - 2013
Critical closing pressure is? (September 2008)
a) Arterial pressure minus venous pressure
b) Capillary pressure minus venous pressure
c) Pressure below which capillaries close
d) None of the above
Answer & Explanation: The correct answer is (c) Pressure below which capillaries close. Critical closing pressure is the minimum pressure needed to keep capillaries open for blood flow. Below this pressure, external tissue pressure exceeds intravascular pressure, causing vessel collapse and cessation of microcirculatory blood flow, affecting tissue perfusion.
Guessed Questions
Critical Closing Pressure: Pressure at which small vessels collapse and stop blood flow.
Capillary Pressure: Pressure within capillaries driving fluid exchange.
Venous Pressure: Pressure in the venous system returning blood to the heart.
Arterial Pressure: Blood pressure in arteries supplying organs.
Which factor increases critical closing pressure?
a) Vasodilation
b) Increased external tissue pressure
c) Low venous pressure
d) Increased arterial pressure
Answer & Explanation: The correct answer is (b) Increased external tissue pressure. External tissue pressure compresses capillaries and raises the critical closing pressure threshold, which can result in capillary closure and reduced perfusion in conditions such as compartment syndrome or edema.
Critical closing pressure concept helps explain:
a) Arterial stenosis
b) Capillary perfusion threshold
c) Venous thrombosis
d) Aortic compliance
Answer & Explanation: The correct answer is (b) Capillary perfusion threshold. The critical closing pressure determines the minimum pressure necessary to maintain capillary blood flow. Below this, capillaries collapse, limiting oxygen delivery to tissues and playing a role in microcirculatory dysfunction during shock or trauma.
Clinical significance of critical closing pressure is most evident in:
a) Hypertension
b) Shock states
c) Hyperthyroidism
d) Diabetes mellitus
Answer & Explanation: The correct answer is (b) Shock states. During shock, reduced arterial pressure may fall below the critical closing pressure, leading to capillary collapse, inadequate tissue perfusion, and organ dysfunction, highlighting its importance in critical care monitoring and management.
Which hormone increases critical closing pressure by vasoconstriction?
a) Epinephrine
b) Insulin
c) Aldosterone
d) Glucagon
Answer & Explanation: The correct answer is (a) Epinephrine. Epinephrine causes vasoconstriction, raising vascular tone and critical closing pressure. This mechanism redirects blood flow to vital organs during stress but may impair tissue perfusion in shock by increasing the pressure threshold for capillary patency.
Critical closing pressure concept was proposed by:
a) Starling
b) Guyton
c) Burton
d) Fick
Answer & Explanation: The correct answer is (c) Burton. Burton described the concept of critical closing pressure in microcirculation, explaining the pressure below which small vessels collapse and no blood flows. This fundamental understanding helps explain phenomena in tissue perfusion and microvascular physiology.
Clinical example of increased critical closing pressure:
a) Exercise
b) Edema
c) Fever
d) Hyperventilation
Answer & Explanation: The correct answer is (b) Edema. In edema, tissue fluid accumulation increases external tissue pressure on capillaries, raising the critical closing pressure and potentially impairing microcirculatory blood flow, which can cause ischemia and hinder nutrient and gas exchange.
Critical closing pressure is most relevant in which organ system?
a) Nervous system
b) Musculoskeletal system
c) Cardiovascular system
d) Respiratory system
Answer & Explanation: The correct answer is (c) Cardiovascular system. It is crucial in microcirculation, influencing capillary perfusion and tissue oxygenation. Understanding it helps manage conditions such as shock, edema, and vascular compression disorders effectively by recognizing the minimal pressures needed for blood flow.
During hypotension, critical closing pressure causes:
a) Increased tissue perfusion
b) Capillary collapse
c) Arterial dilation
d) Venous pooling
Answer & Explanation: The correct answer is (b) Capillary collapse. In hypotension, arterial pressure may fall below the critical closing pressure, leading to capillary closure and impaired tissue perfusion. This contributes to organ hypoxia and dysfunction in severe hypotensive states, like shock or massive hemorrhage.
Which condition lowers critical closing pressure?
a) Inflammation
b) Vasodilation
c) Vasoconstriction
d) Edema
Answer & Explanation: The correct answer is (b) Vasodilation. Vasodilation reduces vascular resistance and lowers critical closing pressure, facilitating capillary blood flow. It occurs during thermoregulation or inflammation, helping enhance tissue perfusion and oxygen delivery in response to metabolic demands or injury.
Measurement of critical closing pressure helps assess:
a) Cardiac output
b) Peripheral vascular resistance
c) Microcirculatory function
d) Pulmonary pressure
Answer & Explanation: The correct answer is (c) Microcirculatory function. Measuring critical closing pressure provides insights into capillary perfusion status and microvascular health, essential in understanding pathophysiological conditions like shock and guiding therapeutic interventions aimed at improving tissue oxygenation and preventing organ failure.
```
Topic: Cardiovascular System
Subtopic: Cardiac Function
Cardiac Reserve: Difference between resting and maximal cardiac output.
Cardiac Output: Volume of blood the heart pumps per minute.
Healthy Adult: An individual without cardiovascular disease, aged 18-65 years.
Percentage Increase: Increase in cardiac output from rest to maximal exertion.
Lead Question - 2013
Healthy adult cardiac reserve is? (September 2008)
a) 50 - 100 %
b) 100 - 200 %
c) 200 - 250 %
d) 300 - 400 %
Answer & Explanation: The correct answer is (c) 200 - 250 %. Cardiac reserve refers to the capacity of the heart to increase its output above resting level during increased activity or stress. In a healthy adult, this reserve typically ranges from 200% to 250%, reflecting strong cardiovascular performance and good physical fitness.
Guessed Questions
Cardiac Reserve: Difference between resting and maximal cardiac output.
Cardiac Output: Volume of blood the heart pumps per minute.
Healthy Adult: An individual without cardiovascular disease, aged 18-65 years.
Percentage Increase: Increase in cardiac output from rest to maximal exertion.
Which factor increases cardiac reserve?
a) Sedentary lifestyle
b) Physical training
c) Heart failure
d) Anemia
Answer & Explanation: The correct answer is (b) Physical training. Regular aerobic exercise enhances cardiac reserve by improving heart muscle function and vascular compliance, leading to a greater ability to increase cardiac output during physical exertion. In contrast, conditions like heart failure decrease cardiac reserve.
Normal resting cardiac output in adults is approximately:
a) 2 L/min
b) 5 L/min
c) 7 L/min
d) 10 L/min
Answer & Explanation: The correct answer is (b) 5 L/min. In a healthy adult, the average resting cardiac output is around 5 liters per minute, determined by the product of stroke volume and heart rate. This value is a standard reference in cardiovascular physiology.
Maximal cardiac output in healthy adults is approximately:
a) 7 - 10 L/min
b) 15 - 20 L/min
c) 25 - 30 L/min
d) 35 - 40 L/min
Answer & Explanation: The correct answer is (b) 15 - 20 L/min. During intense physical activity, a healthy adult's cardiac output can increase up to 15-20 L/min, reflecting a 3-4 fold increase over the resting level, which demonstrates the functional cardiac reserve capacity.
Which condition decreases cardiac reserve?
a) Hypertension
b) Heart failure
c) Physical fitness
d) Normal aging
Answer & Explanation: The correct answer is (b) Heart failure. Heart failure impairs the heart's ability to increase output during stress or exercise, leading to a reduced cardiac reserve. In contrast, physical fitness improves it, while normal aging has a mild effect.
Cardiac reserve is important for:
a) Maintaining constant blood pressure
b) Responding to exercise demands
c) Controlling heart rate
d) Regulating respiratory rate
Answer & Explanation: The correct answer is (b) Responding to exercise demands. Cardiac reserve enables the heart to significantly increase output during exercise or stress, providing more oxygen and nutrients to tissues. It is crucial for athletic performance and coping with increased physical demands.
Which hormone enhances cardiac output during stress?
a) Insulin
b) Cortisol
c) Adrenaline
d) Aldosterone
Answer & Explanation: The correct answer is (c) Adrenaline. Adrenaline (epinephrine) increases heart rate and contractility, thereby enhancing cardiac output and improving cardiac reserve during stress or exercise. This adaptive mechanism helps maintain blood pressure and tissue perfusion under demanding conditions.
Decreased cardiac reserve may present clinically as:
a) Hypertension
b) Hypotension
c) Exercise intolerance
d) Tachypnea
Answer & Explanation: The correct answer is (c) Exercise intolerance. Patients with reduced cardiac reserve experience difficulty during exertion due to inability to sufficiently increase cardiac output. This can lead to symptoms like fatigue, dyspnea, and reduced physical capacity, commonly observed in heart failure.
Which parameter is NOT part of cardiac output calculation?
a) Stroke volume
b) Heart rate
c) Blood pressure
d) All are part
Answer & Explanation: The correct answer is (c) Blood pressure. Cardiac output is calculated as stroke volume multiplied by heart rate (CO = SV × HR). Blood pressure is influenced by cardiac output and vascular resistance but is not directly used in this calculation.
Typical stroke volume in a healthy adult at rest is approximately:
a) 20 ml
b) 70 ml
c) 150 ml
d) 200 ml
Answer & Explanation: The correct answer is (b) 70 ml. Stroke volume is the amount of blood ejected by the left ventricle per beat. In a healthy adult at rest, it averages about 70 ml, and this contributes to the typical cardiac output of 5 L/min.
Topic: Cardiovascular System
Subtopic: Hemodynamics
Pressure: Force exerted per unit area.
SI Unit: International System unit, standard for measurement.
Pascal (Pa): SI unit of pressure, equal to one Newton per square meter.
mmHg: Millimeters of mercury, traditional unit of blood pressure.
Lead Question - 2013
SI unit of pressure is? (September 2008)
a) mmHg
b) cmHg
c) Pascal
d) Torr
Answer & Explanation: The correct answer is (c) Pascal. The SI unit of pressure is Pascal (Pa), defined as one Newton per square meter (N/m²). It standardizes measurement globally, while mmHg and Torr are traditional units commonly used in medicine for measuring blood pressure, but not SI units.
Guessed Questions
Pressure: Force exerted per unit area.
SI Unit: International System unit, standard for measurement.
Pascal (Pa): SI unit of pressure, equal to one Newton per square meter.
mmHg: Millimeters of mercury, traditional unit of blood pressure.
Which instrument is used to measure blood pressure?
a) Thermometer
b) Sphygmomanometer
c) Barometer
d) Stethoscope
Answer & Explanation: The correct answer is (b) Sphygmomanometer. Blood pressure is measured using a sphygmomanometer, which includes an inflatable cuff, a pressure gauge, and a stethoscope. It measures systolic and diastolic pressures, providing critical diagnostic information regarding cardiovascular health in clinical settings.
1 Pascal is equal to:
a) 10 N/m²
b) 1 N/m²
c) 100 N/m²
d) 0.1 N/m²
Answer & Explanation: The correct answer is (b) 1 N/m². One Pascal (Pa) is the pressure exerted by a force of one Newton applied over an area of one square meter. This SI unit is universally accepted and crucial in scientific and engineering calculations involving pressure measurement.
Which unit is commonly used in clinical practice for BP?
a) Pascal
b) mmHg
c) Atm
d) N/m²
Answer & Explanation: The correct answer is (b) mmHg. Millimeters of mercury (mmHg) are traditionally used in clinical practice to express blood pressure readings due to historical use of mercury sphygmomanometers. Despite SI units existing, mmHg remains prevalent in medical settings for practical reasons.
Atmospheric pressure is approximately:
a) 101325 Pa
b) 760 mmHg
c) 1 atm
d) All of the above
Answer & Explanation: The correct answer is (d) All of the above. Standard atmospheric pressure equals 101325 Pa, 760 mmHg, or 1 atmosphere (atm). These units are used in various contexts, but Pascal is the SI unit. Understanding these equivalences is essential in physiology and clinical measurement.
Pressure measurement in respiratory physiology is expressed in:
a) cm H2O
b) mmHg
c) Pascal
d) Both a and b
Answer & Explanation: The correct answer is (d) Both a and b. In respiratory physiology, pressures are often expressed in cm H2O for airway pressures and mmHg in blood gas measurements. Though Pascal is the SI unit, these traditional units remain widely used for clinical relevance.
Which law relates pressure and volume of gases?
a) Boyle's Law
b) Charles's Law
c) Dalton's Law
d) Henry's Law
Answer & Explanation: The correct answer is (a) Boyle's Law. Boyle's Law states that at constant temperature, the pressure and volume of a gas are inversely proportional (P ∝ 1/V). This principle is fundamental in understanding respiratory mechanics and ventilator management in clinical practice.
Unit of pressure equivalent to 1 atm is:
a) 101325 Pa
b) 760 mmHg
c) 1.013 bar
d) All of the above
Answer & Explanation: The correct answer is (d) All of the above. Standard atmospheric pressure equals 101325 Pascal, 760 mmHg, and approximately 1.013 bar. These equivalents are important for conversions and understanding pressures in different contexts, particularly in physiological and clinical measurements.
Normal arterial blood pressure is typically around:
a) 120/80 mmHg
b) 100/70 mmHg
c) 140/90 mmHg
d) 90/60 mmHg
Answer & Explanation: The correct answer is (a) 120/80 mmHg. Normal arterial blood pressure in a healthy adult is approximately 120 mmHg systolic and 80 mmHg diastolic. These values serve as reference standards in medical practice to evaluate cardiovascular health and guide treatment decisions.
Topic: Cardiovascular System
Subtopic: Baroreceptor Reflex
Arterial Baroreceptors: Sensors in carotid sinus and aortic arch monitoring blood pressure changes.
Cardiac Systole: Phase when ventricles contract, ejecting blood into arteries.
Cardiac Diastole: Phase when heart muscles relax, chambers fill with blood.
Baroreceptor Reflex: Negative feedback mechanism to maintain stable blood pressure.
Lead Question - 2013
In healthy person, arterial baroreceptor activity is seen at what stage of cardiac systole? (September 2008)
a) Systole
b) Diastole
c) Both
d) None
Answer & Explanation: The correct answer is (a) Systole. Arterial baroreceptors are most active during the systolic phase of the cardiac cycle because arterial pressure peaks when ventricles contract and eject blood. These receptors sense increased stretch and send signals to the brain to modulate heart rate and vascular resistance, maintaining stable blood pressure.
Guessed Questions
Arterial Baroreceptors: Sensors in carotid sinus and aortic arch monitoring blood pressure changes.
Cardiac Systole: Phase when ventricles contract, ejecting blood into arteries.
Cardiac Diastole: Phase when heart muscles relax, chambers fill with blood.
Baroreceptor Reflex: Negative feedback mechanism to maintain stable blood pressure.
Baroreceptor reflex regulates blood pressure by:
a) Increasing heart rate
b) Decreasing heart rate
c) Increasing peripheral resistance
d) Both b and c
Answer & Explanation: The correct answer is (d) Both b and c. The baroreceptor reflex adjusts blood pressure by decreasing heart rate and increasing peripheral resistance when blood pressure rises. Conversely, if blood pressure drops, it increases heart rate and reduces resistance. This rapid feedback mechanism helps maintain homeostasis.
Carotid sinus baroreceptors primarily monitor pressure in:
a) Aortic arch
b) Left ventricle
c) Carotid artery
d) Pulmonary artery
Answer & Explanation: The correct answer is (c) Carotid artery. The carotid sinus baroreceptors are located at the bifurcation of the common carotid artery and primarily monitor systemic arterial pressure. They provide crucial input for the baroreceptor reflex, influencing heart rate and vascular tone to stabilize blood pressure.
Which cranial nerve carries signals from carotid baroreceptors?
a) Vagus nerve (CN X)
b) Glossopharyngeal nerve (CN IX)
c) Hypoglossal nerve (CN XII)
d) Trigeminal nerve (CN V)
Answer & Explanation: The correct answer is (b) Glossopharyngeal nerve (CN IX). The glossopharyngeal nerve transmits sensory signals from carotid baroreceptors to the brainstem. These signals are essential for autonomic regulation of cardiovascular function, helping to maintain stable blood pressure through reflex adjustments of heart rate and vessel tone.
What is the primary function of baroreceptor reflex?
a) Regulate blood glucose
b) Maintain blood pressure stability
c) Control body temperature
d) Modulate respiratory rate
Answer & Explanation: The correct answer is (b) Maintain blood pressure stability. The baroreceptor reflex provides rapid adjustments to maintain arterial blood pressure within normal limits. It responds to changes in arterial wall stretch, triggering autonomic responses to stabilize heart rate and peripheral resistance, essential for tissue perfusion and preventing hypotension or hypertension.
Which effect occurs during decreased baroreceptor firing?
a) Decreased heart rate
b) Increased vasodilation
c) Increased heart rate and vasoconstriction
d) No change in cardiovascular function
Answer & Explanation: The correct answer is (c) Increased heart rate and vasoconstriction. A drop in arterial pressure reduces baroreceptor firing, triggering sympathetic activation. This leads to increased heart rate (positive chronotropy) and vasoconstriction, thereby elevating blood pressure to restore homeostasis and ensure adequate organ perfusion.
Baroreceptors respond primarily to changes in:
a) Blood volume
b) Arterial wall stretch
c) Blood oxygen levels
d) Blood pH levels
Answer & Explanation: The correct answer is (b) Arterial wall stretch. Baroreceptors are stretch-sensitive mechanoreceptors located in the carotid sinus and aortic arch. They detect changes in arterial wall stretch due to variations in blood pressure and send afferent signals to regulate heart rate and vascular tone appropriately.
Long-term regulation of blood pressure involves:
a) Baroreceptor reflex
b) Chemoreceptor reflex
c) Renin-Angiotensin-Aldosterone System (RAAS)
d) Bainbridge reflex
Answer & Explanation: The correct answer is (c) Renin-Angiotensin-Aldosterone System (RAAS). RAAS plays a crucial role in long-term blood pressure regulation by modulating blood volume and systemic vascular resistance. It responds to decreased perfusion and sodium levels, leading to aldosterone secretion, sodium retention, and vasoconstriction to stabilize pressure.
Which area of brain integrates baroreceptor signals?
a) Hypothalamus
b) Medulla oblongata
c) Cerebellum
d) Midbrain
Answer & Explanation: The correct answer is (b) Medulla oblongata. The medulla oblongata houses the cardiovascular control centers, which integrate baroreceptor signals and adjust autonomic outflow. This regulation is vital for maintaining cardiovascular stability, adjusting heart rate and vessel tone in response to blood pressure changes.
Topic: Electrocardiography
Subtopic: Cardiac Conduction System
P Wave: Represents atrial depolarization in the ECG trace.
Atrial Depolarization: Electrical activation causing atrial contraction, pushing blood into ventricles.
Atrial Repolarization: Electrical recovery phase of the atria, masked by QRS complex.
Ventricular Depolarization: Electrical activation of ventricles, generating QRS complex.
Lead Question - 2013
P wave is due to: (September 2008)
a) Atrial depolarization
b) Atrial repolarization
c) Ventricular depolarization
d) Ventricular repolarization
Answer & Explanation: The correct answer is (a) Atrial depolarization. The P wave in an ECG represents the electrical activity of the atria as they depolarize and contract, pushing blood into the ventricles. This is the initial step in the cardiac cycle, essential for proper heart function and efficient circulation of blood.
Guessed Questions
P Wave: Represents atrial depolarization in the ECG trace.
Atrial Depolarization: Electrical activation causing atrial contraction, pushing blood into ventricles.
Atrial Repolarization: Electrical recovery phase of the atria, masked by QRS complex.
Ventricular Depolarization: Electrical activation of ventricles, generating QRS complex.
Which part of ECG represents ventricular repolarization?
a) P wave
b) QRS complex
c) T wave
d) PR interval
Answer & Explanation: The correct answer is (c) T wave. The T wave in an ECG represents ventricular repolarization, the process by which the ventricles recover electrically after contraction. It is critical for preparing the heart muscle for the next cycle and maintaining rhythm stability, essential in cardiac assessment.
Which structure is the primary pacemaker of the heart?
a) AV node
b) SA node
c) Bundle of His
d) Purkinje fibers
Answer & Explanation: The correct answer is (b) SA node. The sinoatrial (SA) node is the primary pacemaker of the heart, located in the right atrium. It initiates electrical impulses that spread through atria, causing contraction. Its automaticity ensures rhythmic heartbeats, essential for effective blood circulation throughout the body.
Which electrolyte abnormality causes peaked T waves?
a) Hypokalemia
b) Hyperkalemia
c) Hypernatremia
d) Hypocalcemia
Answer & Explanation: The correct answer is (b) Hyperkalemia. Elevated potassium levels in the blood (hyperkalemia) can cause characteristic peaked T waves on ECG. This abnormality reflects accelerated ventricular repolarization, increasing the risk of dangerous arrhythmias. Immediate correction of potassium levels is vital to prevent cardiac arrest.
First-degree heart block is characterized by:
a) Prolonged PR interval
b) Absent P wave
c) Wide QRS complex
d) ST segment elevation
Answer & Explanation: The correct answer is (a) Prolonged PR interval. First-degree heart block presents as a PR interval longer than 200 milliseconds. It indicates a delay in electrical conduction from atria to ventricles, typically asymptomatic but sometimes a sign of underlying cardiac pathology requiring monitoring.
Which condition shortens the QT interval?
a) Hypocalcemia
b) Hypercalcemia
c) Hypokalemia
d) Hyperkalemia
Answer & Explanation: The correct answer is (b) Hypercalcemia. Hypercalcemia, or elevated calcium levels, leads to a shortened QT interval on ECG due to accelerated ventricular repolarization. Clinically significant as it can predispose to arrhythmias, its identification guides appropriate management of calcium levels.
What does PR interval represent in ECG?
a) Atrial depolarization
b) Time from atrial depolarization to ventricular depolarization
c) Ventricular repolarization
d) Ventricular contraction
Answer & Explanation: The correct answer is (b) Time from atrial depolarization to ventricular depolarization. The PR interval reflects the delay caused by the AV node allowing the atria to contract fully before ventricular activation. Its duration provides important clinical information about conduction system health.
Which lead shows electrical activity from the lateral wall of left ventricle?
a) Lead II
b) Lead I
c) V1
d) V6
Answer & Explanation: The correct answer is (d) V6. Lead V6 in ECG monitoring reflects the electrical activity of the lateral wall of the left ventricle. It's particularly useful in diagnosing lateral wall ischemia or infarction, which affects the heart’s pumping efficacy and requires timely intervention.
Which is characteristic of atrial fibrillation?
a) Regular rhythm
b) Absent P waves
c) Prolonged PR interval
d) Peaked T waves
Answer & Explanation: The correct answer is (b) Absent P waves. Atrial fibrillation results in chaotic electrical activity in the atria, leading to absent P waves on ECG. Instead, there are irregular fibrillatory waves. It results in an irregularly irregular ventricular response and increases the risk of thromboembolism.
Topic: Electrocardiography
Subtopic: Cardiac Conduction System
QRS Complex: Represents ventricular depolarization, recorded in ECG.
Ventricular Depolarization: Electrical activation of ventricles causing contraction.
Atrial Depolarization: Electrical activation of atria causing contraction.
AV Node: Conducts impulses from atria to ventricles, with delay.
Lead Question - 2013
QRS complex is due to: (September 2008)
a) Ventricular repolarization
b) Atrial depolarization
c) Conduction through AV node
d) Ventricular depolarization
Answer & Explanation: The correct answer is (d) Ventricular depolarization. The QRS complex in an electrocardiogram represents the rapid depolarization of the right and left ventricles. This electrical activity leads to ventricular contraction and is a crucial indicator of heart health in clinical practice, reflecting the synchronous function of the ventricular myocardium.
Guessed Questions
QRS Complex: Represents ventricular depolarization, recorded in ECG.
Ventricular Depolarization: Electrical activation of ventricles causing contraction.
Atrial Depolarization: Electrical activation of atria causing contraction.
AV Node: Conducts impulses from atria to ventricles, with delay.
Which part of ECG represents atrial depolarization?
a) P wave
b) QRS complex
c) T wave
d) U wave
Answer & Explanation: The correct answer is (a) P wave. The P wave on an electrocardiogram (ECG) represents the electrical activity associated with atrial depolarization. It precedes the QRS complex and indicates the initiation of the heartbeat, essential for effective atrial contraction and efficient blood flow into the ventricles.
Which ion primarily responsible for phase 0 of cardiac action potential?
a) Na+
b) K+
c) Ca2+
d) Cl-
Answer & Explanation: The correct answer is (a) Na+. During phase 0 of the cardiac action potential, there is a rapid influx of sodium ions (Na+) through voltage-gated sodium channels. This influx causes a swift depolarization of the cardiac muscle cell membrane, triggering the contraction of the heart muscle necessary for pumping blood.
In ECG, which interval represents AV nodal conduction time?
a) PR interval
b) QT interval
c) ST segment
d) RR interval
Answer & Explanation: The correct answer is (a) PR interval. The PR interval on an electrocardiogram measures the time from the onset of atrial depolarization to the onset of ventricular depolarization, reflecting the conduction time through the AV node and His-Purkinje system. It is critical for assessing conduction abnormalities.
Bradycardia is defined as heart rate less than:
a) 60 bpm
b) 70 bpm
c) 80 bpm
d) 90 bpm
Answer & Explanation: The correct answer is (a) 60 bpm. Bradycardia refers to a slower than normal heart rate, typically below 60 beats per minute in adults. It can be normal in athletes but may indicate pathological conditions if accompanied by symptoms such as dizziness, fatigue, or syncope, requiring further evaluation.
Which electrolyte imbalance prolongs QT interval in ECG?
a) Hyperkalemia
b) Hypokalemia
c) Hypercalcemia
d) Hypernatremia
Answer & Explanation: The correct answer is (b) Hypokalemia. Hypokalemia, a deficiency of potassium ions in the blood, prolongs the QT interval on an ECG by delaying ventricular repolarization. This prolongation can predispose patients to life-threatening arrhythmias like Torsades de Pointes, requiring careful monitoring and potassium correction.
ST segment elevation is indicative of:
a) Myocardial infarction
b) Atrial fibrillation
c) Heart block
d) Pericarditis
Answer & Explanation: The correct answer is (a) Myocardial infarction. ST segment elevation on an ECG is a hallmark sign of acute myocardial infarction (heart attack). It signifies transmural ischemia, where the entire thickness of the heart muscle is affected, requiring immediate medical intervention to restore blood flow and prevent heart damage.
Which structure connects atria to ventricles?
a) AV node
b) SA node
c) Purkinje fibers
d) Bundle of His
Answer & Explanation: The correct answer is (d) Bundle of His. The Bundle of His is a specialized conduction pathway that transmits electrical impulses from the atrioventricular (AV) node to the ventricles. This ensures coordinated contraction, critical for effective cardiac function and maintenance of adequate circulatory dynamics.
Which part of the ECG represents ventricular repolarization?
a) P wave
b) QRS complex
c) T wave
d) PR interval
Answer & Explanation: The correct answer is (c) T wave. The T wave on an electrocardiogram represents the process of ventricular repolarization. It occurs after the QRS complex and is essential for restoring the resting electrical state of the ventricles, preparing them for the next cardiac cycle.
Which condition leads to a prolonged PR interval?
a) First-degree heart block
b) Myocardial infarction
c) Hyperthyroidism
d) Atrial fibrillation
Answer & Explanation: The correct answer is (a) First-degree heart block. First-degree heart block is characterized by a prolonged PR interval (>200 ms) on an ECG, indicating delayed conduction from atria to ventricles through the AV node. While often asymptomatic, it may signal underlying conduction system disease requiring observation.
Topic: Cardiovascular System
Subtopic: Cardiac Output Measurement
Keywords:
Cardiac Output: Volume of blood the heart pumps per minute, vital for tissue perfusion.
O₂ Content of Arterial Blood: Amount of oxygen carried in arterial blood, important for assessing oxygen delivery.
O₂ Consumption per Unit Time: Amount of oxygen used by tissues per minute, reflecting metabolic activity.
Arteriovenous O₂ Difference: Difference in oxygen content between arterial and venous blood, indicating tissue oxygen extraction.
Lead Question - 2013:
Direct Fick method of measuring cardiac output requires estimation of:
a) O₂ content of arterial blood
b) O₂ consumption per unit time
c) Arteriovenous O₂ difference
d) All of the above
Answer & Explanation:
Correct answer: d) All of the above.
Explanation: The Direct Fick method calculates cardiac output using the formula: CO = VO₂ / (CaO₂ – CvO₂). This requires measurement of oxygen consumption per unit time, arterial oxygen content, and arteriovenous oxygen difference. It provides an accurate assessment of cardiac performance, especially in clinical and research settings.
MCQ 1:
Which device directly measures oxygen consumption?
a) Spirometer
b) Electrocardiograph
c) Sphygmomanometer
d) Pulse oximeter
Answer & Explanation:
Correct answer: a) Spirometer.
Explanation: Spirometer measures respiratory gases, allowing direct assessment of oxygen consumption (VO₂). It helps in determining metabolic rate and pulmonary function. This method is essential in calculating cardiac output via the Fick principle, providing insights into cardiac and respiratory efficiency in health and disease.
MCQ 2 (Clinical):
Fick method is particularly useful in:
a) Measuring peripheral resistance
b) Estimating cardiac output in heart failure
c) Assessing blood pressure
d) Evaluating hemoglobin levels
Answer & Explanation:
Correct answer: b) Estimating cardiac output in heart failure.
Explanation: In heart failure, direct Fick method provides precise cardiac output measurement, aiding diagnosis and treatment planning. It is accurate even when non-invasive methods fail, such as in severe disease or unstable hemodynamics, guiding appropriate management and improving patient outcomes.
MCQ 3:
Arteriovenous O₂ difference increases with:
a) Decreased tissue demand
b) Increased oxygen extraction by tissues
c) High oxygen delivery
d) Low metabolic activity
Answer & Explanation:
Correct answer: b) Increased oxygen extraction by tissues.
Explanation: Greater metabolic demand, as seen in exercise or hypoxia, raises the arteriovenous oxygen difference because tissues extract more oxygen per unit of blood. Monitoring this parameter helps assess tissue perfusion adequacy and oxygen delivery relative to consumption in various clinical scenarios.
MCQ 4 (Clinical):
Fick principle cardiac output measurement is limited by:
a) Inaccurate VO₂ measurement
b) Arterial catheterization
c) Blood sampling errors
d) All of the above
Answer & Explanation:
Correct answer: d) All of the above.
Explanation: The Fick method requires accurate oxygen consumption and blood content measurement, demanding arterial and venous catheterization and precise lab techniques. Errors in gas analysis or sampling compromise accuracy. It remains gold standard in specialized settings despite practical limitations for routine use.
MCQ 5:
The primary advantage of Fick method is:
a) Non-invasive nature
b) High accuracy
c) Requires no special equipment
d) Simple calculation
Answer & Explanation:
Correct answer: b) High accuracy.
Explanation: Fick method is highly accurate for cardiac output estimation, especially in research and critical care. Despite invasiveness and complexity, it is the reference standard, allowing reliable measurement even under extreme conditions, such as severe heart failure or shock, where other methods may be inaccurate.
MCQ 6 (Clinical):
Which patient condition complicates Fick measurement?
a) Anemia
b) Hyperthyroidism
c) Pulmonary disease
d) All of the above
Answer & Explanation:
Correct answer: d) All of the above.
Explanation: Anemia lowers oxygen content, complicating calculations. Pulmonary disease impairs oxygen uptake, and hyperthyroidism increases metabolic demand, making VO₂ estimation variable. These factors impact accuracy, requiring careful interpretation or alternative methods in affected patients.
MCQ 7:
O₂ consumption per unit time is measured in:
a) ml/min
b) L/min
c) g/min
d) mmHg/min
Answer & Explanation:
Correct answer: a) ml/min.
Explanation: Oxygen consumption is expressed in milliliters per minute (ml/min), reflecting the volume of oxygen metabolized by the body per minute. It is essential in the Fick equation to calculate cardiac output, giving insights into metabolic and circulatory efficiency under different conditions.
MCQ 8 (Clinical):
In which situation is Fick method preferred?
a) Routine health checkup
b) Cardiac catheterization labs
c) Office visits
d) School screenings
Answer & Explanation:
Correct answer: b) Cardiac catheterization labs.
Explanation: The Fick method is preferred during invasive cardiac catheterization, where direct arterial and venous samples can be taken along with oxygen consumption measurement. It is not suited for routine or outpatient settings due to complexity but provides precise assessment during detailed cardiac evaluation.
MCQ 9:
Which is NOT needed in Fick method?
a) Hemoglobin concentration
b) Arterial and venous O₂ content
c) Oxygen consumption rate
d) Blood pressure
Answer & Explanation:
Correct answer: d) Blood pressure.
Explanation: Blood pressure is not part of the Fick calculation. Instead, oxygen consumption and arteriovenous O₂ difference are key. While BP affects overall circulation, it is not required for the specific formula CO = VO₂ / (CaO₂ – CvO₂) in cardiac output determination.
MCQ 10 (Clinical):
During exercise, arteriovenous O₂ difference:
a) Decreases
b) Remains constant
c) Increases
d) Fluctuates randomly
Answer & Explanation:
Correct answer: c) Increases.
Explanation: Exercise raises tissue oxygen demand, causing greater extraction of O₂ and an increased arteriovenous O₂ difference. This supports higher metabolic activity. Monitoring this response helps evaluate cardiovascular and respiratory function during stress testing or rehabilitation.
Topic: Circulatory System
Subtopic: Hemodynamics
Keywords:
Arteriole: Small branch of an artery leading to capillaries, regulating blood flow.
Veins: Blood vessels that carry blood toward the heart, with valves preventing backflow.
Capillaries: Smallest blood vessels where exchange of oxygen, nutrients, and waste occurs.
Venules: Small vessels that collect blood from capillaries into veins.
Lead Question - 2013:
Slowest blood flow is seen in ?
a) Arteriole
b) Veins
c) Capillaries
d) Venules
Answer & Explanation:
Correct answer: c) Capillaries.
Explanation: Capillaries exhibit the slowest blood flow due to their enormous total cross-sectional area. This allows optimal time for exchange of gases, nutrients, and metabolic waste between blood and tissues. Although individual capillaries are narrow, their collective area reduces flow velocity, crucial for effective cellular exchange processes.
MCQ 1:
Which factor determines blood flow velocity?
a) Cross-sectional area of vessels
b) Blood viscosity only
c) Blood pressure alone
d) Vessel length
Answer & Explanation:
Correct answer: a) Cross-sectional area of vessels.
Explanation: Blood flow velocity is inversely related to the total cross-sectional area. Capillaries, having the largest cumulative area, have the slowest flow velocity, allowing efficient exchange. Arteries and veins have smaller total areas and higher velocity. This principle underlies efficient tissue perfusion in the body.
MCQ 2 (Clinical):
Slow capillary flow assists in:
a) Rapid oxygen delivery
b) Efficient nutrient and waste exchange
c) Increasing blood pressure
d) Accelerating heart rate
Answer & Explanation:
Correct answer: b) Efficient nutrient and waste exchange.
Explanation: Slow capillary blood flow maximizes time for diffusion of oxygen, nutrients, and removal of CO₂ and metabolic waste. This is critical for tissue viability, especially in high-demand organs like brain and kidneys. Clinical conditions that impair capillary flow cause ischemia and organ dysfunction.
MCQ 3:
Venous blood flow is assisted by:
a) High pressure
b) Skeletal muscle pump
c) Arterial contraction
d) Active capillary contraction
Answer & Explanation:
Correct answer: b) Skeletal muscle pump.
Explanation: Venous return depends on the skeletal muscle pump, especially in extremities, and venous valves that prevent backflow. Low venous pressure means passive return is inadequate, and muscle contractions compress veins, pushing blood toward the heart, critical during physical activity or immobility prevention.
MCQ 4 (Clinical):
Which condition increases capillary hydrostatic pressure?
a) Heart failure
b) Hypotension
c) Arterial stenosis
d) Vasodilation
Answer & Explanation:
Correct answer: a) Heart failure.
Explanation: In heart failure, elevated venous pressure raises capillary hydrostatic pressure, leading to transudation of fluid into interstitial spaces and causing edema. Management includes diuretics and improving cardiac output to reduce capillary pressure and prevent organ dysfunction and discomfort.
MCQ 5:
Capillary permeability increases with:
a) Inflammation
b) Low BP
c) Low heart rate
d) High oxygen levels
Answer & Explanation:
Correct answer: a) Inflammation.
Explanation: Inflammatory mediators (e.g., histamine) increase capillary permeability, allowing proteins and leukocytes to migrate into tissues for immune response. While useful during infection, excessive permeability causes edema and tissue damage, relevant in sepsis or allergic reactions.
MCQ 6 (Clinical):
Venous pooling in lower limbs may cause:
a) Hypertension
b) Orthostatic hypotension
c) Tachycardia
d) Bradycardia
Answer & Explanation:
Correct answer: b) Orthostatic hypotension.
Explanation: Venous pooling reduces venous return when standing, decreasing cardiac output and BP, leading to orthostatic hypotension with symptoms like dizziness. Clinical interventions include gradual posture change, compression stockings, and addressing autonomic dysfunction causes.
MCQ 7:
The primary driving force for capillary exchange is:
a) Osmotic pressure
b) Hydrostatic pressure
c) Cardiac output
d) Blood viscosity
Answer & Explanation:
Correct answer: b) Hydrostatic pressure.
Explanation: Hydrostatic pressure in capillaries pushes plasma through endothelial gaps into interstitial space. Osmotic pressure, mainly due to plasma proteins, opposes this. The balance (Starling forces) regulates net filtration or reabsorption, key to maintaining fluid homeostasis and preventing edema.
MCQ 8 (Clinical):
Which pathological condition reduces capillary exchange?
a) Atherosclerosis
b) Capillary basement membrane thickening
c) Hypertension
d) Low cardiac output
Answer & Explanation:
Correct answer: b) Capillary basement membrane thickening.
Explanation: Diseases like diabetes cause thickened basement membranes, impairing diffusion of oxygen and nutrients. This leads to tissue hypoxia and organ dysfunction, common in diabetic nephropathy and retinopathy, emphasizing strict glycemic control to prevent complications.
MCQ 9:
Which vessel type has valves?
a) Arteries
b) Arterioles
c) Veins
d) Capillaries
Answer & Explanation:
Correct answer: c) Veins.
Explanation: Veins contain valves to prevent backflow, ensuring unidirectional flow toward the heart, especially in lower limbs against gravity. Valve dysfunction causes varicose veins and venous insufficiency, common in elderly and prolonged immobility, with management strategies including compression therapy.
MCQ 10 (Clinical):
Slow capillary flow is particularly important for:
a) Rapid cardiac contraction
b) Effective gas exchange
c) Blood clotting
d) Blood pressure maintenance
Answer & Explanation:
Correct answer: b) Effective gas exchange.
Explanation: Slow capillary flow prolongs contact time between blood and tissues, enhancing oxygen and CO₂ diffusion across capillary walls. Critical for organ function, particularly in lungs and muscles. Disruption leads to hypoxia and metabolic disturbances, stressing the importance in clinical monitoring of perfusion status.
Topic: Cardiovascular System
Subtopic: Coronary Circulation
Keywords:
Coronary Blood Flow: The circulation of blood in the blood vessels of the heart muscle (myocardium).
Isovolumic Relaxation Phase: Period when ventricles relax with all valves closed before ventricular filling.
Ejection Phase: Ventricular contraction phase pushing blood into arteries.
Isovolumic Contraction Phase: Period when ventricles contract with no volume change as valves remain closed.
Lead Question - 2013:
Coronary blood flow is maximum during which phase of cardiac cycle?
a) Isovolumic relaxation phase
b) Isovolumic contraction phase
c) Ejection phase
d) Isovolumic contraction phase
Answer & Explanation:
Correct answer: a) Isovolumic relaxation phase.
Explanation: Coronary blood flow is highest during isovolumic relaxation because intramyocardial pressure falls, relieving vascular compression, allowing maximum perfusion. During contraction, vessels are compressed, reducing flow. This phase is critical for myocardial oxygen delivery, especially in pathological conditions like coronary artery disease where perfusion is compromised.
MCQ 1:
Coronary blood flow is lowest during:
a) Diastole
b) Isovolumic relaxation
c) Systole
d) Isovolumic relaxation
Answer & Explanation:
Correct answer: c) Systole.
Explanation: During systole, high intramyocardial pressure compresses coronary vessels, significantly reducing blood flow. The left coronary artery experiences greater reduction due to thicker myocardium. Coronary perfusion predominantly occurs during diastole, essential for myocardial oxygen supply, particularly under increased workload.
MCQ 2 (Clinical):
Coronary perfusion pressure is mainly determined by:
a) Aortic diastolic pressure minus LVEDP
b) Systolic pressure only
c) Right atrial pressure
d) Pulmonary artery pressure
Answer & Explanation:
Correct answer: a) Aortic diastolic pressure minus LVEDP.
Explanation: Coronary perfusion pressure (CPP) depends on aortic diastolic pressure minus left ventricular end-diastolic pressure (LVEDP). Low aortic pressure or elevated LVEDP (as in heart failure) reduces CPP, impairing myocardial perfusion. Maintaining adequate CPP is crucial during critical care to prevent ischemia.
MCQ 3:
Which coronary artery supplies the interventricular septum?
a) Left anterior descending artery
b) Right coronary artery
c) Circumflex artery
d) Posterior descending artery
Answer & Explanation:
Correct answer: a) Left anterior descending artery.
Explanation: The left anterior descending (LAD) artery supplies the anterior two-thirds of the interventricular septum, essential for electrical conduction and myocardial function. LAD occlusion causes significant infarction with potential arrhythmias. This knowledge is critical for diagnosing and managing coronary artery disease and infarction patterns.
MCQ 4 (Clinical):
During tachycardia, coronary perfusion is impaired because:
a) Diastolic time shortens
b) Systolic time increases
c) Contractility decreases
d) Afterload reduces
Answer & Explanation:
Correct answer: a) Diastolic time shortens.
Explanation: Tachycardia reduces diastolic time, the phase when most coronary perfusion occurs. Shortened diastole leads to insufficient myocardial oxygen delivery, especially in compromised coronary circulation. This explains angina during exertion and emphasizes the importance of heart rate control in ischemic heart disease management.
MCQ 5:
Which factor does NOT influence coronary blood flow?
a) Coronary perfusion pressure
b) Myocardial oxygen demand
c) Intramyocardial pressure
d) Body temperature
Answer & Explanation:
Correct answer: d) Body temperature.
Explanation: Coronary blood flow is primarily regulated by perfusion pressure, myocardial oxygen demand, and intramyocardial pressure. Body temperature changes do not directly alter coronary flow. Autoregulatory mechanisms adjust vascular tone to match oxygen supply to demand, ensuring myocardial function even during temperature variations.
MCQ 6 (Clinical):
Coronary artery disease is primarily due to:
a) Vasospasm
b) Atherosclerosis
c) Embolism
d) Thrombophlebitis
Answer & Explanation:
Correct answer: b) Atherosclerosis.
Explanation: Coronary artery disease (CAD) results mainly from atherosclerotic plaque formation within coronary arteries, narrowing lumen and reducing blood flow. Plaque rupture can cause thrombosis and infarction. Clinical management includes risk factor modification and revascularization to restore myocardial perfusion and prevent complications.
MCQ 7:
Which coronary vessel predominantly supplies the left ventricle?
a) Right coronary artery
b) Left circumflex artery
c) Left anterior descending artery
d) Posterior descending artery
Answer & Explanation:
Correct answer: c) Left anterior descending artery.
Explanation: The left anterior descending artery supplies the anterior wall and most of the left ventricle, a crucial area for cardiac output. LAD occlusion can result in large anterior wall myocardial infarction, necessitating prompt diagnosis and intervention to prevent heart failure.
MCQ 8 (Clinical):
Which clinical sign indicates reduced coronary perfusion?
a) Bradycardia
b) Chest pain (angina)
c) Peripheral edema
d) Tachypnea
Answer & Explanation:
Correct answer: b) Chest pain (angina).
Explanation: Angina pectoris is a hallmark of reduced coronary perfusion due to atherosclerosis or spasm. Pain results from transient myocardial ischemia during increased demand or decreased supply. Clinical evaluation includes ECG, biomarkers, and stress testing to confirm diagnosis and guide treatment.
MCQ 9:
Coronary blood flow autoregulation maintains perfusion by:
a) Adjusting heart rate
b) Modulating coronary vessel diameter
c) Changing blood viscosity
d) Altering blood oxygen content
Answer & Explanation:
Correct answer: b) Modulating coronary vessel diameter.
Explanation: Coronary autoregulation maintains constant blood flow despite perfusion pressure changes by adjusting vessel diameter. Vasodilation increases flow during increased demand (exercise), while vasoconstriction prevents overperfusion at rest. Impaired autoregulation is seen in atherosclerosis, leading to ischemia.
MCQ 10 (Clinical):
Coronary steal phenomenon occurs during:
a) Rest
b) Use of vasodilators
c) Increased cardiac output
d) Decreased myocardial demand
Answer & Explanation:
Correct answer: b) Use of vasodilators.
Explanation: Coronary steal occurs when vasodilators cause healthy coronary vessels to dilate, diverting blood away from stenotic regions, worsening ischemia. This explains why certain vasodilators must be used cautiously in patients with severe coronary artery disease. Recognition prevents adverse outcomes during diagnostics or therapy.
Topic: Cardiovascular System
Subtopic: Cardiac Output Regulation
Keywords:
Cardiac Output: Volume of blood the heart pumps per minute (Heart Rate × Stroke Volume).
Parasympathetic Stimulation: Vagal activation reducing heart rate and cardiac output.
Cardiac Contractility: Strength of heart muscle contraction, influencing stroke volume and cardiac output.
Expiration: Process of exhaling air, minimally affecting cardiac output.
Lead Question - 2013:
Cardiac output increases by?
a) Standing from lying down position
b) Expiration
c) Increased cardiac contractility
d) Parasympathetic stimulation
Answer & Explanation:
Correct answer: c) Increased cardiac contractility.
Explanation: Cardiac output increases significantly when cardiac contractility rises, enhancing stroke volume. Parasympathetic stimulation decreases cardiac output, while standing lowers preload transiently. Expiration has minimal effect. Contractility enhancement via sympathetic stimulation boosts cardiac performance, crucial in stress or exercise situations to meet metabolic demands.
MCQ 1:
Which factor primarily increases cardiac output?
a) Decreased heart rate
b) Increased contractility
c) Parasympathetic stimulation
d) Hypovolemia
Answer & Explanation:
Correct answer: b) Increased contractility.
Explanation: Increased cardiac contractility enhances stroke volume, thereby increasing cardiac output. This is mediated by sympathetic nervous system stimulation and circulating catecholamines, important during exercise or stress to maintain perfusion. Parasympathetic activation lowers heart rate and contractility, reducing output.
MCQ 2 (Clinical):
In heart failure, cardiac output is low due to?
a) Excessive contractility
b) Impaired contractility
c) Increased preload
d) High afterload
Answer & Explanation:
Correct answer: b) Impaired contractility.
Explanation: Heart failure commonly results from reduced myocardial contractility, decreasing stroke volume and cardiac output despite compensatory mechanisms. This leads to insufficient perfusion and symptoms like fatigue and edema. Therapies focus on improving contractility and reducing afterload to support cardiac output.
MCQ 3:
Standing up affects cardiac output by:
a) Increasing preload
b) Decreasing preload
c) Increasing heart rate only
d) No effect
Answer & Explanation:
Correct answer: b) Decreasing preload.
Explanation: When standing from lying position, venous return transiently decreases due to blood pooling in lower extremities, reducing preload and stroke volume. The body compensates by increasing heart rate and vasoconstriction to maintain cardiac output and blood pressure, especially important in preventing orthostatic hypotension.
MCQ 4 (Clinical):
In a patient with parasympathetic overactivity, cardiac output is:
a) Increased
b) Decreased
c) Unchanged
d) Initially increased then decreased
Answer & Explanation:
Correct answer: b) Decreased.
Explanation: Parasympathetic stimulation reduces heart rate and contractility, decreasing cardiac output. This occurs via acetylcholine acting on M2 receptors in the heart. Excess parasympathetic activity may cause bradycardia, hypotension, and syncope, and is managed by atropine in acute clinical settings.
MCQ 5:
Expiration affects cardiac output by:
a) Increasing it significantly
b) Decreasing it
c) No significant change
d) Reversing flow
Answer & Explanation:
Correct answer: c) No significant change.
Explanation: Expiration causes slight increase in intrathoracic pressure, transiently reducing venous return but has negligible effect on cardiac output in healthy individuals. Pathological states may exaggerate this, but in normal physiology, the impact is minimal during quiet breathing.
MCQ 6 (Clinical):
During exercise, cardiac output increases due to:
a) Decreased venous return
b) Increased parasympathetic tone
c) Increased contractility and heart rate
d) Vasoconstriction of skeletal muscles
Answer & Explanation:
Correct answer: c) Increased contractility and heart rate.
Explanation: Exercise triggers sympathetic activation, increasing heart rate and myocardial contractility, boosting cardiac output. Enhanced venous return via muscle pump supports stroke volume. Vasodilation in active muscles ensures adequate perfusion. This physiological adaptation meets elevated metabolic demands during physical activity.
MCQ 7:
Parasympathetic stimulation primarily affects cardiac output by:
a) Increasing contractility
b) Increasing heart rate
c) Decreasing heart rate
d) No effect
Answer & Explanation:
Correct answer: c) Decreasing heart rate.
Explanation: Parasympathetic (vagal) stimulation acts via acetylcholine on M2 receptors, primarily slowing heart rate and slightly reducing contractility, lowering cardiac output. This regulatory mechanism is vital at rest, ensuring energy conservation and preventing excessive workload on the heart.
MCQ 8 (Clinical):
Sympathetic stimulation increases cardiac output by:
a) Decreasing preload
b) Increasing heart rate and contractility
c) Reducing venous return
d) Causing vasodilation in arterioles
Answer & Explanation:
Correct answer: b) Increasing heart rate and contractility.
Explanation: Sympathetic activation releases norepinephrine, enhancing sinoatrial node firing (heart rate) and myocardial contractility, boosting stroke volume and cardiac output. This compensates during stress or exercise, ensuring sufficient oxygen delivery. Sympathetic vasoconstriction redirects blood flow to vital organs.
MCQ 9:
Which is NOT a factor increasing cardiac output?
a) Increased preload
b) Increased afterload
c) Increased contractility
d) Increased heart rate
Answer & Explanation:
Correct answer: b) Increased afterload.
Explanation: Increased afterload impedes ejection of blood from the heart, reducing stroke volume and cardiac output. In contrast, increased preload, contractility, and heart rate enhance cardiac output, maintaining adequate tissue perfusion. Pathological afterload increase contributes to heart failure and reduced performance.
MCQ 10 (Clinical):
Cardiac output measurement helps assess:
a) Pulmonary function
b) Renal function
c) Heart performance and systemic circulation
d) Liver metabolism
Answer & Explanation:
Correct answer: c) Heart performance and systemic circulation.
Explanation: Cardiac output is a key indicator of heart performance and overall circulatory adequacy. Low output may suggest heart failure, shock, or other cardiac dysfunctions. Measurement techniques include thermodilution and Doppler methods, essential for clinical management in critically ill patients.
Topic: Circulatory System
Subtopic: Blood Circulation
Keywords:
Deoxygenated Blood: Blood that carries low oxygen content, typically from body tissues back to the heart and lungs for reoxygenation.
Pulmonary Artery: Vessel carrying deoxygenated blood from the right ventricle to the lungs for oxygenation.
Pulmonary Vein: Vessel carrying oxygenated blood from lungs to the left atrium of the heart.
Umbilical Artery: Vessel carrying deoxygenated blood from fetus to placenta.
Lead Question - 2013:
Deoxygenated blood is not seen in?
a) Pulmonary artery
b) Pulmonary vein
c) Right atrium
d) Umbilical artery
Answer & Explanation:
Correct answer: b) Pulmonary vein.
Explanation: Pulmonary veins carry oxygenated blood from the lungs to the left atrium of the heart, unlike pulmonary arteries and umbilical arteries which carry deoxygenated blood. The right atrium receives deoxygenated blood from systemic circulation. Pulmonary vein uniquely carries oxygen-rich blood despite being termed a "vein".
MCQ 1:
Which vessel carries oxygenated blood to the heart?
a) Pulmonary artery
b) Pulmonary vein
c) Umbilical artery
d) Vena cava
Answer & Explanation:
Correct answer: b) Pulmonary vein.
Explanation: Pulmonary veins carry oxygenated blood from the lungs to the left atrium. Despite being called veins, they are unique in transporting oxygen-rich blood, opposite to systemic veins which carry deoxygenated blood. This is essential for maintaining oxygen supply to body tissues.
MCQ 2 (Clinical):
A newborn with persistent pulmonary hypertension shows decreased oxygen in:
a) Pulmonary vein
b) Umbilical vein
c) Pulmonary artery
d) Systemic artery
Answer & Explanation:
Correct answer: a) Pulmonary vein.
Explanation: In persistent pulmonary hypertension of the newborn, elevated pulmonary vascular resistance leads to poor oxygenation and right-to-left shunting. This causes reduced oxygen content in the pulmonary vein, compromising oxygen delivery to systemic circulation and contributing to cyanosis in neonates.
MCQ 3:
The umbilical artery carries:
a) Oxygenated blood to fetus
b) Deoxygenated blood to placenta
c) Oxygenated blood to placenta
d) Mixed blood to fetus
Answer & Explanation:
Correct answer: b) Deoxygenated blood to placenta.
Explanation: In fetal circulation, the umbilical artery carries deoxygenated blood from the fetus to the placenta for gas exchange. The umbilical vein carries oxygenated blood from the placenta back to the fetus, crucial for fetal development, compensating for immature fetal lungs.
MCQ 4 (Clinical):
Which condition causes abnormal flow in pulmonary veins?
a) Patent ductus arteriosus
b) Total anomalous pulmonary venous return
c) Atrial septal defect
d) Ventricular septal defect
Answer & Explanation:
Correct answer: b) Total anomalous pulmonary venous return.
Explanation: Total anomalous pulmonary venous return is a congenital defect where pulmonary veins drain into systemic venous circulation instead of the left atrium, leading to mixing of oxygenated and deoxygenated blood, causing cyanosis and heart failure in neonates if untreated.
MCQ 5:
Right atrium receives blood from:
a) Pulmonary veins
b) Pulmonary arteries
c) Vena cavae
d) Aorta
Answer & Explanation:
Correct answer: c) Vena cavae.
Explanation: The right atrium receives deoxygenated blood from the systemic circulation via superior and inferior vena cavae. This blood is then pumped to the right ventricle and directed to the lungs for oxygenation, a fundamental step in the cardiac cycle maintaining oxygen supply.
MCQ 6 (Clinical):
Which vessel shows oxygenated blood in fetal circulation?
a) Umbilical artery
b) Pulmonary artery
c) Umbilical vein
d) Vena cava
Answer & Explanation:
Correct answer: c) Umbilical vein.
Explanation: In fetal circulation, the umbilical vein uniquely carries oxygenated blood from the placenta to the fetus, compensating for non-functional fetal lungs. This provides essential oxygen and nutrients to the growing fetus. Postnatally, this vessel closes as the newborn begins pulmonary respiration.
MCQ 7:
Which vessel normally carries deoxygenated blood?
a) Pulmonary vein
b) Pulmonary artery
c) Aorta
d) Coronary vein
Answer & Explanation:
Correct answer: b) Pulmonary artery.
Explanation: The pulmonary artery is unique among arteries as it carries deoxygenated blood from the right ventricle to the lungs for oxygenation. This is essential for gas exchange and distinguishes it from systemic arteries which carry oxygenated blood away from the heart.
MCQ 8 (Clinical):
Which condition increases deoxygenated blood in systemic arteries?
a) Patent foramen ovale
b) Pulmonary embolism
c) Atrial septal defect
d) Total anomalous pulmonary venous return
Answer & Explanation:
Correct answer: d) Total anomalous pulmonary venous return.
Explanation: In total anomalous pulmonary venous return, pulmonary veins drain into systemic veins, causing mixing of oxygenated and deoxygenated blood, leading to low oxygen saturation in systemic arteries. This defect often presents in neonates with cyanosis and requires surgical correction.
MCQ 9:
Pulmonary artery originates from:
a) Left ventricle
b) Right atrium
c) Right ventricle
d) Left atrium
Answer & Explanation:
Correct answer: c) Right ventricle.
Explanation: The pulmonary artery arises from the right ventricle and carries deoxygenated blood to the lungs. Its unique role contrasts systemic arteries, delivering blood for oxygenation rather than distributing oxygenated blood, integral in the pulmonary circulation loop.
MCQ 10 (Clinical):
Why is pulmonary vein an exception among veins?
a) It carries deoxygenated blood
b) It has thicker walls
c) It carries oxygenated blood
d) It lacks valves
Answer & Explanation:
Correct answer: c) It carries oxygenated blood.
Explanation: Pulmonary veins are exceptional because they carry oxygenated blood from lungs to the left atrium, unlike other veins carrying deoxygenated blood. This unique function is critical for systemic circulation, enabling oxygen delivery to organs, and differs from usual vein behavior.
Subtopic: Blood Vessel Structure
Keywords:
Capacitance Vessels: Blood vessels, mainly veins, that act as reservoirs storing large amounts of blood and regulate venous return by changing their capacity.
Elastic Tissue: Connective tissue in vessel walls providing stretch and recoil ability to accommodate blood volume changes.
Muscle Tissue in Vessel Walls: Smooth muscle that adjusts vessel diameter, regulating blood flow and pressure.
Lead Question - 2013:
Capacitance vessels have in their wall ?
a) More elastic tissue and less muscle
b) Less elastic tissue and more muscle
c) More elastic tissue and more muscle
d) Less elastic tissue and less muscle
Answer & Explanation:
Correct answer: a) More elastic tissue and less muscle.
Explanation: Capacitance vessels, primarily veins, contain more elastic tissue and less smooth muscle in their walls compared to arteries. This structure allows them to distend and hold large blood volumes, providing a reservoir function. They accommodate variable blood volume changes with minimal pressure change, essential in circulatory regulation.
MCQ 1:
Which vessels are known as capacitance vessels?
a) Arteries
b) Veins
c) Arterioles
d) Capillaries
Answer & Explanation:
Correct answer: b) Veins.
Explanation: Veins are called capacitance vessels due to their ability to hold large blood volumes at low pressure. Their walls contain more elastic tissue and fewer muscle fibers, allowing them to stretch and store blood. This helps maintain venous return and cardiac output under varying physiological conditions.
MCQ 2 (Clinical):
A patient with chronic venous insufficiency shows dilated veins because of:
a) Increased muscle content
b) Loss of elastic tissue
c) Increased capillary permeability
d) Decreased blood volume
Answer & Explanation:
Correct answer: b) Loss of elastic tissue.
Explanation: Chronic venous insufficiency leads to dilated, incompetent veins due to degradation of elastic tissue and weakening of the venous wall. This reduces vein recoil and causes pooling of blood, especially in lower limbs, contributing to varicose veins and edema commonly seen in such patients.
MCQ 3:
Elastic tissue in blood vessels helps in:
a) Constricting vessels actively
b) Maintaining vessel structure under pressure
c) Reducing blood flow
d) Enhancing oxygen exchange
Answer & Explanation:
Correct answer: b) Maintaining vessel structure under pressure.
Explanation: Elastic tissue in blood vessels provides stretch and recoil capabilities, enabling vessels to accommodate pulsatile blood flow and pressure variations. This is especially important in large arteries and capacitance vessels, allowing them to store and release energy, maintaining continuous blood flow during diastole.
MCQ 4 (Clinical):
In case of severe hemorrhage, which vessel function becomes crucial?
a) Arterial resistance
b) Capacitance vessel recoil
c) Increased capillary permeability
d) Vasodilation of arterioles
Answer & Explanation:
Correct answer: b) Capacitance vessel recoil.
Explanation: During severe hemorrhage, veins constrict (venoconstriction) to push stored blood towards the heart and maintain venous return. Capacitance vessels’ ability to recoil becomes critical for stabilizing cardiac output and blood pressure, especially when circulating volume is compromised, helping to sustain organ perfusion.
MCQ 5:
Compared to arteries, veins have:
a) More smooth muscle
b) Thicker walls
c) Larger lumen and thinner walls
d) More elastic fibers
Answer & Explanation:
Correct answer: c) Larger lumen and thinner walls.
Explanation: Veins have a larger lumen and thinner walls compared to arteries. This allows them to accommodate larger blood volumes at lower pressures. The reduced smooth muscle content and higher compliance make veins suited as capacitance vessels, efficiently storing and returning blood to the heart.
MCQ 6 (Clinical):
Varicose veins are due to failure of:
a) Arterial elastic tissue
b) Venous valves
c) Capillary permeability
d) Smooth muscle contraction
Answer & Explanation:
Correct answer: b) Venous valves.
Explanation: Varicose veins occur when venous valves fail, causing blood to pool and veins to dilate abnormally. Loss of elastic tissue and reduced venous wall tone further exacerbates the problem. The malfunction leads to venous insufficiency, causing leg swelling, heaviness, and potential skin changes or ulceration.
MCQ 7:
Which of the following is a key function of capacitance vessels?
a) Rapidly distribute oxygen
b) Act as blood reservoir
c) Generate high pressure
d) Facilitate filtration
Answer & Explanation:
Correct answer: b) Act as blood reservoir.
Explanation: Capacitance vessels, especially veins, store about 70% of total blood volume. Their high compliance allows them to accommodate large volume changes with minimal pressure increase, serving as a reservoir. This assists in maintaining stable circulation and venous return under different physiological demands.
MCQ 8 (Clinical):
Which histological feature is predominant in capacitance vessels?
a) Thick smooth muscle layer
b) Abundant elastic fibers
c) Prominent internal elastic lamina
d) Dense connective tissue
Answer & Explanation:
Correct answer: b) Abundant elastic fibers.
Explanation: Capacitance vessels contain abundant elastic fibers and relatively less smooth muscle compared to arteries. This structure allows them to stretch and accommodate blood volume changes, adjusting venous return efficiently, especially during postural changes or blood loss situations without significant pressure change.
MCQ 9:
Which statement is true about veins?
a) High pressure vessels
b) Contain valves to prevent backflow
c) Thick muscular walls
d) Primary site for nutrient exchange
Answer & Explanation:
Correct answer: b) Contain valves to prevent backflow.
Explanation: Veins, especially in limbs, contain valves preventing blood backflow due to low pressure. These valves aid venous return towards the heart against gravity. Combined with muscle contractions during movement, they ensure efficient circulation despite low venous pressure.
MCQ 10 (Clinical):
In heart failure, capacitance vessels contribute to:
a) Reducing preload
b) Increasing afterload
c) Venous pooling
d) Increasing cardiac output
Answer & Explanation:
Correct answer: c) Venous pooling.
Explanation: In heart failure, reduced cardiac output and poor venous return lead to blood pooling in capacitance vessels. Impaired recoil and valve dysfunction exacerbate this, contributing to edema and organ congestion. Therapeutic strategies target venous tone improvement to optimize preload and reduce symptoms.
Topic: Cardiovascular System
Subtopic: Blood Pressure Regulation
Keywords:
Chemoreceptor Reflex: A mechanism where chemoreceptors detect changes in blood oxygen, carbon dioxide, and pH levels, adjusting heart rate and vascular tone to maintain homeostasis.
Baroreceptor Reflex: A fast-acting feedback mechanism where baroreceptors in the carotid sinus and aortic arch detect blood pressure changes and adjust heart rate and vascular resistance accordingly.
CNS Ischemic Reflex: A powerful reflex activated when cerebral perfusion falls critically, leading to intense sympathetic activation to restore blood pressure and perfusion to vital organs.
Lead Question - 2013:
BP is less than 40 mm Hg. Which mechanism of regulation is working ?
a) Chemoreceptor reflex
b) Baroreceptor reflex
c) CNS ischemic reflex
d) None of the above
Answer & Explanation:
Correct answer: c) CNS ischemic reflex.
Explanation: When blood pressure falls critically below 40 mm Hg, cerebral perfusion becomes inadequate, activating the CNS ischemic reflex. This powerful reflex triggers intense sympathetic output, causing vasoconstriction and increased heart rate to restore blood pressure and cerebral blood flow, critical in severe hypotension or shock states.
MCQ 1:
Which receptors detect blood pressure changes in baroreceptor reflex?
a) Chemoreceptors
b) Baroreceptors in carotid sinus and aortic arch
c) Thermoreceptors
d) Proprioceptors
Answer & Explanation:
Correct answer: b) Baroreceptors in carotid sinus and aortic arch.
Explanation: Baroreceptors are stretch-sensitive mechanoreceptors located in the carotid sinus and aortic arch. They detect changes in arterial blood pressure and send signals via the glossopharyngeal and vagus nerves to the medulla to modulate heart rate and vascular tone rapidly.
MCQ 2 (Clinical):
A patient with sudden severe hypotension triggers which reflex?
a) Bainbridge reflex
b) CNS ischemic reflex
c) Hering-Breuer reflex
d) Baroreceptor reflex
Answer & Explanation:
Correct answer: b) CNS ischemic reflex.
Explanation: CNS ischemic reflex activates when hypotension critically lowers cerebral perfusion (
MCQ 3:
Baroreceptor reflex primarily modulates blood pressure by:
a) Altering kidney filtration
b) Adjusting heart rate and vascular tone
c) Changing respiratory rate
d) Modulating red blood cell production
Answer & Explanation:
Correct answer: b) Adjusting heart rate and vascular tone.
Explanation: Baroreceptor reflex senses arterial wall stretch via baroreceptors and adjusts cardiac output and systemic vascular resistance. It rapidly compensates for short-term blood pressure fluctuations, increasing heart rate and constricting vessels during hypotension to stabilize systemic perfusion.
MCQ 4 (Clinical):
Chemoreceptor reflex primarily responds to:
a) Blood glucose levels
b) Oxygen, CO2, and pH changes
c) Blood pressure
d) Body temperature
Answer & Explanation:
Correct answer: b) Oxygen, CO2, and pH changes.
Explanation: Chemoreceptors, located in carotid and aortic bodies, sense low oxygen, elevated CO2, or acidosis. They trigger increased respiratory rate and sympathetic activation, indirectly supporting blood pressure by increasing heart rate and systemic vasoconstriction, especially during hypoxia.
MCQ 5:
The CNS ischemic reflex is:
a) Weak and slow
b) Only activated in mild hypotension
c) Powerful and activates during severe hypotension
d) Responsible for respiratory rate adjustment
Answer & Explanation:
Correct answer: c) Powerful and activates during severe hypotension.
Explanation: CNS ischemic reflex is a strong protective mechanism triggered by extreme hypotension (
MCQ 6 (Clinical):
In shock, which mechanism is activated first?
a) CNS ischemic reflex
b) Chemoreceptor reflex
c) Baroreceptor reflex
d) Hormonal RAAS system
Answer & Explanation:
Correct answer: c) Baroreceptor reflex.
Explanation: Baroreceptor reflex is the first line of defense during hypotension. It responds rapidly to maintain blood pressure by adjusting heart rate and vascular tone. CNS ischemic reflex activates only when blood pressure falls critically, while the RAAS system is slower acting.
MCQ 7:
Which of the following is a long-term blood pressure regulation mechanism?
a) Baroreceptor reflex
b) Chemoreceptor reflex
c) Renin-Angiotensin-Aldosterone System (RAAS)
d) CNS ischemic reflex
Answer & Explanation:
Correct answer: c) Renin-Angiotensin-Aldosterone System (RAAS).
Explanation: RAAS regulates blood pressure by controlling sodium, water retention, and vascular resistance. It responds over hours to days, providing sustained correction of hypovolemia or hypotension, unlike baroreceptor and CNS ischemic reflexes, which act quickly for short-term regulation.
MCQ 8 (Clinical):
What happens during the chemoreceptor reflex in severe hypoxia?
a) Decrease in respiratory rate
b) Increase in sympathetic output
c) Vasodilation
d) Bradycardia
Answer & Explanation:
Correct answer: b) Increase in sympathetic output.
Explanation: Chemoreceptors sense hypoxia and trigger increased sympathetic activity, elevating heart rate and systemic vascular resistance. This enhances oxygen delivery by raising cardiac output and redistributing blood flow, critical for maintaining oxygenation in hypoxic states.
MCQ 9:
Baroreceptors send signals to which brain center?
a) Hypothalamus
b) Medulla oblongata
c) Thalamus
d) Pons
Answer & Explanation:
Correct answer: b) Medulla oblongata.
Explanation: Baroreceptors in the carotid sinus and aortic arch send signals via glossopharyngeal and vagus nerves to the medulla oblongata. The cardiovascular center integrates this input and modulates sympathetic and parasympathetic outflow to regulate heart rate and vascular tone.
MCQ 10 (Clinical):
Why is the CNS ischemic reflex considered a last-resort mechanism?
a) It is slow
b) It leads to severe vasodilation
c) It activates only at dangerously low BP
d) It reduces heart rate
Answer & Explanation:
Correct answer: c) It activates only at dangerously low BP.
Explanation: The CNS ischemic reflex activates when cerebral perfusion pressure is critically low (
Chapter: Cardiovascular Physiology
Topic: Reflex Mechanisms
Subtopic: Bezold-Jarisch Reflex
Keywords:
Depressor Reflex: A reflex that causes decreased heart rate, hypotension, and vasodilation, contributing to reduced cardiac workload.
Bezold-Jarisch Reflex: Cardioprotective reflex triggered by mechanoreceptors or chemoreceptors in the heart, especially during myocardial ischemia or ventricular distension, resulting in bradycardia and hypotension.
Ventricular Distension: Excessive stretching of the ventricular walls due to volume overload or impaired ventricular compliance.
Atrial Overload: Increased pressure or volume in the atria, often due to valvular disease or heart failure, impacting cardiac reflexes.
Lead Question - 2013:
Depressor reflex, Bezold-Jarisch reflex, produced by the following stimulus:
a) Atrial overload
b) Myocardial infarction
c) Ventricular distension
d) Isotonic exercise
Answer & Explanation:
Correct answer: b) Myocardial infarction.
Explanation: The Bezold-Jarisch reflex is triggered by myocardial infarction, activating mechanoreceptors and chemoreceptors in the ventricular walls. This leads to bradycardia, hypotension, and peripheral vasodilation as a protective mechanism to limit further myocardial oxygen demand and injury during ischemia.
MCQ 1:
Which receptor primarily mediates the Bezold-Jarisch reflex?
a) Baroreceptors
b) Chemoreceptors
c) Mechanoreceptors in ventricular walls
d) Proprioceptors
Answer & Explanation:
Correct answer: c) Mechanoreceptors in ventricular walls.
Explanation: Mechanoreceptors located in the ventricular walls detect abnormal mechanical stimuli such as ischemia or distension during myocardial infarction. These receptors activate afferent vagal pathways, leading to bradycardia and hypotension via the Bezold-Jarisch reflex, serving to protect the heart from further stress.
MCQ 2 (Clinical):
A patient presents with hypotension and bradycardia after acute inferior myocardial infarction. The likely reflex involved is:
a) Baroreceptor reflex
b) Bainbridge reflex
c) Bezold-Jarisch reflex
d) Hering-Breuer reflex
Answer & Explanation:
Correct answer: c) Bezold-Jarisch reflex.
Explanation: During inferior myocardial infarction, mechanoreceptors in the ventricular walls are stimulated, causing the Bezold-Jarisch reflex. This leads to bradycardia, hypotension, and peripheral vasodilation. The reflex acts to reduce myocardial oxygen consumption and prevent further ischemic damage.
MCQ 3:
The Bezold-Jarisch reflex results in which of the following primary effects?
a) Tachycardia
b) Vasoconstriction
c) Bradycardia
d) Increased cardiac output
Answer & Explanation:
Correct answer: c) Bradycardia.
Explanation: The reflex causes bradycardia, hypotension, and vasodilation. Triggered by chemoreceptors and mechanoreceptors in the ventricular walls during pathological states, it reduces cardiac workload. It does not lead to tachycardia or vasoconstriction, and cardiac output typically decreases due to lower heart rate and reduced vascular tone.
MCQ 4 (Clinical):
Which of the following conditions is LEAST likely to activate the Bezold-Jarisch reflex?
a) Myocardial infarction
b) Ventricular distension
c) Acute atrial overload
d) Profound hypovolemia
Answer & Explanation:
Correct answer: c) Acute atrial overload.
Explanation: The Bezold-Jarisch reflex is primarily mediated by ventricular receptors, not atrial receptors. Myocardial infarction, ventricular distension, and profound hypovolemia can activate this reflex due to altered ventricular mechanics or chemoreceptor activation, but atrial overload is not a direct trigger.
MCQ 5:
During Bezold-Jarisch reflex activation, which autonomic pathway predominates?
a) Sympathetic
b) Parasympathetic
c) Somatic
d) Central nervous system direct activation
Answer & Explanation:
Correct answer: b) Parasympathetic.
Explanation: The reflex is mediated by increased vagal (parasympathetic) activity, causing bradycardia and vasodilation. Afferent signals from ventricular receptors stimulate the medullary centers, enhancing parasympathetic outflow and suppressing sympathetic tone to reduce cardiac workload and protect the heart.
MCQ 6 (Clinical):
Which therapeutic approach may blunt Bezold-Jarisch reflex in myocardial infarction?
a) Beta-blockers
b) Diuretics
c) Calcium channel blockers
d) ACE inhibitors
Answer & Explanation:
Correct answer: a) Beta-blockers.
Explanation: Beta-blockers reduce heart rate and block sympathetic activation but also indirectly dampen reflex pathways, including the Bezold-Jarisch reflex. By decreasing myocardial oxygen demand and inhibiting excessive vagal reflex activation, they help stabilize hemodynamics during myocardial infarction.
MCQ 7:
The Bezold-Jarisch reflex primarily protects against:
a) Hypertension
b) Hypovolemia
c) Myocardial ischemia
d) Arrhythmias
Answer & Explanation:
Correct answer: c) Myocardial ischemia.
Explanation: This reflex decreases heart rate and systemic blood pressure to reduce myocardial oxygen demand during ischemia. It prevents further ischemic injury by limiting cardiac workload, rather than affecting blood volume or systemic hypertension directly.
MCQ 8 (Clinical):
A patient undergoing inferior wall myocardial infarction develops nausea, sweating, and bradycardia. These are due to:
a) Activation of peripheral baroreceptors
b) Bezold-Jarisch reflex
c) Hypovolemia
d) Vasovagal syncope
Answer & Explanation:
Correct answer: b) Bezold-Jarisch reflex.
Explanation: Inferior myocardial infarction stimulates mechanoreceptors in the heart, triggering the Bezold-Jarisch reflex. This leads to parasympathetic activation, resulting in bradycardia, hypotension, nausea, and diaphoresis as compensatory responses to ischemia.
MCQ 9:
The time course of Bezold-Jarisch reflex is typically:
a) Seconds to minutes
b) Hours
c) Days
d) Weeks
Answer & Explanation:
Correct answer: a) Seconds to minutes.
Explanation: The Bezold-Jarisch reflex is a rapid reflex, initiating within seconds of stimulus (like ischemia) and lasting minutes. It provides acute modulation of cardiovascular function, unlike long-term adaptations seen in chronic disease processes.
MCQ 10 (Clinical):
Which clinical test can provoke the Bezold-Jarisch reflex?
a) Tilt table test
b) Valsalva maneuver
c) Cold pressor test
d) Head-up tilt
Answer & Explanation:
Correct answer: a) Tilt table test.
Explanation: The tilt table test can provoke the Bezold-Jarisch reflex in susceptible individuals by sudden changes in venous return and ventricular filling. It helps diagnose neurocardiogenic syncope where exaggerated reflex bradycardia and hypotension occur during postural changes.
Subtopic: Chemoreceptors and Glomus Cells
Keywords:
Glomus Cells: Specialized chemoreceptive cells located in carotid and aortic bodies that detect changes in blood oxygen, carbon dioxide, and pH levels.
Chemoreceptors: Sensory receptors that respond to chemical stimuli such as changes in blood gases and pH.
Carotid Body: A small cluster of chemoreceptors and supporting cells located at the bifurcation of the carotid artery.
Aortic Body: Chemoreceptor located along the aortic arch involved in cardiovascular and respiratory regulation.
Lead Question - 2013:
Glomus cells are found in -
a) Bladder
b) Brain
c) Chemoreceptors
d) Kidney
Answer & Explanation:
Correct answer: c) Chemoreceptors.
Explanation: Glomus cells are specialized chemoreceptor cells present in the carotid and aortic bodies. They detect changes in blood oxygen, carbon dioxide, and pH, and help regulate respiratory and cardiovascular functions by signaling the central nervous system to adjust ventilation or heart rate accordingly.
MCQ 1:
Primary function of glomus cells is?
a) Hormone secretion
b) Mechanical support
c) Detect blood gas changes
d) Produce neurotransmitters
Answer & Explanation:
Correct answer: c) Detect blood gas changes.
Explanation: Glomus cells in carotid and aortic bodies are chemoreceptors sensitive to arterial blood oxygen, carbon dioxide, and pH levels. They transmit signals to respiratory centers to regulate ventilation, essential for maintaining homeostasis and adapting to hypoxic or hypercapnic conditions.
MCQ 2 (Clinical):
Glomus cell tumors are most commonly found in?
a) Carotid body
b) Thyroid gland
c) Adrenal medulla
d) Pituitary gland
Answer & Explanation:
Correct answer: a) Carotid body.
Explanation: Carotid body tumors, also called paragangliomas, arise from glomus cells and are typically benign but can cause local mass effects or catecholamine secretion. Clinical features include neck mass, bruits, and potential cranial nerve palsies.
MCQ 3:
Glomus cells respond primarily to changes in:
a) Blood pressure
b) Blood oxygen and pH
c) Body temperature
d) Plasma glucose
Answer & Explanation:
Correct answer: b) Blood oxygen and pH.
Explanation: Glomus cells in chemoreceptors respond to hypoxia, hypercapnia, and acidosis by stimulating afferent nerves to increase respiratory rate and cardiac output, crucial for maintaining systemic oxygenation and acid-base balance during various physiological and pathological conditions.
MCQ 4 (Clinical):
Which condition may be associated with impaired glomus cell function?
a) Hypertension
b) Hypoventilation syndrome
c) Diabetes mellitus
d) Hypothyroidism
Answer & Explanation:
Correct answer: b) Hypoventilation syndrome.
Explanation: Impaired glomus cell function may blunt chemosensory response to hypoxia or hypercapnia, contributing to conditions like central hypoventilation syndrome (Ondine’s curse), where automatic respiratory regulation fails, particularly during sleep or metabolic challenges.
MCQ 5:
Which nerve carries signals from carotid body glomus cells to brain?
a) Hypoglossal nerve
b) Vagus nerve
c) Glossopharyngeal nerve
d) Trigeminal nerve
Answer & Explanation:
Correct answer: c) Glossopharyngeal nerve.
Explanation: The glossopharyngeal nerve (cranial nerve IX) transmits sensory input from carotid body glomus cells to the brainstem, providing essential feedback for the regulation of respiration and cardiovascular responses to blood gas changes.
MCQ 6 (Clinical):
Carotid body tumor may present with which symptom?
a) Hoarseness
b) Hemiparesis
c) Blurred vision
d) Aphasia
Answer & Explanation:
Correct answer: a) Hoarseness.
Explanation: Carotid body tumors may compress adjacent cranial nerves, such as the vagus nerve, leading to symptoms like hoarseness, dysphagia, or Horner’s syndrome. Early diagnosis is critical to avoid vascular and neurologic complications.
MCQ 7:
Location of aortic body containing glomus cells?
a) At base of brain
b) Along the aortic arch
c) In the renal artery
d) In the pulmonary artery
Answer & Explanation:
Correct answer: b) Along the aortic arch.
Explanation: The aortic body contains glomus cells situated along the aortic arch, where they sense blood oxygen, carbon dioxide, and pH levels. These chemoreceptors complement carotid bodies in regulating respiratory and cardiovascular reflexes.
MCQ 8 (Clinical):
Glomus tumors may secrete which substances?
a) Insulin
b) Catecholamines
c) Thyroid hormones
d) Cortisol
Answer & Explanation:
Correct answer: b) Catecholamines.
Explanation: Some glomus tumors, especially extra-adrenal paragangliomas, can secrete catecholamines, leading to hypertension, palpitations, and headaches. Differentiating these tumors from other masses is important for proper management and prevention of hypertensive crises during surgery.
MCQ 9:
Glomus cells in chemoreceptors detect:
a) Mechanical stretch
b) Chemical changes
c) Temperature changes
d) Light intensity
Answer & Explanation:
Correct answer: b) Chemical changes.
Explanation: Glomus cells are specialized for detecting chemical changes in arterial blood, such as low oxygen (hypoxia), high carbon dioxide (hypercapnia), and acidosis, and send signals to the brainstem to adjust respiratory and cardiovascular responses accordingly.
MCQ 10 (Clinical):
Carotid body tumor treatment primarily involves:
a) Chemotherapy
b) Surgical excision
c) Radiotherapy
d) Observation only
Answer & Explanation:
Correct answer: b) Surgical excision.
Explanation: The main treatment for carotid body tumors is surgical excision, aiming to remove the mass and prevent growth or metastasis. Careful preoperative evaluation is essential to avoid cranial nerve damage and manage catecholamine secretion complications during surgery.
Subtopic: Cerebral Venous System
Keywords:
Cavernous Sinus: A dural venous sinus located on either side of the pituitary gland, draining venous blood from the brain.
Inferior Cerebral Vein: Drains the inferior parts of the cerebral hemispheres into the cavernous sinus.
Central Vein of Retina: Drains the retina and empties into the cavernous sinus, significant in ophthalmology.
Sphenoparietal Sinus: Receives blood from superficial middle cerebral veins and drains into cavernous sinus.
Superior Cerebral Vein: Drains the superior surface of the cerebral hemispheres into the superior sagittal sinus.
Lead Question - 2013:
Tributaries of cavernous sinus are all except?
a) Inferior cerebral vein
b) Central vein of retina
c) Sphenoparietal sinus
d) Superior cerebral vein
Answer & Explanation:
Correct answer: d) Superior cerebral vein.
Explanation: The superior cerebral vein drains into the superior sagittal sinus, not the cavernous sinus. The cavernous sinus receives blood from the inferior cerebral veins, sphenoparietal sinus, ophthalmic veins, and central vein of retina. Knowledge of venous anatomy is vital in managing venous thrombosis and ocular complications.
MCQ 1:
Which vein drains the retina?
a) Superior cerebral vein
b) Central vein of retina
c) Inferior cerebral vein
d) Sphenoparietal sinus
Answer & Explanation:
Correct answer: b) Central vein of retina.
Explanation: The central vein of retina drains venous blood from the retina into the cavernous sinus. Increased pressure or thrombosis in this vein can lead to papilledema and vision loss, making its anatomical course clinically significant.
MCQ 2:
Which structure does NOT drain into the cavernous sinus?
a) Ophthalmic vein
b) Central retinal vein
c) Superior cerebral vein
d) Sphenoparietal sinus
Answer & Explanation:
Correct answer: c) Superior cerebral vein.
Explanation: The superior cerebral vein drains into the superior sagittal sinus. The cavernous sinus receives blood from the ophthalmic veins, central retinal vein, and sphenoparietal sinus. This knowledge is essential in understanding cavernous sinus thrombosis and its ocular manifestations.
MCQ 3:
Clinical significance of cavernous sinus thrombosis includes all EXCEPT:
a) Proptosis
b) Ophthalmoplegia
c) Visual loss
d) Hemiplegia
Answer & Explanation:
Correct answer: d) Hemiplegia.
Explanation: Cavernous sinus thrombosis causes proptosis, ophthalmoplegia, and visual loss due to involvement of cranial nerves III, IV, V1, V2, and VI. Hemiplegia is typically associated with arterial strokes, not venous thrombosis, differentiating the pathology in clinical practice.
MCQ 4 (Clinical):
Infection of which area commonly leads to cavernous sinus thrombosis?
a) Scalp
b) Mid-face (danger triangle)
c) Neck
d) External ear
Answer & Explanation:
Correct answer: b) Mid-face (danger triangle).
Explanation: The danger triangle of the face (from corners of mouth to bridge of nose) is connected to the cavernous sinus via ophthalmic veins. Infections here can cause retrograde thrombophlebitis, leading to cavernous sinus thrombosis and severe neurological deficits.
MCQ 5:
The cavernous sinus is located near which important gland?
a) Thyroid gland
b) Pituitary gland
c) Adrenal gland
d) Pineal gland
Answer & Explanation:
Correct answer: b) Pituitary gland.
Explanation: The cavernous sinus lies lateral to the pituitary gland at the base of the brain. Its anatomical position makes it vulnerable during pituitary adenoma surgery or inflammatory conditions, risking cranial nerve involvement and vascular complications.
MCQ 6 (Clinical):
Which cranial nerve does NOT pass through the cavernous sinus?
a) Oculomotor nerve (III)
b) Optic nerve (II)
c) Trochlear nerve (IV)
d) Abducens nerve (VI)
Answer & Explanation:
Correct answer: b) Optic nerve (II).
Explanation: Cranial nerves III, IV, V1, V2, and VI pass through the cavernous sinus. The optic nerve (II) does not. Cavernous sinus pathology typically presents with ophthalmoplegia, ptosis, and sensory loss but not optic nerve dysfunction.
MCQ 7:
The sphenoparietal sinus drains into:
a) Superior sagittal sinus
b) Inferior sagittal sinus
c) Cavernous sinus
d) Transverse sinus
Answer & Explanation:
Correct answer: c) Cavernous sinus.
Explanation: The sphenoparietal sinus drains into the cavernous sinus, helping collect venous blood from the superficial cerebral veins. Its involvement is important in venous drainage anomalies and thrombosis.
MCQ 8 (Clinical):
What is a key clinical sign of cavernous sinus thrombosis?
a) Hemianopia
b) Periorbital edema and chemosis
c) Contralateral limb weakness
d) Anosmia
Answer & Explanation:
Correct answer: b) Periorbital edema and chemosis.
Explanation: Cavernous sinus thrombosis presents with periorbital edema, chemosis, proptosis, and cranial nerve deficits. Prompt identification prevents life-threatening complications such as meningitis and permanent cranial nerve damage.
MCQ 9:
The cavernous sinus drains into:
a) Internal jugular vein
b) Superior sagittal sinus
c) Inferior petrosal sinus
d) External jugular vein
Answer & Explanation:
Correct answer: c) Inferior petrosal sinus.
Explanation: The cavernous sinus drains into the inferior petrosal sinus and superior petrosal sinus, eventually reaching the internal jugular vein. Understanding this drainage is critical for neurosurgical and radiological interventions to prevent complications.
MCQ 10 (Clinical):
Cavernous sinus thrombosis is most commonly caused by?
a) Viral infection
b) Bacterial infection
c) Fungal infection
d) Trauma only
Answer & Explanation:
Correct answer: b) Bacterial infection.
Explanation: Cavernous sinus thrombosis is most commonly due to bacterial infections from the face (danger triangle). Prompt antibiotics and possible surgical drainage are required to avoid life-threatening complications like sepsis or stroke.
Subtopic: Vertebrobasilar Circulation
Keywords:
Vertebral Arteries: Paired arteries arising from subclavian arteries, supplying posterior brain structures.
Anterior Spinal Artery: Arises from vertebral arteries, supplies anterior two-thirds of spinal cord.
Posterior Spinal Artery: Supplies posterior one-third of spinal cord.
Medullary Arteries: Small arteries supplying the spinal cord segments.
Basilar Artery: Formed by the union of vertebral arteries, supplies brainstem and cerebellum.
Lead Question - 2013:
Vertebral arteries of both sides unite to form?
a) Anterior spinal artery
b) Posterior spinal artery
c) Medullary artery
d) Basilar artery
Answer & Explanation:
Correct answer: d) Basilar artery.
Explanation: The vertebral arteries ascend through the transverse foramina of cervical vertebrae and enter the cranial cavity via the foramen magnum. They unite at the pontomedullary junction to form the basilar artery, which supplies the brainstem, cerebellum, and posterior cerebral circulation, crucial in neurovascular health.
MCQ 1:
Each vertebral artery arises from?
a) Internal carotid artery
b) Subclavian artery
c) Common carotid artery
d) External carotid artery
Answer & Explanation:
Correct answer: b) Subclavian artery.
Explanation: The vertebral arteries are branches of the subclavian arteries. They ascend through transverse foramina of cervical vertebrae, supplying the posterior brain, spinal cord, and cerebellum. Their integrity is vital to prevent ischemic strokes in the posterior circulation.
MCQ 2:
Basilar artery primarily supplies?
a) Anterior cerebral hemispheres
b) Cerebellum and brainstem
c) Spinal cord only
d) Peripheral nerves
Answer & Explanation:
Correct answer: b) Cerebellum and brainstem.
Explanation: The basilar artery is critical in supplying blood to the cerebellum, pons, medulla, and posterior cerebral hemispheres. Occlusion leads to serious deficits like locked-in syndrome, vertigo, or coma, demanding urgent clinical intervention.
MCQ 3:
The anterior spinal artery is formed by?
a) Both vertebral arteries
b) Basilar artery
c) External carotid artery
d) Posterior cerebral artery
Answer & Explanation:
Correct answer: a) Both vertebral arteries.
Explanation: The anterior spinal artery is formed by branches of both vertebral arteries and supplies the anterior two-thirds of the spinal cord. Compromise of this artery causes anterior spinal artery syndrome, characterized by motor deficits and loss of pain/temperature sensation.
MCQ 4 (Clinical):
Occlusion of the basilar artery causes:
a) Hemiplegia
b) Locked-in syndrome
c) Monoplegia
d) Aphasia
Answer & Explanation:
Correct answer: b) Locked-in syndrome.
Explanation: Basilar artery occlusion can result in locked-in syndrome where patients lose voluntary muscle control except for eye movements. Early detection and thrombolysis are essential to reduce morbidity and mortality in such vascular emergencies.
MCQ 5:
Which artery supplies the posterior part of the spinal cord?
a) Anterior spinal artery
b) Posterior spinal artery
c) Vertebral artery
d) Basilar artery
Answer & Explanation:
Correct answer: b) Posterior spinal artery.
Explanation: The posterior spinal arteries supply the posterior one-third of the spinal cord, including dorsal columns responsible for fine touch and proprioception. Compromise leads to posterior spinal artery syndrome, marked by sensory deficits without significant motor loss.
MCQ 6 (Clinical):
Wallenberg syndrome is due to infarction of?
a) Anterior spinal artery
b) Posterior inferior cerebellar artery (PICA)
c) Middle cerebral artery
d) Basilar artery
Answer & Explanation:
Correct answer: b) Posterior inferior cerebellar artery (PICA).
Explanation: Wallenberg syndrome, or lateral medullary syndrome, results from PICA infarction. It causes vertigo, ataxia, dysphagia, and ipsilateral Horner's syndrome. Prompt diagnosis prevents complications like aspiration pneumonia and permanent neurological deficits.
MCQ 7:
The vertebral arteries enter the skull through?
a) Jugular foramen
b) Foramen magnum
c) Carotid canal
d) Optic canal
Answer & Explanation:
Correct answer: b) Foramen magnum.
Explanation: Vertebral arteries ascend through the transverse foramina of cervical vertebrae and enter the cranial cavity via the foramen magnum. Understanding their course is vital during cervical spine surgeries to avoid inadvertent injury and stroke.
MCQ 8 (Clinical):
Clinical consequence of vertebral artery dissection includes:
a) Transient ischemic attack
b) Stroke
c) Headache and neck pain
d) All of the above
Answer & Explanation:
Correct answer: d) All of the above.
Explanation: Vertebral artery dissection can present with headache, neck pain, transient ischemic attacks, or stroke. It’s often seen in young patients after minor trauma. Early diagnosis via imaging prevents permanent deficits.
MCQ 9:
The basilar artery bifurcates into:
a) Anterior cerebral arteries
b) Middle cerebral arteries
c) Posterior cerebral arteries
d) Vertebral arteries
Answer & Explanation:
Correct answer: c) Posterior cerebral arteries.
Explanation: The basilar artery bifurcates into the two posterior cerebral arteries, supplying the occipital lobes and inferior temporal lobes. Their patency is crucial for vision and memory functions, and blockage results in homonymous hemianopia and cortical blindness.
MCQ 10 (Clinical):
Which symptom suggests posterior circulation stroke?
a) Aphasia
b) Hemiplegia
c) Vertigo and ataxia
d) Seizures
Answer & Explanation:
Correct answer: c) Vertigo and ataxia.
Explanation: Posterior circulation stroke affects brainstem and cerebellum, causing vertigo, ataxia, dysarthria, and visual disturbances. Recognizing these signs leads to early intervention, preventing irreversible damage and improving outcomes in stroke management.
Subtopic: Cerebral Arterial Supply
Keywords:
Anterior Cerebral Artery (ACA): Supplies medial portions of frontal lobes and superior medial parietal lobes.
Posterior Cerebral Artery (PCA): Supplies occipital lobe, inferior temporal lobe, and posterior parts of the cerebral hemispheres.
Middle Cerebral Artery (MCA): Main artery supplying lateral surface of the cerebral hemisphere, including primary motor and sensory areas.
Posterior Inferior Cerebellar Artery (PICA): Supplies inferior part of cerebellum and medulla oblongata.
Lead Question - 2013:
Chief artery of lateral surface of cerebral hemisphere?
a) Anterior cerebral artery
b) Posterior cerebral artery
c) Middle cerebral artery
d) Posterior inferior cerebellar artery
Answer & Explanation:
Correct answer: c) Middle cerebral artery.
Explanation: The middle cerebral artery (MCA) is the chief artery supplying the lateral surface of the cerebral hemisphere, including motor and sensory cortex, Broca's and Wernicke's areas. It is the most common site for ischemic stroke, making its clinical importance significant in neurovascular disorders.
MCQ 1:
Which artery supplies the medial surface of cerebral hemisphere?
a) Anterior cerebral artery
b) Middle cerebral artery
c) Posterior cerebral artery
d) Basilar artery
Answer & Explanation:
Correct answer: a) Anterior cerebral artery.
Explanation: The anterior cerebral artery supplies the medial surface of the frontal lobes and superior medial parietal lobes. It plays a major role in supplying motor and sensory areas of the lower limb, critical in stroke and vascular disease presentations.
MCQ 2:
Posterior cerebral artery primarily supplies:
a) Frontal lobe
b) Occipital lobe
c) Parietal lobe
d) Temporal lobe only
Answer & Explanation:
Correct answer: b) Occipital lobe.
Explanation: The posterior cerebral artery supplies the occipital lobe and inferior temporal lobe. Infarction in this artery causes visual field defects like homonymous hemianopia, highlighting its clinical relevance in neurovascular diseases.
MCQ 3:
Which cerebral artery is most frequently affected in ischemic stroke?
a) Anterior cerebral artery
b) Posterior cerebral artery
c) Middle cerebral artery
d) Basilar artery
Answer & Explanation:
Correct answer: c) Middle cerebral artery.
Explanation: The middle cerebral artery (MCA) is most commonly involved in ischemic stroke, affecting contralateral motor and sensory function, language centers, and possibly leading to neglect. Understanding MCA pathology is vital in emergency stroke management.
MCQ 4 (Clinical):
MCA stroke commonly presents with:
a) Contralateral leg weakness
b) Contralateral hemiparesis and sensory loss in face and arm
c) Ipsilateral visual loss
d) Bilateral weakness
Answer & Explanation:
Correct answer: b) Contralateral hemiparesis and sensory loss in face and arm.
Explanation: MCA stroke typically leads to contralateral hemiparesis affecting the face and arm due to the somatotopic representation in the lateral cerebral cortex. It is associated with aphasia if the dominant hemisphere is involved.
MCQ 5:
Which artery forms part of the circle of Willis?
a) Middle cerebral artery
b) Anterior cerebral artery
c) Posterior inferior cerebellar artery
d) External carotid artery
Answer & Explanation:
Correct answer: b) Anterior cerebral artery.
Explanation: The anterior cerebral artery is part of the circle of Willis, which provides collateral circulation between anterior and posterior cerebral circulations. This anatomical feature is crucial in compensating for vascular occlusions and preventing ischemic damage.
MCQ 6 (Clinical):
Occlusion of posterior cerebral artery may lead to:
a) Hemiplegia
b) Visual field defects
c) Aphasia
d) Ataxia
Answer & Explanation:
Correct answer: b) Visual field defects.
Explanation: Occlusion of the posterior cerebral artery often leads to homonymous hemianopia due to occipital lobe infarction. Recognizing these signs helps in localization and management of posterior circulation strokes.
MCQ 7:
The posterior inferior cerebellar artery (PICA) supplies:
a) Anterior cerebellum
b) Superior cerebellum
c) Inferior cerebellum and medulla
d) Midbrain
Answer & Explanation:
Correct answer: c) Inferior cerebellum and medulla.
Explanation: PICA supplies the inferior part of the cerebellum and medulla oblongata. PICA infarction causes lateral medullary (Wallenberg) syndrome, characterized by vertigo, dysphagia, and ipsilateral facial sensory loss.
MCQ 8 (Clinical):
Which clinical syndrome is associated with PICA infarction?
a) Weber’s syndrome
b) Wallenberg syndrome
c) Horner’s syndrome
d) Brown-Sequard syndrome
Answer & Explanation:
Correct answer: b) Wallenberg syndrome.
Explanation: Wallenberg syndrome results from PICA infarction, causing symptoms like vertigo, ipsilateral facial numbness, dysphagia, and Horner's syndrome. Prompt recognition is essential for managing posterior circulation strokes and preventing complications.
MCQ 9:
The middle cerebral artery is a direct continuation of:
a) Internal carotid artery
b) External carotid artery
c) Vertebral artery
d) Basilar artery
Answer & Explanation:
Correct answer: a) Internal carotid artery.
Explanation: The middle cerebral artery (MCA) is a direct continuation of the internal carotid artery. It supplies major functional areas of the brain, making its patency vital for maintaining consciousness and motor function.
MCQ 10 (Clinical):
Which symptom suggests MCA involvement?
a) Hemianopia
b) Contralateral lower limb weakness
c) Aphasia
d) Ataxia
Answer & Explanation:
Correct answer: c) Aphasia.
Explanation: MCA stroke involving the dominant hemisphere (usually left) often results in aphasia (expressive, receptive, or global) due to involvement of Broca's or Wernicke's areas. Recognizing aphasia aids in early stroke diagnosis and targeted treatment.
Subtopic: Venous Drainage of Face
Keywords:
Facial Vein: Major vein draining blood from the face into the internal jugular vein.
External Jugular Vein (EJV): Drains blood from the scalp and face into the subclavian vein.
Angular Vein: Formed at the medial angle of the eye by the union of the supratrochlear and supraorbital veins; drains into the facial vein.
Valveless Veins: Veins in the face lack valves, allowing bidirectional blood flow, which can spread infections.
Lead Question - 2013:
Not true about facial vein is?
a) Drains in EJV
b) Largest vein of face
c) Formed from angular vein
d) Has no valves
Answer & Explanation:
Correct answer: a) Drains in EJV.
Explanation: The facial vein does not drain into the external jugular vein (EJV); rather, it drains into the internal jugular vein (IJV). It is the largest vein of the face, formed from the angular vein, and is valveless. These characteristics are important in understanding facial venous drainage and risk of infection spread.
MCQ 1:
The facial vein drains into which vein?
a) External jugular vein
b) Internal jugular vein
c) Subclavian vein
d) Brachiocephalic vein
Answer & Explanation:
Correct answer: b) Internal jugular vein.
Explanation: The facial vein drains into the internal jugular vein (IJV), facilitating venous return from the face to the heart. Understanding this drainage is essential in clinical procedures and assessing risks of facial infections spreading to the brain via venous connections.
MCQ 2:
The angular vein is formed by the union of which veins?
a) Supratrochlear and supraorbital veins
b) Ophthalmic and infraorbital veins
c) Superficial temporal and maxillary veins
d) Facial and retromandibular veins
Answer & Explanation:
Correct answer: a) Supratrochlear and supraorbital veins.
Explanation: The angular vein is formed by the union of the supratrochlear and supraorbital veins near the medial angle of the eye. It drains into the facial vein, forming a pathway for venous blood and potential infections from the face to the cavernous sinus.
MCQ 3:
Which of the following is true about the facial vein?
a) Has valves preventing backflow
b) Only drains superficial structures
c) Is valveless
d) Drains into external jugular vein
Answer & Explanation:
Correct answer: c) Is valveless.
Explanation: The facial vein is valveless, allowing bidirectional blood flow, which increases the risk of retrograde infection spread into intracranial structures, particularly via the ophthalmic vein and cavernous sinus. This anatomical feature is critical in clinical diagnosis of facial infections.
MCQ 4 (Clinical):
Why are facial infections dangerous due to the valveless facial vein?
a) Increased blood pressure
b) Risk of infection spreading to brain
c) Impaired venous drainage
d) Blockage of lymphatic flow
Answer & Explanation:
Correct answer: b) Risk of infection spreading to brain.
Explanation: The absence of valves in the facial vein allows infections, particularly from the danger triangle of the face, to spread retrogradely to the cavernous sinus and brain, potentially causing cavernous sinus thrombosis, a serious and life-threatening condition requiring urgent medical care.
MCQ 5:
The largest vein of the face is:
a) Angular vein
b) Superficial temporal vein
c) Facial vein
d) Retromandibular vein
Answer & Explanation:
Correct answer: c) Facial vein.
Explanation: The facial vein is the largest vein in the face. It drains blood from superficial and deep structures of the face into the internal jugular vein. Its large size and valveless nature make it clinically significant, especially in facial infection and surgery cases.
MCQ 6 (Clinical):
A patient with cavernous sinus thrombosis may have which sign?
a) Facial swelling
b) Ptosis and ophthalmoplegia
c) Horner’s syndrome
d) Jaw claudication
Answer & Explanation:
Correct answer: b) Ptosis and ophthalmoplegia.
Explanation: Cavernous sinus thrombosis often results from retrograde infection spread via the valveless facial and ophthalmic veins. It presents with ptosis, ophthalmoplegia, proptosis, and potentially life-threatening complications. Recognizing this sign is critical for prompt diagnosis and treatment of intracranial infection.
MCQ 7:
Which vein connects the facial vein to the cavernous sinus?
a) Superior ophthalmic vein
b) Inferior petrosal sinus
c) External jugular vein
d) Middle cerebral vein
Answer & Explanation:
Correct answer: a) Superior ophthalmic vein.
Explanation: The superior ophthalmic vein connects the facial vein to the cavernous sinus. This venous pathway is clinically important because it allows potential retrograde spread of infections from the face to the brain, causing serious complications like cavernous sinus thrombosis.
MCQ 8 (Clinical):
What is the clinical significance of a valveless facial vein?
a) Efficient blood flow
b) Reduced venous pressure
c) Increased risk of intracranial infection
d) Prevents thrombosis
Answer & Explanation:
Correct answer: c) Increased risk of intracranial infection.
Explanation: The valveless nature of the facial vein permits bidirectional flow, which facilitates spread of infections from the face to intracranial structures, especially the cavernous sinus. This explains why infections in the danger triangle are particularly dangerous and require prompt attention.
MCQ 9:
Which is not a tributary of the facial vein?
a) Superior labial vein
b) Inferior labial vein
c) Lingual vein
d) External jugular vein
Answer & Explanation:
Correct answer: d) External jugular vein.
Explanation: The external jugular vein does not drain into the facial vein. The facial vein receives tributaries like superior and inferior labial veins and the lingual vein. The EJV drains scalp and face into the subclavian vein, unrelated to direct facial venous drainage.
MCQ 10 (Clinical):
Which anatomical region is termed the "danger triangle" of the face?
a) Frontal region
b) Cheek region
c) Nasolabial region
d) Mandibular region
Answer & Explanation:
Correct answer: c) Nasolabial region.
Explanation: The nasolabial region, known as the "danger triangle," is clinically important because infections here can spread to the brain via the valveless facial vein and ophthalmic veins, causing serious conditions like cavernous sinus thrombosis. Early recognition is vital for preventing fatal outcomes.
Topic: Cerebral Circulation
Subtopic: Internal Carotid Artery Branches
Keywords:
Internal Carotid Artery: Major artery supplying blood to the brain, entering the skull via the carotid canal.
Anterior Cerebral Artery: Terminal branch of the internal carotid artery supplying medial brain surfaces.
Middle Cerebral Artery: Largest terminal branch of internal carotid, supplying lateral brain regions.
Posterior Communicating Artery: Connects internal carotid artery to posterior cerebral artery, part of circle of Willis.
Cavernous Artery: Small branches supplying structures within the cavernous sinus, not a terminal branch.
Lead Question - 2013:
Terminal branches of internal carotid artery are all except?
a) Anterior cerebral artery
b) Middle cerebral artery
c) Posterior communicating artery
d) Cavernous artery
Answer & Explanation:
Correct answer: d) Cavernous artery.
Explanation: The terminal branches of the internal carotid artery are the anterior cerebral artery and middle cerebral artery. The posterior communicating artery is a branch but not terminal. The cavernous artery supplies structures within the cavernous sinus and is not considered a terminal branch. Accurate knowledge is crucial in vascular neurology.
MCQ 1:
Which artery is part of the circle of Willis?
a) Anterior cerebral artery
b) Middle cerebral artery
c) External carotid artery
d) Cavernous artery
Answer & Explanation:
Correct answer: a) Anterior cerebral artery.
Explanation: The anterior cerebral artery is a critical component of the circle of Willis, supplying medial brain surfaces and connecting via the anterior communicating artery. This anastomotic system provides collateral blood flow to the brain, important in cerebral circulation stability and stroke prevention.
MCQ 2:
The middle cerebral artery supplies which part of the brain?
a) Occipital lobe
b) Medial frontal lobe
c) Lateral cerebral hemisphere
d) Brainstem
Answer & Explanation:
Correct answer: c) Lateral cerebral hemisphere.
Explanation: The middle cerebral artery (MCA) is the largest terminal branch of the internal carotid artery and supplies the lateral cerebral hemispheres, including frontal, parietal, and temporal lobes. It is commonly involved in ischemic strokes, causing contralateral motor and sensory deficits.
MCQ 3:
Which artery connects the internal carotid artery to the posterior cerebral artery?
a) Anterior communicating artery
b) Posterior communicating artery
c) Middle cerebral artery
d) External carotid artery
Answer & Explanation:
Correct answer: b) Posterior communicating artery.
Explanation: The posterior communicating artery links the internal carotid artery to the posterior cerebral artery as part of the circle of Willis. It provides collateral circulation in cases of carotid or vertebral artery stenosis, essential for maintaining cerebral perfusion under pathological conditions.
MCQ 4 (Clinical):
Occlusion of which artery causes contralateral motor and sensory deficits?
a) Anterior cerebral artery
b) Middle cerebral artery
c) Posterior communicating artery
d) Cavernous artery
Answer & Explanation:
Correct answer: b) Middle cerebral artery.
Explanation: Middle cerebral artery (MCA) occlusion leads to contralateral motor and sensory deficits, especially in the face and upper limb, along with aphasia if dominant hemisphere is involved. Rapid identification is crucial in stroke management to prevent permanent neurological damage.
MCQ 5:
The anterior cerebral artery primarily supplies which region?
a) Lateral temporal lobe
b) Medial frontal and parietal lobes
c) Occipital lobe
d) Brainstem
Answer & Explanation:
Correct answer: b) Medial frontal and parietal lobes.
Explanation: The anterior cerebral artery (ACA) supplies the medial surfaces of the frontal and parietal lobes. ACA stroke may cause weakness in the contralateral lower limb and urinary incontinence, making its clinical identification important in cerebrovascular diagnosis.
MCQ 6 (Clinical):
A patient has reduced blood flow in the circle of Willis. Which artery provides collateral flow?
a) Posterior communicating artery
b) External carotid artery
c) Superior thyroid artery
d) Cavernous artery
Answer & Explanation:
Correct answer: a) Posterior communicating artery.
Explanation: The posterior communicating artery forms a critical part of the circle of Willis, allowing collateral circulation between the anterior and posterior circulations. In cases of internal carotid or vertebrobasilar artery stenosis, it ensures adequate cerebral perfusion, preventing ischemic damage.
MCQ 7:
Which of the following is not a branch of the internal carotid artery?
a) Ophthalmic artery
b) Anterior choroidal artery
c) Middle cerebral artery
d) Facial artery
Answer & Explanation:
Correct answer: d) Facial artery.
Explanation: The facial artery arises from the external carotid artery and supplies superficial facial structures. The internal carotid artery primarily supplies intracranial structures, giving branches like the ophthalmic, anterior choroidal, and middle cerebral arteries. Understanding these origins is important in vascular and surgical anatomy.
MCQ 8 (Clinical):
Posterior communicating artery aneurysm typically affects which nerve?
a) Optic nerve (CN II)
b) Oculomotor nerve (CN III)
c) Trigeminal nerve (CN V)
d) Hypoglossal nerve (CN XII)
Answer & Explanation:
Correct answer: b) Oculomotor nerve (CN III).
Explanation: Aneurysm of the posterior communicating artery often compresses the oculomotor nerve, causing ptosis, diplopia, and pupil dilation (blown pupil). Recognizing these clinical signs is crucial in diagnosing intracranial aneurysms, facilitating timely neurosurgical intervention.
MCQ 9:
The ophthalmic artery is a branch of which artery?
a) External carotid artery
b) Internal carotid artery
c) Vertebral artery
d) Subclavian artery
Answer & Explanation:
Correct answer: b) Internal carotid artery.
Explanation: The ophthalmic artery is the first major branch of the internal carotid artery after it enters the cranial cavity. It supplies the eye and orbit, and its compromise may cause visual loss, making it essential in ophthalmologic and neurologic assessments.
MCQ 10 (Clinical):
In carotid artery dissection, which artery is primarily involved?
a) Internal carotid artery
b) External carotid artery
c) Subclavian artery
d) Vertebral artery
Answer & Explanation:
Correct answer: a) Internal carotid artery.
Explanation: Carotid artery dissection often involves the internal carotid artery, leading to stenosis or pseudoaneurysm formation. It can present with ipsilateral headache, Horner’s syndrome, and ischemic stroke. Early diagnosis via imaging and anticoagulation or surgery is vital to prevent complications.
Chapter: Anatomy
Topic: Hepatobiliary System
Subtopic: Portal Vein
Keyword Definitions:
Portal vein: Vein formed by confluence of superior mesenteric and splenic veins, carrying nutrient-rich blood from GI tract to liver.
Hepatic portal circulation: Circulatory system delivering venous blood from intestines, pancreas, spleen, and stomach to liver for metabolism and detoxification.
Splenic vein: Drains spleen, pancreas, and parts of stomach, contributes to portal vein.
Superior mesenteric vein: Drains small intestine, cecum, ascending and transverse colon, joins splenic vein to form portal vein.
Clinical relevance: Portal vein thrombosis or hypertension affects liver function, can cause varices and ascites.
Liver: Receives ~75% of blood via portal vein, ~25% via hepatic artery.
Spleen: Drains into splenic vein, not supplied directly by portal vein.
Lead Question - 2013
Portal vein supplies ?
a) Spleen
b) Liver
c) Pancreas
d) Colon
Explanation: The portal vein carries nutrient-rich venous blood from gastrointestinal organs and spleen to the liver for detoxification and metabolism. It does not supply the spleen, colon, or pancreas directly. Correct answer is b) Liver.
Guessed Question 2
Portal vein is formed by?
a) Superior mesenteric + splenic veins
b) Inferior mesenteric + splenic veins
c) Superior mesenteric + inferior mesenteric veins
d) Hepatic veins
Explanation: Portal vein is formed by the union of the superior mesenteric and splenic veins behind the neck of pancreas. This vein carries blood to the liver. Correct answer is a) Superior mesenteric + splenic veins.
Guessed Question 3
Portal vein carries blood from all except?
a) Spleen
b) Pancreas
c) Kidneys
d) Stomach
Explanation: Portal vein collects blood from GI organs, spleen, and pancreas, but not kidneys. Renal veins drain directly into inferior vena cava. Correct answer is c) Kidneys.
Guessed Question 4
Which vein drains into portal vein?
a) Splenic vein
b) Renal vein
c) Femoral vein
d) Brachial vein
Explanation: Splenic vein drains spleen and joins superior mesenteric vein to form portal vein. Other veins drain into systemic circulation. Correct answer is a) Splenic vein.
Guessed Question 5
Portal vein enters liver at?
a) Porta hepatis
b) Hepatic hilum
c) Both a and b
d) Falciform ligament
Explanation: Portal vein enters the liver at the porta hepatis (hepatic hilum), along with hepatic artery and bile duct. Correct answer is c) Both a and b.
Guessed Question 6
Clinical significance of portal vein thrombosis?
a) Portal hypertension
b) Varices
c) Ascites
d) All of the above
Explanation: Thrombosis of the portal vein obstructs venous flow to the liver, causing portal hypertension, esophageal varices, splenomegaly, and ascites. Correct answer is d) All of the above.
Guessed Question 7
Which artery supplies liver along with portal vein?
a) Hepatic artery
b) Celiac artery
c) Splenic artery
d) Superior mesenteric artery
Explanation: Liver receives blood from portal vein (~75%) and hepatic artery (~25%). Hepatic artery arises from celiac trunk. Correct answer is a) Hepatic artery.
Guessed Question 8
Portal venous blood is rich in?
a) Nutrients
b) Oxygen
c) Hormones
d) Lymph
Explanation: Portal vein carries nutrient-rich blood from intestines, stomach, spleen, and pancreas to liver for metabolism. Oxygen content is less than arterial blood. Correct answer is a) Nutrients.
Guessed Question 9
Which structure is not part of hepatic portal system?
a) Superior mesenteric vein
b) Splenic vein
c) Inferior vena cava
d) Portal vein
Explanation: Inferior vena cava is part of systemic venous circulation, not portal system. Portal system includes portal vein and tributaries. Correct answer is c) Inferior vena cava.
Guessed Question 10
Which organ directly receives portal venous blood?
a) Liver
b) Spleen
c) Pancreas
d) Stomach
Explanation: Portal venous blood is delivered to the liver for metabolism, detoxification, and storage of nutrients. Other organs drain blood into portal vein but do not receive it. Correct answer is a) Liver.
Guessed Question 11
Portal vein is posterior to which structure?
a) Pancreas
b) Duodenum
c) Stomach
d) Gallbladder
Explanation: Portal vein is formed posterior to the neck of pancreas by superior mesenteric and splenic veins. Correct answer is a) Pancreas.
Chapter: Anatomy
Topic: Pelvic Vessels
Subtopic: Obturator Artery Variations
Keyword Definitions:
Obturator artery: Branch of internal iliac artery supplying medial thigh, hip joint, and obturator foramen structures.
Aberrant obturator artery: Variant artery usually arising from external iliac or inferior epigastric artery instead of internal iliac.
Inferior epigastric artery: Branch of external iliac artery supplying anterior abdominal wall, sometimes gives rise to aberrant obturator artery.
Profunda femoris artery: Deep femoral artery, major branch of femoral artery supplying thigh muscles.
Clinical relevance: Aberrant obturator artery crosses superior pubic ramus, risk of severe hemorrhage during hernia or pelvic surgery.
External iliac artery: Continuation of common iliac artery, becomes femoral artery after inguinal ligament.
Herniorrhaphy risk: Aberrant obturator artery injury can occur in laparoscopic or open inguinal hernia repair.
Lead Question - 2013
In case of aberrant obturator artery, it arises most commonly from ?
a) Common iliac artery
b) Femoral artery
c) Profunda femoris artery
d) Inferior epigastric artery
Explanation: The aberrant obturator artery most commonly arises from the inferior epigastric artery, a branch of the external iliac artery. Knowledge of this variant is important to prevent hemorrhage during inguinal hernia repair, pelvic, or orthopedic surgeries. Correct answer is d) Inferior epigastric artery.
Guessed Question 2
Normal obturator artery arises from?
a) Internal iliac artery
b) External iliac artery
c) Femoral artery
d) Profunda femoris
Explanation: Normally, obturator artery arises from the internal iliac artery, supplying medial thigh and hip structures. Variations like aberrant obturator artery increase surgical risk. Correct answer is a) Internal iliac artery.
Guessed Question 3
Obturator artery supplies all except?
a) Hip joint
b) Adductor muscles
c) Quadriceps
d) Obturator foramen region
Explanation: Obturator artery supplies hip joint, adductor muscles, and obturator region, but quadriceps are supplied mainly by femoral artery branches. Correct answer is c) Quadriceps.
Guessed Question 4
Which artery is at risk in laparoscopic inguinal hernia repair?
a) Aberrant obturator artery
b) Internal pudendal
c) Inferior mesenteric
d) Superior gluteal
Explanation: Aberrant obturator artery runs close to superior pubic ramus and may be injured during laparoscopic or open inguinal hernia repair. Surgeons must identify its course to prevent hemorrhage. Correct answer is a) Aberrant obturator artery.
Guessed Question 5
Inferior epigastric artery arises from?
a) External iliac artery
b) Internal iliac artery
c) Femoral artery
d) Profunda femoris
Explanation: Inferior epigastric artery originates from external iliac artery, ascends along abdominal wall, and may give rise to aberrant obturator artery. Correct answer is a) External iliac artery.
Guessed Question 6
Which variant artery crosses superior pubic ramus?
a) Aberrant obturator artery
b) Internal iliac artery
c) Inferior gluteal artery
d) Femoral artery
Explanation: Aberrant obturator artery passes over superior pubic ramus, making it vulnerable during pelvic surgery. Knowledge of its presence is critical for surgical safety. Correct answer is a) Aberrant obturator artery.
Guessed Question 7
Profunda femoris artery supplies?
a) Thigh muscles
b) Abdomen
c) Pelvis
d) Perineum
Explanation: Profunda femoris artery is the deep femoral artery supplying thigh muscles via perforating branches. It rarely gives rise to aberrant obturator artery. Correct answer is a) Thigh muscles.
Guessed Question 8
Common iliac artery divides into?
a) Internal and external iliac arteries
b) Femoral and profunda femoris
c) Superior and inferior epigastric
d) Obturator and femoral
Explanation: Common iliac artery bifurcates into internal and external iliac arteries. External iliac continues as femoral artery, while internal iliac supplies pelvis, including obturator artery. Correct answer is a) Internal and external iliac arteries.
Guessed Question 9
During hernia repair, which artery is most likely injured?
a) Aberrant obturator artery
b) Femoral artery
c) Internal pudendal artery
d) Inferior mesenteric artery
Explanation: Aberrant obturator artery runs close to pubic bone and is prone to injury during hernia repair, potentially causing severe bleeding. Correct answer is a) Aberrant obturator artery.
Guessed Question 10
Which artery forms anastomosis with obturator artery?
a) Inferior epigastric artery
b) Femoral artery
c) Profunda femoris artery
d) Internal pudendal artery
Explanation: Inferior epigastric artery can form anastomosis with obturator artery; this is clinically important in cases of aberrant origin and collateral circulation during surgery. Correct answer is a) Inferior epigastric artery.
Guessed Question 11
Artery variant most relevant in pelvic surgery?
a) Aberrant obturator artery
b) Superior gluteal artery
c) Internal pudendal artery
d) Median sacral artery
Explanation: Aberrant obturator artery is clinically significant in pelvic and hernia surgeries due to its variant origin and course, which may lead to unexpected hemorrhage if unrecognized. Correct answer is a) Aberrant obturator artery.
Chapter: Anatomy
Topic: Thorax
Subtopic: Anterior Intercostal Arteries
Keyword Definitions:
Intercostal arteries: Arteries that supply the thoracic wall including muscles, skin, and parietal pleura.
Anterior intercostal arteries: Branches of internal thoracic and musculophrenic arteries, present in upper nine intercostal spaces.
Posterior intercostal arteries: Branches of thoracic aorta (except 1st and 2nd from superior intercostal artery).
Internal thoracic artery: Branch of subclavian artery, gives anterior intercostal and musculophrenic arteries.
Musculophrenic artery: Terminal branch of internal thoracic artery, supplies lower anterior intercostal arteries.
Azygos system: Venous system draining posterior intercostal veins into SVC.
Clinical relevance: Important in chest surgeries, flap reconstruction, and thoracostomies.
Lead Question - 2013
True about anterior intercostal artery ?
a) Present in 1st to 11th intercostal space
b) Each intercostal space has two anterior intercostal arteries
c) Branch of internal thoracic artery
d) Branch of aorta
Explanation: Anterior intercostal arteries are branches of the internal thoracic and musculophrenic arteries. They are present only in upper nine intercostal spaces, with two arteries per space. They do not arise from the aorta. Correct answers are b) Each intercostal space has two anterior intercostal arteries and c) Branch of internal thoracic artery.
Guessed Question 2
Anterior intercostal arteries are absent in which spaces?
a) 1st and 2nd
b) 9th to 11th
c) 7th and 8th
d) Only 11th
Explanation: Anterior intercostal arteries are present in the first nine spaces. They are absent in the 10th and 11th intercostal spaces because these are “floating ribs” with no anterior articulation. Correct answer is b) 9th to 11th.
Guessed Question 3
Which artery gives rise to anterior intercostal arteries of upper six spaces?
a) Musculophrenic artery
b) Thoracic aorta
c) Internal thoracic artery
d) Superior intercostal artery
Explanation: The internal thoracic artery, a branch of subclavian, gives off anterior intercostal arteries for the first six spaces before dividing into musculophrenic and superior epigastric arteries. Correct answer is c) Internal thoracic artery.
Guessed Question 4
Anterior intercostal arteries of 7th–9th spaces are branches of?
a) Internal thoracic artery
b) Musculophrenic artery
c) Aorta
d) Superior intercostal artery
Explanation: The musculophrenic artery, a terminal branch of internal thoracic artery, supplies the anterior intercostal arteries of the 7th–9th intercostal spaces. Correct answer is b) Musculophrenic artery.
Guessed Question 5
Which intercostal spaces have both anterior and posterior intercostal arteries?
a) 1st–9th
b) 1st–11th
c) Only upper six
d) Only lower six
Explanation: Spaces 1st–9th have both anterior and posterior intercostal arteries. The 10th and 11th spaces have only posterior intercostal arteries due to absence of anterior ends. Correct answer is a) 1st–9th.
Guessed Question 6
Anterior intercostal arteries anastomose with?
a) Coronary arteries
b) Posterior intercostal arteries
c) Bronchial arteries
d) Phrenic arteries
Explanation: Anterior intercostal arteries anastomose with posterior intercostal arteries in each intercostal space, ensuring collateral circulation in thoracic wall. Correct answer is b) Posterior intercostal arteries.
Guessed Question 7
Internal thoracic artery terminates as?
a) Superior epigastric and musculophrenic arteries
b) Anterior intercostal arteries
c) Bronchial and phrenic arteries
d) Superior thyroid and vertebral arteries
Explanation: The internal thoracic artery ends by dividing into musculophrenic and superior epigastric arteries at the level of 6th intercostal space. Correct answer is a) Superior epigastric and musculophrenic arteries.
Guessed Question 8
During CABG (coronary artery bypass graft), which artery is most commonly used?
a) Internal thoracic artery
b) Radial artery
c) Femoral artery
d) Inferior epigastric artery
Explanation: The internal thoracic artery is commonly harvested for CABG due to its excellent long-term patency rates. Correct answer is a) Internal thoracic artery.
Guessed Question 9
Which nerve accompanies anterior intercostal arteries?
a) Vagus nerve
b) Intercostal nerve
c) Phrenic nerve
d) Sympathetic chain
Explanation: Anterior intercostal arteries run along with intercostal nerves and veins in the costal groove between internal and innermost intercostal muscles. Correct answer is b) Intercostal nerve.
Guessed Question 10
Which artery supplies anterior intercostal arteries in upper six spaces?
a) Internal thoracic artery
b) Aorta
c) Superior intercostal artery
d) Axillary artery
Explanation: The internal thoracic artery supplies anterior intercostal arteries for the upper six intercostal spaces before bifurcating. Correct answer is a) Internal thoracic artery.
Guessed Question 11
Anterior intercostal arteries are clinically important in?
a) Drainage of pleural effusion
b) CABG grafting and chest wall surgeries
c) Tracheostomy
d) Vertebral surgery
Explanation: Anterior intercostal arteries are vital in thoracic wall reconstruction, mastectomy, and as collaterals in coronary bypass grafting using internal thoracic artery. Correct answer is b) CABG grafting and chest wall surgeries.
Chapter: Anatomy
Topic: Thorax
Subtopic: Intercostal Arteries
Keyword Definitions:
Intercostal arteries: Arteries supplying muscles, skin, and parietal pleura of thoracic wall.
Posterior intercostal arteries: Branches mainly from thoracic aorta, except upper two from superior intercostal artery.
Anterior intercostal arteries: Branches of internal thoracic (mammary) artery and musculophrenic artery.
Superior intercostal artery: Branch of costocervical trunk from subclavian artery, supplies 1st and 2nd spaces.
Collateral circulation: Network formed between anterior and posterior intercostal arteries.
Clinical relevance: Important in thoracotomy, rib fractures, and chest tube insertion.
Lead Question - 2013
Upper two posterior intercostal arteries arise from ?
a) Aorta
b) Superior intercostal artery
c) Internal mammary artery
d) Bronchial artery
Explanation: The first and second posterior intercostal arteries originate from the superior intercostal artery, a branch of the costocervical trunk of the subclavian artery. The remaining posterior intercostal arteries arise from thoracic aorta. Correct answer is b) Superior intercostal artery.
Guessed Question 2
Most posterior intercostal arteries arise from?
a) Aorta
b) Subclavian artery
c) Internal thoracic artery
d) Vertebral artery
Explanation: Except for the first two, posterior intercostal arteries (3rd–11th) arise directly from the thoracic aorta, ensuring supply to thoracic wall. Correct answer is a) Aorta.
Guessed Question 3
Anterior intercostal arteries are branches of?
a) Thoracic aorta
b) Internal thoracic artery
c) Superior intercostal artery
d) Subclavian vein
Explanation: The anterior intercostal arteries arise from the internal thoracic artery in the upper spaces and from musculophrenic artery in lower spaces. Correct answer is b) Internal thoracic artery.
Guessed Question 4
Which vessel supplies the first two intercostal spaces posteriorly?
a) Thoracic aorta
b) Superior intercostal artery
c) Musculophrenic artery
d) Internal thoracic artery
Explanation: The superior intercostal artery, a branch of costocervical trunk from subclavian, supplies the first two posterior intercostal spaces. Correct answer is b) Superior intercostal artery.
Guessed Question 5
Intercostal arteries lie between?
a) External and internal intercostals
b) Internal and innermost intercostals
c) Pleura and innermost intercostals
d) Skin and external intercostals
Explanation: The intercostal neurovascular bundle (vein, artery, nerve) runs in the costal groove between internal and innermost intercostal muscles. Correct answer is b) Internal and innermost intercostals.
Guessed Question 6
The costocervical trunk is a branch of?
a) Aorta
b) Subclavian artery
c) Internal thoracic artery
d) Axillary artery
Explanation: The costocervical trunk is a branch of the subclavian artery that gives rise to the superior intercostal artery and deep cervical artery. Correct answer is b) Subclavian artery.
Guessed Question 7
The largest branch of posterior intercostal artery is?
a) Dorsal branch
b) Collateral branch
c) Perforating branch
d) Musculophrenic branch
Explanation: Each posterior intercostal artery gives a large dorsal branch that supplies spinal cord, vertebrae, and muscles of back. Correct answer is a) Dorsal branch.
Guessed Question 8
Internal thoracic artery is a branch of?
a) Aorta
b) Subclavian artery
c) Thoracic aorta
d) Axillary artery
Explanation: The internal thoracic (mammary) artery is a direct branch of the subclavian artery, descending along sternum inside thoracic wall. Correct answer is b) Subclavian artery.
Guessed Question 9
During chest tube insertion, intercostal arteries are avoided by placing tube?
a) Along upper border of rib
b) Along lower border of rib
c) Directly in midrib
d) Over sternum
Explanation: The intercostal neurovascular bundle runs along the inferior border of ribs, so procedures like chest tube insertion are performed just above the upper border of rib. Correct answer is a) Along upper border of rib.
Guessed Question 10
Anterior intercostal arteries for lower spaces arise from?
a) Musculophrenic artery
b) Internal thoracic artery
c) Aorta
d) Superior intercostal artery
Explanation: The musculophrenic artery, a terminal branch of internal thoracic artery, supplies anterior intercostal arteries for lower intercostal spaces. Correct answer is a) Musculophrenic artery.
Guessed Question 11
Intercostal space is drained by which veins posteriorly?
a) Azygos and hemiazygos system
b) Pulmonary veins
c) Vertebral veins
d) Coronary sinus
Explanation: Posterior intercostal veins drain into the azygos vein on right and hemiazygos/accessory hemiazygos veins on left, ultimately into superior vena cava. Correct answer is a) Azygos and hemiazygos system.
Chapter: Anatomy
Topic: Thorax
Subtopic: Anterior Relations of Heart
Keyword Definitions:
Sternocostal surface: The anterior surface of the heart facing the sternum and ribs.
Right ventricle: Chamber that pumps deoxygenated blood into the pulmonary trunk, forms anterior surface.
Left ventricle: Pumps oxygenated blood to systemic circulation, forms apex and left border.
Right atrium: Receives venous blood from SVC, IVC, and coronary sinus, forms right border.
Left atrium: Receives pulmonary veins, forms posterior surface, closely related to esophagus.
Clinical relevance: Knowledge of surfaces helps in trauma assessment, imaging, and surgical approaches.
Lead Question - 2013
Posterior to sternum is?
a) Left atrium
b) Left ventricle
c) Right atrium
d) Right ventricle
Explanation: The sternocostal (anterior) surface of the heart lies directly behind the sternum and is formed mainly by the right ventricle, with a small contribution from the right atrium. Thus, the correct answer is d) Right ventricle.
Guessed Question 2
The posterior surface of the heart is formed by?
a) Right ventricle
b) Left atrium
c) Right atrium
d) Left ventricle
Explanation: The posterior surface or base of the heart is formed mainly by the left atrium, which is in relation to the esophagus. The correct answer is b) Left atrium.
Guessed Question 3
The apex of the heart is contributed by?
a) Right ventricle
b) Left ventricle
c) Right atrium
d) Left atrium
Explanation: The apex beat palpable in the left 5th intercostal space is formed by the left ventricle. The correct answer is b) Left ventricle.
Guessed Question 4
The right border of the heart is mainly formed by?
a) Right atrium
b) Left atrium
c) Right ventricle
d) Left ventricle
Explanation: The right atrium forms the right cardiac border visible on chest X-ray. The correct answer is a) Right atrium.
Guessed Question 5
The left border of the heart in chest X-ray is formed by?
a) Left ventricle
b) Right ventricle
c) Right atrium
d) Right ventricle and atrium
Explanation: The left border of the cardiac shadow is formed mainly by the left ventricle, with a small contribution from the left auricle. The correct answer is a) Left ventricle.
Guessed Question 6
The inferior border of the heart is formed largely by?
a) Left ventricle
b) Right ventricle
c) Right atrium
d) Left atrium
Explanation: The inferior border or diaphragmatic surface of the heart is formed largely by the right ventricle with a small contribution from the left ventricle. The correct answer is b) Right ventricle.
Guessed Question 7
Which chamber is most closely related to the esophagus?
a) Left atrium
b) Left ventricle
c) Right ventricle
d) Right atrium
Explanation: The left atrium forms the posterior surface of the heart and lies directly in front of the esophagus. Enlargement of LA can cause dysphagia. The correct answer is a) Left atrium.
Guessed Question 8
The anterior surface of the heart is clinically significant because?
a) It is mostly formed by right ventricle, vulnerable to trauma
b) It is mostly formed by left atrium, related to esophagus
c) It is mostly formed by left ventricle, vulnerable to infarction
d) It is protected by the spine
Explanation: The anterior surface is mainly formed by the right ventricle, which is the most vulnerable chamber in penetrating chest trauma. The correct answer is a) It is mostly formed by right ventricle, vulnerable to trauma.
Guessed Question 9
In mitral stenosis, which chamber enlarges most prominently?
a) Right ventricle
b) Left atrium
c) Right atrium
d) Left ventricle
Explanation: Mitral stenosis causes obstruction to left atrial emptying, leading to marked enlargement of the left atrium. The correct answer is b) Left atrium.
Guessed Question 10
Which chamber is most anterior in location?
a) Right ventricle
b) Left atrium
c) Left ventricle
d) Right atrium
Explanation: The right ventricle lies directly behind the sternum and is the most anterior chamber of the heart. The correct answer is a) Right ventricle.
Guessed Question 11
Which chamber is most posterior in location?
a) Right atrium
b) Left atrium
c) Left ventricle
d) Right ventricle
Explanation: The left atrium is the most posterior chamber, lying directly in front of the esophagus in the posterior mediastinum. The correct answer is b) Left atrium.
Chapter: Anatomy
Topic: Thorax
Subtopic: Heart Surfaces and Borders
Keyword Definitions:
Sternocostal (anterior) surface: Surface of the heart facing sternum and ribs, mainly right ventricle.
Atrium: Right atrium receives systemic venous blood, left atrium receives pulmonary venous blood.
Ventricle: Right ventricle pumps blood to pulmonary circulation, left ventricle to systemic circulation.
Auricle: Small muscular pouch projecting from atrium, increases volume capacity.
Clinical relevance: Knowledge of heart surfaces is essential for interpretation of X-rays, echocardiograms, and cardiac interventions.
Lead Question - 2013
Anterosuperior sternal part of heart is made up of?
a) Right atrium and auricle
b) Left atrium
c) Left ventricle
d) Right ventricle
Explanation: The sternocostal (anterior) surface of the heart is mainly formed by the right ventricle, with a small contribution from the right atrium and auricle. Thus, the correct answer is d) Right ventricle.
Guessed Question 2
The diaphragmatic surface of the heart is formed primarily by?
a) Left atrium
b) Left ventricle
c) Right atrium
d) Right ventricle
Explanation: The diaphragmatic surface of the heart rests on the diaphragm and is formed mainly by the left ventricle with some contribution from the right ventricle. The correct answer is b) Left ventricle.
Guessed Question 3
The posterior surface of the heart is mainly formed by?
a) Left atrium
b) Right atrium
c) Left ventricle
d) Right ventricle
Explanation: The posterior surface (base) of the heart is formed mainly by the left atrium, with minor contribution from the right atrium. The correct answer is a) Left atrium.
Guessed Question 4
The apex of the heart is contributed by?
a) Right ventricle
b) Left ventricle
c) Right atrium
d) Left atrium
Explanation: The apex beat, felt in the left 5th intercostal space, is formed by the left ventricle. The correct answer is b) Left ventricle.
Guessed Question 5
The right border of the heart seen on chest X-ray is formed by?
a) Right atrium
b) Left atrium
c) Right ventricle
d) Left ventricle
Explanation: The right border of the cardiac shadow in a chest X-ray is formed by the right atrium. The correct answer is a) Right atrium.
Guessed Question 6
The left border of the heart seen on chest X-ray is formed by?
a) Right ventricle
b) Left ventricle
c) Right atrium
d) Left atrium
Explanation: The left border of the cardiac shadow on chest radiograph is formed mainly by the left ventricle, with a small contribution by the left auricle. The correct answer is b) Left ventricle.
Guessed Question 7
The inferior border of the heart is formed mainly by?
a) Right ventricle
b) Left ventricle
c) Right atrium
d) Left atrium
Explanation: The inferior (acute) border of the heart is formed largely by the right ventricle with a small contribution by the left ventricle. The correct answer is a) Right ventricle.
Guessed Question 8
Enlargement of which chamber causes a prominent bulge in the right border of cardiac shadow?
a) Left ventricle
b) Left atrium
c) Right ventricle
d) Right atrium
Explanation: Right atrial enlargement causes prominence of the right heart border on X-ray. The correct answer is d) Right atrium.
Guessed Question 9
Enlargement of which chamber is most likely to compress the esophagus?
a) Right atrium
b) Right ventricle
c) Left atrium
d) Left ventricle
Explanation: The left atrium lies immediately in front of the esophagus, and its enlargement can cause dysphagia. The correct answer is c) Left atrium.
Guessed Question 10
The anterior surface of the heart is clinically important because?
a) It is mainly formed by right ventricle, site of trauma injury
b) It is formed by left ventricle, site of infarction
c) It is formed by atria only
d) It is protected by esophagus
Explanation: The anterior surface is mainly formed by the right ventricle, which is vulnerable to trauma and penetrating injuries. The correct answer is a) It is mainly formed by right ventricle, site of trauma injury.
Guessed Question 11
The posterior surface of the heart is closely related to?
a) Trachea
b) Esophagus
c) Sternum
d) Diaphragm
Explanation: The posterior surface of the heart is closely related to the esophagus, which lies in the posterior mediastinum. The correct answer is b) Esophagus.
Chapter: Anatomy
Topic: Thorax
Subtopic: Heart Surfaces and Relations
Keyword Definitions:
Base of heart: Posterior surface formed mainly by left atrium, partly by right atrium.
Atrium: Upper chamber of the heart; right atrium receives venous blood, left atrium receives pulmonary venous blood.
Ventricle: Lower chamber of the heart; right ventricle pumps to lungs, left ventricle to systemic circulation.
Posterior mediastinum: Part of thorax behind the heart containing esophagus and great vessels.
Clinical relevance: Knowledge of cardiac surfaces is essential in imaging, echocardiography, and surgical approaches.
Lead Question - 2013
Posterior surface of heart is formed by -
a) RA
b) LA
c) LV
d) RV
Explanation: The posterior surface of the heart, also called the base, is formed mainly by the left atrium, which receives pulmonary veins. The right atrium contributes slightly. Thus, the correct answer is b) LA.
Guessed Question 2
The diaphragmatic surface of the heart is formed mainly by?
a) Right atrium
b) Right ventricle
c) Left ventricle
d) Left atrium
Explanation: The diaphragmatic surface of the heart rests on the central tendon of the diaphragm and is formed mainly by the left ventricle with a small contribution from the right ventricle. The correct answer is c) Left ventricle.
Guessed Question 3
The right border of the heart is formed by?
a) Right atrium
b) Right ventricle
c) Left atrium
d) Left ventricle
Explanation: The right border of the heart seen on chest X-ray is formed by the right atrium, which receives venous blood from the SVC and IVC. The correct answer is a) Right atrium.
Guessed Question 4
The apex of the heart is formed by?
a) Right ventricle
b) Left ventricle
c) Left atrium
d) Right atrium
Explanation: The apex beat, palpable in the left 5th intercostal space, is formed by the tip of the left ventricle. The correct answer is b) Left ventricle.
Guessed Question 5
Which chamber of the heart lies closest to the esophagus?
a) Right atrium
b) Left atrium
c) Right ventricle
d) Left ventricle
Explanation: The left atrium forms the posterior surface of the heart and lies directly in front of the esophagus in the posterior mediastinum. The correct answer is b) Left atrium.
Guessed Question 6
On a lateral chest X-ray, the posterior border of the heart is formed by?
a) Right ventricle
b) Left ventricle
c) Left atrium
d) Right atrium
Explanation: In lateral chest radiographs, the posterior border of the heart is formed by the left atrium and left ventricle. The correct answer is c) Left atrium.
Guessed Question 7
Which chamber enlargement causes dysphagia by compressing esophagus?
a) Right atrium
b) Left atrium
c) Left ventricle
d) Right ventricle
Explanation: Enlargement of the left atrium can compress the esophagus, leading to dysphagia. This is often seen in mitral stenosis. The correct answer is b) Left atrium.
Guessed Question 8
The anterior surface of the heart is mainly formed by?
a) Left atrium
b) Right ventricle
c) Left ventricle
d) Right atrium
Explanation: The sternocostal or anterior surface of the heart is formed mainly by the right ventricle. The correct answer is b) Right ventricle.
Guessed Question 9
Which chamber of the heart contributes most to the left border in chest X-ray?
a) Right ventricle
b) Left ventricle
c) Right atrium
d) Left atrium
Explanation: The left border of the cardiac shadow on chest X-ray is formed mainly by the left ventricle. The correct answer is b) Left ventricle.
Guessed Question 10
Which cardiac chamber is most posterior in location?
a) Right atrium
b) Right ventricle
c) Left atrium
d) Left ventricle
Explanation: The left atrium forms the base of the heart and is the most posteriorly placed chamber. The correct answer is c) Left atrium.
Guessed Question 11
Atrial fibrillation commonly originates due to abnormal conduction from which heart chamber?
a) Right atrium
b) Left atrium
c) Right ventricle
d) Left ventricle
Explanation: Atrial fibrillation often originates from ectopic foci near the openings of pulmonary veins in the left atrium. The correct answer is b) Left atrium.
Keyword Definitions
• Midinguinal point – Midpoint between anterior superior iliac spine and pubic symphysis; landmark for femoral artery.
• Adductor tubercle – Bony prominence on medial femoral condyle; insertion point for adductor magnus.
• Femoral artery – Main artery of thigh, continuation of external iliac artery.
• Inferior epigastric artery – Branch of external iliac artery, runs upward in anterior abdominal wall.
• Superior epigastric artery – Continuation of internal thoracic artery, supplies anterior abdominal wall.
• Adductor canal (Hunter’s canal) – Intermuscular tunnel through which femoral artery passes before becoming popliteal artery.
• Popliteal artery – Continuation of femoral artery after adductor hiatus.
• Femoral triangle – Triangular space in upper thigh, containing femoral nerve, artery, vein.
• Surface marking – Clinical method of tracing arteries/veins along landmarks.
• Aneurysm – Pathological dilatation of artery, can occur in femoral artery.
Chapter: Anatomy / Lower Limb
Topic: Femoral Region
Subtopic: Surface Marking of Femoral Artery
Lead Question – 2013
Line from midinguinal point to adductor tubercle represent?
a) Inferior epigastric artery
b) Femoral artery
c) Superior epigastric artery
d) None of the above
Explanation: The surface marking of the femoral artery is a line drawn from the midinguinal point to the adductor tubercle. It helps in palpating pulse and guiding surgical procedures. Correct answer: Femoral artery.
Guessed Questions for NEET PG
1) Which structure passes through the adductor hiatus with femoral artery?
a) Femoral vein
b) Femoral nerve
c) Great saphenous vein
d) Obturator nerve
Explanation: The femoral vein accompanies the femoral artery through the adductor hiatus, where they become the popliteal vessels. Correct answer: Femoral vein.
2) Which is not a content of femoral triangle?
a) Femoral nerve
b) Femoral artery
c) Femoral vein
d) Obturator artery
Explanation: The femoral triangle contains femoral nerve, artery, vein, and lymphatics. Obturator artery lies deeper and is not a content. Correct answer: Obturator artery.
3) Which artery is palpated at the midinguinal point?
a) Femoral artery
b) Popliteal artery
c) Posterior tibial artery
d) Dorsalis pedis artery
Explanation: The femoral artery pulse is palpated at the midinguinal point, a key clinical landmark in lower limb examination. Correct answer: Femoral artery.
4) Adductor canal transmits all except:
a) Femoral artery
b) Femoral vein
c) Saphenous nerve
d) Femoral nerve
Explanation: Femoral nerve does not pass through the adductor canal; only its saphenous branch does. Correct answer: Femoral nerve.
5) Which branch of femoral artery supplies head of femur?
a) Profunda femoris
b) Medial circumflex femoral
c) Lateral circumflex femoral
d) Superior gluteal
Explanation: The medial circumflex femoral artery (branch of profunda femoris) supplies most of the femoral head through retinacular branches. Correct answer: Medial circumflex femoral artery.
6) Which artery is continuation of femoral artery beyond adductor hiatus?
a) Popliteal artery
b) Anterior tibial artery
c) Posterior tibial artery
d) Peroneal artery
Explanation: The femoral artery continues as popliteal artery after passing through adductor hiatus. Correct answer: Popliteal artery.
7) Which structure lies medial to femoral artery in femoral sheath?
a) Femoral nerve
b) Femoral vein
c) Great saphenous vein
d) Inguinal ligament
Explanation: Within femoral sheath, femoral vein lies medial to femoral artery. Correct answer: Femoral vein.
8) Profunda femoris artery is a branch of:
a) Femoral artery
b) External iliac artery
c) Internal iliac artery
d) Obturator artery
Explanation: The profunda femoris artery arises from the femoral artery, 3–5 cm below the inguinal ligament. Correct answer: Femoral artery.
9) Which clinical test assesses patency of femoral artery?
a) Trendelenburg test
b) Allen’s test
c) Palpation of femoral pulse
d) Buerger’s test
Explanation: The femoral artery can be directly assessed by palpating its pulse at the midinguinal point. Correct answer: Palpation of femoral pulse.
10) Which nerve accompanies femoral artery in adductor canal?
a) Femoral nerve
b) Saphenous nerve
c) Obturator nerve
d) Tibial nerve
Explanation: The saphenous nerve, a branch of femoral nerve, accompanies femoral artery in the adductor canal. Correct answer: Saphenous nerve.
Chapter: Anatomy
Topic: Lower Limb Vessels
Subtopic: Branches of Femoral Artery
Keyword Definitions:
Superficial External Pudendal Artery: Small branch of femoral artery supplying skin of lower abdomen and external genitalia.
Femoral Artery: Continuation of external iliac artery after passing beneath inguinal ligament, major artery of thigh.
External Iliac Artery: Artery that continues as femoral artery after crossing inguinal ligament.
Internal Iliac Artery: Branch of common iliac artery supplying pelvis, gluteal region, and perineum.
Great Saphenous Vein: Longest superficial vein of body, often used for bypass grafts, runs close to femoral artery branches.
Clinical Relevance: Injury to femoral artery or its branches in groin may cause hemorrhage or ischemia of genital region.
Lead Question - 2013
The superficial external pudendal artery is a branch of?
a) Femoral artery
b) External iliac artery
c) Internal iliac artery
d) Aorta
Explanation: The superficial external pudendal artery arises from the femoral artery near the femoral triangle. It crosses the spermatic cord or round ligament to supply skin of external genitalia. The correct answer is a) Femoral artery.
Guessed Question 2
The profunda femoris artery is a direct branch of?
a) Femoral artery
b) External iliac artery
c) Internal iliac artery
d) Popliteal artery
Explanation: Profunda femoris artery, also called deep artery of thigh, arises from femoral artery in femoral triangle. It supplies deep muscles of thigh and gives perforating branches. The correct answer is a) Femoral artery.
Guessed Question 3
Which artery supplies blood to the head and neck of femur in adults?
a) Medial circumflex femoral artery
b) Lateral circumflex femoral artery
c) Obturator artery
d) Superior gluteal artery
Explanation: Medial circumflex femoral artery provides retinacular branches that supply femoral head and neck in adults. Obturator artery is important in children. The correct answer is a) Medial circumflex femoral artery.
Guessed Question 4
A patient with pelvic fracture injures internal pudendal artery. Which region is mainly affected?
a) Perineum
b) Anterior thigh
c) Lateral leg
d) Lower abdomen
Explanation: Internal pudendal artery, branch of internal iliac, supplies perineum including external genitalia, anal canal, and associated muscles. Its injury leads to perineal bleeding. The correct answer is a) Perineum.
Guessed Question 5
Which branch of femoral artery is commonly used for coronary bypass graft harvesting?
a) Great saphenous vein (vein, not artery)
b) Profunda femoris artery
c) Superficial epigastric artery
d) Superficial circumflex iliac artery
Explanation: The great saphenous vein, closely related to femoral artery branches, is harvested for coronary artery bypass surgery. Among arteries, superficial branches are too small for grafts. The correct answer is a) Great saphenous vein (though a vein, it is clinically relevant).
Guessed Question 6
A stab wound just below inguinal ligament injures femoral artery. Which immediate complication is expected?
a) Profuse bleeding and thigh ischemia
b) Perineal ischemia
c) Pelvic organ necrosis
d) Renal ischemia
Explanation: Femoral artery is main arterial supply of lower limb. Injury below inguinal ligament causes profuse bleeding and acute ischemia of thigh and leg. The correct answer is a) Profuse bleeding and thigh ischemia.
Guessed Question 7
Which artery accompanies the saphenous nerve in adductor canal?
a) Femoral artery
b) Popliteal artery
c) Profunda femoris artery
d) Obturator artery
Explanation: The femoral artery and saphenous nerve run together in adductor canal. Femoral artery continues through adductor hiatus to become popliteal artery. The correct answer is a) Femoral artery.
Guessed Question 8
Which artery gives rise to superficial epigastric artery?
a) Femoral artery
b) Internal iliac artery
c) External iliac artery
d) Inferior epigastric artery
Explanation: The superficial epigastric artery arises from femoral artery just below inguinal ligament, supplying lower anterior abdominal wall skin. The correct answer is a) Femoral artery.
Guessed Question 9
In case of occlusion of femoral artery just distal to inguinal ligament, which artery helps collateral circulation?
a) Inferior gluteal artery
b) Superior epigastric artery
c) Internal pudendal artery
d) Posterior tibial artery
Explanation: Inferior gluteal artery, branch of internal iliac, anastomoses with profunda femoris branches, providing collateral supply when femoral artery is blocked. The correct answer is a) Inferior gluteal artery.
Guessed Question 10
A patient presents with scrotal skin ischemia after groin trauma. Which artery is most likely injured?
a) Superficial external pudendal artery
b) Deep femoral artery
c) Internal pudendal artery
d) Superficial circumflex iliac artery
Explanation: Superficial external pudendal artery supplies skin of scrotum or labia majora. Injury leads to ischemia and necrosis of external genital skin. The correct answer is a) Superficial external pudendal artery.
Guessed Question 11
Which artery is palpated at femoral pulse just below inguinal ligament?
a) Femoral artery
b) External iliac artery
c) Internal iliac artery
d) Profunda femoris artery
Explanation: Femoral artery lies just below inguinal ligament at mid-inguinal point, between anterior superior iliac spine and pubic symphysis. It is the site for femoral pulse palpation and catheter insertion. The correct answer is a) Femoral artery.
Chapter: Anatomy
Topic: Lower Limb
Subtopic: Blood Supply of Femur
Keyword Definitions:
Femoral Head: The upper rounded part of the femur that fits into the acetabulum of the hip bone.
Medial Circumflex Femoral Artery: A branch of the profunda femoris artery supplying the femoral head and neck.
Lateral Circumflex Femoral Artery: Supplies mainly the anterior and lateral aspects of thigh muscles.
Artery of Ligamentum Teres: A small artery supplying minor part of the femoral head in adults.
Avascular Necrosis: Death of bone tissue due to loss of blood supply, commonly affecting femoral head.
Lead Question - 2013
Main blood supply to the head and neck of femur comes from
a) Lateral circumflex femoral artery
b) Medial circumflex femoral artery
c) Artery of ligamentum teres
d) Popliteal artery
Explanation: The main blood supply to the head and neck of femur is from the medial circumflex femoral artery, which provides retinacular branches around the femoral neck. The lateral circumflex and ligamentum teres arteries give minor contributions. The correct answer is b) Medial circumflex femoral artery.
Guessed Question 2
Which artery is most commonly injured in fracture of the femoral neck?
a) Popliteal artery
b) Medial circumflex femoral artery
c) Lateral circumflex femoral artery
d) Femoral artery
Explanation: In fracture of the femoral neck, the medial circumflex femoral artery is commonly injured, leading to avascular necrosis of the femoral head. Its retinacular branches are particularly vulnerable. The correct answer is b) Medial circumflex femoral artery.
Guessed Question 3
Which artery provides a significant blood supply to the femoral head in children?
a) Medial circumflex femoral artery
b) Lateral circumflex femoral artery
c) Artery of ligamentum teres
d) Obturator artery
Explanation: In children, the artery of ligamentum teres (a branch of the obturator artery) significantly contributes to the femoral head blood supply. This diminishes with age. The correct answer is c) Artery of ligamentum teres.
Guessed Question 4
Avascular necrosis of femoral head is most likely due to injury of which artery?
a) Medial circumflex femoral artery
b) Popliteal artery
c) Lateral circumflex femoral artery
d) Profunda femoris artery
Explanation: Damage to the medial circumflex femoral artery is the main cause of avascular necrosis of the femoral head. Its retinacular branches are crucial for blood supply. The correct answer is a) Medial circumflex femoral artery.
Guessed Question 5
During hip dislocation, which artery is at maximum risk of damage?
a) Medial circumflex femoral artery
b) Lateral circumflex femoral artery
c) Inferior gluteal artery
d) Internal pudendal artery
Explanation: Posterior dislocation of the hip may disrupt the medial circumflex femoral artery, compromising femoral head vascularity. This can lead to avascular necrosis. The correct answer is a) Medial circumflex femoral artery.
Guessed Question 6
Artery of ligamentum teres is a branch of which artery?
a) Obturator artery
b) Profunda femoris artery
c) Femoral artery
d) Internal iliac artery
Explanation: The artery of ligamentum teres is usually a branch of the obturator artery. It passes through the ligament of head of femur to supply a minor part of the femoral head. The correct answer is a) Obturator artery.
Guessed Question 7
Which artery predominantly supplies the greater trochanter of femur?
a) Lateral circumflex femoral artery
b) Medial circumflex femoral artery
c) Popliteal artery
d) Artery of ligamentum teres
Explanation: The lateral circumflex femoral artery provides branches to the greater trochanter and anterior thigh region, while medial circumflex mainly supplies the head and neck. The correct answer is a) Lateral circumflex femoral artery.
Guessed Question 8
Profunda femoris artery gives rise to which important arteries supplying femur?
a) Medial and lateral circumflex femoral arteries
b) Obturator artery
c) Inferior gluteal artery
d) Superior gluteal artery
Explanation: Profunda femoris artery, the deep artery of thigh, gives rise to medial and lateral circumflex femoral arteries. These are vital for femoral head and surrounding structures. The correct answer is a) Medial and lateral circumflex femoral arteries.
Guessed Question 9
Which condition is most likely after untreated femoral neck fracture due to arterial compromise?
a) Osteoarthritis
b) Avascular necrosis
c) Osteomyelitis
d) Chondrosarcoma
Explanation: If femoral neck fractures compromise blood supply from the medial circumflex femoral artery, avascular necrosis of the femoral head may develop. The correct answer is b) Avascular necrosis.
Guessed Question 10
Retinacular arteries are branches of which artery?
a) Medial circumflex femoral artery
b) Lateral circumflex femoral artery
c) Femoral artery
d) Popliteal artery
Explanation: Retinacular arteries arise from the medial circumflex femoral artery and travel along the femoral neck to supply the head. These are crucial in maintaining bone viability. The correct answer is a) Medial circumflex femoral artery.
Guessed Question 11
In slipped capital femoral epiphysis, compromised blood supply is mainly from?
a) Medial circumflex femoral artery
b) Lateral circumflex femoral artery
c) Obturator artery
d) Popliteal artery
Explanation: Slipped capital femoral epiphysis (SCFE) endangers the blood supply to femoral head, especially through retinacular branches of the medial circumflex femoral artery. The correct answer is a) Medial circumflex femoral artery.
Keyword Definitions
• Femoral head – Spherical upper end of femur that articulates with acetabulum to form hip joint.
• Medial circumflex femoral artery – Major artery supplying femoral head and neck via retinacular branches.
• Lateral circumflex femoral artery – Smaller contribution to anterior femoral neck and head.
• Ligamentum teres artery – Small artery within ligament of head of femur; significant in children only.
• Profunda femoris artery – Deep femoral artery; gives rise to medial and lateral circumflex branches.
• Retinacular vessels – Arterial branches running along femoral neck beneath joint capsule to supply femoral head.
• Avascular necrosis (AVN) – Bone death due to interruption of blood supply; common complication of femoral neck fracture.
• Intracapsular fracture – Femoral neck fracture within capsule; disrupts blood supply leading to AVN.
• Hip dislocation – Displacement of femoral head; can stretch or rupture retinacular arteries.
• Artery of ligamentum teres – Vestigial in adults but may partially supply femoral head in young children.
Chapter: Anatomy / Lower Limb
Topic: Hip Joint
Subtopic: Blood Supply of Femoral Head
Lead Question – 2013
The blood supply to femoral head is mostly by?
a) Lateral epiphyseal artery
b) Medial epiphyseal artery
c) Ligamentous teres artery
d) Profunda femoris
Explanation: The main supply to the femoral head in adults is from retinacular branches of the medial circumflex femoral artery. The lateral circumflex contributes minimally. The artery of ligamentum teres is significant only in children. Profunda femoris is the parent trunk. Correct answer: Medial epiphyseal artery.
Guessed Questions for NEET PG
1) Which artery is most at risk of injury in intracapsular fracture of femoral neck?
a) Medial circumflex femoral
b) Lateral circumflex femoral
c) Profunda femoris
d) Obturator
Explanation: Intracapsular fractures damage retinacular branches of the medial circumflex femoral artery, the primary supplier of the femoral head. This disruption causes avascular necrosis. Correct answer: Medial circumflex femoral artery.
2) In children, which artery contributes significantly to femoral head supply?
a) Artery of ligamentum teres
b) Medial circumflex femoral
c) Lateral circumflex femoral
d) Inferior gluteal
Explanation: In children, the artery of ligamentum teres (branch of obturator artery) is important for femoral head vascularity. With age, this vessel regresses and becomes less significant. Correct answer: Artery of ligamentum teres.
3) Which complication most commonly follows fracture of femoral neck in elderly?
a) Osteoarthritis
b) Avascular necrosis of femoral head
c) Osteomyelitis
d) Sciatic nerve injury
Explanation: Disruption of medial circumflex femoral artery branches leads to avascular necrosis of the femoral head, particularly after intracapsular neck fractures. Correct answer: Avascular necrosis of femoral head.
4) Lateral circumflex femoral artery is mainly a branch of:
a) Internal iliac artery
b) Profunda femoris artery
c) Common femoral artery
d) External iliac artery
Explanation: The lateral circumflex femoral artery is a branch of profunda femoris artery. It supplies anterior thigh and gives ascending branches to the femoral neck. Correct answer: Profunda femoris artery.
5) Avascular necrosis of femoral head is best diagnosed initially by:
a) Plain X-ray
b) CT scan
c) MRI
d) Ultrasound
Explanation: MRI is the most sensitive modality to detect early avascular necrosis of femoral head before radiographic changes appear. Correct answer: MRI.
6) Which artery forms cruciate anastomosis around hip joint?
a) Superior gluteal
b) Inferior gluteal
c) Medial circumflex femoral
d) All of the above
Explanation: Cruciate anastomosis is formed by inferior gluteal, medial circumflex femoral, lateral circumflex femoral (transverse branch), and first perforator of profunda femoris. Correct answer: All of the above (with contributions).
7) Which ligament encloses the artery of ligamentum teres?
a) Iliofemoral ligament
b) Ischiofemoral ligament
c) Ligament of head of femur
d) Pubofemoral ligament
Explanation: The artery of ligamentum teres passes through the ligament of head of femur (ligamentum teres). Correct answer: Ligament of head of femur.
8) In posterior hip dislocation, blood supply to femoral head is compromised because:
a) Ligamentum teres is torn
b) Retinacular vessels from medial circumflex femoral artery are damaged
c) Obturator nerve compression
d) Venous drainage blocked
Explanation: Posterior dislocation stretches and tears retinacular vessels of medial circumflex femoral artery, causing avascular necrosis. Correct answer: Damage to retinacular vessels from medial circumflex artery.
9) Which artery does not participate in trochanteric anastomosis?
a) Superior gluteal
b) Inferior gluteal
c) Medial circumflex femoral
d) Anterior tibial
Explanation: Trochanteric anastomosis includes superior gluteal, inferior gluteal, medial circumflex femoral, and lateral circumflex femoral arteries. Anterior tibial does not contribute. Correct answer: Anterior tibial artery.
10) Best management for avascular necrosis of femoral head in advanced stage is:
a) Core decompression
b) Bisphosphonates
c) Total hip replacement
d) Bone marrow injection
Explanation: In advanced avascular necrosis with collapse of femoral head, total hip replacement is the definitive treatment. Early stages may respond to decompression. Correct answer: Total hip replacement.
Keyword Definitions
• Anterior compartment (leg) – The muscular compartment on the front of the leg containing tibialis anterior, EDL, EHL, and popliteo-tibial neurovascular structures.
• Tibialis anterior (TA) – Primary dorsiflexor and inverter of foot; tendon medial to bundle at ankle.
• Extensor hallucis longus (EHL) – Extends big toe; its tendon lies between TA and EDL at the dorsum.
• Extensor digitorum longus (EDL) – Extends toes 2–5; lateral to EHL and often associated with peroneus tertius.
• Peroneus tertius – Variable muscle, tendon joins EDL over lateral dorsum, aids eversion.
• Anterior tibial artery – Main artery of anterior compartment, continues as dorsalis pedis on dorsum of foot.
• Deep peroneal (fibular) nerve – Motor to anterior compartment, sensory to web space between 1st and 2nd toes.
• Extensor retinaculum – Holds extensor tendons and neurovascular bundle at the ankle.
• Clinical correlation – Localization of bundle important for dorsalis pedis pulse and ankle compartment surgery.
• Surgical relevance – Avoid injury to anterior tibial vessels and deep peroneal nerve during ankle procedures.
Chapter: Anatomy / Lower Limb
Topic: Leg — Anterior Compartment
Subtopic: Neurovascular relations at the ankle
Lead Question – 2013
Neurovascular bundle of anterior compartment of leg passes between the tendons of ?
a) EHL and EDL
b) EDL and peroneus tertius
c) Tibialis anterior and EHL
d) None of the above
Explanation: The anterior tibial vessels and deep peroneal nerve travel in the anterior compartment and, at the ankle beneath the extensor retinaculum, lie between the tendons of tibialis anterior (medial) and extensor hallucis longus (lateral). Correct answer: (c) Tibialis anterior and EHL. Clinically palpable as dorsalis pedis pulse distal to this region.
Guessed Questions for NEET PG
1) The artery continuing from anterior tibial artery onto the dorsum of the foot is:
a) Posterior tibial artery
b) Dorsalis pedis artery
c) Peroneal artery
d) Medial plantar artery
Explanation: The anterior tibial artery continues over the ankle as the dorsalis pedis artery on the dorsum of the foot. Correct answer: Dorsalis pedis artery. Clinically the dorsalis pedis pulse is used to assess peripheral perfusion and arterial injury in the foot.
2) Sensory supply of the web space between first and second toes is by:
a) Superficial peroneal nerve
b) Deep peroneal nerve
c) Saphenous nerve
d) Tibial nerve
Explanation: The deep peroneal (fibular) nerve supplies the skin between the first and second toes. Correct answer: Deep peroneal nerve. Clinically, numbness here suggests injury to the deep peroneal nerve at the ankle or leg.
3) A patient with foot drop after fibular neck fracture likely has injury to:
a) Superficial peroneal nerve
b) Deep peroneal nerve
c) Tibial nerve
d) Sural nerve
Explanation: Common fibular (peroneal) nerve wraps around fibular neck and its deep branch supplies anterior compartment dorsiflexors. Injury causes foot drop. Correct answer: Deep peroneal (branch of common peroneal). Clinically presents with steppage gait.
4) The extensor retinaculum prevents bowstringing of:
a) Flexor tendons only
b) Extensor tendons only
c) Both flexor and extensor tendons equally
d) Peroneal tendons only
Explanation: The extensor retinaculum secures extensor tendons at the ankle, preventing bowstringing during dorsiflexion. Correct answer: Extensor tendons only. Clinically, tight retinaculum can cause tenosynovitis and pain over the dorsum of ankle.
5) A weak dorsalis pedis pulse with intact posterior tibial pulse suggests occlusion of:
a) Anterior tibial artery
b) Posterior tibial artery
c) Peroneal artery
d) Femoral artery
Explanation: Diminished dorsalis pedis with normal posterior tibial pulse suggests anterior tibial artery compromise. Correct answer: Anterior tibial artery. Clinically important in acute limb ischemia and after tibial fractures.
6) In anterior compartment syndrome, which action is most affected?
a) Plantarflexion of ankle
b) Dorsiflexion of ankle
c) Inversion of foot only
d) Toe abduction
Explanation: Anterior compartment contains dorsiflexors (tibialis anterior, EDL, EHL); raised pressure causes ischemia and loss of dorsiflexion. Correct answer: Dorsiflexion of ankle. Clinically urgent fasciotomy prevents permanent deficit.
7) The tendon order across the dorsum (medial to lateral) just distal to the ankle is:
a) TA, EHL, EDL, peroneus tertius
b) EDL, EHL, TA, peroneus tertius
c) Peroneus tertius, EDL, EHL, TA
d) TA, EDL, EHL, peroneus tertius
Explanation: From medial to lateral the tendons are tibialis anterior, extensor hallucis longus, extensor digitorum longus, then peroneus tertius. Correct answer: TA, EHL, EDL, peroneus tertius. This ordering helps localize neurovascular structures and tendinous pathology.
8) Injury to deep peroneal nerve at the ankle will cause sensory loss where?
a) Lateral foot dorsum
b) Medial sole
c) First web space between toes 1 and 2
d) Posterior calf
Explanation: Deep peroneal nerve supplies sensation to the first web space dorsally. Correct answer: First web space. Clinically, combined motor (dorsiflexion) and this specific sensory loss indicate deep peroneal lesion.
9) The best site to palpate the dorsalis pedis pulse is lateral to which tendon?
a) Tibialis anterior tendon
b) Extensor hallucis longus tendon
c) Extensor digitorum longus tendon
d) Peroneus tertius tendon
Explanation: The dorsalis pedis artery is palpated lateral to the tendon of extensor hallucis longus on the dorsum of the foot. Correct answer: Extensor hallucis longus tendon. Clinically used in vascular exams of the foot.
10) Surgical release of anterior compartment at the leg should avoid damaging which structure running in the compartment?
a) Great saphenous vein
b) Posterior tibial nerve
c) Deep peroneal nerve and anterior tibial vessels
d) Superficial peroneal nerve only
Explanation: The deep peroneal nerve and anterior tibial vessels run in the anterior compartment and must be preserved during fasciotomy or debridement. Correct answer: Deep peroneal nerve and anterior tibial vessels. Clinically, careful technique prevents vascular and motor-sensory loss.
Keyword Definitions
• Great saphenous vein – Longest superficial vein, runs from the medial foot to the groin, drains into femoral vein at saphenous (femoral) opening.
• Dorsal venous arch – Venous network on dorsum of foot; medial end gives origin to great saphenous vein.
• Saphenous nerve – Sensory branch of femoral nerve that accompanies the great saphenous vein along medial leg.
• Small (lesser) saphenous vein – Superficial posterior leg vein draining to popliteal vein; accompanied by sural nerve.
• Saphenous opening (fossa ovalis) – Gap in fascia lata where great saphenous vein passes to join femoral vein.
• Perforator veins – Connect superficial and deep systems (e.g., Cockett perforators); incompetence causes varicose veins.
• Varicose veins – Dilated, tortuous superficial veins due to valvular incompetence, commonly involve great saphenous tributaries.
• Saphenous cutdown – Surgical exposure of great saphenous vein at medial ankle for venous access.
• CABG conduit – Great saphenous vein commonly harvested for coronary artery bypass grafting.
• Clinical relevance – Knowledge of relationships important for varicose vein surgery, saphenous nerve preservation, and venous access.
Chapter: Anatomy / Lower Limb
Topic: Superficial Venous System of Lower Limb
Subtopic: Great Saphenous Vein – Anatomy and Clinical Correlates
Lead Question – 2013
True statement about great saphenous vein
a) It begins at lateral end of dorsal venous arch
b) It runs anterior to medial malleolus
c) It is accompanied by sural nerve
d) Terminates into popliteal vein
Explanation: The great saphenous vein arises from the **medial** end of the dorsal venous arch, ascends **anterior to the medial malleolus**, and is accompanied by the saphenous nerve. It terminates into the femoral vein at the saphenous opening, not the popliteal. Correct answer: (b). Clinically vital for varicose vein surgery and graft harvest.
Guessed Questions for NEET PG
1) The great saphenous vein terminates into the:
a) Popliteal vein
b) Femoral vein at saphenous opening
c) Small saphenous vein
d) Posterior tibial vein
Explanation: The great saphenous vein ascends the medial leg and pierces the fascia lata at the saphenous (femoral) opening to drain into the femoral vein. Correct answer: Femoral vein at saphenous opening. Clinically, this junction is inspected during varicose vein surgery and duplex scanning.
2) Which nerve accompanies the great saphenous vein along the medial leg?
a) Sural nerve
b) Saphenous nerve
c) Superficial peroneal nerve
d) Tibial nerve
Explanation: The saphenous nerve (branch of femoral nerve) runs with the great saphenous vein along the medial aspect of the leg and ankle carrying cutaneous sensation. Correct answer: Saphenous nerve. Clinical: Preserve this nerve during vein harvest to avoid medial leg numbness.
3) The commonest complication of great saphenous vein valve incompetence is:
a) Deep vein thrombosis only
b) Varicose veins of medial leg and thigh
c) Plantar fasciitis
d) Morton's neuroma
Explanation: Incompetence of valves in great saphenous and its tributaries leads to venous hypertension and varicose veins, typically along the medial leg and thigh. Correct answer: Varicose veins. Clinical: Presents with aching, swelling, and skin changes; treated by ablation or stripping.
4) The origin of the great saphenous vein is at the:
a) Lateral end of dorsal venous arch
b) Medial end of dorsal venous arch (near first metatarsal)
c) Posterior aspect of heel
d) Popliteal fossa
Explanation: Great saphenous vein begins at the **medial** end of the dorsal venous arch near the medial side of the foot and first metatarsal region. Correct answer: Medial end of dorsal venous arch. Clinically, this is the landmark for saphenous cutdown access.
5) The small saphenous vein typically drains into the:
a) Femoral vein
b) Popliteal vein
c) Great saphenous vein
d) Anterior tibial vein
Explanation: The small (lesser) saphenous vein ascends posterior calf and usually drains into the popliteal vein in the popliteal fossa. Correct answer: Popliteal vein. Clinical distinction helps in planning venous ablation and thrombosis evaluation.
6) Which perforator group is classically associated with great saphenous reflux in the lower leg?
a) Cockett perforators (lower leg)
b) Dodd perforators (thigh only)
c) Boyd perforators (popliteal crease only)
d) No perforators in lower leg
Explanation: Cockett perforators are located in the lower calf and commonly allow reflux from deep to superficial veins contributing to great saphenous varicosities. Correct answer: Cockett perforators. Clinical: Identified and ligated in surgical treatment of varicose veins.
7) For coronary artery bypass grafting, the great saphenous vein is harvested because it is:
a) Short and deep
b) Long, superficial, and of suitable caliber
c) Accompanied by artery
d) Immune to atherosclerosis
Explanation: The great saphenous vein is long, superficial, readily accessible, and generally of adequate caliber for bypass grafting. Correct answer: Long, superficial, and of suitable caliber. Clinical: Harvest technique must preserve side branches and avoid injury to saphenous nerve.
8) In saphenous vein cutdown for emergency venous access, the incision is usually made just anterior to the:
a) Lateral malleolus
b) Medial malleolus
c) Tibial tuberosity
d) Popliteal fossa
Explanation: Saphenous vein cutdown is performed anterior to the medial malleolus where the great saphenous vein is superficial and fixed, facilitating cannulation. Correct answer: Medial malleolus. Clinical: Useful when peripheral access is difficult; take care to avoid saphenous nerve.
9) Which statement about great saphenous vein valves is correct?
a) Valves prevent flow from superficial to deep veins only
b) Valves are absent in great saphenous vein
c) Valve incompetence leads to retrograde flow and varicosities
d) Valves force blood toward foot
Explanation: Venous valves normally permit unidirectional flow toward the heart; incompetence in the great saphenous system causes retrograde flow and varicose veins. Correct answer: Valve incompetence leads to retrograde flow and varicosities. Clinical: Duplex ultrasound assesses valve function before intervention.
10) A patient with thrombosis limited to the great saphenous vein (superficial thrombophlebitis) most importantly requires evaluation for:
a) Pulmonary embolism risk and extension into deep venous system
b) Immediate limb amputation
c) Coronary artery disease
d) Spinal cord compression
Explanation: Superficial thrombophlebitis of great saphenous vein may extend into deep veins, risking DVT and pulmonary embolism; evaluate with duplex ultrasound and treat accordingly. Correct answer: Pulmonary embolism risk and extension into deep venous system. Clinical: Anticoagulation and surveillance may be needed.
Keyword Definitions
• Interosseous membrane – Fibrous sheet between radius and ulna, provides attachment for muscles and transmits forces.
• Anterior interosseous artery – Branch of common interosseous artery running on anterior surface of interosseous membrane supplying deep forearm muscles.
• Posterior interosseous artery – Branch that reaches the posterior compartment, often passes through/perforates the interosseous membrane to supply extensors.
• Common interosseous artery – Short trunk from ulnar artery dividing into anterior and posterior interosseous arteries.
• Interosseous space – The gap between radius and ulna occupied by membrane and vessels; communicates between compartments.
• Posterior interosseous nerve – Deep branch of radial nerve running in posterior compartment with posterior interosseous vessels.
• Perforating branches – Small vessels that traverse the interosseous membrane to connect anterior and posterior circulations.
• Clinical relevance – Knowledge is vital in forearm fractures and surgical approaches to avoid vascular injury.
• Supination/pronation force transmission – Interosseous membrane transmits axial loads from radius to ulna during weight-bearing.
• Surgical landmark – Interosseous membrane used as reference during forearm reconstructive procedures.
Chapter: Anatomy / Upper Limb
Topic: Forearm Vessels and Membranes
Subtopic: Interosseous Membrane and its Perforators
Lead Question – 2013
Interosseous membrane of forearm is pierced by?
a) Brachial artery
b) Anterior interosseous artery
c) Posterior interosseous artery
d) Ulnar recurrent artery
Explanation: The posterior interosseous artery typically pierces the interosseous membrane to reach the posterior compartment, accompanying the posterior interosseous nerve. The anterior interosseous artery runs on the anterior surface and sends perforators. Correct answer: Posterior interosseous artery. Clinically important in posterior compartment surgeries and fractures.
Guessed Questions for NEET PG
1) The anterior interosseous artery is a branch of:
a) Radial artery
b) Ulnar artery (via common interosseous)
c) Brachial artery directly
d) Posterior interosseous artery
Explanation: The anterior interosseous artery arises from the common interosseous branch of the ulnar artery and runs on the anterior surface of the interosseous membrane. Correct answer: Ulnar artery (via common interosseous). Clinical: AIN and artery are vulnerable in proximal forearm trauma.
2) Which nerve accompanies the posterior interosseous artery in the posterior compartment?
a) Superficial radial nerve
b) Posterior interosseous nerve (deep branch of radial)
c) Median nerve
d) Ulnar nerve
Explanation: The posterior interosseous nerve (deep branch of radial nerve) accompanies the posterior interosseous artery in the posterior compartment to supply extensor muscles. Correct answer: Posterior interosseous nerve. Clinical: Injury causes finger extension weakness without sensory loss.
3) Perforating branches of the interosseous arteries allow communication between:
a) Radial and ulnar arteries only
b) Anterior and posterior compartments of forearm
c) Superficial and deep palmar arches
d) Brachial and radial arteries
Explanation: Perforators through the interosseous membrane connect anterior and posterior interosseous arteries, providing collateral circulation between forearm compartments. Correct answer: Anterior and posterior compartments. Clinical: Important when primary vessels are injured.
4) Injury to posterior interosseous artery in proximal forearm most likely causes:
a) Pure sensory loss in hand
b) Ischemia of posterior compartment muscles
c) Loss of pronation only
d) Thumb adduction loss
Explanation: Damage to posterior interosseous artery reduces blood supply to posterior (extensor) compartment leading to ischemic pain and weakness. Correct answer: Ischemia of posterior compartment muscles. Clinical: May accompany fractures or surgical insults.
5) The common interosseous artery usually arises from:
a) Radial artery
b) Ulnar artery
c) Brachial artery at cubital fossa
d) Profunda brachii artery
Explanation: The common interosseous artery branches from the ulnar artery shortly after the ulnar origin, then divides into anterior and posterior interosseous arteries. Correct answer: Ulnar artery. Clinical: Variant anatomy can affect flap planning.
6) Which structure runs along the anterior surface of the interosseous membrane?
a) Posterior interosseous artery
b) Anterior interosseous artery and nerve
c) Superficial radial nerve
d) Ulnar nerve
Explanation: The anterior interosseous artery and anterior interosseous branch of median nerve run on the anterior surface of the interosseous membrane supplying deep flexors. Correct answer: Anterior interosseous artery and nerve. Clinical: AIN palsy causes pure motor deficits.
7) The posterior interosseous artery usually reaches the posterior compartment via a gap near which landmark?
a) Lister’s tubercle
b) Proximal border of interosseous membrane near supinator
c) Ulnar styloid process
d) Pisiform bone
Explanation: The posterior interosseous artery commonly passes to the posterior compartment near the proximal border of the interosseous membrane in the region of the supinator. Correct answer: Proximal border of interosseous membrane near supinator. Clinical: Supinator syndrome may compromise vessels and nerve.
8) In a Galeazzi fracture (distal radius with DRUJ disruption), which artery's flow might be compromised affecting interosseous communication?
a) Brachial artery
b) Anterior interosseous artery
c) Posterior tibial artery
d) Median artery
Explanation: A distal radius fracture can disturb branches including anterior interosseous artery or its perforators, impairing collateral flow between compartments. Correct answer: Anterior interosseous artery. Clinical: Assess distal perfusion and nerve function in such injuries.
9) The anterior interosseous artery supplies all EXCEPT:
a) Flexor pollicis longus
b) Pronator quadratus
c) Lateral part of flexor digitorum profundus
d) Extensor digitorum communis
Explanation: The AIN supplies FPL, pronator quadratus, and lateral FDP; it does not supply extensor digitorum communis (posterior compartment). Correct answer: Extensor digitorum communis. Clinical: AIN lesions cause weak thumb and index flexion.
10) Surgical exposure of the posterior forearm should avoid injury to which vessel that pierces the interosseous membrane?
a) Radial artery
b) Posterior interosseous artery
c) Ulnar artery
d) Cephalic vein
Explanation: The posterior interosseous artery pierces the interosseous membrane to reach the posterior compartment and must be preserved during surgical approaches to avoid ischemia of extensor muscles. Correct answer: Posterior interosseous artery. Careful dissection around supinator is required.
Chapter: Renal & Cardiovascular Physiology
Topic: Fluid and Electrolyte Homeostasis
Subtopic: Hormonal Control of Sodium and Water
Keyword Definitions:
Renin angiotensin system: Hormonal cascade (renin → angiotensin II → aldosterone) that conserves sodium and water.
ANP / BNP: Atrial and B-type natriuretic peptides that promote natriuresis and reduce volume.
Vasopressin (ADH): Antidiuretic hormone increasing water reabsorption via aquaporins.
Natriuresis: Excretion of sodium in urine.
Aldosterone: Mineralocorticoid increasing distal nephron sodium reabsorption and potassium secretion.
Lead Question - 2012
Which of the following is most important in sodium and water retention ?
a) Renin angiotensin system
b) ANP
c) BNP
d) Vasopressin
Explanation: Renin angiotensin system is the principal regulator of sodium and water retention via angiotensin II mediated aldosterone release, renal arteriolar constriction, and increased proximal sodium reabsorption. It conserves sodium and water during hypovolemia. Therefore correct answer: a) Renin angiotensin system. This mechanism predominates in volume depletion states especially acutely.
Guessed Question 1
ANP primarily causes sodium loss by acting on which site?
a) Proximal tubule
b) Collecting duct
c) Loop of Henle
d) Glomerulus
Explanation: Atrial natriuretic peptide promotes natriuresis and diuresis by inhibiting sodium reabsorption in the collecting duct, decreasing aldosterone and renin release, and increasing GFR through afferent arteriolar dilation. It reduces plasma volume and blood pressure. Therefore correct answer: b) ANP. This hormone is released from atrial myocytes with atrial stretch promptly.
Guessed Question 2
BNP is clinically useful as a marker of?
a) Liver failure
b) Renal tubular injury
c) Heart failure
d) Primary hyperaldosteronism
Explanation: B-type natriuretic peptide is secreted by ventricular myocytes in response to increased wall stress. It promotes natriuresis, vasodilation, and inhibits the renin angiotensin aldosterone system. Elevated plasma levels indicate heart failure severity and help guide management. Therefore correct answer: c) BNP. used in diagnosis and prognostication of heart failure clinically.
Guessed Question 3
Vasopressin (ADH) conserves body water by acting on?
a) Proximal tubule
b) Loop of Henle
c) Distal convoluted tubule
d) Collecting duct
Explanation: Vasopressin (ADH) binds V2 receptors on collecting duct principal cells, stimulating aquaporin 2 insertion into the apical membrane, increasing water permeability and reabsorption, concentrating urine and conserving body water during dehydration. Therefore correct answer: d) Collecting duct. It is released from posterior pituitary in response to hyperosmolality and hypovolemia and hypotension.
Guessed Question 4
Aldosterone increases sodium reabsorption by upregulating?
a) Aquaporin channels
b) ENaC and basolateral Na+/K+ ATPase
c) NKCC2 cotransporter
d) ROMK channels only
Explanation: Aldosterone, released from adrenal zona glomerulosa in response to angiotensin II and hyperkalemia, increases sodium reabsorption in distal nephron by upregulating ENaC and Na+/K+ ATPase, enhancing water retention and potassium secretion. Therefore correct answer: b) ENaC and basolateral Na+/K+ ATPase. This mechanism raises blood pressure and is targeted by ACE inhibitors clinically.
Guessed Question 5
In heart failure with volume overload, which system predominates in causing retention?
a) Renin angiotensin system
b) ANP release predominates
c) BNP secretion predominates
d) Vasopressin alone
Explanation: Despite elevated ANP in heart failure, the renin angiotensin system predominates in promoting sodium and water retention by increasing aldosterone, sympathetic tone, and renal sodium reabsorption; ANP effects are often overwhelmed. Hence correct answer: a) Renin angiotensin system. Consequently, blockade of RAS reduces fluid overload and improves outcomes clinically significantly.
Guessed Question 6
Which hormone mainly conserves water without directly increasing sodium retention?
a) Vasopressin
b) Aldosterone
c) ANP
d) Renin
Explanation: Vasopressin principally conserves water via aquaporin insertion without directly increasing sodium reabsorption, so volume expansion is limited; conversely renin angiotensin system increases both sodium and water retention through aldosterone and proximal reabsorption. Therefore correct answer: a) Vasopressin. This explains why RAS blockade causes natriuresis and blood pressure reduction.
Guessed Question 7
ACE inhibitors reduce sodium and water retention by blocking formation of?
a) Aldosterone directly
b) Angiotensin II
c) Vasopressin
d) ANP
Explanation: ACE inhibitors interrupt conversion of angiotensin I to II, reducing aldosterone secretion, sodium reabsorption, and water retention; they thereby lower blood pressure and reduce edema. Their effect confirms that the renin angiotensin system is central in sodium and water retention. Correct answer: b) Angiotensin II. ACE inhibitor therapy provides therapeutic benefit.
Guessed Question 8
Spironolactone reduces fluid retention by antagonizing which receptor?
a) Vasopressin receptor
b) Mineralocorticoid receptor
c) Beta adrenergic receptor
d) Natriuretic peptide receptor
Explanation: Spironolactone blocks mineralocorticoid receptors, decreasing sodium reabsorption in the distal nephron and promoting natriuresis; it reduces edema and hypertension in conditions of aldosterone excess. This pharmacologic evidence reinforces the dominant role of renin angiotensin system in sodium and water retention. Correct answer: b) Mineralocorticoid receptor. across diverse clinical scenarios.
Guessed Question 9
Which system primarily raises blood pressure and promotes long-term sodium retention?
a) Renin angiotensin system
b) ANP system
c) BNP release
d) Atrial stretch reflex
Explanation: Multiple homeostatic systems regulate body fluids, but the renin angiotensin system, via angiotensin II and aldosterone, exerts the most sustained and potent effects on sodium and water retention. Vasopressin and natriuretic peptides modulate volume acutely. Therefore correct answer: a) Renin angiotensin system. Hence RAS blockade reduces sodium retention clinically significantly.
Guessed Question 10
Which intervention best reduces sodium and water retention in heart failure?
a) ACE inhibitors / ARBs
b) ANP infusion
c) Pure water restriction
d) Vasopressin agonists
Explanation: ACE inhibitors or ARBs block the renin angiotensin system, lowering angiotensin II and aldosterone, reducing renal sodium reabsorption and water retention, improving congestion and mortality in heart failure. Therefore correct answer: a) ACE inhibitors / ARBs. Clinical trials support their central role in managing fluid overload and hypertension.
Chapter: Cardiovascular Physiology
Topic: Cardiac Action Potential
Subtopic: Plateau Phase Mechanism
Keywords
- Plateau phase: Sustained depolarization of cardiac action potential.
- Calcium channels: L-type Ca2+ channels allowing inward current.
- Potassium permeability: Decreased during plateau maintaining depolarization.
- Sodium influx: Occurs early, not during plateau.
- Excitation-contraction coupling: Calcium influx triggers muscle contraction.
Lead Question – 2012
The plateau phase of this graph is due to:
a) The movement of fewer sodium ions across the cell membrane
b) The calcium channels remaining open longer than the sodium channels
c) The increased membrane permeability to potassium ion
d) A decrease in the amount of calcium diffusing across the membrane
Explanation: The plateau phase of the cardiac action potential is primarily maintained by the prolonged opening of L-type calcium channels, while potassium efflux is reduced. This sustained calcium influx prolongs depolarization, essential for contraction. Hence, the correct answer is (b) The calcium channels remaining open longer than the sodium channels.
Guessed Questions
1) Which ion influx is most critical for triggering myocardial contraction?
a) Sodium
b) Calcium
c) Potassium
d) Chloride
Explanation: Myocardial contraction is dependent on calcium influx during the plateau phase, which binds to troponin to initiate contraction. Thus, calcium is the essential ion. Answer: (b) Calcium.
2) A patient receiving verapamil therapy would show reduced:
a) Sodium influx
b) Calcium influx
c) Potassium efflux
d) Chloride influx
Explanation: Verapamil blocks L-type calcium channels, reducing calcium influx, shortening plateau, and decreasing contractility. The answer is (b) Calcium influx.
3) Which phase of the cardiac action potential is most affected by calcium channel blockers?
a) Phase 0
b) Phase 1
c) Phase 2
d) Phase 3
Explanation: Calcium channel blockers reduce calcium entry during phase 2 (plateau phase), altering contraction strength. The correct answer is (c) Phase 2.
4) The refractory period of cardiac muscle is prolonged due to:
a) Rapid sodium influx
b) Calcium influx during plateau
c) Potassium efflux
d) Chloride movement
Explanation: The plateau phase extends depolarization and refractory period, preventing tetany. This is mediated by calcium influx. Answer: (b) Calcium influx during plateau.
5) A drug that increases potassium efflux during phase 2 will:
a) Prolong plateau phase
b) Shorten plateau phase
c) Increase calcium entry
d) Increase refractory period
Explanation: Increased potassium efflux would counter depolarization, leading to early repolarization and shortening of the plateau. The answer is (b) Shorten plateau phase.
6) In pacemaker cells, the plateau phase is:
a) Prominent
b) Absent
c) Longer than in ventricular cells
d) Stronger than atrial cells
Explanation: Pacemaker cells (SA node) do not have a distinct plateau phase; instead, they exhibit gradual depolarization. Thus, the answer is (b) Absent.
7) The plateau phase in ventricular myocytes ensures:
a) Summation of contractions
b) Sustained contraction for effective ejection
c) Prevention of depolarization
d) Immediate relaxation
Explanation: Plateau phase sustains depolarization, allowing sufficient contraction for ejection of blood and preventing rapid re-excitation. The answer is (b) Sustained contraction for effective ejection.
8) Which ion is mainly responsible for repolarization following the plateau?
a) Sodium influx
b) Calcium influx
c) Potassium efflux
d) Chloride efflux
Explanation: Repolarization after the plateau is mediated by increased potassium efflux, which restores resting membrane potential. The answer is (c) Potassium efflux.
9) Digitalis toxicity prolongs plateau by increasing:
a) Calcium influx
b) Sodium influx
c) Potassium efflux
d) Chloride entry
Explanation: Digitalis increases intracellular calcium by inhibiting Na+/K+ ATPase, prolonging the plateau and contraction. The answer is (a) Calcium influx.
10) A mutation causing prolonged opening of calcium channels may lead to:
a) Tachyarrhythmia
b) Short QT syndrome
c) Long QT syndrome
d) Increased potassium clearance
Explanation: Prolonged calcium entry prolongs action potential duration, predisposing to long QT syndrome. Answer: (c) Long QT syndrome.
11) During exercise, sympathetic stimulation increases calcium influx, which results in:
a) Decreased stroke volume
b) Increased contractility
c) Shortened plateau phase
d) Reduced cardiac output
Explanation: Sympathetic activity increases calcium influx, strengthening contraction and raising stroke volume. Answer: (b) Increased contractility.
Chapter: Cardiovascular Physiology | Topic: Cardiac Output and Stroke Volume | Subtopic: Determinants of Stroke Volume
Keywords
Stroke volume — volume of blood ejected by left ventricle per beat.
End-diastolic volume (EDV) — ventricular volume at end of filling.
End-systolic volume (ESV) — ventricular volume at end of contraction.
Ejection fraction (EF) — fraction of EDV ejected, SV/EDV × 100.
Preload — ventricular stretch at end of diastole, approximated by EDV.
Afterload — resistance the ventricle must overcome to eject blood.
Contractility — intrinsic ability of myocardium to contract independent of preload/afterload.
Frank-Starling law — stroke volume rises with increased venous return (EDV) up to a point.
Cardiac output — stroke volume × heart rate.
Clinical relevance — heart failure alters preload, afterload, and contractility, reducing stroke volume.
Lead Question - 2012
Stroke volume is increased by ?
a) Increased end-diastolic and end-systolic volumes
b) Decreased end-diastolic and end-systolic volumes
c) Increased end-diastolic volume and decreased end-systolic volume
d) Decreased end-diastolic volume and increased end-systolic volume
Explanation: Stroke volume depends on preload (EDV), afterload, and contractility. An increased EDV (greater filling) combined with a decreased ESV (stronger ejection) yields maximum stroke volume. Thus the correct answer is c. This mechanism reflects both the Frank-Starling effect and improved myocardial contractility.
Q2. A patient with severe mitral regurgitation typically has increased stroke volume because of:
a) Increased preload
b) Increased afterload
c) Decreased contractility
d) Reduced ejection fraction
Explanation: In mitral regurgitation, left ventricular volume overload increases EDV (preload). The Frank-Starling mechanism initially augments stroke volume despite regurgitation. Correct answer is a. Over time, chronic overload reduces contractility and ejection fraction.
Q3. Stroke volume decreases in which of the following clinical states?
a) Hemorrhagic shock
b) Athlete’s heart
c) Sympathetic stimulation
d) Increased venous return
Explanation: Hemorrhage reduces preload (EDV) due to hypovolemia, leading to reduced stroke volume despite compensatory tachycardia. Correct answer is a. In contrast, exercise, sympathetic drive, and increased venous return augment stroke volume.
Q4. A hypertensive patient with high afterload will most likely have:
a) Increased stroke volume
b) Decreased stroke volume
c) No effect on stroke volume
d) Increased preload
Explanation: Increased afterload (arterial resistance) makes ejection more difficult, raising ESV and lowering stroke volume. Correct answer is b. Chronic high afterload may also lead to concentric LV hypertrophy.
Q5. In septic shock, stroke volume is often:
a) Increased due to reduced afterload
b) Decreased due to low preload
c) Unchanged
d) Increased due to sympathetic stimulation
Explanation: Septic shock features vasodilation and capillary leak. Although afterload decreases, low preload and impaired myocardial contractility often lower stroke volume. Correct answer is b. Fluids and vasopressors are used to restore volume and perfusion.
Q6. Which of the following increases stroke volume physiologically?
a) Beta-adrenergic stimulation
b) Beta-blocker therapy
c) Acidosis
d) Myocardial ischemia
Explanation: Beta-adrenergic stimulation enhances myocardial contractility, reducing ESV and increasing stroke volume. Correct answer is a. Beta-blockers, ischemia, and acidosis depress contractility, reducing stroke volume.
Q7. Stroke volume is most reduced in which of the following?
a) Acute myocardial infarction
b) Athlete’s bradycardia
c) Moderate exercise
d) Positive inotrope infusion
Explanation: In acute myocardial infarction, contractility falls acutely, causing a rise in ESV and a fall in stroke volume. Correct answer is a. In athletes, stroke volume is preserved or enhanced despite slower heart rates.
Q8. Stroke volume increases during exercise due to:
a) Increased EDV and sympathetic stimulation
b) Decreased EDV and afterload
c) Decreased contractility
d) Increased ESV
Explanation: Exercise boosts venous return (increasing preload) and sympathetic activity (enhancing contractility), lowering ESV and raising stroke volume. Correct answer is a. This adaptation supports higher cardiac output during physical activity.
Q9. Which condition decreases stroke volume most significantly?
a) Cardiac tamponade
b) Exercise
c) Sympathetic stimulation
d) Early pregnancy
Explanation: Cardiac tamponade impairs diastolic filling by external compression of the heart, reducing preload and stroke volume. Correct answer is a. Exercise, sympathetic drive, and pregnancy normally increase stroke volume.
Q10. Stroke volume is increased by which pharmacological agent?
a) Dobutamine
b) Beta-blockers
c) Calcium channel blockers (verapamil)
d) Digoxin toxicity
Explanation: Dobutamine, a beta-1 agonist, enhances myocardial contractility and reduces ESV, increasing stroke volume. Correct answer is a. Other listed agents impair contractility or conduction and reduce stroke volume.
Q11. Which of the following is used clinically as a surrogate measure of stroke volume?
a) Pulse pressure
b) Diastolic pressure
c) Heart rate
d) Central venous pressure
Explanation: Pulse pressure (SBP–DBP) is proportional to stroke volume when arterial compliance is constant. Thus it serves as a surrogate measure. Correct answer is a. Diastolic pressure, HR, or CVP do not directly reflect stroke volume.
Chapter: Cardiovascular Physiology | Topic: Cardiac Preload & Hemodynamics | Subtopic: Preload Determinants
Keywords
Preload — ventricular wall stress at end-diastole; approximated clinically by end-diastolic volume or pressure.
End-diastolic volume (EDV) — volume in ventricle before systole; key preload measure.
End-systolic volume (ESV) — volume remaining after systole; used to calculate stroke volume and EF.
Stroke volume (SV) — EDV − ESV; influenced by preload, afterload, and contractility.
Afterload — resistance against which the ventricle ejects; relates to arterial pressure and vascular tone.
Venous return — primary determinant of preload; influenced by blood volume and venous tone.
Frank-Starling law — increased preload increases stroke volume within physiological limits.
Central venous pressure (CVP) — bedside surrogate for right-sided preload.
Pulmonary capillary wedge pressure (PCWP) — surrogate for left-sided preload in invasive monitoring.
Compliance — ventricular compliance affects pressure-volume relationship and preload interpretation.
Lead Question - 2012
Preload measures?
a) End systolic volume
b) End diastolic volume
c) Peripheral resistance
d) Stroke volume
Explanation: Preload reflects ventricular filling at end-diastole and is best approximated by end-diastolic volume (or pressure). End-systolic volume and peripheral resistance are not direct preload measures; stroke volume depends on preload but is not itself the preload. Hence the correct answer is b) End diastolic volume.
Q1. Which bedside measurement approximates left-sided preload in mechanically ventilated patients?
a) Central venous pressure (CVP)
b) Pulmonary capillary wedge pressure (PCWP)
c) Systolic blood pressure
d) Heart rate
Explanation: Pulmonary capillary wedge pressure (PCWP) measured via a pulmonary artery catheter approximates left atrial pressure and left-sided preload. CVP estimates right-sided preload. Systolic BP and heart rate are not preload measures. Correct answer: b) PCWP. PCWP guides fluid vs. inotrope decisions in ICU care.
Q2. According to Frank-Starling mechanism, increasing preload within physiological limits causes:
a) Decreased stroke volume
b) Increased stroke volume
c) No change in stroke volume
d) Immediate decrease in contractility
Explanation: The Frank–Starling law states that increased preload stretches myocardial fibers, augmenting contractile force and stroke volume up to an optimal point. Thus increasing preload typically increases stroke volume. Correct answer: b) Increased stroke volume. Excessive preload, however, may cause congestion without further output gain.
Q3. Which clinical condition is characterized by low preload?
a) Heart failure with volume overload
b) Hypovolemic shock
c) Constrictive pericarditis
d) Cardiac tamponade
Explanation: Hypovolemic shock features reduced intravascular volume and thus low preload, lowering stroke volume and MAP. Constrictive pericarditis and tamponade impair filling but may show elevated venous pressures despite functional low filling. Volume overload increases preload. Correct answer: b) Hypovolemic shock.
Q4. Which intervention will most directly increase preload in a hypotensive patient?
a) Intravenous fluid bolus
b) Intravenous nitroprusside
c) Intravenous furosemide
d) Intravenous beta-blocker
Explanation: An IV fluid bolus increases intravascular volume and venous return, directly increasing preload and stroke volume in responsive patients. Vasodilators and diuretics reduce preload, and beta-blockers reduce heart rate/contractility. Correct answer: a) Intravenous fluid bolus. Fluid responsiveness should be assessed to avoid overload.
Q5. In cardiac tamponade preload is best described as:
a) Increased EDV with normal pressures
b) Reduced effective ventricular filling despite high venous pressures
c) Increased stroke volume
d) Elevated contractility
Explanation: Tamponade restricts diastolic filling producing high venous pressures but reduced effective ventricular volume and low stroke volume. Thus preload is functionally reduced despite raised filling pressures. Correct answer: b). Emergency pericardiocentesis restores filling and improves preload and cardiac output.
Q6. Which measurement is a right-sided preload surrogate often used at bedside?
a) Pulmonary artery diastolic pressure
b) Central venous pressure (CVP)
c) Mean arterial pressure
d) Left atrial pressure
Explanation: Central venous pressure (CVP) reflects right atrial pressure and serves as a bedside surrogate for right-sided preload, albeit with limitations. Pulmonary artery diastolic pressure and left atrial pressure better reflect left-sided filling. MAP is systemic pressure, not preload. Correct answer: b) CVP.
Q7. A patient with sepsis has vasodilation and relative hypovolemia; which change to preload is expected?
a) Increased preload due to fluid shift into vessels
b) Reduced preload due to venous pooling
c) Unchanged preload
d) Sudden increase in EDV
Explanation: Sepsis causes vasodilation and venous pooling, decreasing venous return and preload despite normal total blood volume. This lowers stroke volume and MAP unless corrected. Correct answer: b) Reduced preload due to venous pooling. Treatment includes fluids and vasopressors to restore preload and tone.
Q8. Which statement is true regarding afterload and preload?
a) Preload is the same as afterload
b) Increasing afterload increases stroke volume
c) Preload depends on venous return; afterload depends on arterial resistance
d) Neither affects cardiac output
Explanation: Preload is determined by venous return and filling volumes, while afterload is influenced by arterial resistance and arterial pressure. They are distinct and both modulate stroke volume and cardiac output. Correct answer: c). Therapeutic strategies vary: fluids for preload, vasopressors/vasodilators for afterload.
Q9. A patient with decreased ventricular compliance will have preload characterized by:
a) Low filling pressures at large volumes
b) High filling pressures at relatively small volumes
c) No change in pressure-volume relation
d) Reduced pulse pressure
Explanation: Reduced ventricular compliance means stiffer ventricle: small increases in volume cause large rises in diastolic pressure. Thus preload assessed by pressure may appear high despite modest volumes. Correct answer: b). This occurs in hypertrophy and restrictive cardiomyopathy and affects preload interpretation.
Q10. For fluid responsiveness assessment, which dynamic index best estimates preload responsiveness?
a) Static CVP value
b) Pulse pressure variation during mechanical ventilation
c) Single MAP measurement
d) Heart rate alone
Explanation: Pulse pressure variation (PPV) during controlled mechanical ventilation predicts fluid responsiveness by detecting preload-dependent stroke volume changes. Static CVP is a poor predictor. MAP and heart rate alone do not reliably indicate preload responsiveness. Correct answer: b) Pulse pressure variation.
Chapter: Cardiovascular Physiology | Topic: Blood Pressure Regulation | Subtopic: Mean Arterial Pressure (MAP) and Determinants
Keywords
Mean arterial pressure (MAP) — average arterial pressure during one cardiac cycle.
Systolic blood pressure (SBP) — peak arterial pressure during ventricular systole.
Diastolic blood pressure (DBP) — lowest arterial pressure during diastole.
Pulse pressure — SBP minus DBP; reflects stroke volume and arterial compliance.
Total peripheral resistance (TPR) — resistance to blood flow in systemic circulation.
Cardiac output (CO) — stroke volume × heart rate; determines MAP with SVR.
Autoregulation — intrinsic tissue control of blood flow within a pressure range.
Arterial compliance — arterial elasticity influencing pressure changes.
Hypertensive emergency — sudden BP rise with organ damage.
Vasopressors/vasodilators — drugs altering vascular tone and MAP.
Lead Question - 2012
Mean arterial pressure is calculated as:
a) (SBP + 2DBP) / 3
b) (DBP + 2SBP) / 3
c) (SBP + 3DBP) / 2
d) (DBP + 3SBP) / 2
Explanation: The approximate formula is (SBP + 2×DBP)/3 because diastole occupies two-thirds of the cardiac cycle. This method is valid at normal heart rates and rhythms, making option a correct. MAP reflects tissue perfusion pressure, critical for organ blood supply and often targeted in critical care settings.
Q2. A 45-year-old hypertensive patient suddenly stands and feels dizzy. Which receptor mediates rapid correction?
a) Chemoreceptors
b) Baroreceptors
c) Osmoreceptors
d) Mechanoreceptors in joints
Explanation: Baroreceptors in carotid sinus and aortic arch sense stretch changes, activating reflexes to stabilize MAP. Upon standing, reduced venous return decreases stretch, leading to sympathetic activation and BP restoration. Correct answer is b. Dysfunction causes orthostatic hypotension, often seen in autonomic neuropathies or elderly patients.
Q3. During septic shock, which factor primarily lowers MAP?
a) Increased TPR
b) Decreased TPR
c) Increased cardiac contractility
d) Elevated SBP
Explanation: In septic shock, vasodilation due to inflammatory mediators decreases systemic vascular resistance (TPR), which in turn lowers MAP despite preserved or even elevated cardiac output. Thus, the correct answer is b. Management includes vasopressors to restore vascular tone and adequate tissue perfusion.
Q4. Which parameter has the greatest direct effect on mean arterial pressure?
a) Stroke volume
b) Cardiac output
c) Venous return
d) Systemic vascular resistance
Explanation: Mean arterial pressure is primarily determined by cardiac output × systemic vascular resistance. Both cardiac output and resistance contribute, but b (cardiac output) is the immediate determinant of blood flow and pressure, especially in clinical measurements. Drugs targeting either component affect MAP significantly in critical settings.
Q5. In a patient with chronic renal failure, MAP is often elevated due to:
a) Reduced stroke volume
b) Increased sympathetic tone
c) Decreased vascular resistance
d) Loss of baroreceptor reflex
Explanation: Chronic renal failure leads to sodium and water retention, activating renin-angiotensin and sympathetic pathways, elevating systemic vascular resistance. The result is higher MAP. Correct answer is b. This mechanism explains why antihypertensives targeting the RAAS system are especially effective in renal disease–related hypertension.
Q6. Which organ maintains constant blood flow across a wide MAP range due to autoregulation?
a) Kidney
b) Skin
c) Spleen
d) Muscle
Explanation: The kidney autoregulates blood flow across MAP of 80–180 mmHg, preserving GFR despite systemic fluctuations. This is critical for excretory function. Correct answer is a. Breakdown of autoregulation in shock or severe hypotension leads to acute kidney injury, making MAP support crucial in ICU settings.
Q7. In hypertensive emergencies, lowering MAP too rapidly may cause:
a) Stroke
b) Reflex tachycardia
c) Organ hypoperfusion
d) Increased cerebral perfusion
Explanation: In chronic hypertension, autoregulation shifts to higher MAP levels. Abrupt reduction can reduce perfusion below critical threshold, causing ischemia. The correct answer is c. Guidelines recommend gradual BP lowering by 20–25% within the first hour to prevent hypoperfusion of brain, heart, and kidneys.
Q8. A patient with tachycardia will have MAP estimation error because:
a) Systolic occupies longer cycle time
b) Diastolic shortens disproportionately
c) Formula is independent of HR
d) Stroke volume decreases
Explanation: MAP formula assumes diastole is two-thirds of cardiac cycle. In tachycardia, diastole shortens disproportionately, reducing accuracy of (SBP + 2×DBP)/3. The correct answer is b. In such cases, direct invasive arterial monitoring provides more accurate MAP estimation in intensive care.
Q9. Which drug increases MAP mainly by elevating systemic vascular resistance?
a) Dobutamine
b) Noradrenaline
c) Nitroglycerin
d) Milrinone
Explanation: Noradrenaline (norepinephrine) is a potent α-adrenergic agonist that increases vascular tone, raising systemic vascular resistance and MAP. Thus, the correct answer is b. It is first-line vasopressor in septic shock. Dobutamine and milrinone act more on cardiac contractility and vasodilation, while nitroglycerin reduces MAP.
Q10. MAP is best maintained in shock resuscitation above:
a) 50 mmHg
b) 60 mmHg
c) 65 mmHg
d) 75 mmHg
Explanation: Guidelines recommend maintaining MAP ≥65 mmHg to ensure vital organ perfusion during shock resuscitation. This target balances risks of underperfusion and excessive vasopressor use. Thus, the correct answer is c. Individualization may be required for patients with chronic hypertension or intracranial pathology.
Q11. A 60-year-old with head injury requires MAP support. Adequate MAP is essential primarily to maintain:
a) Cerebral perfusion pressure
b) Pulmonary artery pressure
c) Right atrial pressure
d) Venous return
Explanation: Cerebral perfusion pressure = MAP − intracranial pressure. Adequate MAP is critical to prevent secondary brain ischemia in head injury. Therefore, the correct answer is a. Critical care guidelines recommend maintaining CPP >60 mmHg, which depends on sustaining sufficient MAP alongside reducing raised intracranial pressure.
Chapter: Cardiovascular Physiology
Topic: Cardiac Electrophysiology
Subtopic: Parasympathetic Regulation of Heart Rate
Keywords:
• Acetylcholine – parasympathetic neurotransmitter acting on muscarinic receptors
• SA Node – pacemaker of the heart controlling heart rate
• Diastolic Depolarization – slow rise of membrane potential in pacemaker cells
• Vagal Stimulation – reduces firing rate of SA node
• Chronotropy – effect on heart rate
Lead Question - 2012
Mechanism by which Ach decreases heart rate is by:
a) Delayed diastolic depolarization
b) Increase in plateau
c) Decrease preload
d) Increase afterload
Explanation: Acetylcholine (ACh) released by vagus nerve activates M2 muscarinic receptors in the SA node, increasing K⁺ efflux and reducing slope of diastolic depolarization. This slows pacemaker activity, lowering heart rate. Correct answer: (a).
Guessed Question 1
A 25-year-old athlete presents with episodes of dizziness and bradycardia. Increased vagal tone is suspected. What is the direct ionic mechanism?
a) Increased Ca²⁺ influx
b) Increased K⁺ efflux
c) Decreased Na⁺ influx
d) Increased Cl⁻ influx
Explanation: Vagal stimulation releases ACh, which increases K⁺ efflux via muscarinic K⁺ channels, hyperpolarizing SA node cells and slowing pacemaker firing. Correct answer: (b).
Guessed Question 2
Parasympathetic stimulation primarily affects which cardiac region?
a) Ventricular myocardium
b) SA node and AV node
c) Purkinje fibers
d) Papillary muscles
Explanation: Parasympathetic fibers mainly innervate the SA and AV nodes, altering conduction and rate. Ventricles have minimal vagal innervation. Correct answer: (b).
Guessed Question 3
Increased vagal activity during sleep results in:
a) Tachycardia
b) Bradycardia
c) Increased stroke volume
d) Hypertension
Explanation: During sleep, parasympathetic tone predominates, reducing heart rate and producing physiologic bradycardia without compromising cardiac output. Correct answer: (b).
Guessed Question 4
A patient receives atropine. Which of the following changes is expected?
a) Decreased heart rate
b) Increased heart rate
c) AV nodal delay
d) Enhanced vagal tone
Explanation: Atropine blocks muscarinic receptors, thereby inhibiting vagal effects and increasing heart rate. Correct answer: (b).
Guessed Question 5
Carotid sinus massage produces reflex bradycardia primarily by:
a) Increased vagal discharge
b) Increased sympathetic discharge
c) Reduced baroreceptor firing
d) Decreased preload
Explanation: Carotid massage stretches baroreceptors, enhancing vagal discharge and slowing SA node activity, producing bradycardia. Correct answer: (a).
Guessed Question 6
A 40-year-old man develops AV block due to excessive vagal stimulation. Which interval is prolonged on ECG?
a) PR interval
b) QRS duration
c) QT interval
d) ST segment
Explanation: Excessive vagal stimulation slows AV nodal conduction, prolonging the PR interval. Correct answer: (a).
Guessed Question 7
Which neurotransmitter is responsible for parasympathetic slowing of heart rate?
a) Noradrenaline
b) Dopamine
c) Acetylcholine
d) Adrenaline
Explanation: Acetylcholine released by vagal nerve endings binds muscarinic receptors, slowing the SA node pacemaker. Correct answer: (c).
Guessed Question 8
During vasovagal syncope, the patient faints due to:
a) Sympathetic overactivity
b) Combined bradycardia and vasodilation
c) Hypertension and tachycardia
d) Coronary spasm
Explanation: Vasovagal syncope results from sudden vagal discharge causing bradycardia and systemic vasodilation, leading to transient cerebral hypoperfusion. Correct answer: (b).
Guessed Question 9
Which drug enhances vagal effect on the heart and is contraindicated in bradyarrhythmias?
a) Digoxin
b) Atropine
c) Dobutamine
d) Isoprenaline
Explanation: Digoxin enhances vagal tone, slowing AV nodal conduction, beneficial in supraventricular tachyarrhythmias but contraindicated in bradycardia. Correct answer: (a).
Guessed Question 10
A 32-year-old patient with acute inferior wall MI develops sinus bradycardia. This is most likely due to:
a) Increased sympathetic tone
b) Vagal hyperactivity
c) AV node ischemia
d) Loss of pacemaker cells
Explanation: Inferior wall MI often involves the right coronary artery supplying SA node, leading to vagal hyperactivity and sinus bradycardia. Correct answer: (b).
Chapter: Cardiovascular Physiology
Topic: Electrocardiography
Subtopic: Einthoven’s Law
Keywords:
• Einthoven’s Triangle – imaginary equilateral triangle around the heart formed by limb leads
• Bipolar Limb Leads – leads I, II, III measuring potential difference between limb electrodes
• Augmented Leads – unipolar limb leads (aVR, aVL, aVF)
• Vector – direction and magnitude of electrical activity of the heart
• Lead Axis – orientation of a lead in the frontal plane
Lead Question - 2012
Einthoven’s law -
a) I + III = II
b) I - III = II
c) I + II + III = 0
d) I + III = avL
Explanation: Einthoven’s law states that in a standard ECG, the potential of lead II is equal to the sum of the potentials of leads I and III (II = I + III). This relationship helps in verifying lead placement and ECG recording accuracy. Correct answer: (a).
Guessed Question 1
A 60-year-old male presents with chest pain. His ECG shows ST elevation in leads II, III, and aVF. Which coronary artery is most likely involved?
a) Left anterior descending artery
b) Left circumflex artery
c) Right coronary artery
d) Left main coronary artery
Explanation: ST elevation in leads II, III, and aVF indicates inferior wall myocardial infarction, most commonly due to right coronary artery occlusion. Correct answer: (c).
Guessed Question 2
Lead aVR normally shows:
a) Upright P wave and QRS
b) Negative deflection of P, QRS, and T waves
c) Positive ST segment
d) No consistent wave pattern
Explanation: Lead aVR usually records negative deflections for P, QRS, and T waves because it views the heart from the right shoulder, opposite to the main vector. Correct answer: (b).
Guessed Question 3
PR interval on ECG represents:
a) Atrial depolarization
b) Conduction time from atria to ventricles
c) Ventricular depolarization
d) Atrial repolarization
Explanation: The PR interval corresponds to the time taken for impulse conduction from atria through AV node to ventricles, normally 0.12–0.20 seconds. Correct answer: (b).
Guessed Question 4
In complete heart block, the ECG shows:
a) Prolonged PR interval
b) Progressive PR prolongation
c) Dissociation between P waves and QRS complexes
d) Absent P waves
Explanation: In complete heart block, atrial and ventricular activities are independent, with P waves not related to QRS complexes. Correct answer: (c).
Guessed Question 5
QT interval on ECG corresponds to:
a) Ventricular depolarization
b) Atrial depolarization
c) Ventricular depolarization and repolarization
d) Atrial repolarization
Explanation: The QT interval represents the total duration of ventricular depolarization and repolarization. It is rate-dependent and prolongation predisposes to arrhythmias. Correct answer: (c).
Guessed Question 6
In a patient with hyperkalemia, which ECG change is most characteristic?
a) Flattened T waves
b) Tall peaked T waves
c) Short PR interval
d) Prolonged QT interval
Explanation: Hyperkalemia classically produces tall, peaked T waves due to accelerated repolarization. Severe hyperkalemia can lead to conduction block and cardiac arrest. Correct answer: (b).
Guessed Question 7
Which ECG lead best represents atrial activity?
a) Lead I
b) Lead II
c) Lead III
d) Lead aVL
Explanation: Lead II best shows atrial activity (P waves) because its axis is parallel to the atrial depolarization vector. Correct answer: (b).
Guessed Question 8
A 25-year-old male experiences sudden syncope during exercise. His ECG shows prolonged QT interval. The likely diagnosis is:
a) Brugada syndrome
b) Long QT syndrome
c) WPW syndrome
d) AV nodal reentrant tachycardia
Explanation: Long QT syndrome, congenital or acquired, predisposes to torsades de pointes and sudden death during exertion. Correct answer: (b).
Guessed Question 9
Which component of the ECG corresponds to ventricular depolarization?
a) P wave
b) QRS complex
c) T wave
d) U wave
Explanation: The QRS complex corresponds to ventricular depolarization. Normally < 120 ms, its widening indicates conduction delay or bundle branch block. Correct answer: (b).
Guessed Question 10
A 50-year-old hypertensive patient has left ventricular hypertrophy. Which ECG finding is most consistent?
a) Tall R waves in V1
b) Tall R waves in left chest leads (V5, V6)
c) Deep S waves in V5, V6
d) Low voltage QRS
Explanation: Left ventricular hypertrophy produces tall R waves in left-sided chest leads (V5, V6) and deep S waves in V1, V2 due to increased left ventricular mass. Correct answer: (b).
Chapter: Cardiovascular Physiology
Topic: Coronary Circulation
Subtopic: Regulation of Coronary Blood Flow
Keywords:
• Coronary Blood Flow – blood supply to myocardium through coronary arteries
• Perfusion Pressure – pressure gradient driving blood flow through tissues
• Vascular Resistance – opposition offered by vessels to blood flow
• Autoregulation – intrinsic ability of tissue to maintain constant flow despite pressure changes
• Oxygen Demand – myocardial requirement driving blood flow regulation
Lead Question - 2012
Which one of the following is the CORRECT statement regarding coronary blood flow?
a) Coronary blood flow is directly related to perfusion pressure and inversely related to resistance
b) Coronary blood flow is inversely related to perfusion pressure and directly related to resistance
c) Coronary blood flow is directly related to perfusion pressure and also to resistance
d) Coronary blood flow is inversely related to both pressure and resistance
Explanation: Coronary blood flow is directly proportional to perfusion pressure and inversely proportional to coronary vascular resistance. The myocardium relies on autoregulation and oxygen demand to control flow. Hence, option (a) is correct.
Guessed Question 1
In coronary circulation, maximum blood flow occurs during:
a) Systole
b) Early diastole
c) Mid-diastole
d) Late systole
Explanation: Coronary perfusion predominantly occurs during diastole due to compression of intramyocardial vessels in systole. Peak flow is in early diastole. Correct answer: (b).
Guessed Question 2
Which factor is the most important regulator of coronary blood flow?
a) Perfusion pressure
b) Myocardial oxygen demand
c) Autonomic nervous system
d) Endothelial factors
Explanation: The primary determinant of coronary blood flow is myocardial oxygen demand. Metabolites like adenosine mediate vasodilation. Hence, option (b) is correct.
Guessed Question 3
In left ventricular coronary circulation, systolic flow is reduced because:
a) Aortic valve closure
b) High intramyocardial pressure
c) Reduced perfusion pressure
d) Increased venous return
Explanation: High intramyocardial pressure during systole compresses coronary vessels, especially in the left ventricle, reducing flow. Correct answer: (b).
Guessed Question 4
Coronary flow reserve is:
a) The difference between basal and maximal coronary blood flow
b) Maximum coronary blood flow
c) Flow at rest
d) Myocardial venous return
Explanation: Coronary flow reserve is the ability of coronary circulation to increase flow above basal level in response to demand. Correct answer: (a).
Guessed Question 5
Coronary steal phenomenon occurs due to:
a) Vasodilation in stenosed vessels
b) Preferential blood flow to non-stenosed vessels
c) Coronary spasm
d) Increased venous drainage
Explanation: In stenosed vessels, distal arterioles are already maximally dilated. Vasodilators divert blood to normal vessels, reducing flow to ischemic zones—coronary steal. Correct answer: (b).
Guessed Question 6
Which substance is the most potent coronary vasodilator?
a) Adenosine
b) Nitric oxide
c) Carbon dioxide
d) Prostacyclin
Explanation: Adenosine, generated during hypoxia, is the most powerful coronary vasodilator, matching supply to oxygen demand. Correct answer: (a).
Guessed Question 7
In coronary circulation, autoregulation maintains flow between pressures of:
a) 30–60 mmHg
b) 60–140 mmHg
c) 80–180 mmHg
d) 40–100 mmHg
Explanation: Coronary autoregulation works effectively between mean arterial pressures of 60–140 mmHg, maintaining constant flow despite fluctuations. Correct answer: (b).
Guessed Question 8
Which vessel supplies blood to the sinoatrial (SA) node in most individuals?
a) Right coronary artery
b) Left coronary artery
c) Circumflex artery
d) Anterior interventricular artery
Explanation: In ~60% of cases, the SA node is supplied by the right coronary artery; in others, the circumflex. Correct answer: (a).
Guessed Question 9
A patient develops chest pain at rest due to coronary vasospasm. This condition is termed:
a) Stable angina
b) Unstable angina
c) Prinzmetal’s angina
d) Silent ischemia
Explanation: Prinzmetal’s (variant) angina occurs due to transient coronary vasospasm at rest, often with ST elevation. Correct answer: (c).
Guessed Question 10
The major site of resistance in coronary circulation is:
a) Epicardial arteries
b) Arterioles
c) Capillaries
d) Venules
Explanation: Coronary arterioles are the main resistance vessels controlling coronary flow. Correct answer: (b).
Keywords (for all questions)
Volume (low-pressure) receptors: Mechanosensitive afferents in atria and pulmonary veins sensing central blood volume and atrial distension.
Afferents: Travel mainly in the vagus (cranial nerve X) to medullary and hypothalamic centers; carotid sinus uses glossopharyngeal (IX).
Physiologic effects: Modulate thirst, vasopressin (ADH) secretion, renal sympathetic tone, natriuresis, and diuresis according to central volume status.
Distension signal: Increased atrial stretch → increased firing; decreased stretch → reduced firing and activation of compensatory neurohormonal responses.
Interaction with hormones: Work with angiotensin II, ANP and osmoreceptors to regulate fluid balance and blood pressure.
Clinical relevance: Dysfunction or resetting in heart failure and sepsis alters ADH, RAAS, and thirst responses, contributing to fluid retention or inappropriate diuresis.
Chapter: Cardiovascular & Endocrine Integration | Topic: Volume Receptors & Fluid Homeostasis | Subtopic: Physiology of Low-Pressure Baroreceptors
Lead Question – 2012
True about volume receptors are all, EXCEPT:
a) They are low pressure receptors
b) They provide afferents for thirst control
c) They are located in carotid sinus
d) They mediate vasopressin release
Explanation: Volume receptors are low-pressure mechanoreceptors in atria/pulmonary vessels signaling central blood volume to hypothalamus; they influence thirst and vasopressin release. They are not located in the carotid sinus (a high-pressure arterial baroreceptor site). Answer: c) They are located in carotid sinus which is incorrect here.
1) Where are the primary volume receptors located?
a) Atria
b) Carotid sinus
c) Kidneys
d) Aorta
Explanation: Cardiac volume receptors are low-pressure receptors residing primarily in the atria and pulmonary veins; they detect central venous pressure and blood volume changes. Activation modifies autonomic outflow, thirst, and vasopressin release. They are distinct from high-pressure arterial baroreceptors in carotid sinus and aortic arch. Answer: a) Atria for volume sensing purposes.
2) Which nerve carries afferents from cardiac volume receptors?
a) Vagus nerve (X)
b) Glossopharyngeal nerve (IX)
c) Phrenic nerve
d) Sympathetic cardiac nerves
Explanation: Low-pressure volume receptor afferents travel primarily via the vagus (cranial nerve X) to medullary and hypothalamic centers, conveying central blood volume information. These signals alter sympathetic tone, thirst, and vasopressin secretion. Glossopharyngeal nerves convey high-pressure carotid baroreceptor input, not volume receptor input. Answer: a) Vagus nerve for reflex regulation.
3) Volume receptors respond to which mechanical change?
a) Increase firing with atrial distension
b) Decrease firing with atrial distension
c) Respond only to chemical stimuli
d) Respond only to arterial pressure
Explanation: Volume receptors are mechanosensitive nerve endings that increase firing when atrial and pulmonary venous walls are stretched by increased blood volume or central venous pressure. Elevated afferent activity suppresses vasopressin and stimulates diuresis and natriuresis; reduced firing during hypovolemia promotes thirst and vasopressin release mechanism. Answer: a) Increase with distension.
4) Activation of volume receptors causes which effect on vasopressin (ADH)?
a) Decrease vasopressin secretion
b) Increase vasopressin secretion
c) No effect on vasopressin
d) Variable effect unrelated to volume
Explanation: Atrial volume receptor activation with increased central blood volume inhibits vasopressin secretion from the posterior pituitary via hypothalamic pathways, promoting water excretion. Conversely, reduced receptor firing during hypovolemia removes inhibition, causing vasopressin release. Thus volume receptors indirectly regulate plasma osmolality and volume through antidiuretic hormone modulation. Answer: b) Decrease vasopressin.
5) Do volume receptors contribute afferents for thirst control?
a) Yes
b) No
c) Only in dehydration
d) Only in overhydration
Explanation: Low-pressure volume receptor signals influence hypothalamic osmoregulatory centers and contribute to thirst modulation: decreased atrial filling reduces afferent firing, triggering thirst and drinking behavior, while increased filling suppresses thirst. They act alongside osmoreceptors and angiotensin II to control fluid intake, integrating volume and osmotic signals. Answer: a) Yes they do.
6) Which statement correctly classifies volume receptors?
a) Low-pressure baroreceptors
b) High-pressure arterial baroreceptors
c) Osmoreceptors
d) Chemoreceptors
Explanation: Volume receptors are low-pressure baroreceptors located in the atria and pulmonary veins; they directly sense venous return and central blood volume rather than arterial pressure. High-pressure baroreceptors in carotid sinus and aortic arch monitor arterial pressure. Renal afferents provide signals related to tubular flow and renin release. Answer: a) Atria.
7) Activation of volume receptors promotes which renal effect?
a) Natriuresis and diuresis
b) Antinatriuresis
c) No renal effect
d) Direct renin inhibition only
Explanation: Atrial volume receptor activation during increased central blood volume initiates reflex pathways promoting natriuresis and diuresis by decreasing sympathetic renal tone and facilitating atrial natriuretic peptide release. These responses lower blood volume and pressure to restore homeostasis. Dysfunction in heart failure impairs these reflexes, contributing to fluid retention. Answer: a) True.
8) Volume receptors sense primarily which parameter?
a) Volume/atrial stretch
b) Arterial pressure
c) Plasma osmolarity
d) Blood oxygen tension
Explanation: Volume receptors respond to changes in intravascular volume and atrial distension (stretch), not directly to osmolarity or oxygen tension. They transduce mechanical deformation into neural signals modulating thirst, vasopressin, sympathetic tone, and renal handling. Their sensitivity integrates with hormonal signals like angiotensin II for coordinated volume regulation. Answer: b) Volume/stretch.
9) Which afferent transmits carotid sinus baroreceptor signals?
a) Glossopharyngeal nerve (IX)
b) Vagus nerve (X)
c) Phrenic nerve
d) Sympathetic chain
Explanation: High-pressure baroreceptors in the carotid sinus transmit afferent signals via the glossopharyngeal nerve (cranial nerve IX) to the nucleus tractus solitarius, regulating sympathetic outflow and heart rate. These sensors monitor arterial pressure rapidly, complementing low-pressure volume receptors that monitor venous return and blood volume. Answer: b) Glossopharyngeal nerve (IX) clinically.
10) Reduced firing of volume receptors triggers which hormonal cascade?
a) Renin-angiotensin-aldosterone activation
b) Immediate ANP surge only
c) Direct insulin release
d) Increase in pulmonary surfactant
Explanation: When central volume receptors sense reduced atrial filling they decrease afferent firing, triggering compensatory neurohumoral responses including sympathetic activation and renin release from kidneys, which increases angiotensin II and aldosterone. This conserves sodium and water to restore effective circulating volume. Clinically contributes to fluid retention in hypovolemia. Answer: a) True.
Keywords (for all questions)
Second heart sound (S2): Produced by closure of aortic (A2) and pulmonary (P2) valves; its timing and components reflect great-vessel and ventricular ejection dynamics.
Components A2 and P2: A2 corresponds to aortic valve closure, P2 to pulmonary valve closure; normal respiratory variation alters their relative timing.
Physiologic split: Normal inspiratory widening of S2 due to increased right-sided ejection time and delayed P2.
Fixed split: Unchanged by respiration; classically seen in atrial septal defect (ASD).
Paradoxical split: Split that narrows or reverses on inspiration; occurs with delayed A2 (e.g., LBBB, severe AS).
Pericardial knock: Early diastolic sound in constrictive pericarditis, occurring earlier than S3 and differing in timing and pitch.
Auscultation tips: Use the diaphragm at the base for higher frequency components and the bell at the apex for low-frequency diastolic sounds when appropriate.
Phonocardiography: Instrumental method to record and measure sound timing and duration precisely.
Clinical relevance: Changes in S2 duration, splitting, or intensity help diagnose valvular disease, conduction defects, and hemodynamic abnormalities.
Chapter: Cardiovascular Examination | Topic: Heart Sounds & Murmurs | Subtopic: Second Heart Sound (S2) Duration
Lead Question – 2012
Duration of 2" heart sound is ?
a) 0.15 sec
b) 0.12 sec
c) 0.08 sec
d) 0.1 sec
Explanation: The normal second heart sound (S2) is brief, representing aortic and pulmonary valve closure. Typical duration is about 0.08 seconds due to rapid valve deceleration. Prolongation suggests pathology. Answer: c) 0.08 sec. It is best heard at the cardiac base with the diaphragm during quiet respiration in most healthy adults.
1) S2 components consist of:
a) A2 only
b) A2 & P2
c) P2 only
d) M1 & T1
Explanation: S2 comprises two nearly simultaneous components: A2 from aortic valve closure and P2 from pulmonary valve closure. Their timing varies with respiration. The split widens on inspiration due to increased venous return and delayed P2. Clinically, S2 components reflect great vessel dynamics. Answer: b) A2 & P2 in normal subjects.
2) Physiologic splitting of S2 varies with respiration because:
a) Decreased systemic vascular resistance
b) Increased venous return delays P2
c) Left atrial contraction delays A2
d) Pulmonary embolism accelerates P2
Explanation: Physiologic splitting of S2 widens during inspiration because increased venous return prolongs right ventricular ejection, delaying pulmonary valve closure (P2). Simultaneously, reduced left ventricular filling slightly shortens A2. These dynamic changes produce normal respiratory variation in S2 that distinguishes physiologic from fixed splits. Answer: b) Increased venous return delays P2.
3) Fixed wide splitting of S2 is characteristic of:
a) Ventricular septal defect
b) Left bundle branch block
c) Atrial septal defect (ASD)
d) Mitral stenosis
Explanation: Fixed wide splitting of S2, unchanged by respiration, is characteristic of atrial septal defect due to persistent right ventricular volume overload causing consistently delayed P2. Unlike physiologic splitting, fixed splitting persists during inspiration and expiration, aiding diagnosis. Other causes are rare. Answer: c) Atrial Septal Defect (ASD) on auscultation commonly.
4) Paradoxical splitting of S2 is typically due to:
a) Right bundle branch block
b) Left bundle branch block (LBBB)
c) Atrial septal defect
d) Pulmonary embolism
Explanation: Paradoxical splitting occurs when A2 is delayed, causing the split to narrow or reverse with inspiration; typical causes include left bundle branch block or severe aortic stenosis. The delayed aortic valve closure makes P2 precede A2, producing paradoxical timing that increases with expiration. Answer: b) Left bundle branch block (LBBB).
5) A pericardial knock is most characteristic of:
a) Dilated cardiomyopathy
b) Constrictive pericarditis
c) Mitral regurgitation
d) Atrial fibrillation
Explanation: A pericardial knock is an early diastolic high-amplitude sound occurring in constrictive pericarditis due to sudden cessation of ventricular filling when rigid pericardium limits expansion. It occurs earlier than S3 and is distinguished by timing, high intensity, and association with signs of constriction. Answer: b) Constrictive pericarditis on cardiac auscultation commonly.
6) Inspiration typically causes which change in S2?
a) Split narrows on inspiration
b) Split widens on inspiration
c) S2 disappears on inspiration
d) S2 becomes continuous
Explanation: Inspiration increases venous return to the right heart, prolonging right ventricular ejection and delaying pulmonary valve closure, widening the physiologic split of S2. Simultaneously, reduced left ventricular filling may slightly advance A2. These respiratory mechanics explain normal variation in S2, useful clinically. Answer: b) Split widens on inspiration in healthy.
7) A single (unsplit) S2 is commonly seen in:
a) Aortic stenosis
b) Atrial septal defect
c) Pulmonary hypertension only
d) Mitral stenosis only
Explanation: A single S2 occurs when A2 and P2 are closely synchronous or one component is inaudible. Severe aortic stenosis often produces a single loud A2 because P2 may be soft; alternatively, pulmonary hypertension can accentuate P2. In either case components merge, producing a solitary second sound. Answer: a) Aortic stenosis commonly.
8) Compared to S1, the duration of S2 is generally:
a) Shorter than S1
b) Longer than S1
c) Identical to S1
d) Variable only in children
Explanation: S2 is usually shorter than S1 because semilunar valve closure produces a brisk, brief vibration. S1 arises from mitral and tricuspid valve closure with greater myocardial involvement, yielding longer duration. Thus S2’s brevity reflects rapid valve recoil and minimal sustained tissue vibration. Answer: a) Shorter than S1 in most adults.
9) Prolongation of S2 duration is most suggestive of:
a) Aortic stenosis
b) Mitral stenosis
c) Pericarditis only
d) Tricuspid atresia
Explanation: Prolongation of the second heart sound indicates delayed semilunar valve closure often from prolonged ejection time. Severe aortic stenosis prolongs left ventricular ejection, delaying A2 and lengthening overall S2 duration. Clinical correlation with murmurs and ECG (e.g., LBBB) helps differentiate causes. Answer: a) Aortic stenosis commonly.
10) The best method to measure precise S2 duration is:
a) Phonocardiography
b) Handheld stethoscope
c) Standard ECG
d) Chest X-ray
Explanation: Phonocardiography provides objective visual and temporal measurement of heart sound durations including S2 by converting acoustic signals into tracings, allowing precise measurement in seconds and detection of subtle splitting or knocks. Auscultation is subjective; echocardiography assesses structure not sound timing directly. Answer: a) Phonocardiography is preferred for exact duration measurement.
Keywords (for all questions)
Third heart sound (S3): Low-frequency early diastolic sound produced during rapid ventricular filling; best heard with bell at apex in left lateral position.
Rapid filling phase: Early diastole immediately after opening of AV valves when ventricles fill briskly from atria.
Physiologic vs pathologic S3: Physiologic in young/athletes/pregnancy; pathologic in volume-overload states and systolic dysfunction.
Associated lesions: VSD, ASD, mitral regurgitation, dilated cardiomyopathy often show S3 due to increased flow or reduced compliance.
Pericardial knock: Early diastolic sound in constrictive pericarditis; distinct from S3 by timing and etiology.
S4: Late diastolic (presystolic) low-frequency sound due to atrial contraction into a stiff ventricle; contrasts with S3 timing.
Optimal auscultation: Bell, low pitch, apex in left lateral decubitus increases S3 audibility.
Clinical significance: Pathologic S3 indicates increased filling pressures and reduced systolic function; prognostic for heart failure.
Hemodynamic correlate: Rapid deceleration of inflow or increased compliance with high flow creates vibration of ventricular walls producing S3.
Age considerations: S3 common in children and young adults (normal) but abnormal if new in older adults.
Chapter: Cardiovascular Examination | Topic: Heart Sounds & Murmurs | Subtopic: Third Heart Sound (S3)
Lead Question – 2012
All of the following statements about third Heart sound (S3) are true, except:
a) Occurs due to rapid filling of the ventricles during atrial systole
b) Seen in Constrictive Pericarditis
c) Seen in Atrial Septal Defect (ASD)
d) Seen in Ventricular Septal Defect (VSD)
Explanation: S3 is an early diastolic sound from rapid passive ventricular filling (not atrial systole). It appears in volume-overload states (VSD, ASD) and systolic dysfunction. Constrictive pericarditis typically produces a pericardial knock (early diastolic) rather than a true S3, but S3 can occasionally be heard. Answer: a) Occurs due to rapid filling of the ventricles during atrial systole.
1) The classical timing of S3 is:
a) Late diastole (presystolic)
b) Early diastole (rapid filling)
c) Mid-systole
d) Continuous through systole and diastole
Explanation: S3 occurs in early diastole during the rapid filling phase soon after S2. It is not presystolic (that is S4). S3 reflects ventricular wall vibrations from brisk inflow; clinically significant when new in older adults and associated with heart failure or volume overload. Answer: b) Early diastole (rapid filling).
2) Best method to auscultate an S3 is:
a) Diaphragm at left sternal border
b) Bell at apex with patient in left lateral decubitus
c) Bell at right midclavicular line
d) Diaphragm over epigastrium
Explanation: S3 is a low-frequency sound best detected using the bell at the cardiac apex with the patient lying in the left lateral decubitus position, which brings the left ventricle closer to the chest wall. Diaphragm and other positions are less sensitive for low-frequency S3. Answer: b) Bell at apex with patient in left lateral decubitus.
3) A newly developed S3 in an elderly patient most likely indicates:
a) Normal aging
b) Reduced left ventricular systolic function
c) Atrial fibrillation only
d) Isolated hypertension without dysfunction
Explanation: A new S3 in an older adult commonly signifies impaired left ventricular systolic function and increased filling pressures, often preceding overt congestive heart failure. In contrast, physiologic S3 is typical in young people; atrial fibrillation or hypertension alone do not typically produce an isolated new S3. Answer: b) Reduced left ventricular systolic function.
4) S3 is most commonly heard in which of the following conditions?
a) Hypertrophic cardiomyopathy
b) Dilated cardiomyopathy
c) Aortic stenosis without regurgitation
d) Isolated pericarditis without effusion
Explanation: Dilated cardiomyopathy features increased chamber volumes and reduced systolic function, predisposing to an S3 from vigorous early filling and wall vibration. Hypertrophic cardiomyopathy more often has S4; aortic stenosis and simple pericarditis without volume overload do not classically produce S3. Answer: b) Dilated cardiomyopathy.
5) Which physiologic state can produce a benign S3?
a) Elderly sedentary adult
b) Healthy young athlete
c) Chronic heart failure
d) Acute myocardial infarction
Explanation: In young healthy individuals and well-trained athletes, increased cardiac output and compliant ventricles can produce a physiologic S3 without pathology. In contrast, elderly persons, heart failure, or recent MI with new S3 usually indicate pathology and require evaluation. Answer: b) Healthy young athlete.
6) Which auscultatory feature helps distinguish S3 from a pericardial knock?
a) S3 is high-pitched
b) Pericardial knock occurs earlier in diastole than S3
c) S3 is louder with expiration
d) Pericardial knock is only audible at the apex
Explanation: A pericardial knock typically occurs earlier in diastole, soon after S2, due to abrupt cessation of ventricular filling from a rigid pericardium; S3 follows slightly later in the rapid filling phase. Pitch and positional changes help differentiate them clinically. Answer: b) Pericardial knock occurs earlier in diastole than S3.
7) In mitral regurgitation, S3 occurs because of:
a) Decreased left atrial pressure
b) Volume overload of left ventricle
c) Increased ventricular stiffness only
d) Mitral valve prolapse exclusively
Explanation: Chronic mitral regurgitation causes volume overload of the left ventricle with increased early diastolic filling, producing an S3. Ventricular stiffness relates more to S4; mitral valve prolapse can have varied sounds but is not the exclusive cause of S3. Answer: b) Volume overload of left ventricle.
8) Which statement about S3 frequency and auscultation tool is correct?
a) High-frequency sound heard with diaphragm
b) Low-frequency sound best heard with bell
c) Audible only with electronic stethoscopes
d) Best heard over carotids
Explanation: S3 is a low-frequency vibration best detected using the bell of the stethoscope, placed lightly over the apex in left lateral decubitus. The diaphragm is less sensitive to low-frequency sounds; carotid auscultation is for bruits and systolic murmurs, not S3. Answer: b) Low-frequency sound best heard with bell.
9) A VSD produces an S3 because it causes:
a) Left ventricular outflow obstruction
b) Increased pulmonary venous pressure only
c) Increased volume flow and early diastolic filling
d) Isolated right atrial enlargement
Explanation: A ventricular septal defect increases left-to-right shunt and volume load on ventricles, causing brisk early diastolic filling and vibrations that produce an S3. It is not due to outflow obstruction or isolated atrial changes. Clinical assessment includes murmur plus possible S3. Answer: c) Increased volume flow and early diastolic filling.
10) Which maneuver increases the intensity of a pathologic S3?
a) Valsalva strain phase
b) Leg elevation to increase venous return
c) Standing from supine
d) Deep inspiration only
Explanation: Increasing venous return (leg raise or supine positioning) augments early diastolic filling and may increase S3 intensity in volume-overloaded ventricles. Valsalva and standing decrease venous return and generally lessen S3; deep inspiration has minor effects on left-sided S3. Answer: b) Leg elevation to increase venous return.
11) How does S3 prognosis differ with age?
a) Always ominous regardless of age
b) Benign if young, pathologic if new in older adults
c) Indicative of pulmonary embolism only
d) Only related to valvular calcification
Explanation: An S3 in children or young adults is often physiologic and benign. In older adults, a newly detected S3 usually indicates increased ventricular filling pressures and systolic dysfunction, carrying worse prognosis; it warrants evaluation for heart failure or volume overload causes. Answer: b) Benign if young, pathologic if new in older adults.
Keywords (for all questions)
Microcirculation: Network of arterioles, capillaries, venules (± metarterioles, thoroughfare channels) where exchange occurs.
Precapillary sphincter: Smooth muscle collar regulating capillary perfusion/recruitment.
Starling forces (revised): Filtration depends on hydrostatic/oncotic pressures and endothelial glycocalyx.
Glycocalyx: Endothelial surface layer modulating permeability and oncotic gradient; degraded in sepsis.
Functional hyperemia: ↑Flow to active tissues via metabolites (adenosine, CO₂, H⁺, K⁺).
Reactive hyperemia: Transient ↑Flow after occlusion due to vasodilator washout and myogenic response.
Autoregulation: Local maintenance of flow vs pressure via myogenic/metabolic mechanisms.
Poiseuille’s law: Resistance ∝ viscosity × length / radius⁴; small radius changes drastically alter flow.
Fåhræus–Lindqvist effect: Apparent blood viscosity falls in small vessels (down to ~10–300 μm).
Diffusion distance: Capillaries keep cells within ~100 μm for adequate O₂ exchange.
Capillary types: Continuous (muscle, brain), fenestrated (kidney, intestine), sinusoidal (liver, spleen).
NO/Endothelin: Endothelial vasodilator (NO) and vasoconstrictor (endothelin-1) balancing tone.
Capillary hydrostatic pressure (Pc): ~15–35 mmHg; higher at arteriolar end.
Lymphatics: Return filtered fluid/proteins; failure → edema.
Sepsis microvascular dysfunction: Glycocalyx loss, shunting, ↑permeability → lactate rise.
Chapter: Cardiovascular Physiology | Topic: Microcirculation | Subtopic: Structure and Function
Lead Question – 2012
Microcirculation consists of ?
a) Capillaries
b) Capillaries venules and arterioles
c) Aorta
d) Arteries and veins
Explanation: Microcirculation refers to the smallest vessels involved in exchange and resistance control—terminal arterioles, capillaries, and venules (often including metarterioles/thoroughfare channels). Large arteries, veins, and the aorta are macrocirculatory. Exchange and Starling flux occur at this level. Answer: b) Capillaries venules and arterioles.
1) A runner’s active skeletal muscle shows increased flow. The predominant cause at the microvascular level is:
a) Sympathetic α1 vasoconstriction
b) Local metabolic vasodilation
c) Increased venous pressure
d) Baroreceptor unloading
Explanation: Functional hyperemia is driven by local metabolites (adenosine, CO₂, K⁺, H⁺) opening precapillary sphincters and dilating arterioles, recruiting capillaries and boosting exchange. Sympathetic activity is overridden locally during exercise. Venous pressure/baroreflex are not primary drivers here. Answer: b) Local metabolic vasodilation.
2) Which structure directly regulates the entry of blood into true capillaries?
a) Postcapillary venule
b) Precapillary sphincter
c) Vasa vasorum
d) Sinusoid
Explanation: Precapillary sphincters are smooth muscle cuffs at capillary origins from metarterioles/arterioles. Their tone modulates capillary recruitment and surface area for exchange according to local metabolic demand. Postcapillary venules mainly handle leukocyte trafficking and fluid reabsorption. Answer: b) Precapillary sphincter.
3) A patient with sepsis develops edema despite normal hydrostatic pressures. The best explanation is:
a) Increased plasma oncotic pressure
b) Endothelial glycocalyx degradation
c) Reduced interstitial compliance
d) Lymphatic hyperactivity
Explanation: Sepsis damages the glycocalyx, increasing permeability and altering the effective oncotic gradient in Starling forces, promoting filtration and interstitial edema even with modest pressures. Plasma oncotic rises would oppose, not favor, edema. Lymphatics may be overwhelmed, not “hyperactive.” Answer: b) Endothelial glycocalyx degradation.
4) In which capillary type is bulk protein passage physiologically greatest?
a) Continuous (brain)
b) Fenestrated (glomerulus)
c) Sinusoidal (liver)
d) Continuous (muscle)
Explanation: Liver sinusoids have discontinuous endothelium and incomplete basement membrane, allowing protein exchange between plasma and space of Disse; this supports albumin synthesis. Brain continuous capillaries with tight junctions severely restrict proteins. Fenestrated glomeruli filter little protein due to charge/size selectivity. Answer: c) Sinusoidal (liver).
5) A 65-year-old with uncontrolled hypertension has reduced tissue perfusion in toes. According to Poiseuille’s law, which change most powerfully improves flow?
a) 10% decrease in viscosity
b) 10% increase in vessel radius
c) 10% decrease in vessel length
d) 10% increase in pressure
Explanation: Resistance varies inversely with radius⁴, so small increases in arteriolar radius markedly reduce resistance and enhance flow, dominating effects of viscosity, length, or pressure in microvessels. Thus vasodilation is the most potent intervention. Answer: b) 10% increase in vessel radius.
6) At the arteriolar end of a typical systemic capillary, which relationship favors filtration?
a) Pc < πc
b) Pc > πc
c) Pi > Pc
d) πi > πc
Explanation: Higher capillary hydrostatic pressure (Pc) at the arteriolar end exceeds capillary oncotic pressure (πc), favoring filtration; toward the venular end, πc dominates, favoring reabsorption (modulated by glycocalyx). Interstitial pressure typically remains low/slightly negative. Answer: b) Pc > πc.
7) A limb is occluded for two minutes and then released. The marked flushing is due to:
a) Decreased tissue PCO₂
b) Accumulated vasodilators and myogenic relaxation
c) Increased sympathetic tone
d) Venoconstriction
Explanation: Reactive hyperemia follows brief ischemia: metabolites (adenosine, CO₂, H⁺, K⁺) and myogenic relaxation dilate arterioles; upon release, flow overshoots until vasodilators wash out and tone normalizes. Sympathetic activity is not the primary determinant. Answer: b) Accumulated vasodilators and myogenic relaxation.
8) Which venule segment is the principal site of leukocyte adhesion and increased permeability in inflammation?
a) Terminal arteriole
b) Postcapillary venule
c) Muscular venule
d) Collecting vein
Explanation: Postcapillary venules express adhesion molecules (e.g., selectins, ICAM) and exhibit gap formation with inflammatory mediators, promoting leukocyte transmigration and fluid extravasation—key microcirculatory responses in acute inflammation. Answer: b) Postcapillary venule.
9) In severe anemia, tissue oxygen delivery is maintained partly by microvascular changes including:
a) Increased viscosity raising resistance
b) Fåhræus–Lindqvist effect lowering apparent viscosity
c) Decreased capillary recruitment
d) Diffusion distance increase
Explanation: In small vessels, apparent viscosity falls (Fåhræus–Lindqvist effect), reducing resistance and improving microvascular flow. Capillary recruitment and increased cardiac output also sustain delivery; diffusion distance tends to decrease with recruitment, aiding exchange. Answer: b) Fåhræus–Lindqvist effect lowering apparent viscosity.
10) A patient with nephrotic syndrome develops pitting edema. The most proximate microcirculatory cause is:
a) Increased plasma oncotic pressure
b) Decreased plasma oncotic pressure
c) Decreased capillary hydrostatic pressure
d) Increased lymphatic pumping
Explanation: Hypoalbuminemia lowers capillary oncotic pressure, tipping Starling balance toward filtration across systemic microvessels; lymphatics initially compensate but are overwhelmed, producing interstitial fluid accumulation and pitting edema. Answer: b) Decreased plasma oncotic pressure.
11) Nitric oxide infusion into a limb primarily causes which microvascular change?
a) Arteriolar dilation and capillary recruitment
b) Venular constriction with reduced filtration
c) Precapillary sphincter constriction
d) Increased blood viscosity
Explanation: NO relaxes arteriolar smooth muscle, lowering resistance and opening precapillary sphincters; more capillaries are perfused (recruitment), enlarging surface area for exchange and improving oxygen delivery. Venules are less responsive; viscosity is unaffected. Answer: a) Arteriolar dilation and capillary recruitment.
Keyword Definitions
2,3-DPG - Red cell metabolite that binds deoxygenated hemoglobin and lowers O₂ affinity.
O₂–Hb dissociation curve - Relationship between PaO₂ and hemoglobin saturation; shifts indicate affinity changes.
Right shift - Reduced Hb O₂ affinity; facilitates O₂ release to tissues (favored by ↑2,3-DPG, ↑CO₂, ↑H⁺, ↑T).
Left shift - Increased Hb O₂ affinity; impairs O₂ unloading (favored by ↓2,3-DPG, ↓CO₂, ↓H⁺, ↓T, fetal Hb).
PaO₂ - Partial pressure of oxygen in arterial blood; primary determinant of SaO₂ at physiologic range.
SaO₂ / SvO₂ - Arterial and venous hemoglobin oxygen saturations; reflect oxygen loading/unloading balance.
Hypoxia - Reduced tissue oxygen delivery; stimulates adaptive increases in 2,3-DPG over days.
Anemia - Reduced O₂ content that can trigger higher 2,3-DPG to improve tissue O₂ delivery.
Acidosis - Increased H⁺ promotes right shift (Bohr effect), complementing 2,3-DPG effects.
Transfusion storage - Stored RBCs lose 2,3-DPG; transfused blood may transiently impair O₂ unloading until levels recover.
Chapter: Respiratory Physiology Topic: Hemoglobin & Gas Transport Subtopic: 2,3-DPG and O₂–Hb Affinity
Lead Question – 2012
Which of the following is/are effect of increased 2,3-DPG on oxygen-hemoglobin dissociation curve?
a) ↑ ed affinity of heamoglobin to oxygen
b) ↓ ed affinity of haemoglobin to oxygen
c) Left shift of oxygen-hemoglobin dissociation curve
d) Right shift of oxygen-hemoglobin dissociation curve
e) No change in oxygen-hemoglobin dissociation curve
Explanation: Increased 2,3-DPG binds deoxygenated hemoglobin, reducing O₂ affinity and causing a rightward shift of the O₂–Hb curve; this facilitates oxygen unloading to tissues (eg, chronic hypoxia, anemia). Therefore options b) (decreased affinity) and d) (right shift) are correct together, not a, c, or e.
1) In chronic hypoxemia, rise in erythrocyte 2,3-DPG primarily serves to:
a) Increase arterial PaO₂
b) Enhance tissue O₂ unloading
c) Increase Hb affinity for O₂
d) Promote left shift of O₂–Hb curve
Explanation: Chronic hypoxemia stimulates red cell 2,3-DPG synthesis, lowering hemoglobin O₂ affinity and shifting the curve right. This adaptation increases tissue oxygen delivery despite low PaO₂. Clinically seen in high altitude and chronic lung disease. Answer: b) Enhance tissue O₂ unloading.
2) Stored packed red blood cells may impair immediate O₂ delivery because:
a) They have elevated 2,3-DPG
b) They have reduced 2,3-DPG
c) They shift O₂–Hb curve right
d) They carry more CO₂
Explanation: During storage, RBCs lose 2,3-DPG, increasing hemoglobin O₂ affinity (left shift) and temporarily reducing tissue unloading after transfusion. 2,3-DPG regenerates over 24–72 hours in recipient cells. Therefore answer: b) They have reduced 2,3-DPG.
3) Which combination most strongly produces a right shift similar to ↑2,3-DPG?
a) Alkalosis, hypothermia
b) Acidosis, hypercapnia
c) High fetal hemoglobin, low 2,3-DPG
d) Low CO₂, low temperature
Explanation: Acidosis and hypercapnia increase H⁺ and CO₂, promoting a right shift (Bohr effect) that, like 2,3-DPG, reduces O₂ affinity and enhances unloading in tissues. Clinically present during exercise or sepsis. Answer: b) Acidosis, hypercapnia.
4) A patient with anemia adapts by increasing 2,3-DPG. Expected change in venous O₂ saturation (SvO₂) is:
a) Increased SvO₂
b) Decreased SvO₂
c) No change in SvO₂
d) SvO₂ equals SaO₂
Explanation: Increased 2,3-DPG lowers Hb O₂ affinity causing greater peripheral extraction and lower SvO₂ (larger a–v O₂ difference). Although arterial saturation may be preserved, venous saturation falls, reflecting enhanced unloading in tissues. Answer: b) Decreased SvO₂.
5) Which clinical state would most likely show elevated 2,3-DPG?
a) Recent blood transfusion with old stored blood
b) High-altitude acclimatization over days
c) Acute carbon monoxide poisoning
d) Hypothermia during surgery
Explanation: High-altitude acclimatization stimulates erythrocyte glycolysis and 2,3-DPG production over days to weeks, enhancing tissue O₂ delivery despite low PaO₂. Acute transfusion of stored blood lowers 2,3-DPG transiently; CO poisoning and hypothermia reduce delivery. Answer: b) High-altitude acclimatization.
6) Which hemoglobin variant interaction contrasts with 2,3-DPG effects?
a) Adult HbA has lower O₂ affinity when 2,3-DPG high
b) Fetal Hb (HbF) binds 2,3-DPG more avidly than HbA
c) HbF has higher O₂ affinity and binds less 2,3-DPG than HbA
d) HbS increases 2,3-DPG binding dramatically
Explanation: Fetal hemoglobin (HbF) has reduced binding to 2,3-DPG, giving it increased O₂ affinity (left shift) compared with adult HbA. This facilitates placental O₂ transfer opposite to effects of ↑2,3-DPG. Answer: c) HbF has higher O₂ affinity and binds less 2,3-DPG than HbA.
7) In sepsis with high metabolic demand and tissue hypoxia, what happens to 2,3-DPG and oxygen unloading?
a) 2,3-DPG falls and unloading decreases
b) 2,3-DPG rises and unloading increases
c) 2,3-DPG unchanged
d) 2,3-DPG rises but unloading decreases
Explanation: Tissue hypoxia and increased glycolytic flux in sepsis can elevate 2,3-DPG, diminishing Hb O₂ affinity and enhancing peripheral oxygen unloading. Combined with local acidosis and temperature rise, this improves oxygen delivery to metabolically active tissues. Answer: b) 2,3-DPG rises and unloading increases.
8) Which therapeutic action would counteract a right shift caused by elevated 2,3-DPG?
a) Administer warmed fluids to raise temperature
b) Give 100% oxygen to raise PaO₂ and promote left shift
c) Induce mild acidosis
d) Give transfusion of fresh stored RBCs low in 2,3-DPG
Explanation: Transfusion of fresh (or stored) RBCs low in 2,3-DPG can transiently increase Hb O₂ affinity (left shift), countering right shift effects. High inspired O₂ raises PaO₂ but does not directly reverse biochemical 2,3-DPG effects on Hb binding; answer: d) Give transfusion of fresh stored RBCs low in 2,3-DPG.
9) Which lab feature indirectly suggests increased 2,3-DPG activity clinically?
a) Increased arterial O₂ content with low extraction
b) Normal SaO₂ with low SvO₂ and increased a–v O₂ difference
c) Elevated PaO₂ with increased SaO₂
d) High carboxyhemoglobin
Explanation: Increased 2,3-DPG enhances tissue extraction causing lower venous saturation and a larger a–v O₂ difference while arterial saturation may appear unchanged. Thus b) Normal SaO₂ with low SvO₂ and increased a–v O₂ difference is indicative of enhanced peripheral unloading consistent with elevated 2,3-DPG.
10) Which statement about 2,3-DPG kinetics after transfusion of stored blood is correct?
a) Recipient’s RBCs restore 2,3-DPG in donor cells immediately
b) 2,3-DPG levels in transfused RBCs regenerate over 24–72 hours
c) 2,3-DPG never recovers in stored RBCs after transfusion
d) Storage increases 2,3-DPG so regeneration is unnecessary
Explanation: Stored RBCs have depleted 2,3-DPG that regenerates after transfusion in the recipient’s circulation over 24–72 hours as glycolytic metabolism resumes, restoring normal O₂ unloading capacity. Clinically this transient left shift may modestly impair immediate tissue oxygenation. Answer: b) 2,3-DPG levels regenerate over 24–72 hours.
Keyword Definitions
Hematocrit - Fraction (%) of blood volume occupied by red blood cells; rises with hemoconcentration or polycythemia.
Hemoconcentration - Relative increase in red cell concentration due to reduced plasma volume (eg, dehydration).
Polycythemia - True increase in RBC mass (primary or secondary) causing high hematocrit independent of plasma volume.
Plasma electrolytes - Sodium, potassium, chloride, bicarbonate levels in plasma; reflect volume/status and acid–base balance.
Hemolysis - RBC rupture releasing intracellular K⁺ and other contents into plasma; may artifactually raise plasma K.
Hypernatremia - Increased plasma sodium concentration often from water loss causing hemoconcentration and higher hematocrit.
Metabolic alkalosis - Elevated plasma HCO₃⁻, sometimes from vomiting or diuretics; may associate with volume changes.
Chloride shift (Hamburger shift) - Movement of Cl⁻ into RBCs as HCO₃⁻ leaves during CO₂ transport in tissues.
Venous blood - Blood returning to the heart, higher CO₂, lower O₂ than arterial blood; can show concentration changes with local perfusion.
Plasma volume - Liquid component of blood; decreases in dehydration, increasing hematocrit without change in RBC mass.
Chapter: Clinical Hematology Topic: Blood Composition Subtopic: Hematocrit, Volume Status & Electrolytes
Lead Question – 2012
Venous blood with high hematocrit is seen in ?
a) RBC high chloride
b) Plasma high Na
c) Plasma high HCO3
d) RBC high K
Explanation (Answer: b) Plasma high Na) Elevated hematocrit in venous blood commonly reflects hemoconcentration from reduced plasma volume, as occurs with dehydration or water loss. This concentrates plasma solutes including sodium, so a high plasma Na (hypernatremia due to water deficit) accompanies increased hematocrit. Therefore b) is the correct choice.
1) In dehydration causing hemoconcentration, which laboratory pattern is expected?
a) ↑ Hematocrit, ↑ Plasma Na
b) ↓ Hematocrit, ↑ Plasma Na
c) ↑ Hematocrit, ↓ Plasma Na
d) ↓ Hematocrit, ↓ Plasma Na
Explanation: Dehydration reduces plasma volume leading to relative rise in hematocrit and concentration of solutes including sodium; thus both hematocrit and plasma Na increase. Clinically expect hemoconcentration with hypernatremia when water loss predominates. Answer a) is correct.
2) Which condition causes elevated hematocrit with normal plasma sodium?
a) Primary polycythemia (polycythemia vera)
b) Simple dehydration
c) Acute water intoxication
d) Diuretic-induced hypernatremia
Explanation: Primary polycythemia increases total RBC mass, raising hematocrit while plasma sodium may remain normal. Hemoconcentration from dehydration changes Na. Thus a) polycythemia vera is correct and requires distinct diagnosis (erythropoietin, JAK2 testing) compared with volume-related causes.
3) A lab sample shows high plasma K. Which preanalytical artifact could explain this with normal patient physiology?
a) Hemolysis during phlebotomy
b) Dehydration concentrating plasma
c) Diuretic-induced loss
d) Chronic kidney disease
Explanation: Hemolysis releases intracellular K⁺ into plasma, artifactually elevating measured potassium. This is a common preanalytical error and should be suspected when samples are traumatic. Thus a) hemolysis during phlebotomy explains isolated high plasma K with otherwise normal physiology.
4) Chloride shift in tissue capillaries results in which RBC change?
a) Increased RBC chloride concentration as HCO₃⁻ leaves
b) Increased RBC sodium
c) Increased RBC potassium
d) Decreased RBC chloride
Explanation: As CO₂ diffuses into erythrocytes and is converted to HCO₃⁻, HCO₃⁻ exits the cell while Cl⁻ enters to maintain electroneutrality (Hamburger shift). This increases RBC chloride concentration during CO₂ uptake in tissues. Therefore a) is correct.
5) Which scenario most likely produces low hematocrit in venous blood?
a) Acute hemorrhage with fluid resuscitation
b) Dehydration from vomiting
c) Secondary polycythemia from hypoxia
d) Erythropoietin use
Explanation: Acute hemorrhage followed by rapid crystalloid infusion dilutes remaining RBCs, lowering hematocrit (dilutional anemia). Dehydration does opposite; polycythemia and erythropoietin raise hematocrit. Thus a) is correct clinically in trauma or operative settings.
6) Metabolic alkalosis with high HCO₃⁻ is commonly associated with which volume status?
a) Contraction alkalosis with low plasma volume
b) Hypervolemia with low hematocrit
c) Euvolemia without change in hematocrit
d) Dehydration with low plasma Na
Explanation: Contraction alkalosis (eg, from diuretics or vomiting) reduces plasma volume concentrating HCO₃⁻ and electrolytes; it often accompanies hemoconcentration. So a) contraction alkalosis with low plasma volume is correct and may show relatively elevated hematocrit depending on fluid shifts.
7) Which lab pattern suggests true increased RBC mass rather than hemoconcentration?
a) ↑ Hematocrit with ↑ RBC mass on red cell mass study
b) ↑ Hematocrit with high plasma Na
c) ↑ Hematocrit that normalizes after fluid load
d) ↑ Hematocrit with low reticulocyte count only
Explanation: A red cell mass study showing increased RBC mass confirms true polycythemia rather than hemoconcentration which corrects after fluid repletion. Therefore a) is correct. Fluid-responsive hematocrit indicates volume effect; true polycythemia persists despite rehydration.
8) In diabetic ketoacidosis (DKA), initial labs often show:
a) ↑ Hematocrit due to dehydration
b) ↓ Hematocrit due to hemolysis
c) Normal hematocrit with hyperkalemia only
d) Increased plasma bicarbonate
Explanation: DKA causes osmotic diuresis and marked water loss, producing hemoconcentration and increased hematocrit. Plasma potassium may appear high due to shift from cells, though total body K is depleted. Thus, a) ↑ hematocrit due to dehydration is correct initially in DKA presentation.
9) Which chronic condition raises hematocrit via increased erythropoietin?
a) Chronic hypoxia from COPD
b) Chronic diarrheal dehydration
c) Acute sepsis with vasodilation
d) Cirrhosis with portal hypertension
Explanation: Chronic hypoxia stimulates renal erythropoietin production leading to secondary polycythemia with increased RBC mass and high hematocrit. COPD patients often exhibit this adaptation. Dehydration concentrates cells but erythropoietin-driven increases occur over weeks, so a) is correct.
10) A venous sample shows high hematocrit and high sodium. Best immediate clinical step is:
a) Assess volume status and consider rehydration if hypovolemic
b) Start phlebotomy for polycythemia vera immediately
c) Ignore as lab artifact always
d) Give bicarbonate to correct sodium
Explanation: High hematocrit with hypernatremia suggests hemoconcentration from volume loss; assess clinical volume status and treat dehydration with appropriate fluids. Phlebotomy is for true polycythemia after confirmation. Therefore a) is correct as a pragmatic immediate step in management.
Chapter: Anatomy
Topic: Abdominal Aorta
Subtopic: Branches of Abdominal Aorta
Keywords:
Abdominal aorta: Main arterial trunk of abdomen extending from diaphragm to bifurcation at L4.
Lateral branches: Paired arteries including renal, suprarenal, and gonadal arteries.
Anterior branches: Unpaired visceral arteries such as celiac, superior mesenteric, and inferior mesenteric arteries.
Posterior branches: Paired parietal arteries supplying abdominal wall, like lumbar arteries.
Clinical relevance: Aneurysms, occlusions, and variations in aortic branching cause surgical significance.
Lead Question - 2012
All are lateral branches of abdominal aorta, EXCEPT
a) Right testicular artery
b) Left renal artery
c) Inferior mesenteric artery
d) Middle suprarenal artery
Explanation: The abdominal aorta gives anterior, lateral, and posterior branches. Lateral branches include renal, suprarenal, and gonadal arteries. Inferior mesenteric artery is an anterior visceral branch. Correct answer is Inferior mesenteric artery, as it is not a lateral branch.
Guessed Question 1
Which of the following is an anterior visceral branch of abdominal aorta?
a) Renal artery
b) Suprarenal artery
c) Superior mesenteric artery
d) Gonadal artery
Explanation: The anterior visceral branches of abdominal aorta include celiac trunk, superior mesenteric artery, and inferior mesenteric artery. Renal, gonadal, and suprarenal arteries arise laterally. Correct answer is Superior mesenteric artery.
Guessed Question 2
The abdominal aorta terminates at which vertebral level?
a) L2
b) L3
c) L4
d) L5
Explanation: The abdominal aorta runs from T12 to L4, where it bifurcates into right and left common iliac arteries. Correct answer is L4.
Guessed Question 3
Which of the following arteries arises as a paired lateral branch?
a) Inferior phrenic artery
b) Median sacral artery
c) Gonadal artery
d) Celiac trunk
Explanation: Gonadal arteries (testicular or ovarian) are paired lateral branches of abdominal aorta. Median sacral is unpaired posterior, celiac trunk is anterior, inferior phrenic is paired but superior. Correct answer is Gonadal artery.
Guessed Question 4
Which artery supplies suprarenal glands directly from abdominal aorta?
a) Superior suprarenal artery
b) Middle suprarenal artery
c) Inferior suprarenal artery
d) None
Explanation: Middle suprarenal artery arises directly from the abdominal aorta as a paired lateral branch, while superior comes from inferior phrenic and inferior comes from renal. Correct answer is Middle suprarenal artery.
Guessed Question 5
Which is the first unpaired anterior branch of abdominal aorta?
a) Superior mesenteric artery
b) Inferior mesenteric artery
c) Celiac trunk
d) Gonadal artery
Explanation: The first anterior branch of abdominal aorta is the celiac trunk at T12 level, followed by superior mesenteric and inferior mesenteric arteries. Correct answer is Celiac trunk.
Guessed Question 6
Which branch of abdominal aorta supplies midgut?
a) Inferior mesenteric artery
b) Superior mesenteric artery
c) Celiac trunk
d) Renal artery
Explanation: Midgut (from duodenum distal to bile duct entry to proximal 2/3rd of transverse colon) is supplied by superior mesenteric artery. Correct answer is Superior mesenteric artery.
Guessed Question 7
Aneurysm of abdominal aorta is most common at which level?
a) Above renal arteries
b) Below renal arteries
c) At celiac trunk
d) At bifurcation
Explanation: Abdominal aortic aneurysm most commonly occurs below renal arteries (infrarenal segment). It is clinically important due to risk of rupture. Correct answer is Below renal arteries.
Guessed Question 8
Which artery is a posterior parietal branch of abdominal aorta?
a) Lumbar artery
b) Gonadal artery
c) Superior mesenteric artery
d) Middle suprarenal artery
Explanation: Lumbar arteries are paired posterior parietal branches supplying abdominal wall and muscles. Correct answer is Lumbar artery.
Guessed Question 9
Which abdominal aortic branch supplies hindgut?
a) Superior mesenteric artery
b) Inferior mesenteric artery
c) Celiac trunk
d) Renal artery
Explanation: Inferior mesenteric artery supplies hindgut structures including distal 1/3rd transverse colon, descending colon, sigmoid colon, and rectum. Correct answer is Inferior mesenteric artery.
Guessed Question 10
Occlusion of which artery causes ischemia in kidneys?
a) Renal artery
b) Inferior mesenteric artery
c) Median sacral artery
d) Gonadal artery
Explanation: Renal arteries are paired lateral branches of abdominal aorta supplying kidneys. Their occlusion leads to renal ischemia and hypertension. Correct answer is Renal artery.
Chapter: Neck Anatomy
Topic: Subclavian Artery & Its Branches
Subtopic: Thyrocervical Trunk — Origin and Branches
Keyword Definitions
Subclavian artery (parts): Divided into three parts in relation to anterior scalene: 1st (medial), 2nd (behind), 3rd (lateral).
Thyrocervical trunk: Short branch arising from the 1st part of subclavian; gives inferior thyroid, ascending cervical, transverse cervical, and suprascapular arteries.
Inferior thyroid artery: Branch supplying lower thyroid, parathyroids, and cervical branches; important in thyroid surgery.
Suprascapular artery: Supplies supraspinatus and infraspinatus regions; runs toward scapular notch.
Transverse cervical artery: Supplies trapezius and posterior neck; has superficial and deep branches.
Ascending cervical artery: Small vertical branch accompanying phrenic or vertebral levels.
Vertebral artery: Another branch of 1st part, ascends through transverse foramina to brainstem.
Clinical relevance: Knowledge of thyrocervical trunk anatomy is vital during central line placement, neck dissection, and thyroid surgery to avoid bleeding.
Lead Question - 2012
The thyrocervical trunk is a branch of which part of subclavian artery?
a) 1st
b) 2nd
c) 3rd
d) None
Explanation: The thyrocervical trunk arises from the first (medial) part of the subclavian artery, proximal to the anterior scalene muscle. It is a short, stout trunk giving inferior thyroid, ascending cervical, transverse cervical and suprascapular arteries. Surgeons must note this relation during neck and thyroid operations. Answer: a) 1st.
1. Which of the following is NOT a usual branch of the thyrocervical trunk?
a) Inferior thyroid artery
b) Transverse cervical artery
c) Suprascapular artery
d) Internal thoracic artery
Explanation: The internal thoracic artery is a branch of the subclavian but arises from the first part directly and not from the thyrocervical trunk. Thyrocervical trunk typically gives inferior thyroid, transverse cervical, suprascapular, and sometimes ascending cervical branches. Answer: d) Internal thoracic artery.
2. The inferior thyroid artery supplies all EXCEPT:
a) Lower pole of thyroid
b) Parathyroid glands
c) Larynx (via branches)
d) Supraorbital region
Explanation: Inferior thyroid artery supplies the lower thyroid, parathyroids and gives laryngeal branches but does not supply the supraorbital region which is served by branches of the ophthalmic artery. It is a branch of the thyrocervical trunk. Answer: d) Supraorbital region.
3. The transverse cervical artery commonly supplies which muscle?
a) Sternocleidomastoid
b) Trapezius (superficial branch)
c) Levator scapulae only
d) Diaphragm
Explanation: The superficial branch of the transverse cervical artery supplies the trapezius muscle and overlying skin. The deep branch (dorsal scapular) may supply levator scapulae and rhomboids. It arises from or near the thyrocervical trunk region. Answer: b) Trapezius (superficial branch).
4. During a thyroid lobectomy, which vessel must be carefully ligated to preserve laryngeal blood supply?
a) Superior thyroid artery
b) Inferior thyroid artery
c) Lingual artery
d) Facial artery
Explanation: Ligation of the inferior thyroid artery risks compromising inferior laryngeal branches; surgeons clip branches close to the thyroid capsule to preserve recurrent laryngeal artery supply. The inferior thyroid commonly arises from the thyrocervical trunk. Answer: b) Inferior thyroid artery.
5. The thyrocervical trunk most often arises medial to which muscle?
a) Anterior scalene
b) Middle scalene
c) Posterior scalene
d) Levator scapulae
Explanation: The first part of the subclavian and its branches, including the thyrocervical trunk and vertebral artery, lie medial to the anterior scalene muscle. This anatomical relation is crucial during central venous access and neck surgery. Answer: a) Anterior scalene.
6. A variant artery arising from thyrocervical trunk ascending along cervical vertebrae is called:
a) Ascending cervical artery
b) Superior thyroid artery
c) Deep cervical artery
d) Occipital artery
Explanation: The ascending cervical artery is a small upward branch often arising from the inferior thyroid or thyrocervical trunk, supplying neck muscles and vertebral bodies; it anastomoses with deep cervical branches. Answer: a) Ascending cervical artery.
7. Injury to the suprascapular artery may compromise blood supply to:
a) Supraspinatus and infraspinatus muscles
b) Levator scapulae primarily
c) Pectoralis major
d) Biceps brachii
Explanation: The suprascapular artery travels to the scapular region and supplies supraspinatus and infraspinatus muscles via scapular anastomoses; it commonly originates from the thyrocervical trunk. Damage affects shoulder girdle perfusion. Answer: a) Supraspinatus and infraspinatus muscles.
8. The vertebral artery arises from which part of the subclavian?
a) 1st part
b) 2nd part
c) 3rd part
d) It varies
Explanation: The vertebral artery classically arises from the first part of the subclavian artery and ascends through transverse foramina to supply posterior brain. Its proximity to thyrocervical trunk branches is important in cervical vascular anatomy. Answer: a) 1st part.
9. In posterior triangle bleeding from thyrocervical branches is best controlled by ligating which artery proximally?
a) Subclavian artery (first part)
b) External carotid artery
c) Vertebral artery
d) Internal thoracic artery
Explanation: Major control of bleeding from thyrocervical branches may require proximal control of the subclavian artery (first part) or selective ligation of offending branch at origin; external carotid ligation will not stop these branches. Answer: a) Subclavian artery (first part).
10. Which imaging modality best delineates small branches of thyrocervical trunk preoperatively?
a) Digital subtraction angiography (DSA)
b) Plain X-ray
c) Chest radiograph
d) EEG
Explanation: DSA provides high-resolution dynamic visualization of arterial branches, ideal for planning embolization or surgery involving the thyrocervical trunk and its branches. CT angiography is an alternative noninvasive option, but DSA remains gold standard for small branch delineation. Answer: a) Digital subtraction angiography (DSA).
Chapter: Head and Neck Anatomy
Topic: Arterial supply of Infratemporal Fossa
Subtopic: Maxillary Artery – Branches
Keyword Definitions
Maxillary artery: Terminal branch of external carotid artery, divided into three parts by lateral pterygoid muscle.
Middle meningeal artery: Branch of 1st part, supplies dura mater.
Accessory meningeal artery: Arises from 1st part, supplies cranial dura and trigeminal ganglion.
Inferior alveolar artery: Branch of 1st part, supplies mandible and lower teeth.
Greater palatine artery: Branch of descending palatine artery from 3rd part of maxillary artery, supplies hard palate.
Pterygoid part (2nd): Branches supply muscles of mastication.
Pterygopalatine part (3rd): Branches supply orbit, palate, and nasal cavity.
Lead Question - 2012
Which of the following is NOT a branch of 1st part of maxillary artery?
a) Middle meningeal artery
b) Accessory meningeal artery
c) Inferior alveolar artery
d) Greater palatine artery
Explanation: The 1st part of the maxillary artery gives middle meningeal, accessory meningeal, and inferior alveolar arteries. The greater palatine artery arises from descending palatine artery of 3rd part. Hence the correct answer is d) Greater palatine artery.
Guessed Question 1
The middle meningeal artery enters the cranial cavity through:
a) Foramen ovale
b) Foramen spinosum
c) Foramen rotundum
d) Jugular foramen
Explanation: Middle meningeal artery, branch of 1st part of maxillary artery, enters skull through foramen spinosum to supply dura. Injury causes extradural hematoma. Answer: b) Foramen spinosum.
Guessed Question 2
Inferior alveolar artery before entering mandibular foramen gives branch to:
a) Buccinator
b) Masseter
c) Mylohyoid
d) Temporalis
Explanation: The inferior alveolar artery gives the mylohyoid branch before entering mandibular foramen. It supplies mylohyoid and anterior belly of digastric. Correct answer: c) Mylohyoid.
Guessed Question 3
A fracture at pterion may rupture which artery?
a) Facial artery
b) Middle meningeal artery
c) Superficial temporal artery
d) Ascending pharyngeal artery
Explanation: Middle meningeal artery lies beneath pterion. Trauma causes rupture leading to extradural hematoma. Hence the correct answer is b) Middle meningeal artery.
Guessed Question 4
Accessory meningeal artery commonly enters cranium via:
a) Foramen spinosum
b) Foramen ovale
c) Foramen rotundum
d) Hypoglossal canal
Explanation: Accessory meningeal artery passes through foramen ovale to supply dura and trigeminal ganglion. Answer: b) Foramen ovale.
Guessed Question 5
A patient with bleeding from hard palate after trauma has injury to:
a) Middle meningeal artery
b) Inferior alveolar artery
c) Ascending pharyngeal artery
d) Greater palatine artery
Explanation: Greater palatine artery supplies hard palate through greater palatine foramen. Trauma can cause profuse bleeding. Answer: d) Greater palatine artery.
Guessed Question 6
Branches of 2nd part of maxillary artery mainly supply:
a) Dura mater
b) Muscles of mastication
c) Tongue
d) Nasal septum
Explanation: The pterygoid (2nd) part of maxillary artery mainly gives muscular branches to muscles of mastication. Correct answer: b) Muscles of mastication.
Guessed Question 7
Descending palatine artery is a branch of:
a) Facial artery
b) 3rd part of maxillary artery
c) Lingual artery
d) Ascending pharyngeal artery
Explanation: Descending palatine artery arises from 3rd (pterygopalatine) part of maxillary artery, divides into greater and lesser palatine arteries. Correct answer: b) 3rd part of maxillary artery.
Guessed Question 8
Extradural hematoma is classically due to rupture of:
a) Inferior alveolar vein
b) Middle meningeal artery
c) Anterior cerebral artery
d) Posterior communicating artery
Explanation: Rupture of middle meningeal artery due to skull fracture at pterion causes extradural hematoma, a neurosurgical emergency. Correct answer: b) Middle meningeal artery.
Guessed Question 9
Inferior alveolar artery after entering mandibular canal supplies:
a) Tongue
b) Lower teeth
c) Nasal cavity
d) Upper lip
Explanation: Inferior alveolar artery courses in mandibular canal to supply mandibular teeth and chin via mental branch. Correct answer: b) Lower teeth.
Guessed Question 10
Posterior superior alveolar artery, a branch of maxillary artery, supplies:
a) Lower premolars
b) Upper incisors
c) Maxillary molars
d) Mandibular molars
Explanation: Posterior superior alveolar artery supplies maxillary molars and adjacent gingiva. It is from 3rd part of maxillary artery. Correct answer: c) Maxillary molars.
Keywords:
Ophthalmic artery: First branch of the internal carotid artery after it enters the cranial cavity.
Internal carotid artery (ICA): Major artery supplying the brain and orbit.
Cavernous part of ICA: Segment traversing the cavernous sinus.
Cerebral part of ICA: Also called supraclinoid part, gives rise to ophthalmic artery.
Middle cerebral artery (MCA): Largest branch of ICA, supplies lateral cerebral hemisphere.
Facial artery: Branch of external carotid artery, supplies face.
1) Lead Question - 2012
Ophthalmic artery is a branch of?
a) Cavernous part of ICA
b) Cerebral part of ICA
c) MCA
d) Facial artery
Explanation: The ophthalmic artery arises from the cerebral (supraclinoid) part of the internal carotid artery just after it emerges from the cavernous sinus. It enters the orbit through the optic canal along with the optic nerve. Hence, the correct answer is cerebral part of ICA.
2) A patient presents with sudden monocular blindness. The most likely artery involved is?
a) Ophthalmic artery
b) Middle cerebral artery
c) Posterior communicating artery
d) Basilar artery
Explanation: Monocular blindness is commonly due to embolism or occlusion of the ophthalmic artery, a branch of the internal carotid artery. Retinal artery occlusion can cause sudden painless loss of vision. Correct answer is ophthalmic artery.
3) The central artery of retina is a branch of?
a) Middle cerebral artery
b) Ophthalmic artery
c) Basilar artery
d) External carotid artery
Explanation: The central artery of retina is a crucial end artery supplying the inner retina. It arises from the ophthalmic artery. Occlusion results in irreversible blindness. Correct answer is ophthalmic artery.
4) In cavernous sinus thrombosis, which artery is most closely related?
a) Maxillary artery
b) Ophthalmic artery
c) Internal carotid artery
d) Vertebral artery
Explanation: The cavernous sinus contains the cavernous part of ICA along with cranial nerves. Infection may spread to ICA leading to complications. Correct answer is internal carotid artery.
5) Which artery supplies the extraocular muscles?
a) Lacrimal artery
b) Muscular branches of ophthalmic artery
c) Posterior cerebral artery
d) Anterior cerebral artery
Explanation: Extraocular muscles receive blood supply from muscular branches of the ophthalmic artery. These branches ensure adequate perfusion of recti and oblique muscles. Correct answer is muscular branches of ophthalmic artery.
6) Which artery passes through the optic canal along with optic nerve?
a) Central retinal artery
b) Ophthalmic artery
c) Middle meningeal artery
d) Anterior cerebral artery
Explanation: The ophthalmic artery travels with the optic nerve through the optic canal to enter the orbit. This close relation explains visual loss in ICA occlusion. Correct answer is ophthalmic artery.
7) The lacrimal gland is mainly supplied by?
a) Facial artery
b) Lacrimal branch of ophthalmic artery
c) Posterior auricular artery
d) Maxillary artery
Explanation: The lacrimal gland receives its primary supply from the lacrimal artery, a branch of ophthalmic artery. It also anastomoses with infraorbital and middle meningeal arteries. Correct answer is lacrimal branch of ophthalmic artery.
8) Which artery forms an anastomosis with branches of external carotid artery on the face?
a) Central retinal artery
b) Supraorbital and supratrochlear branches of ophthalmic artery
c) Middle meningeal artery
d) Basilar artery
Explanation: The supraorbital and supratrochlear arteries, branches of ophthalmic artery, anastomose with superficial temporal and facial arteries, forming important ICA–ECA collateral channels. Correct answer is supraorbital and supratrochlear branches of ophthalmic artery.
9) Which part of ICA gives rise to posterior communicating artery?
a) Petrous part
b) Cavernous part
c) Cerebral (supraclinoid) part
d) Cervical part
Explanation: The posterior communicating artery arises from the cerebral (supraclinoid) part of ICA, connecting anterior circulation with posterior circulation. Correct answer is cerebral part.
10) Aneurysm of which artery commonly causes third nerve palsy?
a) Posterior communicating artery
b) Ophthalmic artery
c) Basilar artery
d) Anterior communicating artery
Explanation: Posterior communicating artery aneurysm compresses the oculomotor nerve leading to ptosis, diplopia, and pupillary dilation. Correct answer is posterior communicating artery.
11) A patient with severe facial trauma has massive epistaxis. Which artery is most likely responsible?
a) Ophthalmic artery
b) Sphenopalatine artery
c) Basilar artery
d) Internal carotid artery
Explanation: The sphenopalatine artery (terminal branch of maxillary artery, ECA system) is the main arterial source of severe posterior epistaxis. Correct answer is sphenopalatine artery.
Keywords
* Posterior communicating artery (PCOM) — A vessel connecting the internal carotid artery to the posterior cerebral artery, part of the circle of Willis.
* Internal carotid artery (ICA) — Major intracranial artery that gives rise to the ophthalmic, posterior communicating, anterior cerebral, and middle cerebral branches.
* External carotid artery (ECA) — Supplies extracranial head and neck structures; not a primary intracranial circle of Willis branch.
* Middle cerebral artery (MCA) — Continuation of ICA supplying lateral cerebral convexity; important in stroke syndromes.
* Posterior inferior cerebellar artery (PICA) — Branch of vertebral artery supplying posteroinferior cerebellum; related to Wallenberg syndrome.
* Circle of Willis — Collateral arterial anastomotic ring at the base of the brain linking anterior and posterior circulations.
* PCOM aneurysm — Frequent site for saccular aneurysms; may compress oculomotor nerve causing ptosis and pupil changes.
* Oculomotor nerve palsy — Presents with ptosis, "down and out" eye, pupil involvement suggests compressive lesion (e.g., PCOM aneurysm).
* Subarachnoid hemorrhage (SAH) — Sudden severe headache; common presentation of ruptured intracranial saccular aneurysm including PCOM aneurysms.
* Cerebral angiography — Gold standard imaging for diagnosing aneurysms and arterial anatomy; CT angiography is commonly used as initial test.
Chapter: Neuroanatomy — Topic: Cerebral Circulation — Subtopic: Circle of Willis & Posterior Communicating Artery
Lead Question - 2012
Posterior communicating artery a branch of
a) Internal carotid
b) External carotid
c) Middle cerebral
d) Posterior superior cerebellar
Explanation & answer: The posterior communicating artery arises from the internal carotid artery and connects to the posterior cerebral artery, forming part of the circle of Willis. It is not a branch of the external carotid, MCA, or cerebellar arteries. Correct answer: (a) Internal carotid. This artery is clinically important for PCOM aneurysms and oculomotor palsy. (≈50 words)
1.A patient presents with acute third nerve palsy with pupil involvement. Which vascular lesion is most likely?
a) Posterior communicating artery aneurysm
b) Lacunar infarct in the internal capsule
c) Middle cerebral artery thrombosis
d) Superior sagittal sinus thrombosis
Explanation & answer: A compressive PCOM aneurysm classically produces oculomotor nerve palsy with pupil dilation due to parasympathetic fiber compression. Ischemic microvascular palsies typically spare the pupil. MCA stroke causes cortical deficits, not isolated pupil-involving third nerve palsy. Correct answer: (a) Posterior communicating artery aneurysm. (≈50 words)
2. Which artery completes the posterior circulation connection to the anterior circulation via the PCOM?
a) Posterior cerebral artery
b) Anterior communicating artery
c) Basilar artery branch to PICA
d) Superficial temporal artery
Explanation & answer: The PCOM links the internal carotid system anteriorly to the posterior cerebral artery, which arises from the basilar artery posteriorly; this forms part of the posterior-anterior collateral route in the circle of Willis. The anterior communicating artery links the two anterior cerebral arteries. Correct answer: (a) Posterior cerebral artery. (≈50 words)
3. Best noninvasive initial imaging to detect a suspected PCOM aneurysm after SAH is:
a) CT angiography (CTA)
b) Plain skull X-ray
c) Ultrasound Doppler of carotids only
d) Electroencephalogram (EEG)
Explanation & answer: After subarachnoid hemorrhage, CT angiography is a rapid, noninvasive test to visualize intracranial aneurysms including PCOM aneurysms. Digital subtraction cerebral angiography remains gold standard but CTA is commonly used initially for detection and surgical planning. Correct answer: (a) CT angiography (CTA). (≈50 words)
4. Which embryologic vessel contributes to formation of the posterior communicating artery?
a) Fetal carotid-basilar anastomosis
b) Stapedial artery
c) External maxillary artery
d) Vitelline artery
Explanation & answer: The PCOM represents persistence of embryologic carotid–basilar anastomoses connecting the internal carotid to the posterior circulation. These fetal connections normally regress as posterior communicating and posterior cerebral arteries mature. Stapedial and vitelline arteries are unrelated. Correct answer: (a) Fetal carotid-basilar anastomosis. (≈50 words)
5. A ruptured PCOM aneurysm typically causes SAH with blood deposition in which cistern most prominently?
a) Interpeduncular cistern
b) Cisterna magna only
c) Cavernous sinus
d) Sigmoid sinus
Explanation & answer: A PCOM aneurysm rupture often produces subarachnoid blood in the interpeduncular cistern and basal cisterns around the circle of Willis due to its location at the ICA–PCOM junction. Cavernous sinus or venous sinuses are not primary subarachnoid spaces. Correct answer: (a) Interpeduncular cistern. (≈50 words)
6. Which clinical sign suggests a compressive third nerve palsy rather than ischemic microvascular palsy?
a) Early pupil dilation (mydriasis)
b) Isolated finger weakness
c) Pure sensory loss in a dermatomal pattern
d) Pure cerebellar ataxia
Explanation & answer: Pupil-involving oculomotor palsy with early mydriasis points to compression of peripheral parasympathetic fibers, as in a PCOM aneurysm. Microvascular ischemic palsies typically spare the pupil because central somatic fibers are affected but peripheral parasympathetic fibers are preserved. Correct answer: (a) Early pupil dilation (mydriasis). (≈50 words)
7. Which artery is NOT a direct branch of the internal carotid artery in the intracranial segment?
a) Ophthalmic artery (intracranial origin)
b) Posterior communicating artery
c) Middle cerebral artery
d) External carotid artery
Explanation & answer: The external carotid artery is a separate extracranial terminal branch; it does not arise from the intracranial internal carotid. The ophthalmic artery, PCOM, and MCA are intracranial branches or continuations of the ICA. Correct answer: (d) External carotid artery. (≈50 words)
8. In surgical clipping of a PCOM aneurysm, which neural structure must be protected to avoid postoperative diplopia and ptosis?
a) Oculomotor nerve (III)
b) Facial nerve (VII) extracranial branch
c) Hypoglossal nerve (XII)
d) Vagus nerve (X) trunk
Explanation & answer: The oculomotor nerve runs adjacent to the PCOM and posterior cerebral artery; it can be compressed by aneurysms or injured during clipping, causing ptosis and extraocular movement deficits. Facial, hypoglossal, and vagus nerves are remote from the PCOM region. Correct answer: (a) Oculomotor nerve (III). (≈50 words)
9. Which anatomical variation increases risk of anterior circulation collateral failure if PCOM is hypoplastic?
a) Hypoplastic PCOM with inadequate posterior flow
b) Bilateral large PCOM vessels providing robust collateralization
c) Prominent anterior communicating artery bridging ACAs
d) Redundant ophthalmic artery branches
Explanation & answer: A hypoplastic PCOM limits posterior-to-anterior collateral flow, increasing risk of ischemia if ICA flow is compromised. Large bilateral PCOMs or a robust anterior communicating artery improve collateral resilience. Thus hypoplastic PCOM predisposes to collateral failure. Correct answer: (a) Hypoplastic PCOM with inadequate posterior flow. (≈50 words)
10. Which therapeutic option is commonly considered for a saccular PCOM aneurysm not suitable for clipping?
a) Endovascular coiling (with or without stent-assisted technique)
b) Oral anticoagulation alone
c) High-dose systemic corticosteroids only
d) Carotid endarterectomy
Explanation & answer: Endovascular coiling, sometimes stent-assisted, is a standard treatment for saccular intracranial aneurysms including PCOM aneurysms when clipping is unfeasible. Anticoagulation, steroids, or carotid endarterectomy are inappropriate as primary aneurysm treatments. Correct answer: (a) Endovascular coiling. (≈50 words)
Chapter: Abdomen
Topic: Arterial Supply of the Colon and Rectum
Subtopic: Inferior Mesenteric Artery
Keyword Definitions:
Inferior Mesenteric Artery (IMA): Third unpaired branch of the abdominal aorta supplying the hindgut.
Left Colic Artery: Branch of IMA supplying descending colon.
Sigmoid Arteries: Branches of IMA supplying sigmoid colon.
Superior Rectal Artery: Terminal branch of IMA supplying upper rectum.
Middle Rectal Artery: Branch of internal iliac artery supplying middle rectum.
Lead Question – 2012
All are branches of the inferior mesenteric artery except ?
a) Left colic
b) Sigmoidal artery
c) Middle rectal
d) Superior rectal
Explanation: The middle rectal artery is not a branch of the inferior mesenteric artery. It arises from the internal iliac artery. The inferior mesenteric artery gives left colic, sigmoid, and superior rectal arteries. Clinical relevance: knowledge of arterial supply is vital in colorectal surgeries and controlling hemorrhage. Answer: c) Middle rectal
Guessed Question 1
Which artery supplies the upper part of the rectum?
a) Superior rectal artery
b) Middle rectal artery
c) Inferior rectal artery
d) Median sacral artery
Explanation: The superior rectal artery, terminal branch of the IMA, supplies the upper rectum. It forms an important anastomosis with middle and inferior rectal arteries, relevant in portal hypertension. Answer: a) Superior rectal artery
Guessed Question 2
The middle rectal artery is a branch of?
a) Inferior mesenteric artery
b) Internal iliac artery
c) External iliac artery
d) Median sacral artery
Explanation: The middle rectal artery arises from the internal iliac artery. It supplies the muscular wall of the rectum and the prostate in males. Not from IMA, differentiating pelvic and abdominal arterial supply. Answer: b) Internal iliac artery
Guessed Question 3
Which artery is most important during sigmoid colectomy?
a) Left colic
b) Middle colic
c) Sigmoid arteries
d) Superior mesenteric artery
Explanation: The sigmoid arteries (branches of IMA) are crucial during sigmoid colectomy. Ligation must be done with care to maintain marginal artery circulation. Answer: c) Sigmoid arteries
Guessed Question 4
Marginal artery of Drummond is formed by?
a) Branches of SMA and IMA
b) Branches of celiac and SMA
c) Branches of IMA only
d) Internal iliac and IMA
Explanation: The marginal artery of Drummond is formed by anastomosis between SMA and IMA branches along the colon, important in maintaining collateral circulation during arterial ligation. Answer: a) Branches of SMA and IMA
Guessed Question 5
During rectal cancer surgery, which artery is ligated to control bleeding from the superior rectum?
a) Inferior rectal
b) Middle rectal
c) Superior rectal
d) Median sacral
Explanation: The superior rectal artery is ligated during upper rectal surgeries to control bleeding. It is the terminal continuation of the IMA and the main blood supply to the upper rectum. Answer: c) Superior rectal
Guessed Question 6
Arc of Riolan connects?
a) Left colic and middle colic
b) Superior rectal and inferior rectal
c) Middle rectal and sigmoid
d) Right colic and ileocolic
Explanation: The arc of Riolan is an arterial connection between the left colic artery (IMA) and the middle colic artery (SMA), ensuring collateral circulation of the colon. Answer: a) Left colic and middle colic
Guessed Question 7
Which artery forms the main collateral between SMA and IMA?
a) Middle colic
b) Left colic
c) Marginal artery of Drummond
d) Median sacral
Explanation: The marginal artery of Drummond forms the main collateral between SMA and IMA along the colon, preventing ischemia in case of arterial occlusion. Answer: c) Marginal artery of Drummond
Guessed Question 8
Which rectal artery is involved in hemorrhoids due to portal hypertension?
a) Middle rectal
b) Inferior rectal
c) Superior rectal
d) Median sacral
Explanation: The superior rectal veinAnswer: c) Superior rectal
Guessed Question 9
During abdominal aortic aneurysm surgery, which artery must be preserved for left colon viability?
a) Right colic
b) Middle colic
c) Left colic
d) Ileocolic
Explanation: The left colic artery, a branch of IMA, must be preserved during aortic surgery to maintain blood supply to the descending colon. Answer: c) Left colic
Guessed Question 10
Which of the following is not directly related to IMA branches?
a) Sigmoid arteries
b) Left colic artery
c) Superior rectal artery
d) Inferior rectal artery
Explanation: The inferior rectal artery is not a branch of IMA. It arises from the internal pudendal artery, a branch of the internal iliac, supplying anal canal and perianal region. Answer: d) Inferior rectal artery
Chapter: Abdomen
Topic: Spleen
Subtopic: Surface Anatomy
Keyword Definitions:
Spleen: Largest lymphoid organ, located in the left hypochondrium.
Surface Anatomy: Study of external landmarks that indicate internal structures.
Ribs: Bony framework of thorax, important landmarks for organ projection.
Lead Question - 2012
Spleen extends from ?
a) 5th to 9th rib
b) 9th to 11th rib
c) 2nd to 5th rib
d) 11th to 12th rib
Explanation: The spleen lies in the left hypochondrium, deep to ribs 9–11 along the midaxillary line. Its long axis is parallel to the 10th rib. Correct Answer: b) 9th to 11th rib.
Guessed Question 1
The hilum of the spleen is located on which surface?
a) Diaphragmatic surface
b) Visceral surface
c) Inferior border
d) Superior border
Explanation: The hilum is present on the visceral surface where splenic vessels and lymphatics enter and leave. It is an important landmark for surgical procedures. Correct Answer: b) Visceral surface.
Guessed Question 2
Splenic artery is a branch of?
a) Celiac trunk
b) Superior mesenteric artery
c) Inferior mesenteric artery
d) Renal artery
Explanation: The splenic artery is a tortuous branch of the celiac trunk. It supplies the spleen, pancreas, and part of the stomach. Correct Answer: a) Celiac trunk.
Guessed Question 3
Which ligament connects the spleen to the stomach?
a) Gastrosplenic ligament
b) Splenorenal ligament
c) Phrenicocolic ligament
d) Hepatogastric ligament
Explanation: The gastrosplenic ligament connects the spleen to the greater curvature of the stomach and contains short gastric vessels. Correct Answer: a) Gastrosplenic ligament.
Guessed Question 4
Accessory spleens are most commonly found in?
a) Splenorenal ligament
b) Mesentery
c) Greater omentum
d) Pancreatic tail
Explanation: Accessory spleens are usually found near the splenic hilum or in the splenorenal ligament. They may mimic pathology in imaging. Correct Answer: a) Splenorenal ligament.
Guessed Question 5
In splenomegaly, spleen enlarges along the axis of?
a) 8th rib
b) 9th rib
c) 10th rib
d) 11th rib
Explanation: Splenomegaly causes the spleen to enlarge obliquely downward and medially along the 10th rib. This helps differentiate from renal enlargement. Correct Answer: c) 10th rib.
Guessed Question 6
Splenic vein joins with which vessel to form the portal vein?
a) Superior mesenteric vein
b) Inferior mesenteric vein
c) Left gastric vein
d) Right gastric vein
Explanation: The splenic vein unites with the superior mesenteric vein to form the portal vein behind the neck of the pancreas. Correct Answer: a) Superior mesenteric vein.
Guessed Question 7
Which of the following is NOT a relation of the spleen?
a) Left kidney
b) Stomach
c) Left colic flexure
d) Right adrenal gland
Explanation: The spleen is related to the stomach, left kidney, pancreas, and left colic flexure. The right adrenal gland lies on the opposite side. Correct Answer: d) Right adrenal gland.
Guessed Question 8
During trauma, spleen rupture leads to bleeding into?
a) Peritoneal cavity
b) Pleural cavity
c) Retroperitoneal space
d) Mediastinum
Explanation: Rupture of the spleen results in intraperitoneal hemorrhage, often massive, requiring immediate intervention. Correct Answer: a) Peritoneal cavity.
Guessed Question 9
Splenectomy most commonly predisposes a patient to infections by?
a) Gram-negative bacilli
b) Encapsulated organisms
c) Anaerobic bacteria
d) Mycobacteria
Explanation: Post-splenectomy, patients are more prone to infections with encapsulated bacteria such as Streptococcus pneumoniae, Neisseria meningitidis, and Haemophilus influenzae. Correct Answer: b) Encapsulated organisms.
Guessed Question 10
Which hematological condition is splenectomy most useful in?
a) Thalassemia major
b) Sickle cell anemia
c) Hereditary spherocytosis
d) Iron deficiency anemia
Explanation: Splenectomy is indicated in hereditary spherocytosis as the spleen destroys the abnormal red cells. Correct Answer: c) Hereditary spherocytosis.
Chapter: Abdomen
Topic: Abdominal Aorta and Related Structures
Subtopic: Vertebral Level Anatomy
Keyword Definitions:
Aorta: Main arterial trunk of the body running along the vertebral column.
IVC (Inferior Vena Cava): Largest vein of the body draining blood into the right atrium.
Coeliac trunk: First major branch of abdominal aorta at T12 supplying foregut.
Iliac vessels: Branches of aorta and IVC located at L4–L5.
Lead Question – 2012
Structure not seen at L3 level ?
a) Iliac vessels
b) Aorta
c) Coeliac trunk
d) IVC
Explanation: At L3 level, the aorta and IVC are present. Iliac vessels occur at L4–L5 after bifurcation. The coeliac trunk arises at T12. Thus, coeliac trunk is not seen at L3. Answer: c) Coeliac trunk.
1) Structure present at the level of L1 vertebra is?
a) Coeliac trunk
b) Superior mesenteric artery
c) Inferior mesenteric artery
d) Renal arteries
Explanation: The SMA arises at the level of L1. Coeliac trunk is at T12, renal arteries at L2, and IMA at L3. Hence correct answer is b) Superior mesenteric artery.
2) Inferior mesenteric artery originates at?
a) L1
b) L2
c) L3
d) L4
Explanation: Inferior mesenteric artery arises from the abdominal aorta at L3 vertebral level, supplying hindgut structures. Correct answer is c) L3.
3) At which vertebral level does the aorta bifurcate?
a) L2
b) L3
c) L4
d) L5
Explanation: The abdominal aorta bifurcates into the common iliac arteries at the level of L4. Correct answer is c) L4.
4) The inferior vena cava pierces the diaphragm at?
a) T8
b) T10
c) T12
d) L1
Explanation: The IVC passes through the central tendon of the diaphragm at the T8 vertebral level. Correct answer is a) T8.
5) A patient with bleeding from the foregut most likely has involvement of?
a) Inferior mesenteric artery
b) Superior mesenteric artery
c) Coeliac trunk
d) Renal artery
Explanation: Foregut structures are supplied by branches of the coeliac trunk (T12). Hence the vessel involved is c) Coeliac trunk.
6) Renal arteries arise at which vertebral level?
a) T12
b) L1
c) L2
d) L3
Explanation: Renal arteries arise from the aorta at the level of L2, just below SMA origin. Correct answer is c) L2.
7) Testicular arteries originate from?
a) Common iliac artery
b) Abdominal aorta at L2
c) Inferior mesenteric artery
d) External iliac artery
Explanation: Testicular arteries arise from the abdominal aorta at the L2 vertebral level. Correct answer is b) Abdominal aorta at L2.
8) The cisterna chyli is located at?
a) L1
b) L2
c) L3
d) L4
Explanation: The cisterna chyli, lymphatic dilatation, is located anterior to bodies of L1–L2 vertebrae. Correct answer is b) L2.
9) Which of the following passes through the diaphragm at T10?
a) Esophagus
b) Aorta
c) IVC
d) Thoracic duct
Explanation: The esophagus passes through the esophageal hiatus at T10. IVC at T8 and aorta at T12. Correct answer is a) Esophagus.
10) A patient with ischemia of the midgut is most likely to have occlusion of?
a) Coeliac trunk
b) Inferior mesenteric artery
c) Superior mesenteric artery
d) Median sacral artery
Explanation: Midgut structures are supplied by branches of SMA (arising at L1). Occlusion causes midgut ischemia. Correct answer is c) Superior mesenteric artery.
Chapter: Pelvis and Perineum
Topic: Nerves and Vessels
Subtopic: Relations of Ischial Spine
Keywords:
Ischial spine: A bony projection on the ischium important as a landmark in pelvic anatomy.
Pudendal nerve: Main nerve of perineum, crosses posterior to ischial spine.
Internal pudendal vessels: Artery and vein accompanying pudendal nerve.
Nerve to obturator internus: Motor nerve crossing ischial spine with pudendal bundle.
Obturator nerve: Runs along lateral pelvic wall, does not cross ischial spine.
Lead Question - 2012
Structure crossing dorsal surface of ischial spine are A/E:
a) Internal pudendal vessel
b) Pudendal nerve
c) Obturator nerve
d) Nerve to obturator internus
Explanation: The pudendal nerve, internal pudendal vessels, and nerve to obturator internus cross the ischial spine. The obturator nerve runs along the pelvic sidewall and exits via obturator canal, not over the ischial spine. Answer: c) Obturator nerve
1) A 35-year-old female during childbirth suffered injury to the structure crossing the ischial spine. Which function is most likely affected?
a) Sensation of perineum
b) Hip adduction
c) Quadriceps contraction
d) Knee extension
Explanation: The pudendal nerve crosses ischial spine and provides sensory supply to perineum. Injury causes perineal sensory loss. Hip adduction involves obturator nerve which does not cross spine. Answer: a) Sensation of perineum
2) Which vessel accompanies pudendal nerve while crossing ischial spine?
a) Inferior gluteal artery
b) Internal pudendal artery
c) Superior gluteal artery
d) Obturator artery
Explanation: Pudendal nerve crosses ischial spine along with internal pudendal vessels. Inferior and superior gluteal arteries do not directly cross spine. Answer: b) Internal pudendal artery
3) A surgeon performing pudendal block locates which landmark near ischial spine?
a) Sacral promontory
b) Coccyx tip
c) Ischial spine via vaginal exam
d) Iliac crest
Explanation: Pudendal block is given by palpating ischial spine transvaginally where pudendal nerve passes. Coccyx and iliac crest are not related landmarks. Answer: c) Ischial spine via vaginal exam
4) Which nerve does not cross ischial spine?
a) Pudendal nerve
b) Nerve to obturator internus
c) Internal pudendal vessels
d) Obturator nerve
Explanation: Obturator nerve runs through obturator canal and does not cross ischial spine. Others cross spine dorsally. Answer: d) Obturator nerve
5) Pudendal nerve block provides anesthesia for which procedure?
a) Episiotomy
b) Appendectomy
c) Inguinal hernia repair
d) Cholecystectomy
Explanation: Pudendal block is mainly used in obstetrics for episiotomy and perineal repair by anesthetizing pudendal nerve at ischial spine. Answer: a) Episiotomy
6) Nerve to obturator internus after crossing ischial spine enters which region?
a) Perineum
b) Gluteal region
c) Obturator canal
d) Femoral canal
Explanation: The nerve to obturator internus crosses the ischial spine dorsally and enters the gluteal region before supplying obturator internus muscle. Answer: b) Gluteal region
7) Which muscle acts as close relation of structures crossing ischial spine?
a) Piriformis
b) Coccygeus
c) Obturator externus
d) Psoas major
Explanation: Coccygeus muscle lies in relation to ischial spine, providing support to crossing pudendal bundle. Piriformis is higher, obturator externus and psoas are not related. Answer: b) Coccygeus
8) Clinical feature of pudendal nerve entrapment near ischial spine is:
a) Loss of knee reflex
b) Perineal pain and numbness
c) Foot drop
d) Loss of Achilles reflex
Explanation: Pudendal entrapment near ischial spine causes perineal pain, numbness, and sphincter dysfunction. Reflexes and foot drop are unrelated. Answer: b) Perineal pain and numbness
9) In pelvic fracture involving ischial spine, which function is spared?
a) Perineal sensation
b) Anal sphincter tone
c) Hip adduction
d) External urethral sphincter
Explanation: Hip adduction is mediated by obturator nerve, which does not cross ischial spine, hence spared. Pudendal functions are compromised. Answer: c) Hip adduction
10) The pudendal nerve is derived from which spinal segments?
a) L2-L4
b) L4-S1
c) S2-S4
d) S1-S3
Explanation: Pudendal nerve originates from sacral plexus segments S2, S3, S4. These supply perineum and external sphincters. Answer: c) S2-S4
Chapter: Abdomen
Topic: Portal Vein
Subtopic: Formation and Tributaries
Keyword Definitions
Portal vein: Large vein that carries nutrient-rich blood from the gastrointestinal tract and spleen to the liver.
Superior mesenteric vein (SMV): Vein draining small intestine, ascending and transverse colon.
Splenic vein: Vein draining spleen, pancreas, and part of stomach.
Inferior mesenteric vein (IMV): Vein draining descending colon, sigmoid colon, and rectum.
Hepatic veins: Veins draining processed blood from the liver into the inferior vena cava.
Lead Question – 2012
Portal vein is formed by union of which of the following veins?
a) Superior mesenteric vein & Splenic vein
b) Superior mesenteric vein & Inferior mesenteric vein
c) Inferior mesenteric vein & Splenic vein
d) Inferior mesenteric vein & Hepatic vein
Explanation: The portal vein is formed behind the neck of the pancreas by union of the superior mesenteric vein and the splenic vein. The inferior mesenteric vein usually drains into the splenic vein, not directly forming the portal vein. Correct answer: a) Superior mesenteric vein & Splenic vein.
Guessed Questions for NEET PG
Q1. The inferior mesenteric vein most commonly drains into which vein?
a) Superior mesenteric vein
b) Splenic vein
c) Left renal vein
d) Hepatic vein
Explanation: The inferior mesenteric vein most commonly drains into the splenic vein before joining the superior mesenteric vein to form the portal vein. Variations may occur but splenic vein drainage is the rule. Correct answer: b) Splenic vein.
Q2. At what anatomical location is the portal vein formed?
a) Behind the head of pancreas
b) Behind the neck of pancreas
c) In front of the duodenum
d) Within the liver hilum
Explanation: The portal vein is formed posterior to the neck of the pancreas by union of the SMV and splenic vein. This location is important during pancreatic surgeries. Correct answer: b) Behind the neck of pancreas.
Q3. Which of the following veins is not a direct tributary of the portal vein?
a) Left gastric vein
b) Right gastric vein
c) Superior mesenteric vein
d) Inferior vena cava
Explanation: The inferior vena cava drains systemic circulation and does not contribute to the portal system. Gastric veins and SMV are portal tributaries. Correct answer: d) Inferior vena cava.
Q4. Which of the following conditions is most commonly associated with portal hypertension?
a) Cirrhosis of liver
b) Acute appendicitis
c) Cholecystitis
d) Pancreatitis
Explanation: Cirrhosis of the liver is the most common cause of portal hypertension due to architectural distortion and resistance to portal blood flow. Correct answer: a) Cirrhosis of liver.
Q5. Caput medusae in portal hypertension is due to dilatation of which veins?
a) Superior rectal veins
b) Periumbilical veins
c) Esophageal veins
d) Inferior mesenteric veins
Explanation: Caput medusae is caused by dilatation of paraumbilical veins, which are portosystemic anastomoses around the umbilicus. Correct answer: b) Periumbilical veins.
Q6. A patient with chronic alcoholism develops hematemesis. Which venous anastomosis is responsible?
a) Rectal
b) Esophageal
c) Paraumbilical
d) Retroperitoneal
Explanation: Hematemesis in portal hypertension is commonly due to rupture of dilated esophageal varices, a site of portosystemic anastomosis. Correct answer: b) Esophageal.
Q7. Which structure lies anterior to the portal vein in the hepatoduodenal ligament?
a) Common bile duct
b) Hepatic artery proper
c) Inferior vena cava
d) Gallbladder
Explanation: In the hepatoduodenal ligament, the portal vein lies posteriorly, the hepatic artery proper lies to the left, and the common bile duct lies anteriorly to the right. Correct answer: a) Common bile duct.
Q8. Which vein connects the left gastric vein to the azygos vein system in portal hypertension?
a) Short gastric vein
b) Esophageal vein
c) Inferior phrenic vein
d) Left renal vein
Explanation: The left gastric vein communicates with esophageal veins that drain into the azygos system, creating a portosystemic shunt in portal hypertension. Correct answer: b) Esophageal vein.
Q9. In Budd–Chiari syndrome, obstruction occurs in which vein?
a) Portal vein
b) Inferior vena cava
c) Hepatic veins
d) Splenic vein
Explanation: Budd–Chiari syndrome involves obstruction of hepatic veins, preventing outflow of blood from the liver into the IVC, distinct from portal vein obstruction. Correct answer: c) Hepatic veins.
Q10. A 50-year-old patient presents with bleeding per rectum due to portal hypertension. Which venous communication is involved?
a) Superior rectal vein with middle and inferior rectal veins
b) Splenic vein with gastric vein
c) Left renal vein with gonadal vein
d) Inferior mesenteric vein with renal vein
Explanation: In portal hypertension, rectal varices occur due to anastomosis between superior rectal vein (portal system) and middle/inferior rectal veins (systemic). Correct answer: a) Superior rectal vein with middle and inferior rectal veins.
Chapter: Anatomy & Clinical Correlations
Topic: Venous System
Subtopic: Obstruction of Inferior Vena Cava (IVC)
Keywords and Definitions:
• Inferior vena cava – Large vein carrying deoxygenated blood from lower body to right atrium.
• Paraumbilical veins – Small veins around umbilicus connecting to portal circulation.
• Thoraco-epigastric vein – Vein on thoracoabdominal wall linking superficial epigastric and lateral thoracic veins.
• Esophageal varices – Dilated veins in lower esophagus due to portal hypertension.
• Hemorrhoids – Swollen rectal venous plexus, commonly linked to straining or portal hypertension.
Lead Question – 2012
Obstruction of Inferior vena cava presents:
a) Paraumblical dilatation
b) Thoraco-epigastric dilatation
c) Oesophagus varices
d) Haemorrhoids
Explanation:
The correct answer is (b) Thoraco-epigastric dilatation. Obstruction of IVC causes venous return to bypass through collateral circulation, especially via thoraco-epigastric veins on the abdominal wall, leading to visibly dilated superficial veins. Paraumbilical dilatation is more typical of portal hypertension (caput medusae), while varices and hemorrhoids are linked to portal systemic anastomosis.
Guessed Question 1:
A 45-year-old male presents with distended superficial veins on the anterior abdominal wall. On examination, blood flow is directed from below upward. This finding suggests:
a) Superior vena cava obstruction
b) Inferior vena cava obstruction
c) Portal vein thrombosis
d) Hepatic vein obstruction
Explanation:
Answer: (b) Inferior vena cava obstruction. Direction of venous flow helps differentiate cause. In IVC obstruction, venous blood bypasses blockage via thoraco-epigastric veins, flowing upward toward SVC. In contrast, SVC obstruction would show downward collateral flow. Portal vein thrombosis and hepatic vein obstruction produce other signs like ascites or varices.
Guessed Question 2:
Caput medusae in portal hypertension is due to dilatation of:
a) Thoraco-epigastric veins
b) Paraumbilical veins
c) Inferior epigastric veins
d) Azygos vein
Explanation:
Answer: (b) Paraumbilical veins. Portal hypertension causes reopening and dilation of paraumbilical veins, leading to the classic "caput medusae" around the umbilicus. This differentiates it from IVC obstruction, where thoraco-epigastric veins are predominantly involved. Clinical examination of venous flow direction helps distinguish between the two.
Guessed Question 3:
Which collateral pathway is most important in IVC obstruction?
a) Azygos and hemiazygos veins
b) Superior rectal veins
c) Paraumbilical veins
d) Inferior thyroid veins
Explanation:
Answer: (a) Azygos and hemiazygos veins. In IVC obstruction, azygos-hemiazygos system provides a major collateral route to drain venous blood into SVC. Thoraco-epigastric veins also provide superficial drainage, but the azygos system is the key deep collateral. Superior rectal and thyroid veins are not involved here.
Guessed Question 4:
A patient with chronic Budd-Chiari syndrome develops prominent abdominal wall veins. The mechanism is:
a) Portal vein obstruction
b) IVC obstruction
c) Hepatic arterial thrombosis
d) Splenic vein thrombosis
Explanation:
Answer: (b) IVC obstruction. Budd-Chiari syndrome leads to hepatic vein outflow obstruction. Over time, it may cause IVC compression, resulting in dilated collateral veins on abdominal wall. Portal vein obstruction more commonly produces varices and ascites rather than thoraco-epigastric dilatation.
Guessed Question 5:
Direction of blood flow in dilated abdominal wall veins helps in differential diagnosis. In IVC obstruction, blood flow is:
a) Above downward
b) Below upward
c) Centripedal
d) Random
Explanation:
Answer: (b) Below upward. In IVC obstruction, venous return from lower limbs cannot pass through the IVC, so blood finds alternative pathways to reach the SVC via upward flow in thoraco-epigastric veins. In SVC obstruction, the flow is downward from above.
Guessed Question 6:
A patient presents with esophageal varices and splenomegaly. Which is the most likely underlying cause?
a) IVC obstruction
b) SVC obstruction
c) Portal hypertension
d) Iliac vein thrombosis
Explanation:
Answer: (c) Portal hypertension. Esophageal varices result from increased portal venous pressure causing dilation of left gastric (coronary) veins. Splenomegaly further supports portal hypertension. IVC or SVC obstruction causes abdominal wall vein dilatation instead, not varices.
Guessed Question 7:
In IVC obstruction, the superficial collateral veins seen are:
a) Thoraco-epigastric veins
b) Esophageal veins
c) Paraumbilical veins
d) Hemorrhoidal veins
Explanation:
Answer: (a) Thoraco-epigastric veins. These veins form important collaterals between femoral and axillary venous systems. Their dilation is a hallmark of IVC obstruction. The other options are typical for portal hypertension and not primarily for IVC block.
Guessed Question 8:
Which investigation best confirms IVC obstruction?
a) Abdominal ultrasound with Doppler
b) Chest X-ray
c) Barium swallow
d) ECG
Explanation:
Answer: (a) Abdominal ultrasound with Doppler. Doppler imaging helps visualize venous flow and detect IVC obstruction. CT/MRI venography may also be used. Chest X-ray, barium swallow, and ECG are not definitive for IVC pathology.
Guessed Question 9:
Which of the following is not a cause of IVC obstruction?
a) Retroperitoneal tumor
b) Thrombosis
c) Pregnancy
d) Cirrhosis with portal hypertension
Explanation:
Answer: (d) Cirrhosis with portal hypertension. Cirrhosis causes portal vein obstruction, not IVC obstruction. Retroperitoneal tumors, thrombosis, and gravid uterus may compress or block IVC. Clinical examination and imaging differentiate these conditions.
Guessed Question 10:
Which of the following syndromes is most directly associated with IVC obstruction?
a) Budd-Chiari syndrome
b) Mallory-Weiss syndrome
c) Zollinger-Ellison syndrome
d) Conn’s syndrome
Explanation:
Answer: (a) Budd-Chiari syndrome. It involves hepatic vein outflow obstruction, often extending to IVC. Clinical features include hepatomegaly, ascites, abdominal pain, and dilated abdominal veins. The other syndromes are unrelated to venous obstruction.
Pancreas: A retroperitoneal organ of both endocrine and exocrine function located across the posterior abdominal wall.
Pancreatic head: The widened part of the pancreas that lies in the C-shaped curve of the duodenum.
Inferior vena cava (IVC): Large vein returning blood from the lower body, lies posterior to pancreatic head.
Right renal vein: Vessel draining the right kidney into the IVC, related to the posterior aspect of pancreatic head.
Splenic artery: A tortuous branch of celiac trunk, runs along the superior border of pancreas but not posterior to head.
Inferior mesenteric vein: Drains into splenic vein or SMV, not directly behind pancreatic head.
Celiac trunk: Major abdominal artery arising at T12, located superior to pancreas, not directly posterior to head.
Structure immediately posterior to pancreatic head?
Right renal vein
Splenic artery
Inferior mesenteric vein
Coeliac trunk
Explanation: The correct answer is a) Right renal vein. The structures lying posterior to the head of the pancreas include the inferior vena cava, right renal vessels, and left renal vein (more to the left side). The splenic artery lies superior along the body, the inferior mesenteric vein drains elsewhere, and the celiac trunk lies higher above the pancreas.
The uncinate process of pancreas is related posteriorly to which vessel?
Superior mesenteric vein
Splenic artery
Inferior mesenteric vein
Left renal artery
Explanation: The correct answer is a) Superior mesenteric vein. The uncinate process extends behind the superior mesenteric vessels. This relation is clinically important in pancreatic surgery, as tumors of uncinate process may compress these vessels. Splenic artery runs on superior border, while IMV drains separately, and left renal artery is more lateral.
Which structure lies anterior to the head of pancreas?
Stomach
Second part of duodenum
IVC
Right kidney
Explanation: The answer is b) Second part of duodenum. The head of the pancreas is encircled by the C-shaped duodenum, especially its second part. This is the anterior relation. Stomach lies anterior to the body and tail, not head. IVC and kidney are posterior relations, not anterior.
Which duct joins the common bile duct at the ampulla of Vater in relation to the pancreatic head?
Accessory pancreatic duct
Main pancreatic duct
Cystic duct
Right hepatic duct
Explanation: The answer is b) Main pancreatic duct. The main pancreatic duct joins the common bile duct at the hepatopancreatic ampulla (ampulla of Vater) located within the head of pancreas. This relation is critical in obstructive jaundice due to pancreatic head carcinoma. Accessory duct drains separately into duodenum.
A 55-year-old male with carcinoma head of pancreas presents with jaundice. Which structure is compressed?
Inferior vena cava
Portal vein
Common bile duct
Splenic vein
Explanation: The correct answer is c) Common bile duct. The common bile duct passes through a groove in the posterior surface of pancreatic head. Tumors in this region compress the duct, leading to obstructive jaundice. IVC and portal vein may also be involved but classical clinical sign is jaundice from bile duct obstruction.
Which artery lies superior to the body of the pancreas?
Splenic artery
Superior mesenteric artery
Inferior mesenteric artery
Middle colic artery
Explanation: The answer is a) Splenic artery. The splenic artery runs tortuously along the superior border of pancreas before reaching spleen. It gives off pancreatic branches to body and tail. Superior mesenteric artery lies anterior to uncinate process, not body. Inferior mesenteric artery originates lower, unrelated to pancreas.
Which venous structure runs behind the neck of the pancreas?
Portal vein
Splenic vein
Left renal vein
Superior mesenteric vein
Explanation: The correct answer is a) Portal vein. The portal vein is formed by the union of superior mesenteric vein and splenic vein behind the neck of the pancreas. This anatomical landmark is key in portal hypertension surgeries. Left renal vein runs more posterior and lateral, not behind pancreatic neck.
Which structure is closely related to the posterior surface of pancreatic tail?
Left kidney
Spleen
Left colic flexure
Adrenal gland
Explanation: The answer is b) Spleen. The tail of pancreas extends to the hilum of the spleen within the splenorenal ligament. This relation explains why splenectomy can damage pancreatic tail. Left kidney and adrenal are related but more posterior and medial. Colic flexure is inferior, not posterior.
Which artery runs posterior to the superior part of duodenum near pancreatic head?
Right gastroepiploic artery
Gastroduodenal artery
Inferior pancreaticoduodenal artery
Middle colic artery
Explanation: The answer is b) Gastroduodenal artery. The gastroduodenal artery descends posterior to the first part of duodenum near pancreatic head. Peptic ulcer perforation in this region may erode it, causing massive hemorrhage. Inferior pancreaticoduodenal artery arises from SMA but lies lower, supplying duodenum and pancreas.
A 60-year-old patient with pancreatic head carcinoma develops portal hypertension. Which vessel is compressed?
Splenic vein
Portal vein
Superior mesenteric vein
Hepatic vein
Explanation: The correct answer is b) Portal vein. The portal vein passes behind the neck of the pancreas and is closely related to the pancreatic head. Carcinoma in this region can compress the vein, leading to portal hypertension and its consequences such as varices and splenomegaly. Hepatic vein obstruction is unrelated here.
During Whipple’s procedure, which structure must be carefully preserved near pancreatic head?
Inferior vena cava
Superior mesenteric vessels
Splenic artery
Left renal vein
Explanation: The correct answer is b) Superior mesenteric vessels. In pancreaticoduodenectomy (Whipple’s procedure), the superior mesenteric artery and vein run close to the uncinate process and must be carefully preserved to maintain gut perfusion. IVC and renal vein are nearby but not directly endangered. Splenic artery lies more superiorly on body.
Ovarian artery: A paired branch of the abdominal aorta that supplies the ovary, uterine tube, and part of the uterus.
Abdominal aorta: The largest artery in the abdomen, giving rise to visceral and parietal branches.
Internal iliac artery: Supplies pelvic organs, gluteal region, and perineum.
Common iliac artery: Terminal branch of abdominal aorta dividing into external and internal iliac arteries.
External iliac artery: Continues as femoral artery supplying lower limb.
Gonadal arteries: Paired arteries (ovarian in females, testicular in males) from abdominal aorta below renal arteries.
Right ovarian artery is a branch of ?
Abdominal aorta
Right internal iliac
Common iliac
External iliac
Explanation: The correct answer is a) Abdominal aorta. The ovarian arteries arise directly from the abdominal aorta just below the renal arteries. They descend into the pelvis through the suspensory ligament of ovary to supply ovaries and fallopian tubes. They are not branches of iliac arteries, which mainly supply pelvic and lower limb regions.
At what vertebral level do the ovarian arteries usually arise?
L1
L2
L3
T12
Explanation: The answer is b) L2. Ovarian arteries typically arise from the abdominal aorta at the level of L2, just below the renal arteries. This anatomical relationship is important for surgical approaches to retroperitoneal space. Variations can exist, but L2 origin is the most consistent for both ovarian and testicular arteries.
Which ligament of the ovary carries the ovarian artery?
Ovarian ligament
Suspensory ligament
Round ligament
Broad ligament
Explanation: The correct answer is b) Suspensory ligament. The ovarian artery passes through the suspensory ligament of the ovary (infundibulopelvic ligament), carrying vessels and nerves to the ovary. The ovarian ligament connects ovary to uterus, while round ligament extends to labia majora. Broad ligament is a peritoneal fold, not a vessel carrier.
Which structure does the ovarian artery anastomose with?
Uterine artery
Internal pudendal artery
Inferior mesenteric artery
Renal artery
Explanation: The answer is a) Uterine artery. The ovarian artery forms an important anastomosis with the uterine artery within the mesovarium and broad ligament, ensuring dual blood supply to uterus and ovary. This collateral circulation is essential for maintaining perfusion, especially during pregnancy. Pudendal and mesenteric arteries are unrelated branches.
A 28-year-old woman undergoing oophorectomy is at risk of bleeding from which artery if not ligated properly?
Ovarian artery
Uterine artery
Internal iliac artery
Superior vesical artery
Explanation: The correct answer is a) Ovarian artery. During oophorectomy, the suspensory ligament of ovary must be ligated to control bleeding from the ovarian artery. Uterine artery ligation is important during hysterectomy. Internal iliac branches may be ligated in pelvic hemorrhage, but primary risk in ovary removal is ovarian artery.
Which embryonic structure gives rise to the ovarian artery?
Vitelline arteries
Umbilical arteries
Lateral mesonephric arteries
Arch arteries
Explanation: The answer is c) Lateral mesonephric arteries. The gonadal arteries, including ovarian arteries, develop from the lateral mesonephric arteries in the embryo. These paired vessels supply the mesonephros and later persist as gonadal arteries. Vitelline arteries form celiac and mesenteric vessels, while umbilical arteries contribute to internal iliac branches.
In angiography, an ovarian artery arising from the renal artery is considered:
Normal variant
Pathological finding
Developmental defect
Trauma-induced change
Explanation: The correct answer is a) Normal variant. Though typically from abdominal aorta, the ovarian artery may arise from renal, suprarenal, or common iliac arteries. Such variations are developmental and considered normal variants. They are important to recognize during radiology and surgery to avoid misdiagnosis or accidental vessel injury.
A 32-year-old woman with postpartum hemorrhage undergoes uterine artery ligation. Which collateral maintains uterine blood supply?
Ovarian artery
Superior rectal artery
Middle sacral artery
Inferior epigastric artery
Explanation: The answer is a) Ovarian artery. In cases of uterine artery ligation, the ovarian artery via its anastomosis continues to perfuse the uterus. This collateral circulation helps maintain fertility and uterine viability. Other listed arteries do not have significant uterine anastomoses, making ovarian artery vital clinically in hemorrhage management.
Which vein accompanies the ovarian artery?
Uterine vein
Ovarian vein
Renal vein
Hypogastric vein
Explanation: The correct answer is b) Ovarian vein. The ovarian vein runs with the ovarian artery in the suspensory ligament. The right ovarian vein drains into the inferior vena cava, while the left ovarian vein drains into the left renal vein. Uterine veins accompany uterine arteries, not ovarian arteries.
Which imaging technique best demonstrates ovarian artery during interventional procedures?
Ultrasound
CT angiography
MRI pelvis
X-ray KUB
Explanation: The answer is b) CT angiography. CT angiography allows detailed visualization of ovarian artery origin, course, and anastomoses. It is especially useful in embolization procedures for uterine fibroids or ovarian tumors. Ultrasound and MRI can provide supportive information but not direct arterial mapping. X-ray KUB has no role.
During aortic surgery, ligation of which artery may compromise ovarian function?
Ovarian artery
Inferior mesenteric artery
Middle sacral artery
Superior mesenteric artery
Explanation: The correct answer is a) Ovarian artery. If the ovarian artery is inadvertently ligated during aortic or retroperitoneal surgery, blood supply to the ovary is significantly reduced. This can impair ovarian function and fertility. Mesenteric arteries supply intestines, not gonads. Middle sacral has minor role in pelvic supply.
Chapter: Anatomy
Inferior vena cava (IVC): The large vein that carries deoxygenated blood from the lower body to the right atrium of the heart.
Collateral circulation: Alternate venous pathways that develop when the normal venous route is obstructed.
Paraumbilical veins: Veins connecting portal venous system to systemic veins around the umbilicus.
Thoracoepigastric veins: Superficial veins connecting axillary vein with femoral vein, serving as collaterals.
Esophageal varices: Dilated veins in the esophagus, usually due to portal hypertension.
Hemorrhoids: Dilated veins in the rectum or anus, linked with portal hypertension or increased abdominal pressure.
'Obstruction of Inferior vena cava' presents:
Paraumbilical dilatation
Thoraco-epigastric dilatation
Oesophagus varies
Haemorrhoides
Explanation: The correct answer is b) Thoraco-epigastric dilatation. Inferior vena cava obstruction leads to the development of collateral circulation through superficial abdominal veins, particularly thoraco-epigastric veins. These connect superficial epigastric veins with lateral thoracic veins, bypassing the obstruction. Paraumbilical veins, esophageal varices, and hemorrhoids are associated with portal hypertension, not IVC obstruction.
Which of the following is the most common cause of inferior vena cava obstruction?
Renal cell carcinoma
Portal hypertension
Cirrhosis of the liver
Esophageal carcinoma
Explanation: The answer is a) Renal cell carcinoma. Renal cell carcinoma often invades the renal vein and extends into the IVC. This intraluminal tumor thrombus obstructs venous return. Portal hypertension and cirrhosis are unrelated to IVC obstruction. Esophageal carcinoma leads to dysphagia but not IVC block. Thus, malignancy is a key cause clinically.
Which clinical sign is most characteristic of inferior vena cava obstruction?
Caput medusae
Scrotal varicocele
Ascites
Distended superficial abdominal veins
Explanation: The correct answer is d) Distended superficial abdominal veins. In IVC obstruction, collateral veins in the abdominal wall become prominent, particularly thoracoepigastric veins. Caput medusae occurs due to portal hypertension through the paraumbilical veins. Ascites is also more linked with portal disease. Scrotal varicocele is a localized venous dilation.
Direction of blood flow in superficial abdominal veins in IVC obstruction is:
Upward toward thorax
Downward toward legs
Both upward and downward
Unchanged
Explanation: The correct answer is a) Upward toward thorax. When the IVC is obstructed, blood from lower limbs is redirected via thoracoepigastric veins toward the axillary vein in the thorax. This reversal provides collateral drainage. Downward or unchanged flow would not bypass the block. Both directions are not characteristic.
A patient with IVC obstruction presents with bilateral lower limb edema. What is the underlying mechanism?
Increased lymphatic drainage
Venous stasis due to blocked return
Portal hypertension
Cardiac failure
Explanation: The answer is b) Venous stasis due to blocked return. Obstruction of IVC prevents venous blood from lower limbs returning to the heart, causing venous congestion, edema, and dilated collaterals. Lymphatic changes are secondary. Portal hypertension affects splanchnic circulation, not primarily lower limbs. Cardiac failure causes systemic edema, not isolated leg swelling.
Which investigation best demonstrates IVC obstruction?
Ultrasound with Doppler
X-ray abdomen
Barium swallow
Electrocardiogram
Explanation: The correct answer is a) Ultrasound with Doppler. Doppler ultrasound is the first-line, non-invasive imaging to detect IVC thrombus or obstruction by showing blood flow changes. CT and MRI can also help, but plain X-ray and barium swallow are irrelevant. ECG is for heart rhythm, not IVC pathology.
Collateral pathway in IVC obstruction involves:
Superior mesenteric vein
Thoracoepigastric vein
Paraumbilical vein
Splenic vein
Explanation: The answer is b) Thoracoepigastric vein. This vein connects superficial epigastric veins (femoral territory) to lateral thoracic veins (axillary territory). It becomes a major collateral in IVC obstruction. Paraumbilical and mesenteric veins are linked with portal system. Splenic vein also belongs to portal venous circulation.
In chronic IVC obstruction, which symptom is least likely?
Leg swelling
Abdominal wall venous distension
Hematemesis
Liver enlargement
Explanation: The correct answer is c) Hematemesis. Hematemesis occurs in portal hypertension due to esophageal varices. In IVC obstruction, main symptoms are leg edema, superficial abdominal venous prominence, and sometimes hepatomegaly due to hepatic venous congestion. Bleeding from upper GI tract is not a direct feature of IVC block.
Which clinical maneuver helps differentiate IVC obstruction from portal hypertension?
Direction of flow in superficial veins
Liver palpation
Rectal examination
Spleen percussion
Explanation: The correct answer is a) Direction of flow in superficial veins. In IVC obstruction, superficial abdominal veins show upward flow to thorax, while in portal hypertension, paraumbilical veins flow radially outward. This clinical test differentiates both entities effectively. Liver palpation, rectal exam, and spleen percussion are less specific.
Which surgical approach is preferred for removal of IVC thrombus extending from renal vein?
Midline laparotomy
Posterior approach
Thoracoabdominal approach
Inguinal incision
Explanation: The answer is a) Midline laparotomy. This provides excellent exposure to renal veins, IVC, and surrounding structures. In cases of renal cell carcinoma with thrombus, vascular control is essential. Posterior and inguinal approaches do not provide adequate access. Thoracoabdominal is reserved for extensive suprahepatic extension.
In IVC obstruction below renal veins, which collateral venous pathway is most significant?
Azygos-hemiazygos system
Thoracoepigastric vein
Ovarian/testicular veins
Inferior phrenic veins
Explanation: The correct answer is b) Thoracoepigastric vein. When obstruction is infra-renal, thoracoepigastric veins form the most important superficial collateral pathway, draining into axillary vein. Azygos sys
Topic: Abdominal Arteries
Subtopic: Coeliac Trunk and Its Branches
Keyword Definitions:
Coeliac Trunk: First major branch of the abdominal aorta supplying stomach, spleen, liver, and pancreas.
Left Gastric Artery: Branch of coeliac trunk supplying lesser curvature of the stomach.
Right Gastric Artery: Usually arises from the proper hepatic artery, not coeliac trunk directly.
Splenic Artery: Branch of coeliac trunk supplying spleen, pancreas, and fundus of stomach.
Common Hepatic Artery: Branch of coeliac trunk giving proper hepatic artery to liver and right gastric artery.
Abdominal Aorta: Major artery from heart supplying abdominal organs.
Branches: Major arteries originating from a main trunk.
Gastroepiploic Arteries: Branches supplying stomach greater curvature.
Portal Circulation: System of veins and arteries related to liver blood supply.
Collateral Circulation: Alternative pathways for blood flow if primary arteries are blocked.
Lead Question – 2012
All of the following arteries are the branches of coeliac trunk, EXCEPT?
a) Left gastric artery
b) Right gastric artery
c) Splenic artery
d) Hepatic artery
Explanation: The coeliac trunk has three primary branches: left gastric artery, splenic artery, and common hepatic artery. The right gastric artery usually arises from the proper hepatic artery, not directly from coeliac trunk. Therefore, the correct answer is b) Right gastric artery. Understanding these branches is clinically important in surgeries and liver-stomach blood supply.
1. The common hepatic artery gives rise to which of the following?
a) Left gastric artery
b) Right gastric artery
c) Splenic artery
d) Superior mesenteric artery
Explanation: The common hepatic artery arises from the coeliac trunk and gives off the proper hepatic artery, gastroduodenal artery, and right gastric artery. This is important in liver and stomach surgeries. Correct answer: b) Right gastric artery.
2. Splenic artery branches to which organ?
a) Stomach fundus
b) Spleen
c) Pancreas
d) All of the above
Explanation: The splenic artery supplies the spleen primarily, with branches to the pancreas and short gastric arteries supplying the fundus. This collateral supply is critical during splenectomy. Correct answer: d) All of the above.
3. Which artery arises directly from coeliac trunk and supplies lesser curvature of stomach?
a) Right gastric artery
b) Left gastric artery
c) Gastroduodenal artery
d) Superior mesenteric artery
Explanation: The left gastric artery arises directly from the coeliac trunk and supplies the lesser curvature of the stomach. It anastomoses with the right gastric artery, forming collateral circulation. Correct answer: b) Left gastric artery.
4. Gastroduodenal artery is a branch of:
a) Coeliac trunk directly
b) Common hepatic artery
c) Splenic artery
d) Superior mesenteric artery
Explanation: The gastroduodenal artery arises from the common hepatic artery (branch of coeliac trunk), supplying the pylorus, duodenum, and pancreas. Awareness is important for managing upper GI bleeding. Correct answer: b) Common hepatic artery.
5. Which of the following is NOT a primary branch of coeliac trunk?
a) Left gastric artery
b) Splenic artery
c) Common hepatic artery
d) Superior mesenteric artery
Explanation: The superior mesenteric artery arises from the abdominal aorta below the coeliac trunk, not from it. Coeliac trunk branches are left gastric, splenic, and common hepatic arteries. Correct answer: d) Superior mesenteric artery.
6. The right gastric artery anastomoses with:
a) Left gastric artery
b) Splenic artery
c) Gastroduodenal artery
d) Inferior mesenteric artery
Explanation: The right gastric artery, arising from proper hepatic artery, anastomoses with the left gastric artery along the lesser curvature of the stomach, providing important collateral blood flow. Correct answer: a) Left gastric artery.
7. Clinical importance of splenic artery ligation includes:
a) Control of splenic hemorrhage
b) Pancreatic tumor surgery
c) Gastric fundus surgery
d) All of the above
Explanation: The splenic artery supplies spleen, pancreas, and gastric fundus. Ligating it is necessary in splenectomy, pancreatic surgeries, or upper gastric operations to control bleeding. Correct answer: d) All of the above.
8. Hepatic artery proper arises from:
a) Left gastric artery
b) Common hepatic artery
c) Splenic artery
d) Coeliac trunk directly
Explanation: The hepatic artery proper is a continuation of the common hepatic artery (branch of coeliac trunk), giving off right gastric and cystic arteries. Correct answer: b) Common hepatic artery.
9. Short gastric arteries are branches of:
a) Splenic artery
b) Left gastric artery
c) Right gastric artery
d) Common hepatic artery
Explanation: Short gastric arteries arise from the terminal splenic artery branches to supply the fundus of the stomach. Correct answer: a) Splenic artery.
10. During upper GI bleed, which artery is most commonly involved?
a) Right gastric artery
b) Left gastric artery
c) Splenic artery
d) Superior mesenteric artery
Explanation: The left gastric artery is most commonly implicated in upper GI bleeding, especially along lesser curvature ulcers. Its anastomosis with right gastric artery can also provide collateral circulation. Correct answer: b) Left gastric artery.
Chapter: Thorax
Topic: Venous System of Thorax
Subtopic: Azygos Vein
Keywords
• Azygos vein – unpaired vein draining thoracic wall into SVC
• Superior vena cava – major vein returning blood from upper body
• Hemiazygos vein – tributary of azygos system on left side
• Bronchial veins – drain blood from bronchi
• Mediastinum – central thoracic compartment
• Esophageal veins – tributaries of azygos system
Q1. (2012 – Lead Question)
Azygos vein drains into:
a) Left brachiocephalic vein
b) Inferior vena cava
c) Superior vena cava
d) Right brachiocephalic vein
Answer & Explanation:
Correct answer: c) Superior vena cava.
The azygos vein is a key collateral pathway between the superior and inferior vena cava. It arches over the right main bronchus at the root of the right lung and opens into the superior vena cava. This pathway ensures venous return even when the IVC is obstructed.
Q2.
Which of the following is a tributary of the azygos vein?
a) Right superior intercostal vein
b) Left brachiocephalic vein
c) Inferior thyroid vein
d) External jugular vein
Answer & Explanation:
Correct answer: a) Right superior intercostal vein.
The azygos vein collects venous return from the posterior intercostal veins, right superior intercostal vein, hemiazygos, and accessory hemiazygos veins. These tributaries help drain thoracic walls and act as important venous channels, especially in cases of caval obstruction.
Q3.
The azygos vein arches over which structure before entering the superior vena cava?
a) Right pulmonary artery
b) Right main bronchus
c) Esophagus
d) Thoracic duct
Answer & Explanation:
Correct answer: b) Right main bronchus.
Before draining into the SVC, the azygos vein forms an arch over the root of the right lung, passing above the right main bronchus. This arch is an important radiological landmark on chest X-rays and CT scans.
Q4. Clinical Scenario
A 50-year-old man with obstruction of the inferior vena cava is able to maintain venous return through collateral channels. Which vein plays the most important role?
a) Azygos vein
b) Internal jugular vein
c) Basilic vein
d) Median cubital vein
Answer & Explanation:
Correct answer: a) Azygos vein.
The azygos system forms an alternative pathway connecting SVC and IVC. In IVC obstruction, azygos and hemiazygos veins enlarge and ensure drainage of lower body blood into the superior vena cava. Clinically, this reduces venous congestion.
Q5.
The hemiazygos vein usually drains into the azygos vein at which level?
a) T6
b) T8
c) T9
d) T12
Answer & Explanation:
Correct answer: c) T9.
The hemiazygos vein ascends on the left side of the vertebral column and usually crosses to join the azygos vein around the T9 vertebral level. This anatomical connection allows venous communication between left and right thoracic walls.
Q6. Clinical
On CT chest, a dilated azygos vein is noticed. Which of the following conditions may cause it?
a) Portal hypertension
b) Pulmonary embolism
c) Bronchial asthma
d) Pleural effusion
Answer & Explanation:
Correct answer: a) Portal hypertension.
In portal hypertension, collateral circulation develops via portosystemic anastomosis. One such pathway is the esophageal veins → azygos vein → SVC. This can cause dilatation of the azygos vein and visible varices on imaging.
Q7.
The azygos vein develops embryologically from which structure?
a) Posterior cardinal vein
b) Subcardinal vein
c) Supracardinal vein
d) Vitelline vein
Answer & Explanation:
Correct answer: c) Supracardinal vein.
The azygos system, including azygos, hemiazygos, and accessory hemiazygos, develops from the supracardinal veins. This embryological origin explains their paravertebral location and connection to intercostal veins.
Q8.
Which structure lies anterior to the arch of azygos vein?
a) Right vagus nerve
b) Right phrenic nerve
c) Trachea
d) Esophagus
Answer & Explanation:
Correct answer: c) Trachea.
The azygos arch lies posterior to the superior vena cava and arches over the right main bronchus. The trachea is anterior to it, forming a useful landmark in thoracic imaging.
Q9. Clinical
A patient with carcinoma esophagus develops esophageal varices. Which venous system is primarily involved?
a) Azygos vein
b) Portal vein
c) Inferior thyroid vein
d) Subclavian vein
Answer & Explanation:
Correct answer: a) Azygos vein.
Esophageal varices result from portosystemic anastomosis between left gastric vein (portal system) and esophageal veins draining into azygos (systemic). Enlargement of these collaterals leads to varices and risk of life-threatening bleeding.
Q10.
Accessory hemiazygos vein drains into azygos vein at which level?
a) T5–T6
b) T7–T8
c) T9–T10
d) T11–T12
Answer & Explanation:
Correct answer: a) T5–T6.
The accessory hemiazygos vein descends on the left side, draining mid-thoracic intercostal spaces and typically crosses to join azygos around T5–T6 vertebrae. This provides an alternate channel for thoracic venous return.
Chapter: Cardiovascular System
Topic: Right Atrium Anatomy
Subtopic: Koch’s Triangle
Keyword Definitions
Koch’s Triangle – Anatomical landmark in right atrium used to locate AV node.
Tricuspid valve ring – Fibrous annulus surrounding tricuspid valve, forms part of Koch’s triangle.
Coronary sinus orifice – Opening of coronary sinus into right atrium, key boundary of Koch’s triangle.
Tendon of Todaro – Fibrous extension from Eustachian valve to central fibrous body, part of Koch’s triangle.
Limbus fossa ovalis – Prominent ridge at margin of fossa ovalis, not part of Koch’s triangle.
AV node – Specialized cardiac tissue located within Koch’s triangle, responsible for conduction between atria and ventricles.
Bundle of His – Continuation of AV node, conducting pathway to ventricles.
Eustachian valve – Ridge at IVC opening, gives rise to tendon of Todaro.
Right atrium – Chamber of heart receiving systemic venous blood, site of Koch’s triangle.
Catheter ablation – Clinical procedure often targeting arrhythmias using Koch’s triangle as a landmark.
Lead Question (2012 NEET PG):
Boundary of the Koch's triangle is not formed by?
a) Tricuspid valve ring
b) Coronary sinus
c) Tendon of Todaro
d) Limbus fossa ovalis
Answer & Explanation:
The limbus fossa ovalis is not a boundary of Koch’s triangle. Koch’s triangle is bounded by the tricuspid valve annulus (anteriorly), coronary sinus orifice (posteriorly), and tendon of Todaro (superiorly). Inside this triangle lies the AV node, crucial for conduction. The limbus fossa ovalis is unrelated to this area, hence the correct answer is d).
Q1. The apex of Koch’s triangle corresponds to which structure?
a) AV node
b) SA node
c) Bundle of His
d) Crista terminalis
Answer & Explanation:
The apex of Koch’s triangle corresponds to the AV node, which is the key conduction relay between atria and ventricles. SA node lies near the SVC opening, not in Koch’s triangle. Bundle of His is distal continuation of AV node, and crista terminalis is a ridge in RA. Answer – a).
Q2. During catheter ablation for AV nodal reentry tachycardia, the interventional cardiologist targets which landmark near Koch’s triangle?
a) Near coronary sinus ostium
b) Near SA node
c) Near fossa ovalis
d) Near pulmonary veins
Answer & Explanation:
In AVNRT ablation, the catheter is positioned near the coronary sinus ostium at the posterior boundary of Koch’s triangle. This area provides safe access to slow pathway modification without damaging the compact AV node. Pulmonary veins relate to atrial fibrillation ablation, not AVNRT. Correct answer – a).
Q3. Which of the following is true regarding the tendon of Todaro?
a) Derived from the Thebesian valve
b) Extends from IVC valve to central fibrous body
c) Forms inferior vena cava orifice
d) Lies in left atrium
Answer & Explanation:
The tendon of Todaro is a fibrous band formed by the continuation of the Eustachian valve (valve of IVC) that extends to the central fibrous body. It is an essential boundary of Koch’s triangle. It does not arise from Thebesian valve, nor is it in left atrium. Answer – b).
Q4. The AV node is supplied mainly by which artery in Koch’s triangle?
a) Left coronary artery
b) Right coronary artery
c) Left anterior descending artery
d) Circumflex artery
Answer & Explanation:
The AV nodal artery, usually a branch of the right coronary artery, runs in the region of Koch’s triangle to supply the AV node. In some individuals with left dominant circulation, it arises from the circumflex artery. However, most commonly it is RCA. Correct answer – b).
Q5. Which clinical arrhythmia is classically associated with Koch’s triangle anatomy?
a) AV nodal reentrant tachycardia (AVNRT)
b) Wolff-Parkinson-White syndrome
c) Ventricular fibrillation
d) Atrial flutter
Answer & Explanation:
AVNRT arises due to dual AV nodal pathways located within Koch’s triangle. The slow and fast pathways allow reentrant circuits. WPW involves accessory pathways (Bundle of Kent). Atrial flutter relates to cavotricuspid isthmus. Ventricular fibrillation originates in ventricles. Correct answer – a).
Q6. Which structure forms the posterior boundary of Koch’s triangle?
a) Coronary sinus ostium
b) Tricuspid valve annulus
c) Eustachian ridge
d) Fossa ovalis
Answer & Explanation:
The posterior boundary of Koch’s triangle is formed by the orifice of the coronary sinus. Tricuspid valve annulus is anterior boundary, tendon of Todaro superior boundary. Fossa ovalis is not part of the triangle. Correct answer – a).
Q7. In right atrial dissection, damage to Koch’s triangle risks injury to which conduction structure?
a) SA node
b) AV node
c) Bundle branches
d) Purkinje fibers
Answer & Explanation:
Koch’s triangle houses the AV node, and any surgical or dissection injury here can result in complete heart block. SA node is at SVC-RA junction, not in this region. Bundle branches and Purkinje fibers are distal conduction tissues in ventricles. Correct answer – b).
Q8. In echocardiography, Koch’s triangle is used as a landmark for:
a) Estimating pulmonary artery pressure
b) Localizing the AV node
c) Measuring left atrial size
d) Assessing mitral valve prolapse
Answer & Explanation:
Koch’s triangle serves as an echocardiographic landmark to locate the AV node during procedures and for guiding device placement. It is not used for pulmonary artery pressure, left atrial size, or mitral valve prolapse assessment. Correct answer – b).
Q9. In which chamber of the heart is Koch’s triangle located?
a) Left atrium
b) Right atrium
c) Left ventricle
d) Right ventricle
Answer & Explanation:
Koch’s triangle is an anatomical area within the right atrium, bounded by tricuspid valve annulus, coronary sinus ostium, and tendon of Todaro. Left atrium is relevant for pulmonary veins and mitral valve. Correct answer – b).
Q10. A surgeon operating near Koch’s triangle must avoid injuring which structure to prevent complete heart block?
a) AV node
b) SA node
c) Right bundle branch
d) Moderator band
Answer & Explanation:
The AV node, lying at the apex of Koch’s triangle, is highly vulnerable to injury during surgery or catheterization. Damage can result in AV block requiring pacemaker implantation. SA node is in high right atrium, not this area. Correct answer – a).
Q11. Which surface landmark corresponds to the location of Koch’s triangle?
a) Near septal leaflet of tricuspid valve
b) Near lateral leaflet of tricuspid valve
c) Near anterior mitral valve leaflet
d) Near pulmonary valve annulus
Answer & Explanation:
Koch’s triangle lies adjacent to the septal leaflet of the tricuspid valve inside the right atrium. This relationship is crucial for electrophysiologists to navigate during ablation procedures. It is not related to mitral valve or pulmonary valve areas. Correct answer – a).
Chapter: Anatomy
Topic: Cardiovascular System
Subtopic: Coronary Circulation – Coronary Sinus Tributaries
Keywords:
Coronary sinus: Large venous channel in the posterior atrioventricular groove, drains most venous blood of heart into right atrium.
Great cardiac vein: Major tributary of coronary sinus accompanying LAD artery.
Small cardiac vein: Runs with right marginal artery, drains into coronary sinus.
Middle cardiac vein: Runs in posterior interventricular groove, joins coronary sinus.
Anterior cardiac veins: Drain directly into right atrium, not via coronary sinus.
Thebesian veins: Minute veins draining directly into heart chambers, not into coronary sinus.
Lead Question – 2012
Q1. Tributary of coronary sinus?
a) Anterior cardiac vein
b) Thebesian vein
c) Smallest cardiac vein
d) Great cardiac vein
[Explanation – Light Yellow]
Explanation: The great cardiac vein is the principal tributary of the coronary sinus. The coronary sinus receives blood from great cardiac, middle cardiac, small cardiac, oblique vein of left atrium, and posterior vein of left ventricle. The anterior cardiac and Thebesian veins drain directly into right atrium, not via coronary sinus. Correct answer: d) Great cardiac vein.
Q2. Which of the following veins drains directly into the right atrium, bypassing the coronary sinus?
a) Middle cardiac vein
b) Great cardiac vein
c) Anterior cardiac vein
d) Small cardiac vein
Explanation: The anterior cardiac veins drain the anterior surface of the right ventricle and empty directly into the right atrium, unlike the great, middle, and small cardiac veins that join the coronary sinus. This direct drainage is clinically significant during venous cannulation. Correct answer: c) Anterior cardiac vein.
Q3. The Thebesian veins are characterized by which feature?
a) Drain into coronary sinus
b) Drain directly into heart chambers
c) Always accompany coronary arteries
d) Present only in ventricles
Explanation: Thebesian veins, also known as venae cordis minimae, are minute veins that drain directly into the cardiac chambers, mainly right atrium and ventricles. They do not join the coronary sinus. Their presence allows minimal collateral venous drainage. Correct answer: b) Drain directly into heart chambers.
Q4. Which of the following is the largest tributary of the coronary sinus?
a) Middle cardiac vein
b) Great cardiac vein
c) Small cardiac vein
d) Posterior vein of left ventricle
Explanation: The great cardiac vein is the largest tributary, accompanying the left anterior descending (LAD) artery along the anterior interventricular groove. It drains into the coronary sinus at the left end. This vein is critical in retrograde cardioplegia procedures in cardiac surgery. Correct answer: b) Great cardiac vein.
Q5. Which vein accompanies the posterior interventricular artery and drains into the coronary sinus?
a) Great cardiac vein
b) Middle cardiac vein
c) Small cardiac vein
d) Oblique vein of left atrium
Explanation: The middle cardiac vein runs in the posterior interventricular groove along with the posterior interventricular artery and drains into the coronary sinus. This is an important landmark during posterior heart dissections. Correct answer: b) Middle cardiac vein.
Q6. The oblique vein of left atrium is a remnant of which embryological structure?
a) Left anterior cardinal vein
b) Left superior vena cava
c) Left posterior cardinal vein
d) Right sinus horn
Explanation: The oblique vein of left atrium (Marshall’s vein) is a remnant of the left superior vena cava. It courses obliquely over the left atrium to join the coronary sinus. Its recognition is important during electrophysiological ablation procedures. Correct answer: b) Left superior vena cava.
Q7. A patient undergoing retrograde cardioplegia has a catheter placed in which venous structure?
a) Superior vena cava
b) Coronary sinus
c) Anterior cardiac vein
d) Thebesian veins
Explanation: In retrograde cardioplegia, the cardioplegic solution is delivered via the coronary sinus to perfuse the coronary venous system and myocardium. This method is used when coronary arteries are obstructed. Correct answer: b) Coronary sinus.
Q8. Which vein is most closely related to the right marginal artery and drains into the coronary sinus?
a) Great cardiac vein
b) Small cardiac vein
c) Middle cardiac vein
d) Thebesian vein
Explanation: The small cardiac vein accompanies the right marginal artery along the inferior border of the heart and drains into the coronary sinus near its termination. This association is an important anatomical correlation. Correct answer: b) Small cardiac vein.
Q9. During cardiac catheterization, the opening of the coronary sinus is found in which chamber?
a) Right atrium
b) Left atrium
c) Right ventricle
d) Left ventricle
Explanation: The coronary sinus opens into the right atrium between the orifice of the inferior vena cava and the tricuspid valve. It is guarded by the Thebesian valve. Recognition is important during invasive procedures. Correct answer: a) Right atrium.
Q10. In a stab injury to the posterior atrioventricular groove, which venous structure is most likely to be injured?
a) Great cardiac vein
b) Coronary sinus
c) Anterior cardiac vein
d) Thebesian vein
Explanation: The coronary sinus lies in the posterior part of the atrioventricular groove. Trauma to this region can damage the coronary sinus leading to retroperitoneal cardiac tamponade. Correct answer: b) Coronary sinus.
Mitral Valve Prolapse (MVP): A condition in which the mitral valve leaflets bulge into the left atrium during systole.
Stroke: Acute neurological deficit due to impaired blood flow to the brain.
Infective Endocarditis: Infection of the endocardial surface of the heart, often involving valves.
Mitral Stenosis: Narrowing of the mitral valve opening causing obstructed blood flow from left atrium to ventricle.
Ventricular Arrhythmia: Abnormal rhythm originating from the ventricles, can be life-threatening.
Chapter: Cardiovascular System
Topic: Valvular Heart Disease
Subtopic: Mitral Valve Disorders
Lead Question 2012: Which of the following complications is not seen in mitral valve prolapse?
a) Stroke
b) Infective endocarditis
c) Mitral stenosis
d) Ventricular arrhythmia
Answer: c) Mitral stenosis
Explanation: Mitral valve prolapse (MVP) usually leads to complications such as mitral regurgitation, arrhythmias including ventricular arrhythmias, infective endocarditis, and rarely stroke due to embolic phenomena. However, mitral stenosis, which is narrowing of the mitral valve, is not a complication of MVP. The valve in MVP is typically redundant and floppy, not stenotic. Clinical presentations often include mid-systolic click, palpitations, and occasional thromboembolic events.
1. Which auscultatory finding is most characteristic of mitral valve prolapse?
a) Opening snap
b) Mid-systolic click
c) S3 gallop
d) Continuous murmur
Answer: b) Mid-systolic click
Explanation: The mid-systolic click is characteristic of MVP due to sudden tensing of the redundant mitral leaflet. It is often followed by a late systolic murmur if regurgitation is present. The opening snap is associated with mitral stenosis, S3 with volume overload, and continuous murmur with patent ductus arteriosus.
2. Which arrhythmia is most commonly associated with mitral valve prolapse?
a) Atrial fibrillation
b) Ventricular tachycardia
c) Sinus bradycardia
d) Junctional rhythm
Answer: b) Ventricular tachycardia
Explanation: Ventricular arrhythmias, including ventricular tachycardia, are commonly seen in MVP patients and can rarely lead to sudden cardiac death. Atrial fibrillation is more common in significant mitral regurgitation from other causes. Sinus bradycardia and junctional rhythms are not typical complications of MVP.
3. What is the typical murmur associated with MVP?
a) Holosystolic murmur at apex
b) Late systolic murmur with mid-systolic click
c) Diastolic murmur at apex
d) Continuous murmur at upper sternal border
Answer: b) Late systolic murmur with mid-systolic click
Explanation: The hallmark of MVP is a mid-systolic click due to sudden prolapse of the valve, often followed by a late systolic murmur if regurgitation is present. Holosystolic murmur occurs in primary mitral regurgitation, diastolic murmur in stenosis, and continuous murmur in PDA.
4. Which of the following is the most common symptom of MVP?
a) Dyspnea
b) Palpitations
c) Chest pain
d) Syncope
Answer: b) Palpitations
Explanation: Palpitations are the most common symptom due to arrhythmias in MVP. Chest pain and dyspnea may occur, but syncope is less common. These symptoms are often associated with autonomic dysfunction and ventricular ectopy.
5. Which imaging modality is most definitive for diagnosing MVP?
a) Chest X-ray
b) Echocardiography
c) ECG
d) Cardiac MRI
Answer: b) Echocardiography
Explanation: Echocardiography is the gold standard for diagnosing MVP. It shows leaflet prolapse >2 mm above the mitral annulus in systole. Chest X-ray is nonspecific, ECG detects arrhythmias but not structural changes, and cardiac MRI is rarely required for diagnosis.
6. Which of the following increases the risk of sudden cardiac death in MVP patients?
a) Mild mitral regurgitation
b) Complex ventricular arrhythmias
c) Isolated mid-systolic click
d) Normal ECG
Answer: b) Complex ventricular arrhythmias
Explanation: Complex ventricular arrhythmias in MVP, especially in patients with severe regurgitation or fibrosis, can lead to sudden cardiac death. Mild regurgitation and isolated clicks are low-risk findings. Normal ECG does not rule out MVP but indicates lower arrhythmic risk.
7. Which of the following prophylactic measures is indicated for MVP with prior endocarditis?
a) Beta-blockers
b) Antibiotic prophylaxis before dental procedures
c) ACE inhibitors
d) Anticoagulants
Answer: b) Antibiotic prophylaxis before dental procedures
Explanation: Patients with MVP who have a history of infective endocarditis require antibiotic prophylaxis before invasive dental or surgical procedures. Beta-blockers are used for symptomatic palpitations, ACE inhibitors for heart failure, and anticoagulants only in atrial fibrillation or embolic risk.
8. Which lab test can support the diagnosis of embolic complications in MVP?
a) ESR
b) Blood cultures
c) Brain imaging (CT/MRI)
d) Troponin I
Answer: c) Brain imaging (CT/MRI)
Explanation: Brain imaging is indicated if embolic stroke is suspected in MVP. Blood cultures detect endocarditis. ESR is nonspecific, and troponin I is used in myocardial injury, not embolic events from MVP.
9. Which lifestyle measure is recommended in symptomatic MVP patients?
a) Avoid caffeine and stimulants
b) High-salt diet
c) Sedentary lifestyle
d) Smoking
Answer: a) Avoid caffeine and stimulants
Explanation: MVP symptoms like palpitations can worsen with caffeine, stimulants, and stress. Lifestyle modification, including regular mild exercise and stress management, helps. High-salt diets, sedentary lifestyle, and smoking can worsen cardiac risk and are not advised.
10. Which pharmacological therapy is preferred for symptomatic MVP with palpitations?
a) Beta-blockers
b) Calcium channel blockers
c) Diuretics
d) Nitrates
Answer: a) Beta-blockers
Explanation: Beta-blockers are first-line therapy for symptomatic MVP patients experiencing palpitations or anxiety-related symptoms. They reduce heart rate, decrease arrhythmic burden, and improve quality of life. Calcium channel blockers are secondary options, while diuretics and nitrates are not indicated for arrhythmias in MVP.
IVC Filter: A medical device inserted into the inferior vena cava to prevent pulmonary embolism by trapping large thrombi.
Pulmonary Embolism (PE): Obstruction of pulmonary arteries by blood clots, which can be life-threatening.
Thrombosis: Formation of a blood clot inside a blood vessel, obstructing blood flow.
Native Vessel Disease: Pre-existing disease in the patient’s own veins or arteries, e.g., deep vein thrombosis (DVT).
Symptom Relief: Reduction of pain, swelling, or discomfort associated with vascular obstruction.
Chapter: Cardiovascular System
Topic: Venous Thrombosis Management
Subtopic: IVC Filter
Lead Question 2012: IVC filter is used in following except:
a) To reduces symptoms
b) Negligible size of emboli
c) To prolong life
d) To prevent progress of native blood vessel disease
Answer: b) Negligible size of emboli
Explanation: IVC filters are designed to trap large thrombi from the lower extremities, preventing pulmonary embolism. They do not trap **small or negligible-sized emboli**, which pass through easily. Indications include preventing PE in patients with contraindications to anticoagulation, symptom relief, and occasionally improving survival in high-risk cases, but they do not directly halt progression of underlying vein disease.
1. What is the primary clinical indication for IVC filter placement?
a) Primary prevention in low-risk patients
b) Pulmonary embolism prevention when anticoagulation is contraindicated
c) Routine prophylaxis after surgery
d) Treating leg edema
Answer: b) Pulmonary embolism prevention when anticoagulation is contraindicated
Explanation: IVC filters are primarily indicated for patients at high risk of pulmonary embolism who cannot receive anticoagulants due to bleeding or other contraindications. They physically trap large thrombi traveling from deep veins. They are not used routinely in low-risk patients or solely for edema relief.
2. Which type of thrombi do IVC filters primarily capture?
a) Small microemboli
b) Large thrombi from deep veins
c) Platelet aggregates only
d) Fat emboli
Answer: b) Large thrombi from deep veins
Explanation: IVC filters are designed to trap **large thrombi** traveling from the lower extremities to prevent pulmonary embolism. Microemboli often pass through the filter, and fat emboli or platelet aggregates are not targeted. Correct placement and sizing are critical to maximize effectiveness.
3. Which is a potential complication of IVC filter placement?
a) Migration of the filter
b) Infection
c) Caval perforation
d) All of the above
Answer: d) All of the above
Explanation: Complications of IVC filters include **migration**, infection, caval perforation, thrombosis at the filter site, and filter fracture. Clinicians must weigh the benefits against potential risks and remove temporary filters when no longer indicated.
4. Which patients are considered for temporary or retrievable IVC filters?
a) Patients with permanent contraindication to anticoagulants
b) Patients undergoing high-risk surgery with transient PE risk
c) Patients with chronic DVT
d) All patients regardless of risk
Answer: b) Patients undergoing high-risk surgery with transient PE risk
Explanation: Temporary or retrievable IVC filters are placed for patients with **transient high risk** of pulmonary embolism, such as perioperative periods, and removed once risk subsides. Permanent filters are used for chronic contraindication to anticoagulation.
5. Does IVC filter placement directly prolong survival in all patients?
a) Yes, universally
b) No, mainly in high-risk PE
c) Only in low-risk patients
d) Only with concurrent anticoagulation
Answer: b) No, mainly in high-risk PE
Explanation: IVC filters may prolong survival in select **high-risk pulmonary embolism patients**, particularly those who cannot be anticoagulated. Routine use in all patients does not confer mortality benefit, as small emboli can still pass, and long-term complications may offset benefit.
6. Which imaging modality is used to guide IVC filter placement?
a) Ultrasound
b) Fluoroscopy
c) MRI
d) Chest X-ray
Answer: b) Fluoroscopy
Explanation: Fluoroscopy provides real-time imaging to accurately position the IVC filter at the correct level, typically below the renal veins. Ultrasound may assist in venous access, but MRI and chest X-ray are not used for procedural guidance.
7. Which factor reduces effectiveness of IVC filters?
a) Large thrombi
b) Multiple emboli
c) Small or negligible emboli
d) Correct positioning
Answer: c) Small or negligible emboli
Explanation: IVC filters cannot capture **small thrombi or microemboli**, which pass through the device, potentially causing pulmonary embolism. Correct positioning enhances efficacy for large emboli, but small emboli remain a limitation.
8. Which anticoagulation status typically warrants IVC filter placement?
a) Fully anticoagulated
b) Contraindication to anticoagulation
c) Minor risk of PE
d) None of the above
Answer: b) Contraindication to anticoagulation
Explanation: IVC filters are indicated when **anticoagulation is contraindicated** due to bleeding or other risks. Patients already safely anticoagulated typically do not require filters, as medications effectively prevent PE. Filters are adjunctive therapy, not first-line for all patients.
9. What is a major long-term risk of permanent IVC filters?
a) Filter migration
b) Caval thrombosis
c) Fracture of filter struts
d) All of the above
Answer: d) All of the above
Explanation: Long-term complications of permanent IVC filters include **migration**, **caval thrombosis**, and **filter fracture**. These risks highlight the importance of using filters only when indicated and considering retrievable filters for temporary high-risk scenarios.
10. Which statement is correct regarding IVC filters and native vessel disease?
a) IVC filter prevents progression of native DVT
b) Filter only traps emboli but does not cure underlying vein disease
c) Filter dissolves thrombi
d) Filter replaces anticoagulation permanently
Answer: b) Filter only traps emboli but does not cure underlying vein disease
Explanation: IVC filters **trap large emboli** to prevent PE but do not prevent or treat the underlying thrombosis or progression of native vessel disease. Anticoagulation or other therapies are needed to manage the primary DVT. Filters are an adjunct, not a definitive cure.
Right Coronary Artery (RCA): Supplies blood to the right atrium, right ventricle, SA node, AV node, and part of the ventricular septum.
Ventricular Septum: Wall separating right and left ventricles; blood supply is from both RCA and LAD.
SA Node: Pacemaker of the heart, located in right atrium; receives blood mainly from RCA.
AV Node: Conducting node between atria and ventricles; supplied by RCA in most individuals.
Left Bundle Branch (LBB): Conduction fiber in the left side of interventricular septum; primarily supplied by left anterior descending artery (LAD).
Chapter: Cardiovascular System
Topic: Coronary Circulation
Subtopic: Right Coronary Artery Distribution
Lead Question 2012: Right coronary artery supplies all, except?
a) Anterior 2/3 of ventricular septum
b) SA node
c) AV node
d) LBB
Answer: a) Anterior 2/3 of ventricular septum
Explanation: The **right coronary artery (RCA)** primarily supplies the right atrium, right ventricle, SA node, AV node, and posterior 1/3 of the ventricular septum. The **anterior 2/3 of the septum** and left bundle branch (LBB) are mainly supplied by the **left anterior descending (LAD) artery**, making option (a) the correct choice as it is not supplied by RCA.
1. Which part of the conduction system is mainly supplied by RCA?
a) SA node
b) AV node
c) Bundle of His
d) Purkinje fibers
Answer: a) SA node
Explanation: The **SA node**, located in the right atrium, is the primary pacemaker of the heart. It receives blood from the **RCA in 60% of individuals**, highlighting the artery’s role in maintaining normal sinus rhythm.
2. RCA dominance refers to:
a) RCA supplying LBB
b) RCA giving posterior descending artery
c) RCA supplying LAD territory
d) RCA supplying SA node only
Answer: b) RCA giving posterior descending artery
Explanation: **RCA dominance** occurs when the RCA gives rise to the **posterior descending artery (PDA)**, supplying the posterior interventricular septum. This influences myocardial infarct patterns and clinical outcomes.
3. Posterior 1/3 of ventricular septum is supplied by:
a) RCA
b) LAD
c) Circumflex artery
d) LBB
Answer: a) RCA
Explanation: The **posterior 1/3 of the interventricular septum** is typically supplied by the **RCA** via the posterior descending artery. LAD supplies the anterior 2/3, while the circumflex artery contributes variably. Knowledge of this is critical in managing septal infarctions.
4. Which artery primarily supplies the left bundle branch?
a) RCA
b) LAD
c) Circumflex
d) PDA
Answer: b) LAD
Explanation: The **left bundle branch** is located in the anterior 2/3 of the interventricular septum, which is primarily supplied by the **left anterior descending artery (LAD)**. RCA rarely contributes, highlighting the anatomical separation of conduction system blood supply.
5. RCA supplies AV node in approximately what percentage of individuals?
a) 40%
b) 65%
c) 80%
d) 95%
Answer: b) 65%
Explanation: The **AV node** is supplied by the **RCA in about 65% of individuals**, depending on coronary dominance. Knowledge of this is crucial in planning surgeries or managing conduction abnormalities following RCA occlusion or infarction.
6. Inferior wall myocardial infarction is usually due to occlusion of:
a) LAD
b) RCA
c) Circumflex
d) LBB
Answer: b) RCA
Explanation: **Inferior wall MI** is most commonly caused by occlusion of the **RCA**, which supplies the inferior portion of the right ventricle and posterior interventricular septum. Recognizing artery involvement guides ECG interpretation and intervention.
7. Which coronary artery supplies the posterior descending artery in left-dominant hearts?
a) RCA
b) LAD
c) Circumflex
d) AV node branch
Answer: c) Circumflex
Explanation: In **left-dominant circulation**, the **posterior descending artery (PDA)** arises from the **circumflex artery**, not RCA. This affects myocardial infarct localization and surgical planning, especially in coronary artery bypass grafting.
8. RCA primarily supplies which chamber of the heart?
a) Left atrium
b) Left ventricle
c) Right ventricle
d) Left atrial appendage
Answer: c) Right ventricle
Explanation: The **right coronary artery** predominantly supplies the **right ventricle** and parts of the right atrium. Its occlusion can cause right ventricular infarction, hypotension, and conduction disturbances due to AV node involvement.
9. Which conduction abnormality is commonly seen in RCA infarction?
a) Left bundle branch block
b) AV block
c) Sinus tachycardia
d) Right bundle branch block
Answer: b) AV block
Explanation: RCA infarction may compromise the **AV node**, leading to various degrees of **AV block**. Recognizing this helps in ECG interpretation and deciding temporary pacing in acute inferior myocardial infarction.
10. Anterior 2/3 of interventricular septum is supplied by:
a) RCA
b) LAD
c) Circumflex
d) Right atrial branch
Answer: b) LAD
Explanation: The **left anterior descending artery** supplies the **anterior 2/3 of the interventricular septum**, including the left bundle branch. This is why anterior MI often causes septal and conduction abnormalities, distinguishing it from RCA occlusion.
Transverse Sinus: A pericardial space posterior to the aorta and pulmonary trunk, allowing surgical access during cardiac procedures.
Pericardial Sinuses: Spaces within the pericardial sac, including transverse and oblique sinuses.
Right Atrium: Receives deoxygenated blood from systemic veins.
Left Atrium: Receives oxygenated blood from pulmonary veins.
Aorta: Main artery leaving the left ventricle, distributing oxygenated blood to the body.
Pulmonary Artery: Artery carrying deoxygenated blood from the right ventricle to the lungs.
Chapter: Heart Anatomy
Topic: Pericardial Sinuses
Subtopic: Transverse Sinus Location
Lead Question 2012: The transverse sinus is present posterior to which structures?
a) Right atrium
b) Left atrium
c) Upper pulmonary artery
d) Aorta
Answer: d) Aorta
Explanation: The transverse pericardial sinus lies posterior to the ascending aorta and pulmonary trunk, and anterior to the superior vena cava. It is a critical landmark during cardiac surgery, allowing surgeons to pass ligatures around the great arteries. It is not located behind the atria or pulmonary artery itself.
1. Which pericardial sinus is located posterior to the left atrium?
a) Transverse sinus
b) Oblique sinus
c) Coronary sinus
d) Superior sinus
Answer: b) Oblique sinus
Explanation: The oblique pericardial sinus is a blind recess posterior to the left atrium, bounded by the pulmonary veins and inferior vena cava. Understanding sinus locations is important during pericardial fluid drainage or cardiac surgery.
2. The transverse sinus is clinically important because:
a) It houses the SA node
b) Surgeons can pass ligatures around great arteries
c) It drains coronary veins
d) It surrounds the atrioventricular valves
Answer: b) Surgeons can pass ligatures around great arteries
Explanation: The transverse sinus lies between the arterial and venous ends of the heart, allowing safe passage of ligatures around the aorta and pulmonary artery during procedures such as cardiopulmonary bypass. It does not house the SA node or valves.
3. The transverse sinus is bounded anteriorly by:
a) Superior vena cava
b) Pulmonary veins
c) Ascending aorta and pulmonary trunk
d) Inferior vena cava
Answer: c) Ascending aorta and pulmonary trunk
Explanation: The transverse sinus lies posterior to the aorta and pulmonary trunk, and anterior to the superior vena cava. This anatomical position is significant during procedures like clamping or bypass.
4. Which structure lies posterior to the transverse sinus?
a) Aorta
b) Superior vena cava
c) Left atrium
d) Pulmonary artery
Answer: c) Left atrium
Explanation: The transverse sinus separates the arterial trunks (aorta and pulmonary trunk) anteriorly from the venous structures and left atrium posteriorly. Surgeons use this space to manipulate vessels during cardiac procedures.
5. The coronary sinus drains into which chamber of the heart?
a) Right atrium
b) Left atrium
c) Right ventricle
d) Left ventricle
Answer: a) Right atrium
Explanation: The coronary sinus is located in the posterior atrioventricular groove and drains deoxygenated blood from the myocardium into the right atrium. Its position is close to the oblique sinus but not the transverse sinus.
6. In cardiac surgery, clamping the aorta can be facilitated using which sinus?
a) Oblique sinus
b) Transverse sinus
c) Coronary sinus
d) Superior sinus
Answer: b) Transverse sinus
Explanation: The transverse sinus lies behind the aorta and pulmonary trunk. Surgeons pass tapes or clamps through this space to isolate these arteries during cardiopulmonary bypass, making it a key anatomical landmark.
7. Which sinus is a blind-ended recess located posterior to the left atrium?
a) Transverse sinus
b) Oblique sinus
c) Coronary sinus
d) Sinus of Valsalva
Answer: b) Oblique sinus
Explanation: The oblique sinus is a pericardial recess posterior to the left atrium. Its blind-ended nature makes it important for pericardial effusion accumulation. Unlike the transverse sinus, it is not used for surgical clamping.
8. The superior vena cava is located relative to the transverse sinus:
a) Anterior
b) Posterior
c) Lateral
d) Within
Answer: b) Posterior
Explanation: The transverse sinus lies anterior to the superior vena cava, with the sinus running between the arterial trunks and venous inflow. This spatial relationship allows surgical manipulation without damaging major veins.
9. Which sinus is important for placing a retrograde cardioplegia cannula?
a) Transverse sinus
b) Oblique sinus
c) Coronary sinus
d) Superior sinus
Answer: c) Coronary sinus
Explanation: Retrograde cardioplegia cannula is inserted into the coronary sinus to deliver cardioplegia solution. Although near the pericardial sinuses, it is distinct from transverse or oblique sinuses.
10. The transverse sinus separates which two structures?
a) Arterial trunks anteriorly and atria posteriorly
b) Left and right ventricles
c) Pulmonary veins and aorta
d) Coronary arteries and veins
Answer: a) Arterial trunks anteriorly and atria posteriorly
Explanation: The transverse sinus acts as a passage behind the aorta and pulmonary trunk (arterial trunks) and anterior to the left atrium, providing surgical access. This anatomical separation is crucial during open-heart procedures.
Keyword Definitions:
Native Aortic Valve Disease: A condition affecting the aortic valve without prior valve replacement or prosthetic material.
Right Hemiparesis: Weakness of the muscles on the right side of the body, usually due to a brain lesion or stroke.
Antiplatelet: Drugs that prevent platelet aggregation, reducing thrombus formation in arteries.
Anticoagulant: Drugs that reduce blood clotting by inhibiting clotting factors in the coagulation cascade.
Low Molecular Weight Heparin (LMWH): Injectable anticoagulant that prevents clot extension and embolization.
Dual Antiplatelet Therapy: Combination of two antiplatelet drugs (e.g., aspirin and clopidogrel) for enhanced platelet inhibition.
Stroke Prevention: Measures to reduce the risk of recurrent cerebrovascular events in at-risk individuals.
Lead Question (2012):
A patient with native aortic valve disease came with right hemiparesis. What will you do to prevent further stroke?
a) Antiplatelet only
b) Anticoagulant only
c) Both antiplatelet and anticoagulant
d) One dose of low molecular weight heparin sub-cutaneously followed by dual antiplatelet therapy
Explanation: The correct answer is b) Anticoagulant only. Native aortic valve disease with embolic stroke usually indicates a cardioembolic source, often requiring anticoagulation to prevent further events. Antiplatelets are more effective in atherosclerotic stroke, whereas anticoagulants like warfarin or DOACs are preferred for cardioembolic strokes. LMWH is used initially in certain cases but not routinely in chronic prevention.
Q2. A patient with mitral stenosis presents with sudden-onset left-sided weakness. Best drug for secondary prevention?
a) Aspirin
b) Warfarin
c) Clopidogrel
d) Dabigatran
Explanation: The correct answer is b) Warfarin. Mitral stenosis, especially with atrial fibrillation, is a high-risk source of cardioembolic stroke. Long-term anticoagulation with warfarin reduces stroke risk. Antiplatelets are insufficient for this type of embolic prevention.
Q3. Which investigation confirms cardioembolic stroke origin in aortic valve disease?
a) ECG
b) Echocardiography
c) CT Brain
d) Carotid Doppler
Explanation: The correct answer is b) Echocardiography. Echocardiography can detect valvular vegetations, thrombi, and structural abnormalities, confirming the source of embolism in cardioembolic strokes.
Q4. Which is NOT an indication for anticoagulation in valvular heart disease?
a) Prosthetic heart valve
b) Native valve with atrial fibrillation
c) Native valve with sinus rhythm but embolic stroke history
d) Native valve disease with mild regurgitation and no history of embolism
Explanation: The correct answer is d). Mild regurgitation without embolism or atrial fibrillation does not require anticoagulation.
Q5. In acute ischemic stroke within 4.5 hours, which therapy is considered?
a) Aspirin immediately
b) IV thrombolysis
c) Heparin bolus
d) Clopidogrel only
Explanation: The correct answer is b) IV thrombolysis. Alteplase is used within 4.5 hours of onset in eligible patients, improving outcomes. Antiplatelets are started after ruling out hemorrhage.
Q6. For preventing recurrent stroke in atrial fibrillation, which is preferred?
a) Aspirin
b) Apixaban
c) Ticagrelor
d) Clopidogrel
Explanation: The correct answer is b) Apixaban. Direct oral anticoagulants like apixaban reduce stroke risk in AF without the monitoring needs of warfarin.
Q7. Which side of the brain lesion causes right hemiparesis?
a) Left hemisphere
b) Right hemisphere
c) Brainstem only
d) Cerebellum
Explanation: The correct answer is a) Left hemisphere. The motor tracts decussate in the medulla, so a lesion in the left hemisphere causes contralateral (right-sided) weakness.
Q8. Which is the mainstay drug class for secondary prevention after non-cardioembolic stroke?
a) Anticoagulants
b) Antiplatelets
c) Statins
d) Beta-blockers
Explanation: The correct answer is b) Antiplatelets. Aspirin, clopidogrel, or combination therapy is standard for preventing recurrence in atherosclerotic stroke.
Q9. Which LMWH is most commonly used for initial anticoagulation?
a) Heparin sodium
b) Enoxaparin
c) Fondaparinux
d) Dalteparin
Explanation: The correct answer is b) Enoxaparin. It is widely used due to predictable pharmacokinetics and subcutaneous administration.
Q10. In a patient with recent stroke, anticoagulation should be started:
a) Immediately in all cases
b) After 1–2 weeks in large infarcts
c) Never
d) Only if carotid stenosis present
Explanation: The correct answer is b). In large infarcts, early anticoagulation increases hemorrhage risk; it’s usually delayed 1–2 weeks unless urgent indication exists.
Chapter: Cardiovascular System / Topic: Blood Vessels / Subtopic: Vasa Vasorum & Aortic Wall
Vasa vasorum — small blood vessels that supply the outer part of large vessel walls (tunica adventitia and outer media) providing nutrients and oxygen to cells beyond diffusion limits.
Tunica intima/media/adventitia — three layers of arterial wall; intima (endothelium + subendothelial), media (smooth muscle, elastic fibres) and adventitia (connective tissue, vasa vasorum, nerves).
Ascending aorta — proximal segment of the aorta arising from the left ventricle; closely related to coronary ostia and receives coronary arterial flow in systole/diastole dynamics.
Aortic vasa vasorum origin — vasa vasorum origins vary with aortic segment; in the ascending aorta they commonly arise from coronary arteries whereas in the descending thoracic aorta they arise from intercostal/bronchial branches.
Aortic pathology (dissection, aneurysm) — diseases of the aortic wall often relate to medial degeneration, vasa vasorum ischemia, inflammation (e.g., syphilitic vasa vasorum endarteritis) or hypertension-related stress.
Lead Question - 2012: Vasa Vasorum of the ascending aorta arises from?
a) Left coronary artery
b) Anterior interventricular artery
c) Posterior interventricular artery
d) Left atrium
Explanation (answer included): The vasa vasorum supplying the wall of the proximal ascending aorta typically originate from the coronary arteries because the coronary ostia lie at the root of the ascending aorta; these small vessels enter the adventitia and outer media. While both right and left coronary arteries contribute, among the choices given the most appropriate single answer is (a) Left coronary artery. Clinically this relationship explains why coronary disease or compromised coronary flow can affect the nutrition of the aortic root wall and contribute to local wall pathology.
Q2. Which layer of the aortic wall is primarily nourished by vasa vasorum?
a) Tunica intima only
b) Outer media and adventitia
c) Entire wall uniformly by diffusion
d) Endothelium via luminal blood only
Explanation (answer included): Vasa vasorum penetrate from the adventitial side to supply oxygen and nutrients primarily to the outer portion of the media and the adventitia because diffusion from the lumen is insufficient for thick vessel walls. The intima and inner media rely more on luminal diffusion. Therefore the correct answer is (b) Outer media and adventitia. Impaired vasa vasorum perfusion can cause medial ischemia contributing to aortic disease such as dissection or aneurysm formation.
Q3. Syphilitic (tertiary) aortitis primarily affects which structure leading to a 'tree-bark' aorta?
a) Vasa vasorum endarteritis causing medial ischemia
b) Intimal cholesterol deposition
c) Adventitial lymphatic hyperplasia
d) Endothelial plaque rupture
Explanation (answer included): Tertiary syphilis produces an obliterative endarteritis of the vasa vasorum, reducing blood supply to the media and causing medial necrosis with replacement fibrosis; the resulting scarring and wrinkled intima produce the classic 'tree-bark' appearance. Hence the key pathogenic event is (a) Vasa vasorum endarteritis causing medial ischemia. This explains why syphilitic aortitis involves the ascending aorta and aortic root leading to aneurysm and aortic regurgitation.
Q4. In which aortic segment are vasa vasorum most abundant and clinically most important?
a) Ascending aorta and aortic root
b) Abdominal aorta only
c) All segments have equal vasa vasorum density
d) Pulmonary trunk but not aorta
Explanation (answer included): Vasa vasorum are most plentiful in the proximal thoracic aorta (ascending aorta and arch) because the wall is thick and high-demand; these vessels supply the outer media and adventitia. The abdominal aorta has fewer vasa vasorum comparatively. Therefore the best choice is (a) Ascending aorta and aortic root. This concentration partly explains why syphilitic and other inflammatory processes target the ascending aorta and cause root aneurysms.
Q5. Which mechanism best explains how hypertension predisposes to aortic dissection?
a) Increased shear stress leading to intimal tear and medial degeneration
b) Decreased cardiac output causing stasis
c) Hypotension-induced proteolysis of vasa vasorum
d) Enhanced lymphatic drainage of the media
Explanation (answer included): Chronic hypertension increases intraluminal pressure and shear forces on the aortic wall, promoting intimal tears and progressive medial degeneration (cystic medial necrosis) that permit blood to dissect between layers. Vasa vasorum ischemia may contribute but the primary driver is mechanical stress. Therefore the best answer is (a) Increased shear stress leading to intimal tear and medial degeneration. Clinically, controlling blood pressure reduces risk of dissection progression.
Q6. The coronary ostia are located at which aortic sinus?
a) Right and left aortic sinuses (sinuses of Valsalva)
b) Non-coronary sinus only
c) At the aortic isthmus
d) Within the ascending aortic adventitia
Explanation (answer included): The coronary arteries arise from the right and left aortic sinuses (sinuses of Valsalva) immediately above the aortic valve cusps. The non-coronary sinus does not give rise to a coronary artery. Therefore the correct response is (a) Right and left aortic sinuses (sinuses of Valsalva). Because the coronaries originate at the root, vasa vasorum relationships are important for the aortic root and ascending aorta physiology and pathophysiology.
Q7. On contrast CT, an intimal flap in the ascending aorta indicates which condition?
a) Aortic dissection
b) Aortic coarctation
c) Aortic valve stenosis
d) Intramural hematoma only without intimal tear
Explanation (answer included): An intimal flap seen on contrast-enhanced CT denotes separation between true and false lumens and is diagnostic of aortic dissection. Intramural hematoma may lack an obvious flap initially; coarctation and valve stenosis show different imaging features. Thus the finding indicates (a) Aortic dissection. Timely recognition is vital because ascending (Type A) dissections often need urgent surgical repair to prevent rupture and coronary compromise.
Q8. Cystic medial degeneration (myxomatous change) of the aorta is characterized by:
a) Loss of elastic fibres and accumulation of proteoglycan matrix
b) Intimal fibrous plaque only
c) Proliferation of endothelial cells
d) Calcification of the adventitia
Explanation (answer included): Cystic medial degeneration involves fragmentation and loss of elastic fibres, smooth muscle cell dropout, and accumulation of mucopolysaccharide-rich proteoglycan matrix in the media. This weakens the wall and predisposes to aneurysm and dissection. The best description is (a) Loss of elastic fibres and accumulation of proteoglycan matrix. It is commonly seen in aging, Marfan syndrome, and other connective tissue disorders affecting the aorta.
Q9. During repair of an ascending aortic aneurysm involving the root, why must the coronary ostia be reimplanted carefully?
a) To restore coronary blood flow and avoid myocardial ischemia
b) To prevent pulmonary embolism
c) To increase venous return to the heart
d) To prevent aortic valve prolapse only
Explanation (answer included): Reimplantation of the coronary ostia after root replacement is essential to re-establish coronary perfusion; failure or malposition can cause myocardial ischemia, infarction or arrhythmia. This is the primary reason for meticulous coronary button reattachment during surgery. Thus the correct choice is (a) To restore coronary blood flow and avoid myocardial ischemia. The vasa vasorum supply to the root is also considered during repair to preserve wall viability.
Q10. Which of the following interventions most directly reduces shear stress on the aortic wall acutely?
a) Lowering blood pressure with intravenous beta-blocker
b) Starting high-dose statin therapy
c) Giving broad-spectrum antibiotics
d) Low molecular weight heparin
Explanation (answer included): Beta-blockers reduce heart rate and the force of left ventricular contraction, thereby lowering the rate of rise of aortic pressure and peak shear stress on the aortic wall; they are used acutely in suspected aortic dissection to limit extension. Statins and other measures are important long term but do not acutely reduce shear. Therefore the best acute intervention is (a) Lowering blood pressure with intravenous beta-blocker.
Q11. Which pathological feature is most suggestive of vasa vasorum-mediated ischemic injury in the aortic media?
a) Medial necrosis with loss of smooth muscle cells and elastic fibres
b) Intimal lipid core formation only
c) Endothelial proliferation with thrombosis of lumen
d) Adventitial calcification exclusively
Explanation (answer included): Ischemia from vasa vasorum occlusion causes medial degeneration characterised by smooth muscle cell loss and fragmentation of elastic fibres (medial necrosis), weakening the wall and predisposing to aneurysm or dissection. Intimal lipid cores describe atherosclerosis, not specifically vasa vasorum ischemia. Therefore the most suggestive feature is (a) Medial necrosis with loss of smooth muscle cells and elastic fibres. Recognizing this pattern helps pathologists link vasa vasorum pathology to aortic disease.
End of set. Each explanation contains the correct option and is ≥50 words. Use this HTML in Blogger's HTML editor — it preserves the light red block for the lead question and light yellow blocks for explanations while keeping body text dark black at 16px for SEO and readability.
Subtopic: Nutrient Artery
Keywords & Definitions:
Nutrient artery: The main artery that enters the diaphysis of long bones through the nutrient foramen supplying the bone marrow and inner two-thirds of the cortex.
Metaphysis: The neck portion of a long bone between the epiphysis and diaphysis, rich in blood supply for bone growth.
Epiphysis: The rounded end of a long bone, initially separated from the diaphysis by the growth plate.
Diaphysis: The shaft or central part of a long bone.
Bone vasculature: The network of blood vessels supplying bones, critical for nutrition and repair.
Foramen: An opening or hole in bone for passage of vessels and nerves.
Clinical relevance: Knowledge of nutrient artery direction is vital during orthopedic surgeries to avoid ischemia.
Growth plate: Also called the epiphyseal plate, site of longitudinal bone growth.
Blood supply: Nutrient arteries supply inner compact bone and marrow, while periosteal arteries supply outer bone.
Long bones: Bones longer than they are wide, such as femur, tibia, humerus.
Lead Question - 2012:
Nutrient artery runs?
a) Towards metaphysis
b) Away from metaphysis
c) Away from epiphysis
d) None
Explanation & Answer:
The correct answer is b) Away from metaphysis. Nutrient arteries enter the diaphysis and typically run away from the metaphysis towards the epiphysis. This ensures the metaphysis, which has a rich blood supply from other sources, is not primarily dependent on the nutrient artery. This is crucial in fracture healing and orthopedic interventions.
Q2. Which part of the bone does the nutrient artery primarily supply?
a) Outer periosteum
b) Inner two-thirds of cortex and marrow
c) Epiphyseal cartilage
d) Articular cartilage
Explanation & Answer:
The nutrient artery mainly supplies the inner two-thirds of the cortex and the bone marrow (option b). The outer one-third of the cortex is supplied by periosteal arteries.
Q3. The nutrient foramen is usually located on which surface of long bones?
a) Posterior
b) Anterior
c) Lateral
d) Variable but often on the shaft's anterior surface
Explanation & Answer:
The nutrient foramen is variable but most commonly located on the anterior surface of the shaft (option d). Its position is important during surgical approaches to avoid damaging vessels.
Q4. Which artery supplements the blood supply to the epiphysis?
a) Nutrient artery
b) Metaphyseal artery
c) Periosteal artery
d) Epiphyseal artery
Explanation & Answer:
The epiphyseal artery (option d) supplies the epiphysis, which is separate from the nutrient artery that mainly supplies the diaphysis and medullary cavity.
Q5 (Clinical). Damage to the nutrient artery during fracture fixation can result in?
a) Delayed union or nonunion
b) Increased bone growth
c) Immediate bone healing
d) No clinical significance
Explanation & Answer:
Injury to the nutrient artery can cause delayed union or nonunion (option a) due to compromised blood supply essential for bone healing.
Q6. Which embryological layer forms the periosteum that carries periosteal arteries?
a) Endoderm
b) Mesoderm
c) Ectoderm
d) Neural crest
Explanation & Answer:
The periosteum, containing periosteal arteries, is derived from the mesoderm (option b), which also forms most skeletal components.
Q7. Which of the following is NOT a source of blood supply to long bones?
a) Nutrient artery
b) Periosteal arteries
c) Metaphyseal and epiphyseal arteries
d) Pulmonary arteries
Explanation & Answer:
Pulmonary arteries (option d) do not supply bones; they are part of the lung circulation. Bones receive blood from nutrient, periosteal, metaphyseal, and epiphyseal arteries.
Q8. The nutrient artery enters the bone via?
a) Epiphyseal plate
b) Nutrient foramen
c) Metaphyseal artery
d) Periosteum
Explanation & Answer:
The nutrient artery enters the bone through the nutrient foramen (option b), a small opening in the diaphyseal cortex.
Q9 (Clinical). Nutrient artery injury is most critical in which type of bone fracture?
a) Greenstick fracture
b) Comminuted fracture
c) Spiral fracture
d) Transverse fracture of diaphysis
Explanation & Answer:
Transverse diaphyseal fractures (option d) can disrupt the nutrient artery, risking compromised blood supply and delayed healing.
Q10. The nutrient artery usually arises from which blood vessel?
a) Periosteal arteries
b) Major systemic arteries near the bone
c) Venous system
d) Lymphatic vessels
Explanation & Answer:
The nutrient artery typically arises from major systemic arteries near the bone (option b), such as the femoral artery for the femur
Subtopic: Coronary Sinus Development
Keywords & Definitions:
Coronary sinus: A large venous channel collecting blood from coronary veins and draining into the right atrium.
Truncus arteriosus: Embryonic arterial outflow tract that later divides into aorta and pulmonary artery.
Conus: Also called conus arteriosus, part of the embryonic outflow tract of the heart.
Sinus venosus: Embryonic structure receiving systemic venous blood, contributes to parts of atria and venous valves.
Atrioventricular (AV) canal: The embryonic heart canal between atria and ventricles, precursor to AV valves.
Embryonic heart: The developing heart in the fetus, undergoing morphogenesis to form adult heart structures.
Venous system: Network of veins returning deoxygenated blood to the heart.
Right atrium: Chamber of the heart receiving systemic venous blood.
Cardiac morphogenesis: Process of heart development and formation of chambers and vessels.
Endocardial cushions: Tissue masses important for septation and valve formation.
Q1 (2012). Coronary sinus develops from?
a) Truncus arteriosus
b) Conus
c) Sinus venosus
d) AV canal
Explanation & Answer:
The correct answer is c) Sinus venosus.
The coronary sinus arises from the left horn of the sinus venosus during embryonic heart development. It serves as the major venous drainage channel for the heart muscle, emptying into the right atrium. The truncus arteriosus and conus are involved in arterial outflow tract formation, while the AV canal develops into the atrioventricular valves.
Q2. The left horn of the sinus venosus forms which adult structure?
a) Coronary sinus
b) Left atrium
c) Pulmonary vein
d) Right atrium
Explanation & Answer:
The left horn of the sinus venosus develops into the coronary sinus, which collects venous blood from the heart muscle. The right horn contributes mainly to the smooth part of the right atrium.
Q3. The smooth part of the right atrium develops from:
a) Truncus arteriosus
b) Left horn of sinus venosus
c) Right horn of sinus venosus
d) AV canal
Explanation & Answer:
The smooth portion of the right atrium (sinus venarum) is derived from the right horn of the sinus venosus as it becomes incorporated into the atrial wall during development.
Q4 (Clinical). Persistent left superior vena cava draining into the coronary sinus is due to failure of regression of:
a) Left anterior cardinal vein
b) Right anterior cardinal vein
c) Sinus venosus
d) AV canal
Explanation & Answer:
Persistent left superior vena cava results from failure of the left anterior cardinal vein to regress. It typically drains into the coronary sinus, which is derived from the left horn of sinus venosus, potentially causing coronary sinus enlargement.
Q5. The atrioventricular canal contributes to:
a) Formation of AV valves
b) Formation of coronary sinus
c) Formation of pulmonary veins
d) Formation of truncus arteriosus
Explanation & Answer:
The AV canal gives rise to endocardial cushions that participate in forming the atrioventricular septum and valves, but it does not contribute to the coronary sinus.
Q6. The truncus arteriosus gives rise to:
a) Pulmonary artery and aorta
b) Coronary sinus
c) Left atrium
d) Right atrium
Explanation & Answer:
The truncus arteriosus divides into the ascending aorta and pulmonary trunk, forming the arterial outflow tracts of the heart, unrelated to venous structures like the coronary sinus.
Q7 (Clinical). An absent coronary sinus may be due to:
a) Agenesis of left horn of sinus venosus
b) Abnormal AV canal development
c) Truncus arteriosus defect
d) Conus malformation
Explanation & Answer:
Agenesis or maldevelopment of the left horn of sinus venosus can result in absent or hypoplastic coronary sinus, affecting venous drainage from the heart muscle.
Q8. The sinus venosus initially receives blood from all except:
a) Common cardinal veins
b) Umbilical veins
c) Vitelline veins
d) Pulmonary veins
Explanation & Answer:
The sinus venosus receives blood from common cardinal, vitelline, and umbilical veins, but pulmonary veins drain separately into the left atrium.
Q9. The venous valves of the heart develop from:
a) Endocardial cushions
b) Sinus venosus
c) AV canal
d) Truncus arteriosus
Explanation & Answer:
Venous valves such as the valve of the inferior vena cava develop from tissues near the sinus venosus region during cardiac development.
Q10. The conus arteriosus contributes to:
a) Right ventricular outflow tract
b) Coronary sinus
c) Left atrium
d) AV valves
Explanation & Answer:
The conus arteriosus forms the smooth part of the right ventricular outflow tract, directing blood into the pulmonary trunk.
Keywords & Definitions
Pharyngeal arches: Transient embryonic structures forming head–neck skeleton, muscles, arteries, and nerves.
Aortic arch arteries: Arterial segments within each pharyngeal arch that remodel into definitive great vessels.
1st arch artery (Mandibular): Largely regresses; persistent part forms maxillary artery.
2nd arch artery (Hyoid/Stapedial): Forms stapedial and hyoid arteries; mostly regresses postnatally.
3rd arch artery: Forms common carotids and proximal internal carotids.
4th arch artery: Left contributes to aortic arch; right to proximal right subclavian.
6th arch artery: Forms proximal pulmonary arteries; left forms ductus arteriosus.
Stapedial artery: Embryonic vessel through stapes; persistence may cause pulsatile tinnitus/bleeding in ear surgery.
Maxillary artery: Terminal branch of external carotid; derivative of 1st arch artery.
Ductus arteriosus: Left 6th arch derivative connecting pulmonary trunk to aorta in fetus.
Aberrant right subclavian: Vessel coursing behind esophagus due to right 4th arch regression.
Common carotid artery: Major neck artery from 3rd arch; external carotid sprouts from ventral aorta.
Recurrent laryngeal nerve: Loops under 6th arch derivatives; left under ductus/arch, right under subclavian.
Reichert’s cartilage: Cartilage of 2nd arch (stapes, styloid process, lesser hyoid horn).
Meckel’s cartilage: 1st arch cartilage (malleus, incus, sphenomandibular ligament).
Chapter: Embryology – Cardiovascular Development
Topic: Pharyngeal (Branchial) Arch Arteries
Subtopic: Derivatives of Individual Arch Arteries & Clinical Correlates
Lead Question – 2012
Artery of 2nd pharyngeal arch is ?
a) Maxillary artery
b) Stapedial artery
c) Subclavian artery
d) Common carotid artery
Explanation (Answer inside):
The 2nd pharyngeal arch gives rise to the stapedial (and hyoid) arteries, which largely regress postnatally. The maxillary artery is from the 1st arch; the common carotid from the 3rd arch; the subclavian (proximal right) from the 4th arch. Hence, the correct option is b) Stapedial artery.
1) The definitive derivative of the 1st pharyngeal arch artery is:
a) Superior thyroid artery
b) Maxillary artery
c) Stapedial artery
d) Lingual artery
Explanation: The 1st arch artery largely disappears; its remnant persists as the maxillary artery, a major branch of the external carotid. Superior thyroid and lingual arteries arise from the external carotid but are not arch derivatives. Stapedial belongs to the 2nd arch. Correct option: b) Maxillary artery.
2) The 3rd pharyngeal arch artery predominantly forms:
a) External carotid artery entirely
b) Common carotid and proximal internal carotid arteries
c) Vertebral arteries
d) Distal internal carotid artery
Explanation: Third arch arteries become the common carotids and the proximal internal carotids. The external carotid sprouts from the ventral aorta; distal internal carotid derives from dorsal aorta. Vertebral arteries form from intersegmental arteries. Correct option: b) Common carotid and proximal internal carotid arteries.
3) The right 4th pharyngeal arch artery contributes to the:
a) Entire right subclavian artery
b) Proximal segment of right subclavian artery
c) Distal right subclavian via dorsal aorta only
d) Brachiocephalic trunk entirely
Explanation: The right 4th arch forms the proximal right subclavian artery. The distal subclavian arises from the right dorsal aorta and right 7th intersegmental artery. The brachiocephalic trunk develops from the aortic sac. Correct: b) Proximal segment of right subclavian artery.
4) The left 4th pharyngeal arch artery becomes part of the:
a) Left common carotid artery
b) Left pulmonary artery
c) Arch of aorta between left common carotid and left subclavian
d) Descending thoracic aorta
Explanation: The left 4th arch contributes to the arch of aorta segment between left common carotid and left subclavian origins. Left pulmonary is from 6th arch; descending thoracic aorta is dorsal aorta. Correct: c) Arch of aorta between left common carotid and left subclavian.
5) The 6th pharyngeal arch arteries form which key fetal structure?
a) Ductus venosus
b) Ductus arteriosus
c) Ligamentum teres
d) Coronary arteries
Explanation: The sixth (pulmonary) arches produce the proximal pulmonary arteries. On the left, the distal part persists as the ductus arteriosus, later becoming ligamentum arteriosum. Ductus venosus and ligamentum teres are hepatic fetal channels; coronary arteries arise from aortic sinuses. Correct: b) Ductus arteriosus.
6) A patient with pulsatile tinnitus has a vascular middle-ear mass due to a persistent embryonic artery. The vessel most likely persisting is:
a) Stapedial artery
b) Hyoid artery
c) Inferior tympanic artery
d) Ascending pharyngeal artery
Explanation: Persistence of the stapedial artery (2nd arch derivative) may traverse the middle ear, causing pulsatile tinnitus and bleeding risk during otologic surgery. Hyoid artery accompanies stapedial embryologically but usually regresses. Inferior tympanic/ascending pharyngeal are adult branches. Correct: a) Stapedial artery.
7) Aberrant right subclavian artery (dysphagia lusoria) most commonly results from:
a) Persistence of right 4th arch and right dorsal aorta proximally
b) Regression of right 4th arch with persistence of distal right dorsal aorta
c) Persistence of right 6th arch distal segment
d) Failure of left dorsal aorta to form
Explanation: An aberrant right subclavian arises when the right 4th arch regresses and the distal right dorsal aorta persists, so the vessel originates distal to the left subclavian and passes posterior to the esophagus, causing dysphagia. Correct: b) Regression of right 4th arch with persistence of distal right dorsal aorta.
8) The facial nerve is the cranial nerve of which pharyngeal arch, whose artery is the stapedial?
a) First arch
b) Second arch
c) Third arch
d) Fourth arch
Explanation: The second (hyoid) arch is supplied by the facial nerve (CN VII) and has the stapedial/hyoid arteries. The first arch is trigeminal (CN V), third is glossopharyngeal (CN IX), and fourth–sixth are vagus (CN X) branches. Correct: b) Second arch.
9) Interruption of the aortic arch (type B) is classically due to abnormal development of which arch artery?
a) Third arch
b) Fourth arch
c) Fifth arch
d) Sixth arch
Explanation: Type B interruption typically reflects failure involving the left fourth arch segment that normally contributes to the aortic arch between left carotid and subclavian. Third arch forms carotids; sixth forms pulmonary/ductus; a true fifth arch is controversial/transient. Correct: b) Fourth arch.
10) Patent ductus arteriosus represents persistence of a derivative of which embryonic arch?
a) Left 6th arch
b) Right 6th arch
c) Left 4th arch
d) Right 4th arch
Explanation: The ductus arteriosus is the distal portion of the left sixth arch artery. Postnatally, closure forms the ligamentum arteriosum; failure results in PDA with left-to-right shunt and continuous murmur. Right sixth arch does not form a ductus. Correct: a) Left 6th arch.
11) Which vessel is formed by sprouting from the ventral aorta near the third arch rather than directly from an arch artery?
a) External carotid artery
b) Internal carotid artery (proximal)
c) Vertebral artery
d) Subclavian artery (proximal right)
Explanation: The external carotid artery develops as a sprout from the ventral aorta near the third arch, while the proximal internal carotid derives from the third arch itself. Vertebral arteries form from cervical intersegmentals; proximal right subclavian is from the right 4th arch. Correct: a) External carotid artery.
Subtopic: Interatrial Septum Formation
Keywords & Definitions:
Fossa ovalis: Oval depression in the right atrium, remnant of fetal foramen ovale.
Septum primum: First interatrial septum growing downward during fetal heart development.
Septum secundum: Second crescent-shaped septum overlapping septum primum, forming the foramen ovale.
Ductus arteriosus: Vessel connecting pulmonary artery to aorta in fetus, bypassing lungs.
Ductus venosus: Vessel shunting blood from umbilical vein to inferior vena cava in fetus.
Foramen ovale: Opening in fetal heart allowing blood flow between atria, closes after birth.
Remnant: Vestigial structure from fetal life.
Interatrial septum: Wall between left and right atria.
Embryology: Study of prenatal development.
Patent foramen ovale (PFO): Incomplete closure of foramen ovale after birth.
Lead Question - 2012:
Fossa ovalis is a remnant of?
a) Septum primum
b) Septum secundum
c) Ductus arteriosus
d) Ductus venosus
Explanation & Answer:
The correct answer is a) Septum primum. The fossa ovalis represents the closed remnant of the foramen ovale, which is originally formed by the septum primum during fetal development. After birth, increased left atrial pressure causes fusion of septum primum and septum secundum, closing the foramen ovale and leaving the fossa ovalis.
Q2. What is the function of foramen ovale in fetal circulation?
a) Connects left and right ventricles
b) Allows blood to bypass lungs by flowing between atria
c) Connects pulmonary artery and aorta
d) Connects umbilical vein to inferior vena cava
Explanation & Answer:
Foramen ovale allows oxygenated blood to bypass the non-functioning fetal lungs by flowing directly from right atrium to left atrium, essential for fetal circulation.
Q3. Which structure closes shortly after birth due to increased left atrial pressure?
a) Foramen ovale
b) Ductus arteriosus
c) Ductus venosus
d) Umbilical vein
Explanation & Answer:
The foramen ovale closes functionally shortly after birth as increased pulmonary blood flow raises left atrial pressure, pressing septum primum against septum secundum.
Q4 (Clinical). Patent foramen ovale (PFO) may cause which clinical condition?
a) Stroke due to paradoxical embolism
b) Pulmonary hypertension
c) Aortic stenosis
d) Coarctation of aorta
Explanation & Answer:
PFO can allow venous emboli to bypass lungs and enter systemic circulation causing paradoxical embolism and ischemic stroke, especially after activities increasing right atrial pressure.
Q5. Which fetal vessel connects pulmonary artery to descending aorta?
a) Ductus arteriosus
b) Ductus venosus
c) Foramen ovale
d) Umbilical artery
Explanation & Answer:
The ductus arteriosus shunts blood from pulmonary artery to aorta, bypassing fetal lungs. It normally closes after birth to become ligamentum arteriosum.
Q6. The septum secundum forms which part of the interatrial septum?
a) Thin membranous part
b) Thick muscular ridge
c) Valve of foramen ovale
d) None of the above
Explanation & Answer:
The septum secundum is a thick muscular ridge that overlaps the foramen ovale and works with septum primum to prevent backflow after birth.
Q7. Ductus venosus connects which vessels in fetal circulation?
a) Umbilical vein to inferior vena cava
b) Pulmonary artery to aorta
c) Right atrium to left atrium
d) Umbilical artery to placenta
Explanation & Answer:
Ductus venosus bypasses the liver by connecting umbilical vein directly to the inferior vena cava, facilitating rapid oxygenated blood flow to the heart.
Q8 (Clinical). Which congenital defect results from failure of fusion of septum primum and secundum?
a) Atrial septal defect (ostium secundum type)
b) Ventricular septal defect
c) Patent ductus arteriosus
d) Tetralogy of Fallot
Explanation & Answer:
Failure of septum primum and secundum fusion causes atrial septal defect of ostium secundum type, leading to left-to-right shunt and possible right heart overload.
Q9. Which adult structure corresponds to the ductus arteriosus?
a) Ligamentum arteriosum
b) Ligamentum venosum
c) Fossa ovalis
d) Median umbilical ligament
Explanation & Answer:
After birth, the ductus arteriosus closes and becomes ligamentum arteriosum, a fibrous remnant connecting pulmonary artery and aorta.
Q10. Which statement about fetal circulation is TRUE?
a) Foramen ovale allows blood flow from left atrium to right atrium
b) Ductus arteriosus carries blood from aorta to pulmonary artery
c) Ductus venosus bypasses fetal liver
d) Umbilical arteries carry oxygenated blood
Explanation & Answer:
The ductus venosus connects the umbilical vein to inferior vena cava, allowing oxygen-rich blood to bypass the liver and enter systemic circulation rapidly.